Sei sulla pagina 1di 619

8/22/2016

MyPastest

Back to Filters (/Secure/TestMe/Filter/429893/QA)

Question 1 of 298

A 26-year-old woman presents to her GP complaining of intermittent abdominal distension


and bloating, which changes with her menstrual cycle, and is interspersed with bouts of loose
motions. She works as a trader in a busy office and finds work stressful: she has previously
taken a course of Prozac for depression/anxiety. Examination, bloods and sigmoidoscopy
were all normal.
What is the best-fit diagnosis?
A

Chronic pancreatitis

Ulcerative colitis

Peptic ulcer disease

Diverticulitis

Irritable bowel syndrome

Explanation
Irritable bowel syndrome
Irritable bowel syndrome has a female to male preponderance of 2:1 and frequently occurs in
patients with underlying problems of anxiety. Examination and investigations are invariably
normal. Any history of weight loss, bleeding, onset > 40 years of age or faecal incontinence
would not fit with this picture, however, and these features should trigger other investigations
if the basic examination/investigations are unremarkable.
Management
Management is with antispasmodics such as peppermint oil (eg Colpermin), a high-fibre diet
and avoidance of trigger foods (a significant proportion of patients report improvement
when cutting out dairy foods). In many cases, serotonin-reuptake inhibitors such as Prozac
also have positive effects on symptoms.
1406

Next Question

https://mypastest.pastest.com/Secure/TestMe/Browser/429893

1/2

8/22/2016

MyPastest

Previous Question

Tag Question

Feedback

End Review

Difficulty: Easy
Peer Responses

Session Progress
Responses Correct:

Responses Incorrect:

298

Responses Total:

298

Responses - % Correct:

0%

Blog (https://www.pastest.com/blog) About Pastest (https://www.pastest.com/about-us)


Contact Us (https://www.pastest.com/contact-us) Help (https://www.pastest.com/help)
Pastest 2016

https://mypastest.pastest.com/Secure/TestMe/Browser/429893

2/2

8/22/2016

MyPastest

Back to Filters (/Secure/TestMe/Filter/429893/QA)

Question 2 of 298

A 48-year-old publican presents with acute-onset confusion and a mild fever. On examination
he has signs of chronic liver disease and ascites and is generally tender over his abdomen.
Blood tests reveal mildly raised aspartate aminotrasferase (AST) and alanine
aminotransferase (ALT) levels and a bilirubin of 186 mol/l. His creatinine is 145 mol/l. His
international normalised ratio (INR) is 2 and he has a mixed-picture anaemia with a
haemoglobin of 9.8 g/dl, low platelets and an elevated neutrophil count. Ascitic tap reveals
fluid with a polymorphonuclear cell count of > 250/mm3.
What is the most likely diagnosis?
A

Spontaneous bacterial peritonitis

Perforated duodenal ulcer

Cholangitis

Cholecystitis

Acute pancreatitis

Explanation
Spontaneous bacterial peritonitis
This man clearly has alcoholic cirrhosis, which is decompensated with ascites. The ascites has
become infected and spontaneous bacterial peritonitis has developed. Diagnosis is made on
the basis of a white count of > 250 cells/mm3, the presence of bacteria on Gram staining and
a positive ascitic fluid culture. Pathogens are usually Gram negative, and include Escherichia
coli, Klebsiella pneumoniae and enterococci. The treatment of choice includes
piperacillin/tazobactam for severe disease, although ciprofloxacin or ofloxacin might be used
in selected patients.
1407

Next Question

Previous Question

Tag Question

https://mypastest.pastest.com/Secure/TestMe/Browser/429893

Feedback

End Review
1/2

8/22/2016

MyPastest

Difficulty: Easy
Peer Responses

Session Progress
Responses Correct:

Responses Incorrect:

298

Responses Total:

298

Responses - % Correct:

0%

Blog (https://www.pastest.com/blog) About Pastest (https://www.pastest.com/about-us)


Contact Us (https://www.pastest.com/contact-us) Help (https://www.pastest.com/help)
Pastest 2016

https://mypastest.pastest.com/Secure/TestMe/Browser/429893

2/2

8/22/2016

MyPastest

Back to Filters (/Secure/TestMe/Filter/429893/QA)

Question 3 of 298

An 82-year-old woman is admitted from a nursing home with profuse diarrhoea. She was
discharged 2 weeks earlier from the orthopaedic ward, where she was treated for a fractured
hip. There was some evidence of osteomyelitis during that admission and she was treated
with clindamycin and discharged on tablets. On examination she is drowsy and dehydrated,
with lower abdominal tenderness. She soils the bed with watery diarrhoea during the
examination. Blood tests confirm pre-renal failure.
What is the most likely diagnosis?
A

Salmonellosis

Ulcerative colitis

Enteric parasitic infection

Pseudomembranous colitis

Colonic malignancy

Explanation
Pseudomembranous colitis
Pseudomembranous colitis can occur in up to 10% of patients who have received a course of
clindamycin. In addition, it is thought that many nursing-home residents show chronic
carriage of Clostridium difficile (the causative pathogen). Sigmoidoscopy will usually reveal
raised, white-yellow exudative plaques adherent to the colonic mucosa (the
pseudomembrane). The diagnosis is made by the presence of clostridium toxin in the stool.
Treatment is with oral metronidazole or vancomycin for 1014 days, accompanied by
appropriate rehydration therapy. The mortality rate is as high as 10% in the elderly.
Salmonellosis would not be impossible here but the osteomyelitis associated with this tends
to affect the long bones and typically occurs in patients with sickle cell disease.
1408

Next Question

Previous Question

Tag Question

https://mypastest.pastest.com/Secure/TestMe/Browser/429893

Feedback

End Review
1/2

8/22/2016

MyPastest

Difficulty: Easy
Peer Responses

Session Progress
Responses Correct:

Responses Incorrect:

298

Responses Total:

298

Responses - % Correct:

0%

Blog (https://www.pastest.com/blog) About Pastest (https://www.pastest.com/about-us)


Contact Us (https://www.pastest.com/contact-us) Help (https://www.pastest.com/help)
Pastest 2016

https://mypastest.pastest.com/Secure/TestMe/Browser/429893

2/2

8/22/2016

MyPastest

Back to Filters (/Secure/TestMe/Filter/429893/QA)

Question 4 of 298

Which one of the following statements best describes a feature of irritable bowel syndrome?
A

Characterised by nocturnal diarrhoea

If there is nausea and vomiting the diagnosis should be reconsidered

Weight loss becomes more evident as the disease runs a chronic course

Sigmoidoscopy findings are often diagnostic

A diet high in soluble fibre is often prescribed for the treatment of the syndrome

Explanation
Irritable bowel syndrome
Irritable bowel syndrome is a functional disorder of the alimentary tract that is characterised
by altered bowel function, constipation and diarrhoea, with or without abdominal pain,
nausea and vomiting, with no significant physical, laboratory or histological findings.
Anaemia, occult blood in the stool, weight loss or nocturnal symptoms cannot be attributed
to irritable bowel syndrome. A diet high in soluble fibre can be useful in some patients and
others seem to gain benefit from excluding dairy foods.
1609

Next Question

Previous Question

Tag Question

Feedback

End Review

Difficulty: Average
Peer Responses

https://mypastest.pastest.com/Secure/TestMe/Browser/429893

1/2

8/22/2016

MyPastest

Session Progress
Responses Correct:

Responses Incorrect:

298

Responses Total:

298

Responses - % Correct:

0%

Blog (https://www.pastest.com/blog) About Pastest (https://www.pastest.com/about-us)


Contact Us (https://www.pastest.com/contact-us) Help (https://www.pastest.com/help)
Pastest 2016

https://mypastest.pastest.com/Secure/TestMe/Browser/429893

2/2

8/22/2016

MyPastest

Back to Filters (/Secure/TestMe/Filter/429893/QA)

Question 5 of 298

Which one of the following pathological changes favours a diagnosis of ulcerative colitis over
Crohns disease?
A

Ileal involvement

Crypt abscesses

Transmural involvement

Granulomas

Skip lesions

Explanation
Inflammatory bowel diseases
Crohns disease
Crohns disease can involve any segment in the alimentary canal but distal ileum involvement
is characteristic. The inflammatory process involves all layers of the bowel with the formation
of non-caseating granulomas, ulcers and fistulae. Discontinuity of the inflammatory process
across the bowel (skip lesions) is also characteristic.
Ulcerative colitis
In ulcerative colitis there is diffuse, continuous involvement of the colon with proctitis as an
early feature in 90% of cases. The inflammation is confined to the mucosa and lamina propria
with crypt abscess formation. Ileal involvement is not a common feature of ulcerative colitis
but the distal segment of the ileum can be involved in the inflammatory process from
adjacent inflamed colonic segment (backwash ileitis).
Table of pathological findings;
Crohn's Disease

Ulcerative colitis

Transmural inflammation

Mucosa and submucosa only involved

Mucosal ulcers (in 30% only) Inflammatory cell infiltrate

https://mypastest.pastest.com/Secure/TestMe/Browser/429893

1/2

8/22/2016

MyPastest

Fissuring ulcers

Crypt abscesses

Lymphoid aggregates
Neutrophil infiltrates
1610

Next Question

Previous Question

Tag Question

Feedback

End Review

Difficulty: Average
Peer Responses

Session Progress
Responses Correct:

Responses Incorrect:

298

Responses Total:

298

Responses - % Correct:

0%

Blog (https://www.pastest.com/blog) About Pastest (https://www.pastest.com/about-us)


Contact Us (https://www.pastest.com/contact-us) Help (https://www.pastest.com/help)
Pastest 2016

https://mypastest.pastest.com/Secure/TestMe/Browser/429893

2/2

8/22/2016

MyPastest

Back to Filters (/Secure/TestMe/Filter/429893/QA)

Question 6 of 298

Which one of the following conditions is most likely to be associated with gastric acid
hypersecretion?
A

Pernicious anaemia

Large-bowel resection

Vasoactive intestinal polypeptide- (VIP-) secreting tumour

Systemic mastocytosis

Cushing syndrome

Explanation
Gastric acid secretion
In the stomach, parietal cell acid secretion is stimulated by one of the three principal
mediators: gastrin, acetylcholine and histamine.
Several hormones in the small intestine inhibit gastrin and gastric acid secretion in vivo.
Resection of the small bowel leads to the removal of this inhibition and gastric acid
hypersecretion results. (Large-bowel resection has no effect on gastric acid secretion.)
Systemic mastocytosis is associated with increased histamine production.
In pernicious anaemia, gastrin levels are elevated in the presence of mucosal atrophy in
the body of the stomach; acid production is therefore reduced.
Steroid therapy and Cushing syndrome have been associated with peptic ulcer disease;
it has not been demonstrated that this possible relationship is due to gastric acid
hypersecretion, however.
Vasoactive intestinal polypeptide (VIP) inhibits gastric acid secretion and achlorhydria
is a feature of VIP-secreting tumours.
1611

Next Question
https://mypastest.pastest.com/Secure/TestMe/Browser/429893

1/2

8/22/2016

MyPastest

Previous Question

Tag Question

Feedback

End Review

Difficulty: Difficult
Peer Responses

Session Progress
Responses Correct:

Responses Incorrect:

298

Responses Total:

298

Responses - % Correct:

0%

Blog (https://www.pastest.com/blog) About Pastest (https://www.pastest.com/about-us)


Contact Us (https://www.pastest.com/contact-us) Help (https://www.pastest.com/help)
Pastest 2016

https://mypastest.pastest.com/Secure/TestMe/Browser/429893

2/2

8/22/2016

MyPastest

Back to Filters (/Secure/TestMe/Filter/429893/QA)

Question 7 of 298

Which one of the following clinical findings is most suggestive of a pyogenic liver abscess
rather than an amoebic liver abscess?
A

Patient usually aged over 60

Recent bowel surgery

Raised white cell count

History of recent biliary colic and fever

Solitary abscess in the right lobe of the liver

Explanation
Liver abscess
In developed countries, liver abscess (commonly of bacterial origin) usually complicates
pre-existing biliary and gastrointestinal tract infections.
Pyogenic liver abscess is caused by enteric flora (Escherichia coli,Klebsiella) and
Staphylococcus aureus.
Unlike amoebic liver abscess, the symptoms of pyogenic abscess are those of a
systemic febrile illness lasting for only days to weeks and multiple abscesses are usually
identified on ultrasound examination of the liver.
A raised white cell count and other acute-phase reactants are common in both
conditions.
A solitary abscess in the right lobe of the liver is typical of amoebic liver abscess.
A history of chronic diarrhoea might be elicited in patients with amoebic liver abscess.
A history of recent biliary colic and fever is much more suggestive of cholecystitis.
1613

Next Question

https://mypastest.pastest.com/Secure/TestMe/Browser/429893

1/2

8/22/2016

MyPastest

Previous Question

Tag Question

Feedback

End Review

Difficulty: Difficult
Peer Responses

Session Progress
Responses Correct:

Responses Incorrect:

298

Responses Total:

298

Responses - % Correct:

0%

Blog (https://www.pastest.com/blog) About Pastest (https://www.pastest.com/about-us)


Contact Us (https://www.pastest.com/contact-us) Help (https://www.pastest.com/help)
Pastest 2016

https://mypastest.pastest.com/Secure/TestMe/Browser/429893

2/2

8/22/2016

MyPastest

Back to Filters (/Secure/TestMe/Filter/429893/QA)

Question 8 of 298

Which one of the following disorders is most strongly associated with Helicobacter pylori
infection?
A

Non-ulcer dyspepsia

Reflux oesophagitis

Coeliac disease

Gastric lymphoma

Achalasia of the cardia

Explanation
Associations of Helicobacter pylori infection
Consequences of Helicobacter pylori infection include duodenal and gastric ulcer and their
complications (eg bleeding and perforation), atrophic gastritis, gastric cancer and mucosaassociated lymphoid tissue (MALT) lymphoma. Epidemiological studies have shown that 95%
of low-grade gastric MALT lymphomas are associated with H. pylori, and these lymphomas
have been shown to arise from B-cell clones at the site of H. pylori gastritis. Eradication of H.
pylori can produce clinical and histological remission of these tumours in 7080% of cases,
but treated patients must be followed closely for residual or recurrent lymphoma.
Patients with a variety of upper gastrointestinal symptoms that have been called non-ulcer
dyspepsia may or may not be infected with H. pylori; at present, however, there is no
generally recognised association of non-ulcer dyspepsia with H. pylori infection.
Several mechanisms operate in the pathogenesis of reflux oesophagitis but there is no
recognised association with H. pylori infection. More recently, it has also become evident that
individuals without H. pylori are at greater risk for gastroesophageal reflux disease and its
sequelae, Barretts oesophagus and adenocarcinoma of the oesophagus. Achalasia of the
cardia is a motility disorder leading to failure of relaxation of the lower end of the
oesophagus and is not associated with H. pylori infection. Coeliac disease is a malabsorption
syndrome due to gluten sensitivity; it is an autoimmnune disorder and is not associated with
H. pylori infection.
1614

https://mypastest.pastest.com/Secure/TestMe/Browser/429893

1/2

8/22/2016

MyPastest

Next Question

Previous Question

Tag Question

Feedback

End Review

Difficulty: Average
Peer Responses

Session Progress
Responses Correct:

Responses Incorrect:

298

Responses Total:

298

Responses - % Correct:

0%

Blog (https://www.pastest.com/blog) About Pastest (https://www.pastest.com/about-us)


Contact Us (https://www.pastest.com/contact-us) Help (https://www.pastest.com/help)
Pastest 2016

https://mypastest.pastest.com/Secure/TestMe/Browser/429893

2/2

8/22/2016

MyPastest

Back to Filters (/Secure/TestMe/Filter/429893/QA)

Question 9 of 298

Which one of the following is accurate with regard to alcoholic liver disease?
A

Men are more susceptible than women

In alcoholic hepatitis the aspartate aminotransferase to alanine aminotransferase


(AST:ALT) ratio is 2:1

Hepatic iron overload is indicative of concomitant heterozygote haemochromatosis

Alcoholic fatty infiltration is irreversible once established

Unlike other causes of liver cirrhosis, alcoholic cirrhosis does not progress to
hepatoma

Explanation
Alcoholic liver disease
Alcoholic liver diseases include acute alcoholic hepatitis, chronic active hepatitis and
alcoholic cirrhosis.
Alcoholic liver disease is the most common cause of cirrhosis in developed countries.
Women are more susceptible to alcohol-related liver disease than men, even when
consumption is corrected for body weight.
Unlike viral hepatitis, alcoholic hepatitis is associated with a reversed AST:ALT ratio of
2:1.
Transferrin saturation and serum ferritin are commonly increased in alcoholic liver
disease and minor degrees of iron overload are common.
Alcoholic hepatitis and alcoholic fatty infiltration are reversible with abstinence and
adequate nutrition.
1615

Next Question

https://mypastest.pastest.com/Secure/TestMe/Browser/429893

1/2

8/22/2016

MyPastest

Previous Question

Tag Question

Feedback

End Review

Difficulty: Average
Peer Responses

Session Progress
Responses Correct:

Responses Incorrect:

298

Responses Total:

298

Responses - % Correct:

0%

Blog (https://www.pastest.com/blog) About Pastest (https://www.pastest.com/about-us)


Contact Us (https://www.pastest.com/contact-us) Help (https://www.pastest.com/help)
Pastest 2016

https://mypastest.pastest.com/Secure/TestMe/Browser/429893

2/2

8/22/2016

MyPastest

Back to Filters (/Secure/TestMe/Filter/429893/QA)

Question 10 of 298

Which one of the following conditions is expected to be associated with normal urinary Dxylose test findings?
A

Coeliac disease

Chronic pancreatitis

Blind loop syndrome

Chronic renal failure

Liver cirrhosis with ascites

Explanation
D-xylose test
This test distinguishes between malabsorption due to small-intestinal diseases and
malabsorption due to pancreatic exocrine insufficiency. A 5-hour urinary excretion of 5 g or
greater is normal following the oral administration of 25 g of D-xylose to a well-hydrated
subject.
Decreased xylose absorption and excretion are found:
In patients with damage to the proximal small intestine
When there is bacterial overgrowth in the small intestine (the bacteria catabolise the
xylose)
Patients with pancreatic steatorrhoea usually have normal xylose absorption. Abnormal
results might be encountered in renal failure, in the elderly and in patients with ascites due to
an excretion defect rather than malabsorption.
1616

Next Question

Previous Question

Tag Question

https://mypastest.pastest.com/Secure/TestMe/Browser/429893

Feedback

End Review
1/2

8/22/2016

MyPastest

Difficulty: Difficult
Peer Responses

Session Progress
Responses Correct:

Responses Incorrect:

298

Responses Total:

298

Responses - % Correct:

0%

Blog (https://www.pastest.com/blog) About Pastest (https://www.pastest.com/about-us)


Contact Us (https://www.pastest.com/contact-us) Help (https://www.pastest.com/help)
Pastest 2016

https://mypastest.pastest.com/Secure/TestMe/Browser/429893

2/2

8/22/2016

MyPastest

Back to Filters (/Secure/TestMe/Filter/429893/QA)

Question 11 of 298

In a patient with liver cirrhosis, which one of the features listed below is characteristic of
portal hypertension?
A

Jaundice

Gynaecomastia

Spider telangiectases

Hepatomegaly

Oesophageal varices

Explanation
Portal hypertension
The liver receives approximately 1500 ml of blood each minute, two-thirds of which is
provided by the portal vein. Portal hypertension is present when the wedged hepatic vein
pressure is more than 5 mmHg higher than the inferior vena cava pressure. Because the veins
in the portal system lack valves, increased resistance to flow at any point between the
splanchnic venules and the heart will increase the pressure in all vessels on the intestine site
of the obstruction. This is manifest clinically by the development of porto-systemic collaterals
(oesophageal varices), splenomegaly and/or ascites.
Spider telangiectases, jaundice, hepatomegaly and gynaecomastia are manifestations of
abnormal liver cell function.
1617

Next Question

Previous Question

Tag Question

Feedback

End Review

Difficulty: Easy
Peer Responses
https://mypastest.pastest.com/Secure/TestMe/Browser/429893

1/2

8/22/2016

MyPastest

Session Progress
Responses Correct:

Responses Incorrect:

298

Responses Total:

298

Responses - % Correct:

0%

Blog (https://www.pastest.com/blog) About Pastest (https://www.pastest.com/about-us)


Contact Us (https://www.pastest.com/contact-us) Help (https://www.pastest.com/help)
Pastest 2016

https://mypastest.pastest.com/Secure/TestMe/Browser/429893

2/2

8/22/2016

MyPastest

Back to Filters (/Secure/TestMe/Filter/429893/QA)

Question 12 of 298

You are asked by a GP to review a 16-year-old girl who appears tremulous, with some
evidence of ataxia. She also has dysarthria, which has developed over time. Otherwise she
appears relatively well. You carry out some screening tests: the alanine aminotransferase
(ALT) is elevated, the serum caeruloplasmin is low and there is increased urinary copper
excretion.
Which diagnosis fits best with this clinical picture?
A

Abuse of alcohol

Wilsons disease

Menkes disease

Drug abuse

Haemochromatosis

Explanation
Wilsons disease
Wilsons disease has a prevalence of 1 in 30,000, with an equal sex distribution. The onset of
symptoms has been described in patients aged between 3 and 40 years. They can present
acutely with so-called fulminant Wilsons disease, with a hepatitic picture, malaise, anorexia,
nausea and jaundice. Alternatively, they can present with a more chronic picture, with
neurological symptoms (as in this case), symptoms of chronic cirrhosis, or occasionally with
psychiatric disorders such as depression or obsessive-compulsive disorder.
Diagnosis and treatment
The diagnosis is based on abnormal liver function tests, increased urinary copper excretion
and decreased serum caeruloplasmin. Liver biopsy at an early stage might reveal focal
necrosis and hepatic steatosis. Late biopsy reveals cirrhosis. The liver copper content is
usually more than five times the upper limit of normal. Wilsons disease is treated with
penicillamine, which acts as a copper chelator.
2187

Next Question
https://mypastest.pastest.com/Secure/TestMe/Browser/429893

1/2

8/22/2016

MyPastest

Previous Question

Tag Question

Feedback

End Review

Difficulty: Easy
Peer Responses

Session Progress
Responses Correct:

Responses Incorrect:

298

Responses Total:

298

Responses - % Correct:

0%

Blog (https://www.pastest.com/blog) About Pastest (https://www.pastest.com/about-us)


Contact Us (https://www.pastest.com/contact-us) Help (https://www.pastest.com/help)
Pastest 2016

https://mypastest.pastest.com/Secure/TestMe/Browser/429893

2/2

8/22/2016

MyPastest

Back to Filters (/Secure/TestMe/Filter/429893/QA)

Question 13 of 298

A 34-year-old publican was admitted from the Emergency Department. His wife said that he
had been suffering from confusion for around 2448 hours. On examination, there were
obvious signs of chronic liver disease as well as nystagmus and cerebellar ataxia. He
appeared very confused. Investigations showed an abnormal alanine aminotransferase (ALT),
mildly raised bilirubin levels and an alkaline phosphatase level just above the upper limit of
normal. His full blood count and glucose are normal.
Which diagnosis fits best with this clinical picture?
A

Wernickes encephalopathy

Acute alcohol poisoning

Drug abuse

Urinary sepsis

Subdural haematoma

Explanation
Wernickes encephalopathy
This neurological picture, with no localising signs but in the presence of signs of chronic liver
disease, is likely to be related to Wernickes encephalopathy. The precipitating cause in this
case is probably chronic liver disease secondary to alcohol abuse. If there had been a history
of head injury, subdural haematoma would have been an alternative diagnosis. Computed
tomography often reveals evidence of cerebral atrophy secondary to chronic alcoholism in
patients with Wernickes encephalopathy.
Management is 100 mg thiamine, intravenously or intramuscularly, followed by oral thiamine
replacement to correct the thiamine deficiency. Untreated, this condition can become
chronic.
2188

Next Question

https://mypastest.pastest.com/Secure/TestMe/Browser/429893

1/2

8/22/2016

MyPastest

Previous Question

Tag Question

Feedback

End Review

Difficulty: Easy
Peer Responses

Session Progress
Responses Correct:

Responses Incorrect:

298

Responses Total:

298

Responses - % Correct:

0%

Blog (https://www.pastest.com/blog) About Pastest (https://www.pastest.com/about-us)


Contact Us (https://www.pastest.com/contact-us) Help (https://www.pastest.com/help)
Pastest 2016

https://mypastest.pastest.com/Secure/TestMe/Browser/429893

2/2

8/22/2016

MyPastest

Back to Filters (/Secure/TestMe/Filter/429893/QA)

Question 14 of 298

A 43-year-old man has been referred to the Gastroenterology Clinic by his GP. There is a long
history of arthralgia and more recently this man has begun to complain of diarrhoea with up
to eight semi-formed oily stools per day. He also complains of excessive abdominal cramps
and bloating, and a general loss of appetite.
Examination reveals signs of weight loss and anaemia, there is mild oedema and evidence of
ascites and a pericardial rub is heard on auscultation. A 72-hour faecal fat collection reveals
10 g fat/24 hours. There is a mixed-picture anaemia, hypocalcaemia, hypokalaemia and
decreased serum albumin. Antigliadin and anti-endomysial antibodies are negative. A smallbowel follow-through study reveals evidence of mucosal oedema.
Which diagnosis fits best with this clinical picture?
A

Coeliac disease

Whipples disease

Ulcerative colitis

Laxative abuse

Giardiasis

Explanation
Whipples disease
Coeliac disease is a possibility with such a history, but negative antigliadin and antiendomysial antibodies make this possibility remote. Whipples disease is a very uncommon
condition, occurring slightly more commonly in men and peaking in the 3060-year age
group.
Diagnosis of Whipples disease is based on biopsy of the small-intestinal lamina propria,
which reveals infiltration by periodic acidSchiff- (PAS-) positive macrophages containing
Gram-positive bacilli. The causative organism of Whipples disease is the bacterium
Tropheryma whipplei.
Standard therapy involves a prolonged period of treatment (some months) with Septrin or a
penicillin-, streptomycin- or tetracycline-based regimen.
2189

https://mypastest.pastest.com/Secure/TestMe/Browser/429893

1/2

8/22/2016

MyPastest

Next Question

Previous Question

Tag Question

Feedback

End Review

Difficulty: Average
Peer Responses

Session Progress
Responses Correct:

Responses Incorrect:

298

Responses Total:

298

Responses - % Correct:

0%

Blog (https://www.pastest.com/blog) About Pastest (https://www.pastest.com/about-us)


Contact Us (https://www.pastest.com/contact-us) Help (https://www.pastest.com/help)
Pastest 2016

https://mypastest.pastest.com/Secure/TestMe/Browser/429893

2/2

8/22/2016

MyPastest

Back to Filters (/Secure/TestMe/Filter/429893/QA)

Question 15 of 298

A 54-year-old woman is brought to the Emergency Department by her relatives. The family
had returned from a holiday in Morocco some 2 weeks earlier. She is intermittently confused,
but a history taken from her relatives confirms that she has suffered a prolonged fever,
myalgia, headaches and cough for some days. Apparently, just after returning to the UK there
was a history of diarrhoea. On examination you notice some faint rose spots, which blanch, on
her chest. Blood testing reveals neutropenia. You send blood, stool and urine samples for
culture.
Which diagnosis fits best with this clinical picture?
A

Malaria

Tuberculosis

Brucellosis

Amoebic liver abscess

Typhoid fever

Explanation
Typhoid fever
Typhoid fever can have an incubation period of anything from a few days to a few weeks.
Diarrhoea or constipation is common at the outset of the illness, but often settles. Later
symptoms include fever, malaise, headache, cough, anorexia, sore throat and the
characteristic maculopapular rose spots, which blanch on pressure. Laboratory testing might
reveal raised transaminases, and neutropenia is common. Multiple blood, stool and urine
cultures are sometimes needed to identify the causative organism, Salmonella typhi.
Acute treatment is with a 14-day course of ciprofloxacin. Chronic carriage is possible, and up
to a 4-week course of ciprofloxacin may be required in this case. The disease is rare in this
country, but it occurs more commonly in parts of the world where there is poor hygiene.
2191

Next Question

https://mypastest.pastest.com/Secure/TestMe/Browser/429893

1/2

8/22/2016

MyPastest

Previous Question

Tag Question

Feedback

End Review

Difficulty: Easy
Peer Responses

Session Progress
Responses Correct:

Responses Incorrect:

298

Responses Total:

298

Responses - % Correct:

0%

Blog (https://www.pastest.com/blog) About Pastest (https://www.pastest.com/about-us)


Contact Us (https://www.pastest.com/contact-us) Help (https://www.pastest.com/help)
Pastest 2016

https://mypastest.pastest.com/Secure/TestMe/Browser/429893

2/2

8/22/2016

MyPastest

Back to Filters (/Secure/TestMe/Filter/429893/QA)

Question 16 of 298

You are asked to review a nursing-home resident who has generalised inflammation of his
oropharynx and is finding it difficult to eat. His past history of note includes the use of a
steroid inhaler for chronic obstructive pulmonary disease. On examination there are areas of
erythema and a number of white plaques accompanied by some white, curd-like material.
Which diagnosis fits best with this clinical picture?
A

Oropharyngeal candidiasis

Human immunodeficiency virus (HIV) infection

Hairy leukoplakia

Dariers disease

Leukoedema

Explanation
Oral thrush
This man is in a nursing home and is using a steroid inhaler for his chronic obstructive
pulmonary disease. It is likely that he also has inadequate oral hygiene and a Candida
infection has taken hold. The best treatment is to encourage mouth-swilling after using the
inhaler and a nystatin mouthwash to get rid of the acute infection. It is worth noting that
dentures can harbour Candida spp., so they should be soaked overnight in a dilute nystatin
solution. Resistant infections can be treated with a short course of fluconazole.
2192

Next Question

Previous Question

Tag Question

Feedback

End Review

Difficulty: Easy
Peer Responses
https://mypastest.pastest.com/Secure/TestMe/Browser/429893

1/2

8/22/2016

MyPastest

Session Progress
Responses Correct:

Responses Incorrect:

298

Responses Total:

298

Responses - % Correct:

0%

Blog (https://www.pastest.com/blog) About Pastest (https://www.pastest.com/about-us)


Contact Us (https://www.pastest.com/contact-us) Help (https://www.pastest.com/help)
Pastest 2016

https://mypastest.pastest.com/Secure/TestMe/Browser/429893

2/2

8/22/2016

MyPastest

Back to Filters (/Secure/TestMe/Filter/429893/QA)

Question 17 of 298

A 26-year-old woman is referred by her GP. She has recently returned from her honeymoon in
Africa. While there she ate food bought from a number of roadside stalls. There is a history of
abdominal cramps, bloating and diarrhoea. Examination of three stool specimens revealed
cysts in two out of the three specimens. The condition responded to a course of
metronidazole.
What is the most likely clinical diagnosis in this case?
A

Salmonella infection

Typhoid fever

Giardiasis

Cryptosporidium

Tapeworm infection

Explanation
Giardiasis
Giardiasis is caused by the protozoal parasite Giardia lamblia and is transmitted by poor
hygiene practices. Giardia infection occurs more commonly in families with X-linked
agammaglobulinaemia and in sexually active homosexual men.
Clinical features
About 70% of infected patients have one or more intestinal symptoms (diarrhoea, flatulence,
cramps, bloating, nausea), but fever occurs in fewer than 20%. Malaise, anorexia, chronic
diarrhoea, malabsorption and weight loss can occur in chronic sufferers. Between 20% and
25% are asymptomatic.
Three or more stool specimens yield a result with 90% sensitivity. Treatment is with a 1-week
course of metronidazole.
2193

Next Question

https://mypastest.pastest.com/Secure/TestMe/Browser/429893

1/2

8/22/2016

MyPastest

Previous Question

Tag Question

Feedback

End Review

Difficulty: Average
Peer Responses

Session Progress
Responses Correct:

Responses Incorrect:

298

Responses Total:

298

Responses - % Correct:

0%

Blog (https://www.pastest.com/blog) About Pastest (https://www.pastest.com/about-us)


Contact Us (https://www.pastest.com/contact-us) Help (https://www.pastest.com/help)
Pastest 2016

https://mypastest.pastest.com/Secure/TestMe/Browser/429893

2/2

8/22/2016

MyPastest

Back to Filters (/Secure/TestMe/Filter/429893/QA)

Question 18 of 298

A 21-year-old student visits his GP complaining of a flu-like illness. He has not been eating for
around 48 hours. There is mild jaundice on examination but no other physical findings of
note. His serum bilirubin is raised at 60 mol/l, but the other liver function tests are normal.
Full blood count and U&Es are normal and haptoglobins are normal.
Which diagnosis fits best with this clinical picture?
A

Haemolytic anaemia

Gilbert syndrome

Hepatitis A

Hepatitis B

Cholecystitis

Explanation
Gilbert syndrome
Gilberts syndrome is an autosomal condition characterised by impaired glucuronyltransferase
activity. There is a male to female preponderance of 3:1, and a prevalence of around 5%.
Excess alcohol or a period of fasting can precipitate jaundice. The diagnosis is made on the
basis of the history, together with an isolated rise in unconjugated bilirubin without evidence
of haemolysis (the haptoglobins are normal). No intervention is required and the jaundice
usually subsides over the course of a few days.
2194

Next Question

Previous Question

Tag Question

Feedback

End Review

Difficulty: Easy
Peer Responses
https://mypastest.pastest.com/Secure/TestMe/Browser/429893

1/2

8/22/2016

MyPastest

Session Progress
Responses Correct:

Responses Incorrect:

298

Responses Total:

298

Responses - % Correct:

0%

Blog (https://www.pastest.com/blog) About Pastest (https://www.pastest.com/about-us)


Contact Us (https://www.pastest.com/contact-us) Help (https://www.pastest.com/help)
Pastest 2016

https://mypastest.pastest.com/Secure/TestMe/Browser/429893

2/2

8/22/2016

MyPastest

Back to Filters (/Secure/TestMe/Filter/429893/QA)

Question 19 of 298

A 54-year-old man presents to his GP with symptoms of burning retrosternal pain. He reports
occasional sticking of food. There is a past history of asthma but nothing else of note. He
smokes 20 cigarettes per day. Examination reveals some epigastric tenderness but only to
deep palpation.
Which of the following represents the best clinical management?
A

Lifestyle advice

Lifestyle advice with a proton-pump inhibitor

Lifestyle advice with antacids

Upper gastrointestinal endoscopy

Lifestyle advice with an H2-antagonist

Explanation
Heartburn
This mans symptoms, age and smoking history mean that oesophageal or gastric carcinoma
need to be excluded. His symptoms are suggestive of gastro-oesophageal reflux disease,
which might be associated with asthma and chronic bronchospasm.
The occasional sticking of food does, however, flag a warning signal and confirms the need
for diagnostic endoscopy. Heartburn is known to occur in up to 60% of adult. Lifestyle advice,
including alcohol avoidance and giving up smoking, is important, but proton-pump inhibitors
are highly effective in symptom relief.
Severe long-term reflux disease can result in Barretts oesophagus (columnisation of the
oesophageal squamous epithelium), which is known to predispose to oesophageal carcinoma.
People with Barretts oesophagus should undergo surveillance endoscopy at least once every
2 years.
2195

Next Question

https://mypastest.pastest.com/Secure/TestMe/Browser/429893

1/2

8/22/2016

MyPastest

Previous Question

Tag Question

Feedback

End Review

Difficulty: Average
Peer Responses

Session Progress
Responses Correct:

Responses Incorrect:

298

Responses Total:

298

Responses - % Correct:

0%

Blog (https://www.pastest.com/blog) About Pastest (https://www.pastest.com/about-us)


Contact Us (https://www.pastest.com/contact-us) Help (https://www.pastest.com/help)
Pastest 2016

https://mypastest.pastest.com/Secure/TestMe/Browser/429893

2/2

8/22/2016

MyPastest

Back to Filters (/Secure/TestMe/Filter/429893/QA)

Question 20 of 298

You are asked to review a 75-year-old woman. She has been referred by her GP because of
increasing weight loss, early satiety and increasing anorexia. She admits to two or three
episodes of vomiting blood. He feels there is an epigastric mass. Investigations have revealed
a microcytic anaemia and abnormal liver enzymes. Her past history, which might be of
importance, includes excess consumption of sherry and spirits, and a 30 pack-year smoking
history.
Which diagnosis fits best with this clinical picture?
A

Pancreatic carcinoma

Gastric lymphoma

Benign gastric ulcers

Helicobacter gastritis

Gastric carcinoma

Explanation
Gastric carcinoma
The annual incidence of gastric carcinoma in the Western world is around 7/100,000. In
Japan, the incidence is much higher at around 80/100,000, and it is thought to be dietrelated. Most gastric cancers (35%) occur in the antrum. There is a slight male preponderance
(3:2) and the disease is more common in the over-65 age group. Metastasis at presentation of
gastric carcinoma is common, with the liver the commonest site of metastasis. Around 5% of
gastric tumours are gastric lymphomas.
Risk factors for gastric carcinoma include:
Chronic Helicobacter pylori infection
Smoking
Alcohol
Food additives
Chronic atrophic gastritis
https://mypastest.pastest.com/Secure/TestMe/Browser/429893

1/2

8/22/2016

MyPastest

The 5-year survival rate is poor at 12% overall. Early gastric carcinoma detected prior to
metastasis might be associated with higher rates of survival (35% in one case series).
Pancreatic cancer could produce a similar clinical picture; however, the presence of
haematemesis here points more towards gastic cancer.
2196

Next Question

Previous Question

Tag Question

Feedback

End Review

Difficulty: Average
Peer Responses

Session Progress
Responses Correct:

Responses Incorrect:

298

Responses Total:

298

Responses - % Correct:

0%

Blog (https://www.pastest.com/blog) About Pastest (https://www.pastest.com/about-us)


Contact Us (https://www.pastest.com/contact-us) Help (https://www.pastest.com/help)
Pastest 2016

https://mypastest.pastest.com/Secure/TestMe/Browser/429893

2/2

8/22/2016

MyPastest

Back to Filters (/Secure/TestMe/Filter/429893/QA)

Question 21 of 298

A 72-year-old man is referred with tiredness. His GP notices that he is jaundiced and suspects
liver disease. He also appears to have angular stomatitis. There is a history of steroid inhaler
use for asthma, but nothing else of note. A blood film reveals features of a megaloblastic
anaemia, his serum bilirubin is raised but other biochemistry is unremarkable. There are no
gastrointestinal symptoms.
Which diagnosis fits best with this clinical picture?
A

Pernicious anaemia

Chronic myeloid leukaemia

Iron deficiency anaemia

Crohns disease

Autoimmune hepatitis

Explanation
Pernicious anaemia
Pernicious anaemia is common in the elderly, occurring in 1 in 8000 of the over-60 age group
(there is also a raised incidence of gastric carcinoma in this age group, 13%). Increased
bilirubin results from an increased turnover of immature red blood cells.
The disease is usually insidious in onset, presenting with tiredness and lethargy. Very low
serum levels of vitamin B12 can result in a symmetrical peripheral polyneuropathy,
progressively involving the posterior and then the lateral columns of the spinal cord
(subacute combined degeneration of the cord).
Anti-parietal cell antibodies are present in 90% of patients with pernicious anaemia. Antithyroid antibodies are present in up to 50% of patients. Vitamin B12 absorption can be
measured using the Schilling test, and treatment is with intramuscular injections of vitamin
B12.
There are no symptoms here to suggest Crohns disease, coeliac disease or a leukaemic
picture.
2197

https://mypastest.pastest.com/Secure/TestMe/Browser/429893

1/2

8/22/2016

MyPastest

Next Question

Previous Question

Tag Question

Feedback

End Review

Difficulty: Average
Peer Responses

Session Progress
Responses Correct:

Responses Incorrect:

298

Responses Total:

298

Responses - % Correct:

0%

Blog (https://www.pastest.com/blog) About Pastest (https://www.pastest.com/about-us)


Contact Us (https://www.pastest.com/contact-us) Help (https://www.pastest.com/help)
Pastest 2016

https://mypastest.pastest.com/Secure/TestMe/Browser/429893

2/2

8/22/2016

MyPastest

Back to Filters (/Secure/TestMe/Filter/429893/QA)

Question 22 of 298

A 62-year-old man with inflammatory bowel disease was diagnosed with primary sclerosing
cholangitis 5 years ago. He now presents with weight loss and a more rapid deterioration of
his liver function. On examination he is cachectic, liver function tests reveal a profoundly
obstructive picture with raised bilirubin, gamma-glutamyltransferase and alkaline
phosphatase levels, accompanied by a lesser increase in alanine aminotransferase (ALT).
There is nothing to suggest an acute infective process.
Which diagnosis fits best with this clinical picture?
A

Hepatocellular carcinoma

Primary carcinoma of the gallbladder

Cholangiocarcinoma

Ascending cholangitis

Primary biliary cirrhosis

Explanation
Cholangiocarcinoma
Cholangiocarcinoma has a well-recognised association with primary sclerosing cholangitis. It
can also occur in association with a choledochal cyst and chronic infection of the biliary tree.
Patients usually present with rapidly worsening jaundice. The diagnosis can usually be made
with a combination of ultrasound and spiral computed tomography or magnetic resonance
cholangiopancreatography (MRCP). Proximal lesions are usually not readily resectable at
presentation because of local spread. Patients can, however, usually be offered palliation with
metal stenting to maintain patency of the bile duct.
2198

Next Question

Previous Question

Tag Question

https://mypastest.pastest.com/Secure/TestMe/Browser/429893

Feedback

End Review

1/2

8/22/2016

MyPastest

Difficulty: Easy
Peer Responses

Session Progress
Responses Correct:

Responses Incorrect:

298

Responses Total:

298

Responses - % Correct:

0%

Blog (https://www.pastest.com/blog) About Pastest (https://www.pastest.com/about-us)


Contact Us (https://www.pastest.com/contact-us) Help (https://www.pastest.com/help)
Pastest 2016

https://mypastest.pastest.com/Secure/TestMe/Browser/429893

2/2

8/22/2016

MyPastest

Back to Filters (/Secure/TestMe/Filter/429893/QA)

Question 23 of 298

A 57-year-old publican is referred by his GP with chronic left-sided/central upper abdominal


pain. He admits to enjoying three or four pints of beer during an evening in the pub. Over the
past 18 months he has lost about 12.5 kg (2 stones) in weight, and his wife says he prefers
alcohol to food. He has intermittent diarrhoea, which he reports as being oily and difficult to
flush away on occasions.
Examination reveals a slim man with some tenderness to deep palpation in the epigastrium.
Blood testing reveals a mild normochromic normocytic anaemia and a raised alanine
aminotransferase (ALT) level to twice the upper limit of normal. Amylase and antigliadin
antibodies are normal. Upper abdominal ultrasound is performed, which shows diffuse
pancreatic calcification but nothing else of note.
Which diagnosis fits best with this clinical picture?
A

Acute pancreatitis

Chronic pancreatitis

Coeliac disease

Pancreatic carcinoma

Recurrent cholecystitis

Explanation
Chronic pancreatitis
There is a history of excess alcohol consumption and chronic upper abdominal pain in this
patient. The diarrhoea suggests a deficiency of pancreatic enzymes. These findings, coupled
with diffuse pancreatic calcification, are highly suggestive of chronic pancreatitis.
Causes
Alcohol accounts for 6080% of chronic pancreatitis cases in the developed world. Alcohol is
thought to alter the balance of trypsinogen in the pancreas and this may be one factor
involved in the association with alcohol. Other causes include cystic fibrosis and an autosomal
dominant familial pancreatitis syndrome.
Diagnosis and treatment
https://mypastest.pastest.com/Secure/TestMe/Browser/429893

1/2

8/22/2016

MyPastest

Where the diagnosis is unclear, magnetic resonance cholangiopancreatography (MRCP) or


spiral computed tomography might be useful. Treatment includes pain relief and pancreatic
enzyme replacement, with withdrawal of alcohol.
2199

Next Question

Previous Question

Tag Question

Feedback

End Review

Difficulty: Easy
Peer Responses

Session Progress
Responses Correct:

Responses Incorrect:

298

Responses Total:

298

Responses - % Correct:

0%

Blog (https://www.pastest.com/blog) About Pastest (https://www.pastest.com/about-us)


Contact Us (https://www.pastest.com/contact-us) Help (https://www.pastest.com/help)
Pastest 2016

https://mypastest.pastest.com/Secure/TestMe/Browser/429893

2/2

8/22/2016

MyPastest

Back to Filters (/Secure/TestMe/Filter/429893/QA)

Question 24 of 298

A 79-year-old woman has been seen twice by her GP during the past 8 months complaining
of dull abdominal pain radiating through to her back. The GP diagnosed wear and tear on
the spine and prescribed analgesics. The pain is partially relieved by sitting forwards. Her
daughter, who says she has not been eating for weeks, has brought her to the Emergency
Department.
Examination reveals a cachectic woman. She has a normochromic normocytic anaemia and
liver function tests reveal mildly elevated transaminases and grossly elevated bilirubin and
alkaline phosphatase levels. An ultrasound scan reveals bile duct obstruction with suspicion
of a mass in the epigastrum.
Which diagnosis fits best with this clinical picture?
A

Pancreatic carcinoma

Hepatocellular carcinoma

Cholecystitis

Gastrointestinal lymphoma with bile-duct obstruction

Gastric carcinoma with local spread

Explanation
Pancreatic carcinoma Pancreatic carcinoma has an incidence of 9 per 100,000, with the peak incidence occurring
above 60 years of age. There is a 3:2 male preponderance. Symptoms include epigastric pain
radiating to the back, which is partially relieved by sitting forwards, and jaundice (occurring
late and often the presenting feature). Pancreatic carcinoma can also be associated with
thrombophlebitis migrans, and some patients present with thromboembolic phenomena.
Diagnosis Spiral computed tomography, magnetic resonance cholangiopancreatography (MRCP) and
endoscopic retrograde cholangiopancreatography (ERCP) can be useful adjuncts to
investigations where the diagnosis is unclear. The CA-19.9 tumour marker can also be helpful.
Management https://mypastest.pastest.com/Secure/TestMe/Browser/429893

1/2

8/22/2016

MyPastest

The majority of these tumours are unsuitable for resection and the survival rate at 5 years is
only 2%. About 20% of patients have a technically resectable tumour, but surgical resection is
sometimes impossible due to co-morbidities.
2200

Next Question

Previous Question

Tag Question

Feedback

End Review

Difficulty: Easy
Peer Responses

Session Progress
Responses Correct:

Responses Incorrect:

298

Responses Total:

298

Responses - % Correct:

0%

Blog (https://www.pastest.com/blog) About Pastest (https://www.pastest.com/about-us)


Contact Us (https://www.pastest.com/contact-us) Help (https://www.pastest.com/help)
Pastest 2016

https://mypastest.pastest.com/Secure/TestMe/Browser/429893

2/2

8/22/2016

MyPastest

Back to Filters (/Secure/TestMe/Filter/429893/QA)

Question 25 of 298

A 52-year-old woman presents for review. She has been complaining of tiredness and
lethargy for some months, but her GP initially put this down to the menopause. There is a
past history of pernicious anaemia, which has been adequately treated. She is of normal
weight. The GP has now found that she has elevated transaminases, with alkaline
phosphatase and bilirubin levels just above the upper limit of normal. An autoimmune profile
shows raised antinuclear and anti-smooth muscle antibodies.
Which diagnosis fits best with this clinical picture?
A

Type II autoimmune hepatitis

Type I autoimmune hepatitis

Hepatitis A infection

Hepatitis B infection

Non-alcoholic steatohepatitis (NASH)

Explanation
Autoimmune hepatitis
Type I autoimmune hepatitis tends to occur in association with other autoimmune diseases
(eg pernicious anaemia, thyroiditis), and there are often antinuclear and anti-smooth muscle
autoantibodies detectable. Type II autoimmune hepatitis occurs in association with antiliver/kidney microsomal antibodies, most frequently in girls and young women. Liver biopsy
usually shows a necro-inflammatory process that eventually results in fibrosis and cirrhosis.
Around a third of patients have cirrhosis at the time of presentation.
Management
Treatment is with oral prednisolone 30 mg daily for 2 weeks, followed by a slow reduction
and maintenance dose accompanied by a steroid-sparing agent such as azathioprine.
Remission occurs in up to 80% of cases. Liver transplantation may be offered in advanced
disease where medical therapy has failed, but the disease can recur in the transplanted liver.
2201

Next Question
https://mypastest.pastest.com/Secure/TestMe/Browser/429893#Top

1/2

8/22/2016

MyPastest

Previous Question

Tag Question

Feedback

End Review

Difficulty: Average
Peer Responses

Session Progress
Responses Correct:

Responses Incorrect:

298

Responses Total:

298

Responses - % Correct:

0%

Blog (https://www.pastest.com/blog) About Pastest (https://www.pastest.com/about-us)


Contact Us (https://www.pastest.com/contact-us) Help (https://www.pastest.com/help)
Pastest 2016

https://mypastest.pastest.com/Secure/TestMe/Browser/429893#Top

2/2

8/22/2016

MyPastest

Back to Filters (/Secure/TestMe/Filter/429893/QA)

Question 26 of 298

An obese 36-year-old woman has been referred by her GP. Her past history of note includes
gestational diabetes during her last pregnancy 2 years ago. She also has a strong family
history of type 2 diabetes. There is no history of excess alcohol consumption.
Her GP checked her liver function tests as part of a routine health screen and found a raised
alanine aminotransferase (ALT) level. An infective hepatitis screen and an autoimmune profile
were normal. Abdominal ultrasound reveals evidence of fatty infiltration of the liver. Biopsy
reveals fat infiltration with some evidence of fibrosis.
Which diagnosis fits best with this clinical picture?
A

Non-alcoholic steatohepatitis (NASH)

Autoimmune hepatitis

Alcoholic cirrhosis

Cholecystitis

Hepatocellular carcinoma

Explanation
Non-alcoholic steatohepatitis
Non-alcoholic steatohepatitis (NASH) is associated with obesity, diabetes mellitus and
hyperlipidaemia. It is characterised by deposition of fat within the liver and hepatic fibrosis.
The inflammatory process is thought to result from initial oxidative stress, coupled with
further oxidation of intrahepatic lipids, resulting eventually in a fibrotic picture. Eventually, a
percentage of patients with NASH progress to full-blown cirrhosis.
NASH is also associated with increased insulin resistance per se and is a risk factor for
impaired glucose tolerance and diabetes mellitus. Recently published data has shown that an
elevated alanine aminotransferase (ALT) is an independent predictor of type 2 diabetes.
Weight loss is associated with a reduction in intrahepatic fat, and glitazones (which reduce
intrahepatic fat) might be useful as therapy for NASH in the long term.
2202

Next Question
https://mypastest.pastest.com/Secure/TestMe/Browser/429893

1/2

8/22/2016

MyPastest

Previous Question

Tag Question

Feedback

End Review

Difficulty: Easy
Peer Responses

Session Progress
Responses Correct:

Responses Incorrect:

298

Responses Total:

298

Responses - % Correct:

0%

Blog (https://www.pastest.com/blog) About Pastest (https://www.pastest.com/about-us)


Contact Us (https://www.pastest.com/contact-us) Help (https://www.pastest.com/help)
Pastest 2016

https://mypastest.pastest.com/Secure/TestMe/Browser/429893

2/2

8/22/2016

MyPastest

Back to Filters (/Secure/TestMe/Filter/429893/QA)

Question 27 of 298

A 45-year-old bar owner from Tenerife presents for review. Although he has been in the UK
for 3 months during the winter season, he still looks deeply tanned. He is tired and feels
'washed out'. He attends with his girlfriend, who is concerned that he is impotent and has lost
interest in sex. He has a past history of joint pains and mild arthritis, particularly affecting his
knees. There is a family history of autoimmune disease, with type 1 diabetes in one firstdegree relative and hypothyroidism in another.
On examination, he is deeply pigmented, there is loss of body hair, and testicular atrophy. His
fasting blood glucose concentration is 8.4 mmol/l, and alkaline phosphatase and
transaminase levels are raised.
Which diagnosis fits best with this clinical picture?
A

Alcoholic cirrhosis

Diabetes mellitus

Primary adrenal failure

Haemochromatosis

Wilsons disease

Explanation
Haemochromatosis
Haemochromatosis generally presents in the fifth decade in men, but women present later
because menstruation acts as a natural form of venesection. The incidence is around 1 in 300
in white populations. It has an autosomal recessive pattern of inheritance, and the
commonest gene mutation can now be screened for. The cause is excess iron deposition in
the liver due to increased iron absorption despite the existence of excessive iron stores; this
results in the accumulation of iron, particularly in the liver.
Clinical features
So-called bronze diabetes, it is associated with arthropathy, diabetes mellitus, impotence
and eventual cirrhosis of the liver. Phlebotomy is the treatment of choice, generally aiming to
reduce the haematocrit to below 40%. Prognosis is good if treatment is commenced before
the development of advanced cirrhosis.
https://mypastest.pastest.com/Secure/TestMe/Browser/429893#Top

1/2

8/22/2016

MyPastest
2203

Next Question

Previous Question

Tag Question

Feedback

End Review

Difficulty: Easy
Peer Responses

Session Progress
Responses Correct:

Responses Incorrect:

298

Responses Total:

298

Responses - % Correct:

0%

Blog (https://www.pastest.com/blog) About Pastest (https://www.pastest.com/about-us)


Contact Us (https://www.pastest.com/contact-us) Help (https://www.pastest.com/help)
Pastest 2016

https://mypastest.pastest.com/Secure/TestMe/Browser/429893#Top

2/2

8/22/2016

MyPastest

Back to Filters (/Secure/TestMe/Filter/429893/QA)

Question 28 of 298

A 57-year-old man who has undergone previous treatment for alcohol addiction is brought to
the Emergency Department by his wife, who reports that he has become increasingly drowsy
and difficult to rouse. There is no history of head injury. He has had problems sleeping
recently and was given zopiclone (Zimovane) by his GP. His pills have been counted and it
does not appear that he has taken an overdose.
On examination he clearly has ascites and is generally tender. Blood tests reveal an iron
deficiency picture, elevated transaminases and a bilirubin of 145 mol/l; a random blood
glucose concentration is 6.7 mmol/l. He has a mildly elevated white blood cell count.
Which of the following is the most likely cause of his decreased conscious level?
A

Zimovaneoverdose

Subdural haematoma

Hepatic encephalopathy

Hypoglycaemia

Metastatic carcinoma

Explanation
Hepatic encephalopathy
In this case, either the Zimovane or a possible spontaneous bacterial peritonitis could be
responsible for precipitating hepatic encephalopathy. Hepatic encephalopathy is usually
graded from 1 to 4, with 1 being mild obtundation and 4 being deep coma.
Precipitating factors for encephalopathy in patients with underlying cirrhosis include:
Upper gastrointestinal tract bleeding
Hypokalaemia
Hypomagnesaemia
Analgesic and sedative drugs
Sepsis
Alkalosis
https://mypastest.pastest.com/Secure/TestMe/Browser/429893#Top

1/2

8/22/2016

MyPastest

Increased dietary protein


Management involves:
Identification and treatment of any precipitating factors
Oral lactulose solution or neomycin in an attempt to reduce bacterial production of
ammonia within the GI tract
2204

Next Question

Previous Question

Tag Question

Feedback

End Review

Difficulty: Easy
Peer Responses

Session Progress
Responses Correct:

Responses Incorrect:

298

Responses Total:

298

Responses - % Correct:

0%

Blog (https://www.pastest.com/blog) About Pastest (https://www.pastest.com/about-us)


Contact Us (https://www.pastest.com/contact-us) Help (https://www.pastest.com/help)
Pastest 2016

https://mypastest.pastest.com/Secure/TestMe/Browser/429893#Top

2/2

8/22/2016

MyPastest

Back to Filters (/Secure/TestMe/Filter/429893/QA)

Question 29 of 298

You are asked to review an 18-year-old student who has just returned from a gap-year trip to
India. He reports having profound tiredness and a lack of appetite for the last week of his
travels, and he had noticed that he had jaundice just before he was due to return home. He
had also felt fevered, but this appeared to subside once the jaundice appeared. He is not
homosexual and is not an intravenous drug abuser. He did not have a blood transfusion or
any tattoos during his trip.
Liver function tests reveal an alanine aminotransferase (ALT) of 950 U/l, a bilirubin of
240 mol/l and an elevated alkaline phosphatase that was just outside the upper limit of
normal. White blood cell count, albumin level and prothrombin times are all normal.
Which of the following is the most likely diagnosis, given this clinical picture?
A

Hepatitis B

Hepatitis C

Cytomegalovirus

Leptospirosis

Hepatitis A

Explanation
Hepatitis A
This young man gives a history suggestive of hepatitis A, with initial fever, anorexia and
malaise, subsiding at the onset of jaundice. Anti-hepatitis A IgM antibody will confirm the
diagnosis (IgG antibody would suggest a previous hepatitis A infection or another underlying
cause such as cytomegalovirus). Treatment is conservative, with advice about maintaining a
high-calorie intake. Cholestasis is only prolonged in rare cases, and fatal fulminant hepatitis is
thought to occur in fewer than 0.1% of patients.
Leptospirosis usually occurs with a history of exposure to rat-infested water. There is nothing
in his history to suggest exposure to either hepatitis B or hepatitis C.
2205

Next Question
https://mypastest.pastest.com/Secure/TestMe/Browser/429893#Top

1/2

8/22/2016

MyPastest

Previous Question

Tag Question

Feedback

End Review

Difficulty: Easy
Peer Responses

Session Progress
Responses Correct:

Responses Incorrect:

298

Responses Total:

298

Responses - % Correct:

0%

Blog (https://www.pastest.com/blog) About Pastest (https://www.pastest.com/about-us)


Contact Us (https://www.pastest.com/contact-us) Help (https://www.pastest.com/help)
Pastest 2016

https://mypastest.pastest.com/Secure/TestMe/Browser/429893#Top

2/2

8/22/2016

MyPastest

Back to Filters (/Secure/TestMe/Filter/429893/QA)

Question 30 of 298

A 61-year-old man with known cirrhosis secondary to hepatitis C infection attends for review.
There is a past history of intravenous heroin abuse and alcoholism. He has been feeling
progressively more unwell during the past 6 months, with weight loss and worsening ascites.
A pre-clinic alpha-fetoprotein is elevated. He is on long-term sick leave and has been closely
monitored by his live-in partner, who maintains there has been no further drug abuse or
consumption of alcohol. Which of the following is the most likely diagnosis, given this clinical
picture?
A

Superimposed hepatitis B infection

Alcoholism

Chronic active hepatitis

Spontaneous bacterial peritonitis

Hepatocellular carcinoma

Explanation
Hepatocellular carcinoma The -fetoprotein (AFP) is elevated in 70% of patients with hepatocellular carcinoma. The
history of worsening weight loss and ascites occurring over a relatively short period is
suggestive of this diagnosis.
Risk factors for hepatocellular carcinoma include:
Chronic liver disease
Cirrhosis
Chronic hepatitis B or C infection
Hepatotoxins (alcohol, aflatoxin, anabolic steroid abuse, vinyl chloride exposure)
Alpha1-antitrypsin deficiency
Haemochromatosis

https://mypastest.pastest.com/Secure/TestMe/Browser/429893#Top

1/2

8/22/2016

MyPastest

The definitive diagnosis is generally made using ultrasound followed by computed


tomography-guided biopsy. Screening is difficult, but surveillance of AFP levels can result in
the earlier detection of cases. Management, unfortunately, is usually palliative. Even after
surgical resection, in the rare cases where this is possible, the survival rate is only 2530%.
2206

Next Question

Previous Question

Tag Question

Feedback

End Review

Difficulty: Easy
Peer Responses

Session Progress
Responses Correct:

Responses Incorrect:

298

Responses Total:

298

Responses - % Correct:

0%

Blog (https://www.pastest.com/blog) About Pastest (https://www.pastest.com/about-us)


Contact Us (https://www.pastest.com/contact-us) Help (https://www.pastest.com/help)
Pastest 2016

https://mypastest.pastest.com/Secure/TestMe/Browser/429893#Top

2/2

8/22/2016

MyPastest

Back to Filters (/Secure/TestMe/Filter/429893/QA)

Question 31 of 298

A 24-year-old student presents with bloody diarrhoea. She says that she has been passing up
to 12 motions per day for the past 23 weeks. She now presents to the Emergency
Department complaining of abdominal pain and distension. On examination she is dehydrated
with a clearly distended, tender abdomen. There is anaemia with raised plasma viscosity, the
potassium is mildly decreased at 3.2 mmol/l and the urea is raised in keeping with the
dehydration. Liver function testing reveals a decreased albumin level. Autoantibody screen is
positive for perinuclear antineutrophil cytoplasmic antibody (pANCA). Sigmoidoscopy shows
a friable mucosa with a uniform pattern of inflammation and loss of normal mucosa. Stool
culture is negative.
Which diagnosis fits best with this clinical picture?
A

Crohns disease

Coeliac disease

Ischaemic colitis

Ulcerative colitis

Diverticulitis

Explanation
Ulcerative colitis
This is the typical presentation of ulcerative colitis. Extraintestinal manifestations such as
arthropathy, uveitis and pyoderma gangrenosum can also occur. The annual incidence of
ulcerative colitis is said to be 50150 cases/100,000 of the population, with the commonest
age at presentation being between 14 and 38 years. Perinuclear antineutrophil cytoplasmic
antibody (pANCA) is positive in 45% of cases.
Management
Management includes correction of dehydration and subcutaneous heparin for patients who
are inactive. The acute management of inflammation involves a combination of intravenous
hydrocortisone and 5-aminosalicylic acid compounds such as mesalazine. Between 15% and
20% of patients eventually require colectomy for disease that is resistant to medical therapy.
2386

https://mypastest.pastest.com/Secure/TestMe/Browser/429893#Top

1/2

8/22/2016

MyPastest

Next Question

Previous Question

Tag Question

Feedback

End Review

Difficulty: Easy
Peer Responses

Session Progress
Responses Correct:

Responses Incorrect:

298

Responses Total:

298

Responses - % Correct:

0%

Blog (https://www.pastest.com/blog) About Pastest (https://www.pastest.com/about-us)


Contact Us (https://www.pastest.com/contact-us) Help (https://www.pastest.com/help)
Pastest 2016

https://mypastest.pastest.com/Secure/TestMe/Browser/429893#Top

2/2

8/22/2016

MyPastest

Back to Filters (/Secure/TestMe/Filter/429893/QA)

Question 32 of 298

A 64-year-old woman presents with increasing epigastric pain and waterbrash of some
months' duration. She has been taking alendronic acid tablets for osteoporosis. There is no
history of food-sticking or weight loss, and her general examination is unremarkable. Full
blood count, urea and electrolytes and liver function tests are all normal.
Which diagnosis fits best with this clinical picture?
A

Gastric carcinoma

Peptic ulcer disease

Pancreatic carcinoma

Oesophagitis

Oesophageal carcinoma

Explanation
Oesophagitis
Causes
Drugs oesophagitis is a well-recognised side-effect of bisphosphonate therapy. Some
of the newer agents, such as risedronate, are thought to have a slightly better
gastrointestinal side-effect profile. Non-steroidal anti-inflammatory drugs (NSAIDs) can
also contribute to oesophagitis.
Alcohol and smoking are contributory factors
Infective causes include Candida albicans.
Functional reflux can occur when there is significant abdominal obesity, resulting in
oesophagitis.
Management
Heartburn occurs in up to 60% of adults, 20% of whom use over-the-counter products to
relieve their indigestion at least once per week. Treatment is based around eliminating
precipitating causes such as drugs, smoking, infection or obesity, and the use of protonhttps://mypastest.pastest.com/Secure/TestMe/Browser/429893#Top

1/2

8/22/2016

MyPastest

pump inhibitors such as omeprazole.


2387

Next Question

Previous Question

Tag Question

Feedback

End Review

Difficulty: Average
Peer Responses

Session Progress
Responses Correct:

Responses Incorrect:

298

Responses Total:

298

Responses - % Correct:

0%

Blog (https://www.pastest.com/blog) About Pastest (https://www.pastest.com/about-us)


Contact Us (https://www.pastest.com/contact-us) Help (https://www.pastest.com/help)
Pastest 2016

https://mypastest.pastest.com/Secure/TestMe/Browser/429893#Top

2/2

8/22/2016

MyPastest

Back to Filters (/Secure/TestMe/Filter/429893/QA)

Question 33 of 298

A 27-year-old woman attends for review. She has a past history of perianal abscess but
nothing else of note. During the past few months she has twice presented to the Emergency
Department complaining of grumbling abdominal pain. In addition, she has had intermittent
episodes of bloody diarrhoea.
Blood tests reveal microcytic anaemia and she has mild hypokalaemia; the albumin is reduced
but other liver function tests are unremarkable. Barium imaging reveals a small-bowel
stricture with evidence of mucosal ulceration extending into the colon, interspersed with
normal-looking mucosa (skip lesions).
Given this clinical picture, which is the most likely diagnosis?
A

Ulcerative colitis

Small-bowel lymphoma

Coeliac disease

Tropical sprue

Crohns disease

Explanation
Crohns disease
Crohns disease has a prevalence of around 1 in 1000, and is most commonly seen in people
of white European or Jewish descent. Extraintestinal manifestations at diagnosis can include
small-joint polyarthritis (seronegative), erythema nodosum, clubbing and sacroiliitis.
Endoscopic features of Crohns disease include asymmetric disease, deep longitudinal
fissuring, a cobblestone appearance and the presence of strictures. Crypt distortion,
inflammation and granulomas are also sometimes seen.
Acute therapy revolves around the use of corticosteroids, with the addition of 5aminosalycilic acid compounds, with or without azathioprine as a steroid-sparing agent.
Injectable anti-tumour necrosis factor (anti-TNF) compounds are also now finding an
important role in the treatment of Crohns disease. Surgical intervention should be avoided if
at all possible.
https://mypastest.pastest.com/Secure/TestMe/Browser/429893#Top

1/2

8/22/2016

MyPastest

2388

Next Question

Previous Question

Tag Question

Feedback

End Review

Difficulty: Easy
Peer Responses

Session Progress
Responses Correct:

Responses Incorrect:

298

Responses Total:

298

Responses - % Correct:

0%

Blog (https://www.pastest.com/blog) About Pastest (https://www.pastest.com/about-us)


Contact Us (https://www.pastest.com/contact-us) Help (https://www.pastest.com/help)
Pastest 2016

https://mypastest.pastest.com/Secure/TestMe/Browser/429893#Top

2/2

8/22/2016

MyPastest

Back to Filters (/Secure/TestMe/Filter/429893/QA)

Question 34 of 298

A 22-year-old woman is sent to the Gastroenterology Clinic for review. She has a confirmed
pregnancy and is in the third month of gestation but has been having trouble with excessive
morning sickness. Her GP has checked some routine bloods and has found an alanine
aminotransferase (ALT) of 64 U/l and a bilirubin of 45 mol/l. He is now concerned about
liver disease of pregnancy.
Which diagnosis fits best with this clinical picture?
A

Intrahepatic cholestasis of pregnancy

Acute fatty liver of pregnancy

Hyperemesis gravidarum

Pre-eclampsia

Biliary tract disease

Explanation
Liver abnormalities associated with pregnancy
Severe hyperemesis (hyperemesis gravidarum) is common in pregnancy, and is associated
with mild liver abnormalities in around 0.51% of pregnancies. ALT of 64 and bilirubin of 45
are only marginally above the upper limit of normal, and given only morning sickness is out of
keeping with the other diagnoses listed. Jaundice is always mild. If hyperemesis continues for
a prolonged period it can result in a lower birthweight baby, but this is rare.
Intrahepatic cholestasis and acute fatty liver of pregnancy are the two most serious liver
abnormalities in pregnancy, both occurring most commonly during the third trimester. Acute
fatty liver is associated with fetal and maternal mortalities of up to 20%. Intrahepatic
cholestasis is associated with stillbirth, prematurity and a fetal mortality rate of up to 3.5%.
2389

Next Question

Previous Question

Tag Question

https://mypastest.pastest.com/Secure/TestMe/Browser/429893#Top

Feedback

End Review
1/2

8/22/2016

MyPastest

Difficulty: Average
Peer Responses

Session Progress
Responses Correct:

Responses Incorrect:

298

Responses Total:

298

Responses - % Correct:

0%

Blog (https://www.pastest.com/blog) About Pastest (https://www.pastest.com/about-us)


Contact Us (https://www.pastest.com/contact-us) Help (https://www.pastest.com/help)
Pastest 2016

https://mypastest.pastest.com/Secure/TestMe/Browser/429893#Top

2/2

8/22/2016

MyPastest

Back to Filters (/Secure/TestMe/Filter/429893/QA)

Question 35 of 298

A 23-year-old woman is in her 29th week of pregnancy. She has suffered from itching for
3 weeks and is concerned. She now has mild jaundice. Her bilirubin is raised at around
80 mol/l and her alanine aminotransferase (ALT) is raised at 82 U/l; the alkaline
phosphatase is markedly raised.
Which diagnosis fits best with this clinical picture?
A

Cholecystitis

Acute fatty liver of pregnancy

Hyperemesis gravidarum

Intrahepatic cholestasis of pregnancy

HELLP syndrome

Explanation
Intrahepatic cholestasis of pregnancy
Intrahepatic cholestasis of pregnancy occurs in around 0.10.2% of pregnancies. The
commonest symptom is itching, and jaundice appears some 14 weeks after the itching. Both
the alanine aminotransferase (ALT) and bilirubin levels are raised. Liver biopsy is not
indicated, but the underlying pathology would show centrilobular cholestasis.
The disease is associated with increased rates of stillbirth, prematurity and a fetal mortality
rate of around 3.5%. Treatment is symptomatic, using ursodeoxycholic acid. There is a
tendency for the cholestasis to recur in subsequent pregnancies or after using the oral
contraceptive pill.
2390

Next Question

Previous Question

Tag Question

Feedback

End Review

Difficulty: Easy
https://mypastest.pastest.com/Secure/TestMe/Browser/429893#Top

1/2

8/22/2016

MyPastest

Peer Responses

Session Progress
Responses Correct:

Responses Incorrect:

298

Responses Total:

298

Responses - % Correct:

0%

Blog (https://www.pastest.com/blog) About Pastest (https://www.pastest.com/about-us)


Contact Us (https://www.pastest.com/contact-us) Help (https://www.pastest.com/help)
Pastest 2016

https://mypastest.pastest.com/Secure/TestMe/Browser/429893#Top

2/2

8/22/2016

MyPastest

Back to Filters (/Secure/TestMe/Filter/429893/QA)

Question 36 of 298

A patient with Crohns disease attends for his annual review. He has heard about infliximab
and wonders if it might be suitable for him.
Which of the following statements best describes infliximab?
A

It is a new selective corticosteroid

It is an oral anti-TNF (anti-tumour necrosis factor alpha) monoclonal antibody

It is an injectable anti-TNF monoclonal antibody

It is an oral 2b,3a inhibitor

It can be given safely to the patient at home

Explanation
Infliximab
Infliximab is an injectable anti-TNF (tumour necrosis factor alpha) monoclonal antibody
licensed for the treatment of Crohns disease or ulcerative colitis that is refractory to steroids
and conventional immunomodulatory agents. TNF is a key cytokine involved in the
pathophysiology of Crohns disease and TNF levels are reduced by infliximab. As it is a
monoclonal antibody it can precipitate anaphylaxis and therefore must be given by a
specialist in a setting where resuscitation facilities are available.
2391

Next Question

Previous Question

Tag Question

Feedback

End Review

Difficulty: Easy
Peer Responses

https://mypastest.pastest.com/Secure/TestMe/Browser/429893#Top

1/2

8/22/2016

MyPastest

Session Progress
Responses Correct:

Responses Incorrect:

298

Responses Total:

298

Responses - % Correct:

0%

Blog (https://www.pastest.com/blog) About Pastest (https://www.pastest.com/about-us)


Contact Us (https://www.pastest.com/contact-us) Help (https://www.pastest.com/help)
Pastest 2016

https://mypastest.pastest.com/Secure/TestMe/Browser/429893#Top

2/2

8/22/2016

MyPastest

Back to Filters (/Secure/TestMe/Filter/429893/QA)

Question 37 of 298

A 34-year-old man presents with symptoms of reflux oesophagitis. You elect to start
lansoprazole because he has already made changes to his lifestyle without complete
resolution of the reflux symptoms.
Which of the following statements best describes the mode of action of lansoprazole?
A

It binds to the histamine H1 receptor

It binds to the histamine H2 receptor

It inhibits the hydrogensodium-ATP proton pump

It inhibits the hydrogenpotassium-ATP proton pump

It inhibits the hydrogencalcium-ATP proton pump

Explanation
Proton-pump inhibitors
Lansoprazole is a proton-pump inhibitor. There are a number of other drugs in the class,
including omeprazole, and they work by blocking the hydrogenpotassium-ATP pump that
allows the gastric parietal cells to secrete acid. These drugs are indicated for the treatment of
peptic ulceration and oesophagitis and for Helicobacter pylori eradication as part of a tripletherapy regime.
Histamine H2-receptor blocking drugs, such as ranitidine, have a lesser effect on gastric acid
reduction and many are now sold over the counter in pharmacies.
2392

Next Question

Previous Question

Tag Question

Feedback

End Review

Difficulty: Average
Peer Responses
https://mypastest.pastest.com/Secure/TestMe/Browser/429893#Top

1/2

8/22/2016

MyPastest

Session Progress
Responses Correct:

Responses Incorrect:

298

Responses Total:

298

Responses - % Correct:

0%

Blog (https://www.pastest.com/blog) About Pastest (https://www.pastest.com/about-us)


Contact Us (https://www.pastest.com/contact-us) Help (https://www.pastest.com/help)
Pastest 2016

https://mypastest.pastest.com/Secure/TestMe/Browser/429893#Top

2/2

8/22/2016

MyPastest

Back to Filters (/Secure/TestMe/Filter/429893/QA)

Question 38 of 298

A 40-year-old woman was diagnosed with Crohns disease 3 months ago. At the time of
diagnosis she was initiated on sulfasalazine. Her bowel symptoms are now much improved,
but she is now attending the Emergency Department because of easy bruising. On
examination she has a number of bruises. Blood testing reveals a pancytopenia.
Which of the following is the most likely cause of her pancytopenia?
A

Acute leukaemia

Worsening of her Crohns disease

Sulfasalazine therapy

Viral infection

Idiopathic thrombocytopenic purpura

Explanation
Sulfasalazine
Sulfasalazine therapy can lead to isolated falls in the numbers of red blood cells, white cells or
platelets, or can lead to a pancytopenia, as in this case. Patients are recommended to report
any unexplained bleeding, bruising, purpura, sore throat, fever or malaise, so that their doctor
can carry out a full blood count the medication should be suspended pending the result.
Other side-effects of sulfasalazine therapy include:
Hypersensitivity reactions
Periorbital oedema
Stomatitis
Parotitis
Cautions for sulfasalazine use include:
Renal and hepatic insufficiency
Glucose-6-phosphate dehydrogenase (G6PD) deficiency
https://mypastest.pastest.com/Secure/TestMe/Browser/429893#Top

1/2

8/22/2016

MyPastest

Slow acetylator status


2393

Next Question

Previous Question

Tag Question

Feedback

End Review

Difficulty: Easy
Peer Responses

Session Progress
Responses Correct:

Responses Incorrect:

298

Responses Total:

298

Responses - % Correct:

0%

Blog (https://www.pastest.com/blog) About Pastest (https://www.pastest.com/about-us)


Contact Us (https://www.pastest.com/contact-us) Help (https://www.pastest.com/help)
Pastest 2016

https://mypastest.pastest.com/Secure/TestMe/Browser/429893#Top

2/2

8/22/2016

MyPastest

Back to Filters (/Secure/TestMe/Filter/429893/QA)

Question 39 of 298

A 72-year-old man presents with left-sided lower abdominal pain. He is obese and admits to a
dislike of high-fibre foods. The pain has been grumbling for the past couple of weeks and is
partially relieved by defecation. He has suffered intermittent diarrhoea. Blood testing reveals
neutrophilia and microcytic anaemia. Barium enema shows multiple diverticulae, more
marked on the left-hand side of the colon.
Which diagnosis fits best with this clinical picture?
A

Irritable bowel syndrome

Ulcerative colitis

Ischaemic colitis

Lactose intolerance

Diverticular disease

Explanation
Diverticular disease
The incidence of diverticular disease in the general population is said to be as high as 35
50%. It is common in Western countries, affecting more than 50% of the over-70 age group.
Management
Management involves increasing water and fibre intake to reduce intracolonic pressure
and the risk of further diverticulae developing.
Acute diverticulitis is usually managed conservatively with intravenous antibiotics, such
as ciprofloxacin and metronidazole, or co-amoxiclav.
Occasionally a diverticular abscess forms, which often requires surgical intervention. (It
has been reported that up to 30% of patients with diverticulitis will eventually require
some form of surgical intervention.)
2394

https://mypastest.pastest.com/Secure/TestMe/Browser/429893#Top

1/2

8/22/2016

MyPastest

Next Question

Previous Question

Tag Question

Feedback

End Review

Difficulty: Easy
Peer Responses

Session Progress
Responses Correct:

Responses Incorrect:

298

Responses Total:

298

Responses - % Correct:

0%

Blog (https://www.pastest.com/blog) About Pastest (https://www.pastest.com/about-us)


Contact Us (https://www.pastest.com/contact-us) Help (https://www.pastest.com/help)
Pastest 2016

https://mypastest.pastest.com/Secure/TestMe/Browser/429893#Top

2/2

8/22/2016

MyPastest

Back to Filters (/Secure/TestMe/Filter/429893/QA)

Question 40 of 298

A 75-year-old man with a history of atrial fibrillation and peripheral vascular disease presents
to the Emergency Department. His abdomen is distended and tender. A plain abdominal film
shows thumb-printing at the site of the splenic flexure. Blood testing reveals evidence of mild
dehydration, and a full blood count shows a mildly raised neutrophil count.
Which diagnosis fits best with this clinical picture?
A

Ischaemic colitis

Diverticulitis

Colonic carcinoma

Ulcerative colitis

Diverticular abscess

Explanation
Ischaemic colitis
Ischaemic colitis occurs with increasing frequency in:
Women who are taking the contraceptive pill
People with a thrombophilia syndrome
People with a history of pre-existing vascular disease (as in this case)
The plain abdominal film is characteristic, when thumbprinting occurs at the site of the
splenic flexure.
Management is usually conservative, with intravenous rehydration and supportive
management, but the condition can progress to gangrene and perforation and require
surgical intervention (partial colectomy). A long-standing consequence of a resolved episode
is stricture formation in the previously ischaemic area.
2395

Next Question

https://mypastest.pastest.com/Secure/TestMe/Browser/429893#Top

1/2

8/22/2016

MyPastest

Previous Question

Tag Question

Feedback

End Review

Difficulty: Easy
Peer Responses

Session Progress
Responses Correct:

Responses Incorrect:

298

Responses Total:

298

Responses - % Correct:

0%

Blog (https://www.pastest.com/blog) About Pastest (https://www.pastest.com/about-us)


Contact Us (https://www.pastest.com/contact-us) Help (https://www.pastest.com/help)
Pastest 2016

https://mypastest.pastest.com/Secure/TestMe/Browser/429893#Top

2/2

8/22/2016

MyPastest

Back to Filters (/Secure/TestMe/Filter/429893/QA)

Question 41 of 298

A 30-year-old business executive presents complaining of heartburn for the past 6 months.
You suspect that he could be suffering from gastro-oesophageal reflux disease.
What is the most important physiological mechanism that prevents reflux?
A

Valve-like mechanism of the short portion of the oesophagus

Diaphragmatic muscle surrounding the oesophagus

Parasympathetic stimulation of the lower circular smooth-muscle fibres of the


oesophagus

Increased intrathoracic compared with intra-abdominal pressure

Formation of an anatomical sphincter by the lower circular smooth-muscle fibres of


the oesophagus

Explanation
Physiological anti-reflux mechanisms in the oesophagus The lower circular smooth-muscle fibres of the oesophagus are under neural control. The
parasympathetic efferent fibres passing through the vagus nerve contain vagal excitatory
fibres (VEF) and vagal inhibitory fibres (VIF). When VEF are stimulated, the muscle fibres
contract. The lower oesophageal muscle fibres do not form a true anatomical sphincter.
Other anti-reflux mechanisms (of lesser importance) are:
Valve-like mechanism of the short portion of the oesophagus that extends into the
diaphragm. This caves inwards and closes the oesophagus when gastric pressure
increases.
Fibres of the crural portion of the diaphragm surround the lower end of the
oesophagus and exert a pinch-cock action.
2814

Next Question

https://mypastest.pastest.com/Secure/TestMe/Browser/429893#Top

1/2

8/22/2016

MyPastest

Previous Question

Tag Question

Feedback

End Review

Difficulty: Difficult
Peer Responses

Session Progress
Responses Correct:

Responses Incorrect:

298

Responses Total:

298

Responses - % Correct:

0%

Blog (https://www.pastest.com/blog) About Pastest (https://www.pastest.com/about-us)


Contact Us (https://www.pastest.com/contact-us) Help (https://www.pastest.com/help)
Pastest 2016

https://mypastest.pastest.com/Secure/TestMe/Browser/429893#Top

2/2

8/22/2016

MyPastest

Back to Filters (/Secure/TestMe/Filter/429893/QA)

Question 42 of 298

A 55-year-old alcoholic is admitted with portal hypertension. The wedged hepatic venous
pressure is recorded.
This pressure reflects the pressure in which part of the hepatic vascular system?
A

Portal vein

Hepatic artery

Hepatic vein

Central vein radicles

Hepatic sinusoids

Explanation
Wedged hepatic venous pressure
The wedged hepatic venous pressure is the pressure recorded by a catheter wedged in a
hepatic vein. It reflects the portal venous pressure in the hepatic sinusoids. Wedged hepatic
venous pressure is elevated in sinusoidal and post-sinusoidal portal hypertension, but remains
normal in pre-sinusoidal portal hypertension.
2815

Next Question

Previous Question

Tag Question

Feedback

End Review

Difficulty: Difficult
Peer Responses

https://mypastest.pastest.com/Secure/TestMe/Browser/429893#Top

1/2

8/22/2016

MyPastest

Session Progress
Responses Correct:

Responses Incorrect:

298

Responses Total:

298

Responses - % Correct:

0%

Blog (https://www.pastest.com/blog) About Pastest (https://www.pastest.com/about-us)


Contact Us (https://www.pastest.com/contact-us) Help (https://www.pastest.com/help)
Pastest 2016

https://mypastest.pastest.com/Secure/TestMe/Browser/429893#Top

2/2

8/22/2016

MyPastest

Back to Filters (/Secure/TestMe/Filter/429893/QA)

Question 43 of 298

A 25-year-old cook applies for a job at a cafeteria. He gives a history of having had enteric
fever 2 years ago.
Which of the following investigations is most likely to indicate a chronic carrier status?
A

Vi agglutination test

Widal antigen test

Blood culture

Full blood count

Culture of intestinal secretions

Explanation
Salmonella carriage
Salmonella typhi can be cultured from intestinal secretions, faeces or urine in chronic carriers
and this investigation is recommended to confirm the diagnosis. Vi agglutination
demonstration is a screening test that can be positive in around 80% of cases. The Vi
(virulence) antigen is a polysaccharide on the exterior of the cell wall. The Vi antigen prevents
O antibodies from binding to the O antigen.
Serological tests such as the Widal antigen test are not useful for detecting chronic carriage.
Blood cultures will invariably be negative in chronic carriers because the organism resides
mainly in the gallbladder. Leucopenia occurs in acute infection.
2816

Next Question

Previous Question

Tag Question

Feedback

End Review

Difficulty: Difficult
Peer Responses
https://mypastest.pastest.com/Secure/TestMe/Browser/429893#Top

1/2

8/22/2016

MyPastest

Session Progress
Responses Correct:

Responses Incorrect:

298

Responses Total:

298

Responses - % Correct:

0%

Blog (https://www.pastest.com/blog) About Pastest (https://www.pastest.com/about-us)


Contact Us (https://www.pastest.com/contact-us) Help (https://www.pastest.com/help)
Pastest 2016

https://mypastest.pastest.com/Secure/TestMe/Browser/429893#Top

2/2

8/22/2016

MyPastest

Back to Filters (/Secure/TestMe/Filter/429893/QA)

Question 44 of 298

A 24-year-old man presents with malaise, mild fever, loss of weight and anorexia. On
examination, his sclerae appear yellow. Serum bilirubin is elevated at 85 mol/l (normal range
122 mol/l), enzyme-linked immunosorbent assay (ELISA) for IgG anti-HEV (anti-hepatitis E
virus) is positive and HEV RNA is detectable in serum by polymerase chain reaction (PCR).
What would the characteristic finding on liver biopsy be in this case?
A

Ground-glass hepatocytes

Marked cholestasis

Lymphoid aggregates

Microvesicular steatosis

Marked increase in the activation of sinusoidal lining cells

Explanation
Hepatitis histology
Marked cholestasis is the hallmark histological finding in hepatitis E virus infection.
Ground-glass hepatocytes are large hepatocytes containing surface antigen. They are
seen in chronic hepatitis.
Lymphoid aggregates and a marked increase in the activation of sinusoidal lining cells
are seen in hepatitis C infection.
Microvesicular steatosis occurs in hepatitis D.
2817

Next Question

Previous Question

Tag Question

https://mypastest.pastest.com/Secure/TestMe/Browser/429893#Top

Feedback

End Review

1/2

8/22/2016

MyPastest

Difficulty: Difficult
Peer Responses

Session Progress
Responses Correct:

Responses Incorrect:

298

Responses Total:

298

Responses - % Correct:

0%

Blog (https://www.pastest.com/blog) About Pastest (https://www.pastest.com/about-us)


Contact Us (https://www.pastest.com/contact-us) Help (https://www.pastest.com/help)
Pastest 2016

https://mypastest.pastest.com/Secure/TestMe/Browser/429893#Top

2/2

8/22/2016

MyPastest

Back to Filters (/Secure/TestMe/Filter/429893/QA)

Question 45 of 298

A 35-year-old man presents with a history of jaundice. The underlying cause is suspected to
be acute hepatitis B infection.
Which of the following immunological test results when found would most support the
diagnosis?
A

HBsAg

HBeAg

HBV DNA

IgM anti-HBc

Anti-HBeAg

Explanation
Hepatitis B serology
The presence of HBsAg indicates a hepatitis B infection, but it is less helpful than IgM in
indicating the time period over which infection has occurred. IgM anti-HBc best
indicate an acute infection. Absence of IgM anti-HBc indicates a chronic infection.
HBeAg appears shortly after the appearance of HBsAg. Its principal clinical usefulness
is as an indicator of relative infectivity. It denotes viral replication. HBeAg testing is
primarily indicated during the follow-up of chronic infection.
HBV DNA has the same role as HBeAg. This test is more quantitative and sensitive, ie
HBV DNA can be detected at levels at which HBeAg is usually undetectable.
Anti-HBeAg appears after the disappearance of HBeAg. It indicates diminished viral
replication and decreased infectivity.
2818

Next Question

https://mypastest.pastest.com/Secure/TestMe/Browser/429893#Top

1/2

8/22/2016

MyPastest

Previous Question

Tag Question

Feedback

End Review

Difficulty: Difficult
Peer Responses

Session Progress
Responses Correct:

Responses Incorrect:

298

Responses Total:

298

Responses - % Correct:

0%

Blog (https://www.pastest.com/blog) About Pastest (https://www.pastest.com/about-us)


Contact Us (https://www.pastest.com/contact-us) Help (https://www.pastest.com/help)
Pastest 2016

https://mypastest.pastest.com/Secure/TestMe/Browser/429893#Top

2/2

8/22/2016

MyPastest

Back to Filters (/Secure/TestMe/Filter/429893/QA)

Question 46 of 298

A 26-year-old schoolteacher was diagnosed with hepatitis B infection 2 years ago. She is now
worried about whether the infection is still active. Which of the following test results is most
sensitive in confirming continued viral replication?
A

HBV DNA

IgM anti-HBc

Anti-HBeAg

HBeAg

Anti-HBs

Explanation
Hepatitis B activity assessment
HBV DNA is the most sensitive index of viral replication and is found without e-antigen
in hepatitis due to mutants.
The presence of IgM anti-HBc indicates acute hepatitis.
HBeAg testing is indicated in the follow-up of chronic infection.
Anti-HBeAg indicates diminished viral replication and decreased infectivity.
2819

Next Question

Previous Question

Tag Question

Feedback

End Review

Difficulty: Average
Peer Responses

https://mypastest.pastest.com/Secure/TestMe/Browser/429893#Top

1/2

8/22/2016

MyPastest

Session Progress
Responses Correct:

Responses Incorrect:

298

Responses Total:

298

Responses - % Correct:

0%

Blog (https://www.pastest.com/blog) About Pastest (https://www.pastest.com/about-us)


Contact Us (https://www.pastest.com/contact-us) Help (https://www.pastest.com/help)
Pastest 2016

https://mypastest.pastest.com/Secure/TestMe/Browser/429893#Top

2/2

8/22/2016

MyPastest

Back to Filters (/Secure/TestMe/Filter/429893/QA)

Question 47 of 298

A resident doctor who was infected with hepatitis B a year ago now presents with jaundice,
weight loss and malaise. Her IgM anti-HBc titre is not elevated but her serum IgM anti-delta is
raised, along with IgG anti-HBc.
What is the most likely diagnosis?
A

Hepatitis B

Hepatitis C

Hepatitis D

Hepatitis A

Hepatitis E

Explanation
Hepatitis D serology
Hepatitis D viral infection occurs as a superinfection in an HBsAg-positive patient.
Superinfection results in an acute flare-up of a previously quiescent chronic HBV infection.
Diagnosis is by finding serum IgM anti-delta at the same time as IgG anti-HBc. Patients are
usually negative for IgM anti-HBc.
2820

Next Question

Previous Question

Tag Question

Feedback

End Review

Difficulty: Average
Peer Responses

https://mypastest.pastest.com/Secure/TestMe/Browser/429893#Top

1/2

8/22/2016

MyPastest

Session Progress
Responses Correct:

Responses Incorrect:

298

Responses Total:

298

Responses - % Correct:

0%

Blog (https://www.pastest.com/blog) About Pastest (https://www.pastest.com/about-us)


Contact Us (https://www.pastest.com/contact-us) Help (https://www.pastest.com/help)
Pastest 2016

https://mypastest.pastest.com/Secure/TestMe/Browser/429893#Top

2/2

8/22/2016

MyPastest

Back to Filters (/Secure/TestMe/Filter/429893/QA)

Question 48 of 298

A social worker has been diagnosed with hepatitis C virus (HCV) infection.
Which test will conclusively establish the presence of this infection?
A

Anti-HCV

ELISA-3

HBV DNA

HCV RNA

IgM anti-HAV

Explanation
Hepatitis C serology
HCV RNA is the most sensitive test for detecting hepatitis C infection it is detected 1
2 weeks after infection.
Anti-HCV is usually positive 6 weeks from infection.
A tentative diagnosis of hepatitis C can also be made in hepatitis with negative markers
for HAV, HBV and other viruses.
Antigens from the nucleocapsid regions have been used to develop enzyme-linked
immunosorbent assays (ELISA). Only certain antigens are incorporated in the currently
available assays.
2821

Next Question

Previous Question

Tag Question

Feedback

End Review

Difficulty: Easy
https://mypastest.pastest.com/Secure/TestMe/Browser/429893#Top

1/2

8/22/2016

MyPastest

Peer Responses

Session Progress
Responses Correct:

Responses Incorrect:

298

Responses Total:

298

Responses - % Correct:

0%

Blog (https://www.pastest.com/blog) About Pastest (https://www.pastest.com/about-us)


Contact Us (https://www.pastest.com/contact-us) Help (https://www.pastest.com/help)
Pastest 2016

https://mypastest.pastest.com/Secure/TestMe/Browser/429893#Top

2/2

8/22/2016

MyPastest

Back to Filters (/Secure/TestMe/Filter/429893/QA)

Question 49 of 298

A 10-year-old Egyptian boy who has recently come to live in the UK is found to have
hepatitis C infection. His father is unsure about how he could have acquired this disease. His
mother died of jaundice 10 years ago. He was treated in Egypt 3 years ago for a bladder
infection and the passage of blood in his urine.
What is the most likely method of transmission in this case?
A

Contact with the local population

Vertical transmission

Sexual transmission

Contaminated drinking water

Intramuscular injections

Explanation
Parenteral antimony treatment and hepatitis C infection
While vertical transmission is rare, it is the most likely cause here. Previously, when antimony
injections were used for treatment of schistosomiasis the intramuscular injections option
would have been the correct answer, though now praziquantel is the treatment of choice.
2822

Next Question

Previous Question

Tag Question

Feedback

End Review

Difficulty: Easy
Peer Responses

https://mypastest.pastest.com/Secure/TestMe/Browser/429893#Top

1/2

8/22/2016

MyPastest

Session Progress
Responses Correct:

Responses Incorrect:

298

Responses Total:

298

Responses - % Correct:

0%

Blog (https://www.pastest.com/blog) About Pastest (https://www.pastest.com/about-us)


Contact Us (https://www.pastest.com/contact-us) Help (https://www.pastest.com/help)
Pastest 2016

https://mypastest.pastest.com/Secure/TestMe/Browser/429893#Top

2/2

8/22/2016

MyPastest

Back to Filters (/Secure/TestMe/Filter/429893/QA)

Question 50 of 298

A 47-year-old patient with maturity-onset diabetes is being advised about his diet.
Which of the following foods should he be most careful to avoid as far as possible?
A

Banana

Peanuts

Carrots

Cornflakes

Yoghurt

Explanation
The glycaemic index
Cornflakes have the highest glycaemic index (84) in this list. The glycaemic index is a
measure of a foods ability to raise blood sugar levels. The indexing is achieved by comparing
a foods digestion rate with that of glucose (which has a glycaemic index of 100). The higher
the glycaemic index, the faster the food will enter the bloodstream and raise the blood
glucose level. The other foods listed all have a lower glycaemic index banana (50), carrot
(49), yoghurt (33) and peanuts (14).
2823

Next Question

Previous Question

Tag Question

Feedback

End Review

Difficulty: Difficult
Peer Responses

https://mypastest.pastest.com/Secure/TestMe/Browser/429893#Top

1/2

8/22/2016

MyPastest

Session Progress
Responses Correct:

Responses Incorrect:

298

Responses Total:

298

Responses - % Correct:

0%

Blog (https://www.pastest.com/blog) About Pastest (https://www.pastest.com/about-us)


Contact Us (https://www.pastest.com/contact-us) Help (https://www.pastest.com/help)
Pastest 2016

https://mypastest.pastest.com/Secure/TestMe/Browser/429893#Top

2/2

8/22/2016

MyPastest

Back to Filters (/Secure/TestMe/Filter/429893/QA)

Question 51 of 298

A 41-year-old man presents with a 5-year history of recurrent episodes of bloody diarrhoea.
He has been diagnosed with ulcerative colitis. Despite regular treatment with adequate doses
of sulfasalazine, he has had several exacerbations of his disease and has required several
weeks of steroids to control the flare-ups.
What is the best next line of treatment for him?
A

Methotrexate

Azathioprine

Ciclosporin

Cyclophosphamide

Subtotal colectomy

Explanation
Second-line treatment of ulcerative colitis
This patient has ulcerative colitis that is not controlled with sulfasalazine or steroids. The
treatment of choice now is azathioprine. This would reduce the requirement of steroids as
well as maintaining the patient in remission. Ciclosporin is useful in ulcerative colitis but it
does not reduce the relapse rate and is not effective as maintenance therapy.
In acute disease, subtotal colectomy with end-ileostomy and preservation of the rectum is the
operation of choice where medical treatment has failed or if complications occur (eg
haemorrhage, perforation or toxic dilatation).
Methotrexate is useful in steroid-dependent Crohns disease as an alternative to azathioprine,
but not in ulcerative colitis. Cyclophosphamide has no role in the management of either
Crohns disease or ulcerative colitis.
2824

Next Question

Previous Question

Tag Question

https://mypastest.pastest.com/Secure/TestMe/Browser/429893#Top

Feedback

End Review
1/2

8/22/2016

MyPastest

Difficulty: Average
Peer Responses

Session Progress
Responses Correct:

Responses Incorrect:

298

Responses Total:

298

Responses - % Correct:

0%

Blog (https://www.pastest.com/blog) About Pastest (https://www.pastest.com/about-us)


Contact Us (https://www.pastest.com/contact-us) Help (https://www.pastest.com/help)
Pastest 2016

https://mypastest.pastest.com/Secure/TestMe/Browser/429893#Top

2/2

8/22/2016

MyPastest

Back to Filters (/Secure/TestMe/Filter/429893/QA)

Question 52 of 298

A 37-year-old man presented complaining of severe epigastric pain that has become more or
less constant. A screening test for Helicobacter pylori is positive.
Which of the following conditions is most strongly associated with this infection?
A

Erosive gastropathy

Gastric stromal tumour

Gastric B-cell lymphoma

Gastro-oesophageal reflux disease

Autoimmune gastritis

Explanation
Helicobacter pylori-associated conditions
Over 70% of patients with gastric B-cell lymphomas (mucosal-associated lymphoid tissue
lymphoma or MALT lymphoma) have Helicobacter pylori infection. H. pylori gastritis has been
shown to contain the clonal B cell that eventually gives rise to the MALT lymphoma. Some
low-grade tumours regress with H. pylori eradication alone. Gastric stromal tumour is not
associated with H. pylori infection.
The eradication of H. pylori in gastro-oesophageal reflux disease is controversial. Some
believe that after eradication acid secretion increases and worsens reflux symptoms.
However, H. pylori is not known to play a role in the pathogenesis of this disease.
Gastropathy is a term used where there is mucosal damage but little or no accompanying
inflammation. The commonest cause is the use of aspirin, other non-steroidal antiinflammatory drugs (NSAIDs) and alcohol. Autoimmune gastritis leads to atrophic gastritis
with the loss of parietal cells. This is not associated with H. pylori.
http://www.ncbi.nlm.nih.gov/pubmed/11025354
(http://www.ncbi.nlm.nih.gov/pubmed/11025354)
2825

Next Question

https://mypastest.pastest.com/Secure/TestMe/Browser/429893#Top

1/2

8/22/2016

MyPastest

Previous Question

Tag Question

Feedback

End Review

Difficulty: Average
Peer Responses

Session Progress
Responses Correct:

Responses Incorrect:

298

Responses Total:

298

Responses - % Correct:

0%

Blog (https://www.pastest.com/blog) About Pastest (https://www.pastest.com/about-us)


Contact Us (https://www.pastest.com/contact-us) Help (https://www.pastest.com/help)
Pastest 2016

https://mypastest.pastest.com/Secure/TestMe/Browser/429893#Top

2/2

8/22/2016

MyPastest

Back to Filters (/Secure/TestMe/Filter/429893/QA)

Question 53 of 298

A junior doctor from Nigeria is being investigated following a needlestick injury sustained
while taking a blood sample from a patient infected with hepatitis B virus, the doctor's
vaccination status is unknown.
Which test will provide the earliest diagnosis of hepatitis B infection in the junior doctor?
A

HBeAg

IgM anti-HBc

Anti-HBeAg

HBsAg

IgG anti-HBc

Explanation
Hepatitis B testing
Following hepatitis B infection, the first virological marker detectable in the serum is HBsAg.
IgM anti-HBc would confirm the diagnosis of an acute infection. HBeAg and anti-HBeAg
appear in the serum later. IgG anti-HBc indicates past exposure to hepatitis B (HBsAgnegative). Anti-HBsAg antibody is the marker associated with vaccination.
2826

Next Question

Previous Question

Tag Question

Feedback

End Review

Difficulty: Average
Peer Responses

https://mypastest.pastest.com/Secure/TestMe/Browser/429893#Top

1/2

8/22/2016

MyPastest

Session Progress
Responses Correct:

Responses Incorrect:

298

Responses Total:

298

Responses - % Correct:

0%

Blog (https://www.pastest.com/blog) About Pastest (https://www.pastest.com/about-us)


Contact Us (https://www.pastest.com/contact-us) Help (https://www.pastest.com/help)
Pastest 2016

https://mypastest.pastest.com/Secure/TestMe/Browser/429893#Top

2/2

8/22/2016

MyPastest

Back to Filters (/Secure/TestMe/Filter/429893/QA)

Question 54 of 298

A 3-year-old boy presents with a 2-year history of constipation and abdominal distension. A
plain radiograph of the abdomen reveals faecal matter-containing distended bowel loops. A
barium enema study shows a transition zone at the rectosigmoid junction with reversal of the
rectosigmoid ratio. What is the most probable diagnosis?
A

Malrotation of the gut

Anal atresia

Hirschsprungs disease

Idiopathic slow transit

Megarectum

Explanation
Hirschsprungs disease
This boy most probably has Hirschsprungs disease. The barium enema demonstrates an
aganglionic segment that appears narrow in comparison with the dilated proximal bowel. This
area is called the transition zone. Normally the rectum is wider than the rest of the colon
(except for the caecum). When the rectum is seen to be narrower than the proximal colon,
this is pathognomonic of Hirschsprungs disease. These features are not seen in any of the
other conditions that also cause constipation.
2827

Next Question

Previous Question

Tag Question

Feedback

End Review

Difficulty: Average
Peer Responses

https://mypastest.pastest.com/Secure/TestMe/Browser/429893#Top

1/2

8/22/2016

MyPastest

Session Progress
Responses Correct:

Responses Incorrect:

298

Responses Total:

298

Responses - % Correct:

0%

Blog (https://www.pastest.com/blog) About Pastest (https://www.pastest.com/about-us)


Contact Us (https://www.pastest.com/contact-us) Help (https://www.pastest.com/help)
Pastest 2016

https://mypastest.pastest.com/Secure/TestMe/Browser/429893#Top

2/2

8/22/2016

MyPastest

Back to Filters (/Secure/TestMe/Filter/429893/QA)

Question 55 of 298

An 18-month-old boy is suspected of having Hirschsprungs disease.


Which of the following investigations would provide a definitive diagnosis?
A

Barium enema

Rectal manometry

Colonoscopy

Rectal biopsy

Proctoscopy

Explanation
Diagnosis of Hirschsprungs disease
The definitive diagnosis of Hirschsprungs disease is made by rectal biopsy. This would show:
Absence of ganglion cells in Meissners and myenteric plexuses
Hypertrophy of the nerve trunks
Increased staining for acetylcholinesterase: this enzyme is elevated in Hirschsprungs
disease and would be evident in a full-thickness biopsy on frozen section
All the other investigations would be only suggestive of Hirschsprungs disease.
2828

Next Question

Previous Question

Tag Question

Feedback

End Review

Difficulty: Average
Peer Responses
https://mypastest.pastest.com/Secure/TestMe/Browser/429893#Top

1/2

8/22/2016

MyPastest

Session Progress
Responses Correct:

Responses Incorrect:

298

Responses Total:

298

Responses - % Correct:

0%

Blog (https://www.pastest.com/blog) About Pastest (https://www.pastest.com/about-us)


Contact Us (https://www.pastest.com/contact-us) Help (https://www.pastest.com/help)
Pastest 2016

https://mypastest.pastest.com/Secure/TestMe/Browser/429893#Top

2/2

8/22/2016

MyPastest

Back to Filters (/Secure/TestMe/Filter/429893/QA)

Question 56 of 298

A 72-year-old patient has hepatocellular carcinoma.


Of the following, which one is the most significant feature in the epidemiology of this
disease?
A

It has a high incidence in the West

The most common aetiological factor is alcoholic cirrhosis

Primary biliary cirrhosis is associated with a higher incidence of hepatocellular


carcinoma than haemochromatosis

Over 80% of tumours are surgically resectable

The risk of hepatocellular carcinoma is increased in patients with hepatitis B who are
antigen positive

Explanation
Hepatocellular carcinoma
The development of hepatocellular carcinoma (HCC) is related to the integration of
viral DNA into the genome of the host hepatocyte. The risk of HCC is elevated in both
hepatitis B (HBV) and hepatitis C (HCV). E-antigen positivity in association with HBV is
associated with increased risk of hepatocellular carcinoma.
HCC has a high incidence in East Africa and South-East Asia.
Because of its multicentricity, bilobar involvement, portal vein invasion and lymphatic
metastases, only 1520% of cases are resectable.
Some 30% of patients with cirrhosis due to haemochromatosis will develop primary
HCC.
Males are affected more than females, which might account for the high incidence seen
in haemochromatosis and low incidence in primary biliary cirrhosis.
2830

https://mypastest.pastest.com/Secure/TestMe/Browser/429893#Top

1/2

8/22/2016

MyPastest

Next Question

Previous Question

Tag Question

Feedback

End Review

Difficulty: Average
Peer Responses

Session Progress
Responses Correct:

Responses Incorrect:

298

Responses Total:

298

Responses - % Correct:

0%

Blog (https://www.pastest.com/blog) About Pastest (https://www.pastest.com/about-us)


Contact Us (https://www.pastest.com/contact-us) Help (https://www.pastest.com/help)
Pastest 2016

https://mypastest.pastest.com/Secure/TestMe/Browser/429893#Top

2/2

8/22/2016

MyPastest

Back to Filters (/Secure/TestMe/Filter/429893/QA)

Question 57 of 298

A 50-year-old woman presents with a 1-year history of recurrent episodes of right upper
abdominal pain. She has now had jaundice and fever for the past 4 days. On examination she
appears toxic. Her blood pressure is 90/60 mmHg. Abdominal ultrasonography demonstrates
stones in the common bile duct.
What is the best next step once she is adequately stabilised?
A

ERCP

Laparoscopic cholecystectomy

Laparotomy and stone extraction

Lithotripsy

Open cholecystectomy

Explanation
<h2>Acute cholangitis</h2>
This patient has cholangitis presenting as Charcots triad fever, pain and jaundice. She is
also toxic. The definitive management is to relieve the pressure in the obstructed biliary
system. Endoscopic bile duct clearance is the preferred technique. Cholecystectomy is
indicated in symptomatic gallstones but if possible not in the stage of acute cholecystitis.
Extracorporeal shock-wave lithotripsy (ESWL) might not be useful in this case because the
stones have to be evaluated for size and composition the greater the calcium content of the
stone, the less likely the success of fragmentation. The stones also have to be greater than 10
mm in diameter; common bile duct stones can be smaller than this.
2831

Next Question

Previous Question

Tag Question

Feedback

End Review

Difficulty: Average
https://mypastest.pastest.com/Secure/TestMe/Browser/429893#Top

1/2

8/22/2016

MyPastest

Peer Responses

Session Progress
Responses Correct:

Responses Incorrect:

298

Responses Total:

298

Responses - % Correct:

0%

Blog (https://www.pastest.com/blog) About Pastest (https://www.pastest.com/about-us)


Contact Us (https://www.pastest.com/contact-us) Help (https://www.pastest.com/help)
Pastest 2016

https://mypastest.pastest.com/Secure/TestMe/Browser/429893#Top

2/2

8/22/2016

MyPastest

Back to Filters (/Secure/TestMe/Filter/429893/QA)

Question 58 of 298

A 60-year-old man with coronary artery disease was found to have gallstones on routine
abdominal ultrasonography. He has no history of biliary colic or jaundice.
What is the best treatment for this patient?
A

Laparoscopic cholecystectomy

Endoscopic removal of stones

Open cholecystectomy

No treatment

Lithotripsy

Explanation
Asymptomatic cholelithiasis
Therapeutic intervention is not required for a patient with asymptomatic cholelithiasis. Less
than 25% of patients with asymptomatic cholelithiasis will develop symptoms that require
intervention over a 5-year follow-up period.
2832

Next Question

Previous Question

Tag Question

Feedback

End Review

Difficulty: Easy
Peer Responses

https://mypastest.pastest.com/Secure/TestMe/Browser/429893#Top

1/2

8/22/2016

MyPastest

Session Progress
Responses Correct:

Responses Incorrect:

298

Responses Total:

298

Responses - % Correct:

0%

Blog (https://www.pastest.com/blog) About Pastest (https://www.pastest.com/about-us)


Contact Us (https://www.pastest.com/contact-us) Help (https://www.pastest.com/help)
Pastest 2016

https://mypastest.pastest.com/Secure/TestMe/Browser/429893#Top

2/2

8/22/2016

MyPastest

Back to Filters (/Secure/TestMe/Filter/429893/QA)

Question 59 of 298

A 47-year-old publican presents with abnormal liver function tests. Which of the following
would be most suggestive of a diagnosis of chronic active hepatitis?
A

Positive smooth-muscle antibodies

On concurrent methyldopa

A past history of hepatitis A infection

Minimal alcohol ingestion

Pruritus

Explanation
Chronic active hepatitis
Chronic active hepatitis has four major causes:
Hepatitis B virus (HBV)
Hepatitis C virus (HCV)
Autoimmune
Drugs (methyldopa, isoniazid)
In autoimmune hepatitis there is a characteristic elevation of serum antibodies smoothmuscle antibodies (SMA) 70%, antibodies to nuclear antigen (ANF) 70%, antimitochondrial
antibodies (AMA) 35%.
Associations are with peripheral arthritis, thyroid disease, haemolytic anaemia and
glomerulonephritis. Metabolic causes include Wilson's disease and 1-antitrypsin deficiency.
Pruritus is a symptom of cholestasis, a very late sign in inflammatory liver disease.
3194

Next Question

https://mypastest.pastest.com/Secure/TestMe/Browser/429893#Top

1/2

8/22/2016

MyPastest

Previous Question

Tag Question

Feedback

End Review

Difficulty: Average
Peer Responses

Session Progress
Responses Correct:

Responses Incorrect:

298

Responses Total:

298

Responses - % Correct:

0%

Blog (https://www.pastest.com/blog) About Pastest (https://www.pastest.com/about-us)


Contact Us (https://www.pastest.com/contact-us) Help (https://www.pastest.com/help)
Pastest 2016

https://mypastest.pastest.com/Secure/TestMe/Browser/429893#Top

2/2

8/22/2016

MyPastest

Back to Filters (/Secure/TestMe/Filter/429893/QA)

Question 60 of 298

A patient with chronic liver disease presents with abdominal swelling. On examination he is
jaundiced and has moderate ascites but does not have any peripheral oedema.
Which one of the following is true concerning management of his ascites?
A

A low-salt diet is unnecessary if the patient is on diuretics

Loop diuretics are preferable to potassium-sparing agents

Weight loss greater than 1.5 kg/day is the ideal

Paracentesis is less likely to be successful if the patient has peripheral oedema

Paracentesis is contraindicated if the patients prothrombin time is prolonged

Explanation
Management of ascites
A major reason for so-called diuretic-resistant ascites is an excess sodium intake
Spironolactone is more effective than furosemide because the site of sodium retention
in cirrhosis is the distal nephron
The ideal weight loss is 0.5 kg/day
Paracentesis is relatively contraindicated if the patient is encephalopathic, but
prolonged clotting is the norm in these patients and those without leg oedema do
better with respect to response to therapy in terms of abdominal symptoms versus
those with peripheral oedema
3195

Next Question

Previous Question

Tag Question

https://mypastest.pastest.com/Secure/TestMe/Browser/429893#Top

Feedback

End Review

1/2

8/22/2016

MyPastest

Difficulty: Difficult
Peer Responses

Session Progress
Responses Correct:

Responses Incorrect:

298

Responses Total:

298

Responses - % Correct:

0%

Blog (https://www.pastest.com/blog) About Pastest (https://www.pastest.com/about-us)


Contact Us (https://www.pastest.com/contact-us) Help (https://www.pastest.com/help)
Pastest 2016

https://mypastest.pastest.com/Secure/TestMe/Browser/429893#Top

2/2

8/22/2016

MyPastest

Back to Filters (/Secure/TestMe/Filter/429893/QA)

Question 61 of 298

A diagnosis of biliary stones in a 43-year-old executive with right-sided abdominal pain is


supported by which one of the following?
A

A past history of a right hemicolectomy for Crohns disease

An increase in dietary cholesterol

Thyrotoxicosis

Irritable bowel syndrome

Raised aspartate aminotransferase (AST) and alanine aminotransferase (ALT) levels

Explanation
Risk factors for biliary stones
Cholesterol gallstones are thought to arise as a result of a triple defect:
Supersaturation of gallbladder bile (high in cholesterol, low in bile salts)
Increased rate of cholesterol nucleation in the gallbladder
Reduction in gallbladder contractility
Predisposing factors to gallstone formation:
Older age
Female sex (oestrogens)
Oral contraceptive use
Cirrhosis (bile pigment stones)
Ileal resection (by reducing entero-hepatic circulation and increasing bile salt loss)
Clofibrate (by increasing biliary supersaturation)
Colestyramine (by binding bile salts)
Crohns disease

https://mypastest.pastest.com/Secure/TestMe/Browser/429893#Top

1/2

8/22/2016

MyPastest

The relative risk of cholesterol containing gallstones is much greater for ileal resection than
for high cholesterol diet.
3196

Next Question

Previous Question

Tag Question

Feedback

End Review

Difficulty: Average
Peer Responses

Session Progress
Responses Correct:

Responses Incorrect:

298

Responses Total:

298

Responses - % Correct:

0%

Blog (https://www.pastest.com/blog) About Pastest (https://www.pastest.com/about-us)


Contact Us (https://www.pastest.com/contact-us) Help (https://www.pastest.com/help)
Pastest 2016

https://mypastest.pastest.com/Secure/TestMe/Browser/429893#Top

2/2

8/22/2016

MyPastest

Back to Filters (/Secure/TestMe/Filter/429893/QA)

Question 62 of 298

A 56-year-old retired primary schoolteacher gives a 5-month history of pruritus and lethargy.
On examination she is jaundiced and has a palpable liver and a just-palpable spleen. She
admits to drinking two units of alcohol per day and twenty-four hours after admission she has
a haematemesis and her haemoglobin drops to to 8.8 g/dl. Upper gastrointestinal endoscopy
reveals oesophageal varices. Anti-mitochondrial antibodies are noted.
What is the most likely diagnosis?
A

Chronic active hepatitis

Haemochromatosis

Alcoholic cirrhosis

Primary biliary cirrhosis

Cryptogenic cirrhosis

Explanation
Primary biliary cirrhosis (PBC)
The history of pruritus and the finding of jaundice as well as evidence of cirrhosis in the form
of varices raises the possibility of PBC. Her very modest alcohol history counts against
alcoholic cirrhosis, and the anti-mitochondrial antibodies are strongly suggestive of PBC.
3197

Next Question

Previous Question

Tag Question

Feedback

End Review

Difficulty: Average
Peer Responses

https://mypastest.pastest.com/Secure/TestMe/Browser/429893#Top

1/2

8/22/2016

MyPastest

Session Progress
Responses Correct:

Responses Incorrect:

298

Responses Total:

298

Responses - % Correct:

0%

Blog (https://www.pastest.com/blog) About Pastest (https://www.pastest.com/about-us)


Contact Us (https://www.pastest.com/contact-us) Help (https://www.pastest.com/help)
Pastest 2016

https://mypastest.pastest.com/Secure/TestMe/Browser/429893#Top

2/2

8/22/2016

MyPastest

Back to Filters (/Secure/TestMe/Filter/429893/QA)

Question 63 of 298

A 22-year-old patient presents with unexplained jaundice that occurs repeatedly during
episodes of starvation.
What is the most likely diagnosis?
A

Haemochromatosis

Hepatitis B virus infection

Gilbert syndrome

Wilsons disease

Alcohol abuse

Explanation
Gilbert syndrome
Gilbert syndrome is a mild unconjugated hyperbilirubinaemia. Factors that raise serum
bilirubin are fasting, infection and haemolysis. Symptoms such as fatigue are loosely linked
but many consider them incidental findings. The male to female ratio is 7:2.
3199

Next Question

Previous Question

Tag Question

Feedback

End Review

Difficulty: Easy
Peer Responses

https://mypastest.pastest.com/Secure/TestMe/Browser/429893#Top

1/2

8/22/2016

MyPastest

Session Progress
Responses Correct:

Responses Incorrect:

298

Responses Total:

298

Responses - % Correct:

0%

Blog (https://www.pastest.com/blog) About Pastest (https://www.pastest.com/about-us)


Contact Us (https://www.pastest.com/contact-us) Help (https://www.pastest.com/help)
Pastest 2016

https://mypastest.pastest.com/Secure/TestMe/Browser/429893#Top

2/2

8/22/2016

MyPastest

Back to Filters (/Secure/TestMe/Filter/429893/QA)

Question 64 of 298

A 29-year-old secretary is admitted to hospital with a haematemesis. She has been drinking
'several' cans of cider a day for more than 10 years. On examination, she is deeply jaundiced
but alert, with stigmata of chronic liver disease. Blood pressure is 95/50 mmHg, pulse 130
bpm. After appropriate resuscitation, her blood pressure rises to 125/70 mmHg and her pulse
to 100 bpm. She is still vomiting small amounts of fresh blood.
What would be your next course of action?
A

Give intravenous nitrates

Intravenous Terlipressin

Intravenous Somatostatin

Endoscopy and banding

Sengstaken/Blakemore tube

Explanation
Management of variceal bleeding
The priorities of management are:
Resuscitate
Stabilise and stop bleeding Terlipressin > Octreotide
Endoscope and banding of varices or sclerotherapy
Sengstaken tube reserved for failure to respond to these three measures
3201

Next Question

Previous Question

Tag Question

https://mypastest.pastest.com/Secure/TestMe/Browser/429893#Top

Feedback

End Review

1/2

8/22/2016

MyPastest

Difficulty: Difficult
Peer Responses

Session Progress
Responses Correct:

Responses Incorrect:

298

Responses Total:

298

Responses - % Correct:

0%

Blog (https://www.pastest.com/blog) About Pastest (https://www.pastest.com/about-us)


Contact Us (https://www.pastest.com/contact-us) Help (https://www.pastest.com/help)
Pastest 2016

https://mypastest.pastest.com/Secure/TestMe/Browser/429893#Top

2/2

8/22/2016

MyPastest

Back to Filters (/Secure/TestMe/Filter/429893/QA)

Question 65 of 298

A 55-year-old geography teacher is referred for management of obesity. He has a body mass
index (BMI) of 36 kg/m2 and investigations show: cholesterol 7.7 mmol/l, fasting triglycerides
of 3.1 mmol/l, alanine aminotransferase (ALT) 150 U/l. He denies alcohol excess.
What is the significance of his raised liver enzymes?
A

Negligible he can be reassured

Likely autoimmune hepatitis

Low significance should be monitored but will improve with weight reduction

Probably has NASH, which can include fibrosis

Highly significant has cirrhosis

Explanation
Non-alcoholic steatohepatitis
This is NASH non-alcoholic steatohepatitis. The patient has all the known risk factors and
has a high probability of having already developed fibrosis. NASH is now considered to one of
the main underlying cause of so-called cryptogenic cirrhosis.
3202

Next Question

Previous Question

Tag Question

Feedback

End Review

Difficulty: Easy
Peer Responses

https://mypastest.pastest.com/Secure/TestMe/Browser/429893#Top

1/2

8/22/2016

MyPastest

Session Progress
Responses Correct:

Responses Incorrect:

298

Responses Total:

298

Responses - % Correct:

0%

Blog (https://www.pastest.com/blog) About Pastest (https://www.pastest.com/about-us)


Contact Us (https://www.pastest.com/contact-us) Help (https://www.pastest.com/help)
Pastest 2016

https://mypastest.pastest.com/Secure/TestMe/Browser/429893#Top

2/2

8/22/2016

MyPastest

Back to Filters (/Secure/TestMe/Filter/429893/QA)

Question 66 of 298

A 36-year-old nurse with a 15-year history of ulcerative colitis develops abnormal liver
enzymes: alanine aminotransferase (ALT) 154 U/l, alkaline phosphatase 354 U/l, bilirubin
12 mol/l. An ultrasound is normal. She is antineutrophil cytoplasmic antibody- (ANCA-)
positive.
What would you be most likely to suspect?
A

Gallstones

Mesalazine hepatitis

Primary sclerosing cholangitis

Chronic active hepatitis

Primary biliary cirrhosis

Explanation
Primary sclerosing cholangitis
Primary sclerosing cholangitis classically occurs with inflammatory bowel disease, especially
ulcerative colitis and is associated with a high risk of cholangiocarcinoma and colon cancer.
3203

Next Question

Previous Question

Tag Question

Feedback

End Review

Difficulty: Easy
Peer Responses

https://mypastest.pastest.com/Secure/TestMe/Browser/429893#Top

1/2

8/22/2016

MyPastest

Session Progress
Responses Correct:

Responses Incorrect:

298

Responses Total:

298

Responses - % Correct:

0%

Blog (https://www.pastest.com/blog) About Pastest (https://www.pastest.com/about-us)


Contact Us (https://www.pastest.com/contact-us) Help (https://www.pastest.com/help)
Pastest 2016

https://mypastest.pastest.com/Secure/TestMe/Browser/429893#Top

2/2

8/22/2016

MyPastest

Back to Filters (/Secure/TestMe/Filter/429893/QA)

Question 67 of 298

A 35-year-old office worker presents with a 5-month history of intermittent right-sided


abdominal pain, watery but not bloody diarrhoea, intermittent vomiting and weight loss of 1
stone. On examination she is clubbed. Abdominal examination confirms right sided
tenderness, and rectal examination is unremarkable. Her haemoglobin is 11.1 g/dl, white cell
count (WCC) 9.8 109/l and C-reactive protein (CRP) 15 mg/l (normal range 010 mg/l).
What is the most likely diagnosis?
A

Large-bowel Crohns disease

Ulcerative colitis

Subacute appendicitis

Small-bowel Crohns disease

Irritable bowel syndrome

Explanation
<h2>Crohns disease</h2>
The combination of watery diarrhoea, minimally raised acute-phase proteins and a normal
rectum exclude rectal disease. Clubbing is seen in active small-bowel Crohns disease. The
lack of bloody diarrhoea and the right-sided pain point more towards proximal rather than
distal disease.
3204

Next Question

Previous Question

Tag Question

Feedback

End Review

Difficulty: Average
Peer Responses

https://mypastest.pastest.com/Secure/TestMe/Browser/429893#Top

1/2

8/22/2016

MyPastest

Session Progress
Responses Correct:

Responses Incorrect:

298

Responses Total:

298

Responses - % Correct:

0%

Blog (https://www.pastest.com/blog) About Pastest (https://www.pastest.com/about-us)


Contact Us (https://www.pastest.com/contact-us) Help (https://www.pastest.com/help)
Pastest 2016

https://mypastest.pastest.com/Secure/TestMe/Browser/429893#Top

2/2

8/22/2016

MyPastest

Back to Filters (/Secure/TestMe/Filter/429893/QA)

Question 68 of 298

A 55-year-old mechanic presents with a 4-week history of tenesmus and rectal bleeding. His
bowel habit has not changed significantly. Rectal examination reveals a granular mucosa and
a sigmoidoscopy reveals touch bleeding on a background of diffuse erythema. Above 10 cm,
the mucosa appears to be normal. Rectal biopsies show generalised mucosal inflammation
with crypt abscesses.
What would the most appropriate initial therapy be?
A

Oral prednisolone

Oral mesalazine

Oral sulfasalazine

Rectal steroids

Rectal mesalazine

Explanation
Distal proctitis variant of ulcerative colitis
This is the distal proctitis variant of ulcerative colitis. An important management step is to
treat anterior proximal constipation. In this situation where there is not a significant change in
bowel habit, locally delivered 5-aminosalicylic acid (5-ASA) preparations may be tried initially.
3206

Next Question

Previous Question

Tag Question

Feedback

End Review

Difficulty: Difficult
Peer Responses

https://mypastest.pastest.com/Secure/TestMe/Browser/429893#Top

1/2

8/22/2016

MyPastest

Session Progress
Responses Correct:

Responses Incorrect:

298

Responses Total:

298

Responses - % Correct:

0%

Blog (https://www.pastest.com/blog) About Pastest (https://www.pastest.com/about-us)


Contact Us (https://www.pastest.com/contact-us) Help (https://www.pastest.com/help)
Pastest 2016

https://mypastest.pastest.com/Secure/TestMe/Browser/429893#Top

2/2

8/22/2016

MyPastest

Back to Filters (/Secure/TestMe/Filter/429893/QA)

Question 69 of 298

A 23-year-old art student presents with a 5-week history of bloody diarrhoea, which has
become more severe in the past 48 hours. She feels tired and depressed. On examination she
has a pulse rate of 120 bpm, blood pressure of 95/50 mmHg, temperature 37.8 C, and is
tender to palpation in the left iliac fossa. Rectal examination reveals a granular mucosa.
Investigations show: haemoglobin is 9.2 g/dl, white cell count 11 109/l, albumin 29 g/l, Creactive protein (CRP) 54 mg/l. What would be your next step?
A

Sigmoidoscopy and biopsy

Plain abdominal X-ray

Stool culture

Intravenous hydrocortisone

IV antibiotics

Explanation
Toxic dilatation of the colon
This is a classic presentation of severe toxic dilatation of the colon. The most reliable sign is
the pulse rate; the most helpful investigation is a plain abdominal X-ray. The treatment of
choice for established dilatation is colectomy. If the X-ray shows less severe changes, then the
other investigations and management steps become appropriate.
3207

Next Question

Previous Question

Tag Question

Feedback

End Review

Difficulty: Average
Peer Responses

https://mypastest.pastest.com/Secure/TestMe/Browser/429893#Top

1/2

8/22/2016

MyPastest

Session Progress
Responses Correct:

Responses Incorrect:

298

Responses Total:

298

Responses - % Correct:

0%

Blog (https://www.pastest.com/blog) About Pastest (https://www.pastest.com/about-us)


Contact Us (https://www.pastest.com/contact-us) Help (https://www.pastest.com/help)
Pastest 2016

https://mypastest.pastest.com/Secure/TestMe/Browser/429893#Top

2/2

8/22/2016

MyPastest

Back to Filters (/Secure/TestMe/Filter/429893/QA)

Question 70 of 298

A 49-year-old man is sent to the Emergency Department with a 6-hour history of abdominal
pain and vomiting.
What is the first investigation that should be carried out to confirm bowel obstruction?
A

Abdominal ultrasound

Computed tomography of the abdomen

Abdominal X-ray

Exploratory surgery

C-reactive protein

Explanation
Diagnosis of intestinal obstruction
Intestinal obstruction is a medical emergency requiring urgent therapy and surgical
involvement. The history is the most useful clue; the most helpful early diagnostic tool is the
plain abdominal X-ray. The physical findings are generally non-specific, and acidosis is more
suggestive of gut ischaemia.
3208

Next Question

Previous Question

Tag Question

Feedback

End Review

Difficulty: Easy
Peer Responses

https://mypastest.pastest.com/Secure/TestMe/Browser/429893#Top

1/2

8/22/2016

MyPastest

Session Progress
Responses Correct:

Responses Incorrect:

298

Responses Total:

298

Responses - % Correct:

0%

Blog (https://www.pastest.com/blog) About Pastest (https://www.pastest.com/about-us)


Contact Us (https://www.pastest.com/contact-us) Help (https://www.pastest.com/help)
Pastest 2016

https://mypastest.pastest.com/Secure/TestMe/Browser/429893#Top

2/2

8/22/2016

MyPastest

Back to Filters (/Secure/TestMe/Filter/429893/QA)

Question 71 of 298

In a patient with ulcerative colitis the risk of developing colonic cancer is greatest in the
presence of which one of the following features?
A

Proctitis

Left-sided colitis

Onset of disease in childhood

Poor compliance with therapy

Annual relapses

Explanation
Risk factors for colon cancer in ulcerative colitis
Risk factors for colon cancer in ulcerative colitis are a combination of:
Onset at an early age
Extensive disease
Long duration (> 10years)
Unremitting disease
There is a suggestion (not yet proved), that strict adherence to 5-ASA medication might
reduce the risk.
3209

Next Question

Previous Question

Tag Question

Feedback

End Review

Difficulty: Average

https://mypastest.pastest.com/Secure/TestMe/Browser/429893#Top

1/2

8/22/2016

MyPastest

Peer Responses

Session Progress
Responses Correct:

Responses Incorrect:

298

Responses Total:

298

Responses - % Correct:

0%

Blog (https://www.pastest.com/blog) About Pastest (https://www.pastest.com/about-us)


Contact Us (https://www.pastest.com/contact-us) Help (https://www.pastest.com/help)
Pastest 2016

https://mypastest.pastest.com/Secure/TestMe/Browser/429893#Top

2/2

8/22/2016

MyPastest

Back to Filters (/Secure/TestMe/Filter/429893/QA)

Question 72 of 298

A 23-year-old woman develops severe watery diarrhoea 10 days after a complicated


childbirth. She is unwell, with a temperature of 38 C and pulse of 105 bpm, and is very
thirsty. On examination there is widespread non-specific abdominal tenderness. A plain
abdominal X-ray reveals gas throughout a non-dilated left-sided colon. Proctoscopy reveals
an inflamed rectum. Stool cultures and histology are awaited. She is given intravenous fluids,
but is still tachycardic and unwell.
What would be the next most appropriate management step?
A

Await further results

Start ciprofloxacin

Start metronidazole

Start hydrocortisone

Blood transfusion

Explanation
Pseudomembranous colitis
The abnormal rectal mucosa seen on proctoscopy and the recent complicated childbirth
where antibiotics are likely to have been prescribed, make Clostridium difficile
(pseudomembranous) colitis the most likely diagnosis. Ulcerative colitis is less likely as there
is no bleeding. Other acute bacterial infections do not usually cause a proctitis.
3210

Next Question

Previous Question

Tag Question

Feedback

End Review

Difficulty: Average
Peer Responses
https://mypastest.pastest.com/Secure/TestMe/Browser/429893#Top

1/2

8/22/2016

MyPastest

Session Progress
Responses Correct:

Responses Incorrect:

298

Responses Total:

298

Responses - % Correct:

0%

Blog (https://www.pastest.com/blog) About Pastest (https://www.pastest.com/about-us)


Contact Us (https://www.pastest.com/contact-us) Help (https://www.pastest.com/help)
Pastest 2016

https://mypastest.pastest.com/Secure/TestMe/Browser/429893#Top

2/2

8/22/2016

MyPastest

Back to Filters (/Secure/TestMe/Filter/429893/QA)

Question 73 of 298

A 58-year-old company director has a 6-month history of central abdominal pain


accompanied by 5 kg of weight loss. On direct questioning he admits to smoking
20 cigarettes a day for 30 years and to 'social drinking'. Investigations reveal: haemoglobin
13.4 g/dl, white cell count 7.8 109/l, C-reactive protein (CRP) 9 mg/l, amylase 78 U/l, albumin
39 g/l. Ultrasound and upper gastrointestinal endoscopy are both normal. An endoscopic
retrograde cholangiopancreatography (ERCP) shows calcification on the control film and
irregularity of pancreatic side branches.
Which one of the following would give him symptomatic relief?
A

Proton-pump inhibitor

Creon

Opiates

Dietary supplements

Distal pancreatectomy

Explanation
Chronic pancreatitis
The presence of pancreatic calcification on plain X-rays is pathognomonic of chronic
pancreatitis. Causes of chronic pancreatitis include alcohol, malnutrition and hyperlipidaemia.
It presents with pain, diabetes and steatorrhoea. Diagnosis is by the appearance of pancreatic
calcification on abdominal X-ray and by transabdominal ultrasonography, computed
tomography or endoscopic retrograde cholangiopancreatography (ERCP). An oral glucose
tolerance test (OGTT) will be abnormal in approximately 70%, and 50% will have steatorrhoea
(normal stool weight 200 g/day, containing 6 g fat).
The pain is treated with analgesics, antioxidants and surgery for duct obstruction or localised
disease. One of the difficulties of pain management of this condition is that a number of
factors may contribute to the pain in chronic pancreatitis, including anatomical/surgical
factors (e.g. pseudocyst, obstruction of biliary tree, pancreatic nerve inflammation). A trial of
non-coated pancreatic enzymes may be considered if pain is exacerbated by eating (but of

https://mypastest.pastest.com/Secure/TestMe/Browser/429893#Top

1/2

8/22/2016

MyPastest

limited evidence, benefit appears limited to those with early, non-alcoholic disease).
Pancreatic enzyme supplements plus proton-pump inhibitors are used to treat the
steatorrhoea.
3212

Next Question

Previous Question

Tag Question

Feedback

End Review

Difficulty: Difficult
Peer Responses

Session Progress
Responses Correct:

Responses Incorrect:

298

Responses Total:

298

Responses - % Correct:

0%

Blog (https://www.pastest.com/blog) About Pastest (https://www.pastest.com/about-us)


Contact Us (https://www.pastest.com/contact-us) Help (https://www.pastest.com/help)
Pastest 2016

https://mypastest.pastest.com/Secure/TestMe/Browser/429893#Top

2/2

8/22/2016

MyPastest

Back to Filters (/Secure/TestMe/Filter/429893/QA)

Question 74 of 298

A 42-year-old maths teacher has returned to the UK after living in Trinidad for 15 years. She
has a 3-year history of intermittent diarrhoea, suggestive of steatorrhoea, weight loss of 6 kg,
cramps in her calves and marked lethargy. Investigations reveal: haemoglobin 8.7 g/dl, mean
corpuscular volume (MCV) 77 fl, white cell count 9.8 109/l, albumin 29 g/l, corrected
calcium 1.9 mmol/l. Endomysial and gliadin antibodies are negative. A jejunal biopsy shows
abnormal villi with an inflammatory cell infiltrate of lymphocytes, plasma cells and
eosinophils.
What is the most likely diagnosis?
A

Coeliac disease

Crohns disease

Tropical sprue

Giardia lamblia infection

Whipples disease

Explanation
Tropical sprue
The jejunal biopsy can be abnormal in all the conditions listed, but it is the combination
of time in the tropics, malabsorption and the resulting deficits, plus the acute
inflammatory epithelial infiltrate that makes tropical sprue the diagnosis here
Without a history of working in the tropics, coeliac would be the next logical choice
here
Giardiasis is associated with symptoms of irritable bowel with increased foul smelling
bowel gas and abdominal bloating
Whipple's is associated with flitting arthralgia
Crohn's would be associated with more severe symptoms of diarrhoea, and other signs
such as apthous ulceration
3213

https://mypastest.pastest.com/Secure/TestMe/Browser/429893#Top

1/2

8/22/2016

MyPastest

Next Question

Previous Question

Tag Question

Feedback

End Review

Difficulty: Average
Peer Responses

Session Progress
Responses Correct:

Responses Incorrect:

298

Responses Total:

298

Responses - % Correct:

0%

Blog (https://www.pastest.com/blog) About Pastest (https://www.pastest.com/about-us)


Contact Us (https://www.pastest.com/contact-us) Help (https://www.pastest.com/help)
Pastest 2016

https://mypastest.pastest.com/Secure/TestMe/Browser/429893#Top

2/2

8/22/2016

MyPastest

Back to Filters (/Secure/TestMe/Filter/429893/QA)

Question 75 of 298

A 29-year-old patient has a 6-month history of intermittent diarrhoea and weight loss.
What is the most appropriate first-line investigation if coeliac disease is suspected?
A

Anti-reticulin antibodies

Anti-enterocyte antibodies

Anti-gluten antibodies

Anti-TTG antibodies

Small-bowel biopsy

Explanation
Diagnosis of coeliac disease
There are potenially both false negatives and false positives in coeliac screening. Antireticulin is too non-specific, anti-enterocyte is a marker for T-cell lymphoma, anti-gluten lacks
specificity but has a role in IgA deficiency (seen in 1 in 700). Anti-TTG antibody is the most
sensitive test, but it is an IgA antibody so can cause false-negatives in IgA-deficient patients.
Anti-endomyseal antibodies are far more specific for coeliac than anti-TTG but are less
sensitive. Where there is IgA deficiency the IgG anti-gliadin antibody would have been the
most reliable test. Small-bowel biopsy is still the gold standard, but is not suitable as a
screening test.
3214

Next Question

Previous Question

Tag Question

Feedback

End Review

Difficulty: Easy
Peer Responses

https://mypastest.pastest.com/Secure/TestMe/Browser/429893#Top

1/2

8/22/2016

MyPastest

Session Progress
Responses Correct:

Responses Incorrect:

298

Responses Total:

298

Responses - % Correct:

0%

Blog (https://www.pastest.com/blog) About Pastest (https://www.pastest.com/about-us)


Contact Us (https://www.pastest.com/contact-us) Help (https://www.pastest.com/help)
Pastest 2016

https://mypastest.pastest.com/Secure/TestMe/Browser/429893#Top

2/2

8/22/2016

MyPastest

Back to Filters (/Secure/TestMe/Filter/429893/QA)

Question 76 of 298

In patients suspected of HIV/AIDS infection the most likely explanation of persisting watery
diarrhoea is which one of the following?
A

Anal cancer

Cryptococcus infection other than neoformans

Cryptococcus neoformans

Microsporidium

Herpes enteritis

Explanation
Microsporidia
Microsporidia should be considered as a cause of HIV associated diarrhoea where no other
causative organisms are found. In the difficult to diagnose category (i.e. where multiple
negative tests for other organisms are found), it is responsible for 15-34% of cases.
AIDS and the GI Tract
Retrosternal pain/discomfort

Candidiasis
CMV
HSV
Diarrhoea/weight loss/malabsorption

Cryptosporidium
Isospora belli
Microsporidia
MV/HSV
Mycobacteria
Enteric bacteria (salmonella)
https://mypastest.pastest.com/Secure/TestMe/Browser/429893#Top

1/3

8/22/2016

MyPastest

Neoplasia
Hepatitis/cholestasis/mycobacteria

Cryptosporidium
CMV
Cryptococcus
Drugs
Neoplasia/miscellaneous

Kaposi's sarcoma
Lymphoma
Hairy leucoplakia
Anal warts
Squamous oral/anal carcinoma
3215

Next Question

Previous Question

Tag Question

Feedback

End Review

Difficulty: Difficult
Peer Responses

Session Progress
Responses Correct:

Responses Incorrect:

298

Responses Total:

298

Responses - % Correct:

0%

Blog (https://www.pastest.com/blog) About Pastest (https://www.pastest.com/about-us)


Contact Us (https://www.pastest.com/contact-us) Help (https://www.pastest.com/help)
https://mypastest.pastest.com/Secure/TestMe/Browser/429893#Top

2/3

8/22/2016

MyPastest

Pastest 2016

https://mypastest.pastest.com/Secure/TestMe/Browser/429893#Top

3/3

8/22/2016

MyPastest

Back to Filters (/Secure/TestMe/Filter/429893/QA)

Question 77 of 298

A patient presents with inflammatory bowel disease.


Which of the following conditions would treatment with 5-aminosalicylic acid (5-ASA) be
most appropriate for?
A

Acute therapy for ulcerative colitis

Maintenance therapy for ulcerative colitis

Acute therapy for Crohns disease

Maintenance therapy for Crohns disease

Prevention of colon cancer

Explanation
Therapy with 5-aminosalicylic acid
The greatest benefit with 5-aminosalicylic acid (5-ASA) is seen with maintenance in ulcerative
colitis (70% response compared with 30% for placebo). It is less valuable for active disease
(60% response in ulcerative colitis, 45% response in Crohns disease) and for maintenance in
Crohns disease (40% response rate).
3216

Next Question

Previous Question

Tag Question

Feedback

End Review

Difficulty: Average
Peer Responses

https://mypastest.pastest.com/Secure/TestMe/Browser/429893#Top

1/2

8/22/2016

MyPastest

Session Progress
Responses Correct:

Responses Incorrect:

298

Responses Total:

298

Responses - % Correct:

0%

Blog (https://www.pastest.com/blog) About Pastest (https://www.pastest.com/about-us)


Contact Us (https://www.pastest.com/contact-us) Help (https://www.pastest.com/help)
Pastest 2016

https://mypastest.pastest.com/Secure/TestMe/Browser/429893#Top

2/2

8/22/2016

MyPastest

Back to Filters (/Secure/TestMe/Filter/429893/QA)

Question 78 of 298

A cancer patient has been referred for a nutritional assessment.


What is the most reliable measurement?
A

Body mass index

Body weight

Clinical observation

Serum albumin

Triceps skin-fold thickness

Explanation
Nutritional status
Subjective global assessment (SGA) is the most useful of a series of less than ideal methods.
The serum albumin is a negative acute phase-protein; the triceps skin fold measures fat stores
and is very observer-dependent. Weight and body mass index (BMI) are more useful as
indicators of change rather than as absolute values.
3217

Next Question

Previous Question

Tag Question

Feedback

End Review

Difficulty: Difficult
Peer Responses

https://mypastest.pastest.com/Secure/TestMe/Browser/429893#Top

1/2

8/22/2016

MyPastest

Session Progress
Responses Correct:

Responses Incorrect:

298

Responses Total:

298

Responses - % Correct:

0%

Blog (https://www.pastest.com/blog) About Pastest (https://www.pastest.com/about-us)


Contact Us (https://www.pastest.com/contact-us) Help (https://www.pastest.com/help)
Pastest 2016

https://mypastest.pastest.com/Secure/TestMe/Browser/429893#Top

2/2

8/22/2016

MyPastest

Back to Filters (/Secure/TestMe/Filter/429893/QA)

Question 79 of 298

A slim 39-year-old deputy head teacher has a 3-month history of dyspepsia, particularly at
night, that is relieved by milk. He had a trial of proton-pump inhibitor 1 month ago and his
symptoms have returned. He has not lost any weight. On examination there is some mild
epigastric tenderness.
The most useful next management step would be?
A

Heliobacter pylori antibodies and eradication therapy if positive

Course of H2 antagonists/proton-pump inhibitors

Upper gastrointestinal endoscopy

13

H.pylori eradication therapy

C urea breath test and eradicate if positive

Explanation
Helicobacter testing
The risk of upper gastrointestinal cancer in this patient is low. If he is Helicobacter pyloripositive, it would be reasonable to try eradication therapy and only investigate further if his
symptoms do not improve or are recurrent. H. pylori antibodies are cheap but non-specific.
Breath testing is the most specific and sensitive non-invasive way of establishing the
presence of current H. pylori infection and costs about the same as serology testing.
3218

Next Question

Previous Question

Tag Question

Feedback

End Review

Difficulty: Average
Peer Responses

https://mypastest.pastest.com/Secure/TestMe/Browser/429893#Top

1/2

8/22/2016

MyPastest

Session Progress
Responses Correct:

Responses Incorrect:

298

Responses Total:

298

Responses - % Correct:

0%

Blog (https://www.pastest.com/blog) About Pastest (https://www.pastest.com/about-us)


Contact Us (https://www.pastest.com/contact-us) Help (https://www.pastest.com/help)
Pastest 2016

https://mypastest.pastest.com/Secure/TestMe/Browser/429893#Top

2/2

8/22/2016

MyPastest

Back to Filters (/Secure/TestMe/Filter/429893/QA)

Question 80 of 298

A 79-year-old woman is admitted with a 24-hour history of abdominal pain, vomiting and
diarrhoea. Her past medical history includes a myocardial infarction 10 years ago and an
irregular heart beat noted by her GP some 5 years earlier for which he commenced her on
Clopidogrel. Her bowel habit was usually regular. On examination, her temperature is 37.5 C,
blood pressure 120/80 mmHg, pulse 120 bpm and irregularly irregular. Abdominal
examination reveals a generally tender abdomen with more marked pain in the left iliac fossa
and suprapubic area. Bowel sounds are sparse but of normal pitch. Rectal examination is
normal. Investigations show: haemoglobin 13.7 g/dl, sodium 139 mmol/l, potassium 5.1 mmol/l,
urea 8.2 mmol/l, white cell count 15 109/l, platelets 452 109/l. A plain abdominal X-ray is
unremarkable.
What is the most likely diagnosis?
A

Inferior myocardial infarction

Sigmoid volvulus

Diverticulitis

Inferior mesenteric artery occlusion

Ulcerative proctitis

Explanation
Inferior mesenteric artery occlusion
The presence of atrial fibrillation and lack of any pre-existing bowel symptoms would suggest
an acute mesenteric vascular occlusion as the cause of this patients illness rather than
diverticulitis, which would be the next most likely diagnosis.
3219

Next Question

Previous Question

Tag Question

https://mypastest.pastest.com/Secure/TestMe/Browser/429893#Top

Feedback

End Review

1/2

8/22/2016

MyPastest

Difficulty: Average
Peer Responses

Session Progress
Responses Correct:

Responses Incorrect:

298

Responses Total:

298

Responses - % Correct:

0%

Blog (https://www.pastest.com/blog) About Pastest (https://www.pastest.com/about-us)


Contact Us (https://www.pastest.com/contact-us) Help (https://www.pastest.com/help)
Pastest 2016

https://mypastest.pastest.com/Secure/TestMe/Browser/429893#Top

2/2

8/22/2016

MyPastest

Back to Filters (/Secure/TestMe/Filter/429893/QA)

Question 81 of 298

A 56-year-old man presents with abdominal pain, diarrhoea and heartburn. He has lost about
2 kg in weight in the last 3 months. Clinical examination does not reveal anything further.
Investigations show: haemoglobin 12.3 g/dl, white cell count 8.5 109/l, platelets 198 109/l,
mean corpuscular volume (MCV) 102 fl. Upper gastrointestinal endoscopy reveals multiple
gastroduodenal ulcers.
What is the most likely diagnosis?
A

Crohns disease

Chronic alcohol abuse

Gastric adenocarcinoma

Helicobacter pylori-associated peptic ulceration

ZollingerEllison syndrome

Explanation
ZollingerEllison syndrome
Gastrinomas are malignant tumours, usually sited in the pancreas. They secrete gastrin, which
causes hyperchlorhydria and ulceration, as well as diarrhoea. The gastric acid increases
breakdown of vitamin B12, resulting in a B12 deficiency in severe cases, hence the elevated
mean corpuscular volume (MCV).
Crohns disease very rarely presents with B12 deficiency (iron deficiency is more common).
Alcohol abuse is usually associated with a lower platelet count than this and he has no
stigmata of chronic disease.
3220

Next Question

Previous Question

Tag Question

Feedback

End Review

Difficulty: Easy
https://mypastest.pastest.com/Secure/TestMe/Browser/429893#Top

1/2

8/22/2016

MyPastest

Peer Responses

Session Progress
Responses Correct:

Responses Incorrect:

298

Responses Total:

298

Responses - % Correct:

0%

Blog (https://www.pastest.com/blog) About Pastest (https://www.pastest.com/about-us)


Contact Us (https://www.pastest.com/contact-us) Help (https://www.pastest.com/help)
Pastest 2016

https://mypastest.pastest.com/Secure/TestMe/Browser/429893#Top

2/2

8/22/2016

MyPastest

Back to Filters (/Secure/TestMe/Filter/429893/QA)

Question 82 of 298

A 33-year-old nursery nurse is admitted with abdominal pain, diarrhoea and severe
malnutrition and her BMI is 14 kg/m2. Investigatins show: haemoglobin 10 g/dl, whte cell count
12.5 109/l, platelets 675 109/l, albumin 30 g/l, calcium 1.9 mmol/l, phosphate 0.2 mmol/l,
sodium 130 mmol/l, potassium 2.9 mmol/l. Because it is the weekend she given an 'off the
shelf' standard bag of total parenteral nutrition (TPN), giving her 2200 calories and 9 g of
nitrogen. On Monday she develops severe congestive cardiac failure.
This might have been prevented by pre-treating her before the start of TPN with:
A

Intravenous potassium

Intravenous sodium

Intravenous calcium

Intravenous phosphate

Intravenous vitamins

Explanation
Refeeding syndrome
This is the refeeding syndrome, caused by phosphate deficiency. Refeeding with high
concentrations of glucose without restoring phosphate levels results in further falls in plasma
concentrations of phosphate, driven into cells by insulin. This problem is exacerbated by any
other cation deficiency and requires cautious replacement because the refeeding syndrome
has a significant mortality rate.
3221

Next Question

Previous Question

Tag Question

Feedback

End Review

Difficulty: Easy

https://mypastest.pastest.com/Secure/TestMe/Browser/429893#Top

1/2

8/22/2016

MyPastest

Peer Responses

Session Progress
Responses Correct:

Responses Incorrect:

298

Responses Total:

298

Responses - % Correct:

0%

Blog (https://www.pastest.com/blog) About Pastest (https://www.pastest.com/about-us)


Contact Us (https://www.pastest.com/contact-us) Help (https://www.pastest.com/help)
Pastest 2016

https://mypastest.pastest.com/Secure/TestMe/Browser/429893#Top

2/2

8/22/2016

MyPastest

Back to Filters (/Secure/TestMe/Filter/429893/QA)

Question 83 of 298

A 48-year-old man with a 2-year history of ulcerative colitis has been receiving parenteral
nutrition for 4 months. He develops a dermatitis and has noticed some loss of hair. Serum
biochemistry shows a marginally raised glucose concentration and a reduction in alkaline
phosphatase activity.
Which of the following is the most likely explanation?
A

Chromium deficiency

Copper deficiency

Magnesium deficiency

Selenium deficiency

Zinc deficiency

Explanation
Mineral deficiencies
Dermatitis and alopecia are typical of zinc deficiency and patients with inflammatory
bowel disease can lose considerable quantities of zinc from the gut.
Selenium deficiency causes a cardiomyopathy, seen as Keshan disease in areas where
soil has a low selenium content.
Hypomagnesaemia causes hypocalcaemia and neuromuscular excitability.
Copper deficiency in adults is very rare features include cardiac dysrhythmias and
altered lipoprotein metabolism.
Chromium deficiency is even rarer, the most consistent feature being glucose
intolerance.
3231

Next Question

https://mypastest.pastest.com/Secure/TestMe/Browser/429893#Top

1/2

8/22/2016

MyPastest

Previous Question

Tag Question

Feedback

End Review

Difficulty: Average
Peer Responses

Session Progress
Responses Correct:

Responses Incorrect:

298

Responses Total:

298

Responses - % Correct:

0%

Blog (https://www.pastest.com/blog) About Pastest (https://www.pastest.com/about-us)


Contact Us (https://www.pastest.com/contact-us) Help (https://www.pastest.com/help)
Pastest 2016

https://mypastest.pastest.com/Secure/TestMe/Browser/429893#Top

2/2

8/22/2016

MyPastest

Back to Filters (/Secure/TestMe/Filter/429893/QA)

Question 84 of 298

You are carrying out a colonoscopy on a 42-year-old man with a family history of colon
cancer. You find a number of polyps, which are biopsied.
What features of colonic adenomas are most associated with an increased risk of malignant
change?
A

Size less than 1 cm

Sessile or flat polyps

Tubular architecture

Single polyp only found

Pedunculated polyp

Explanation
Polyps and colorectal cancer
Colorectal cancer is the second commonest cause of cancer death in the UK, with an all-age
prevalence of 53.5/100,000 for men and 36.7/100,000 for women. The average age at
diagnosis is 6065 years.
Polyp characteristics: polyps greater than 1.5 cm, which are sessile or flat, are associated with
a higher risk of malignant change. Histology demonstrating severe dysplasia, predominantly
villous architecture or squamous metaplasia, is also associated with a higher risk of malignant
change.
Family history is an extremely important risk factor for colon cancer. Familial adenomatous
polyposis (FAP) is the best-recognised syndrome predisposing to colorectal cancer, but in
practice it is only related to around 1% of all colorectal cancers. Hereditary non-polyposis
cancer (HNPCC) arises from germline mutations in any one of five DNA mismatch repair
genes.
3723

Next Question

https://mypastest.pastest.com/Secure/TestMe/Browser/429893#Top

1/2

8/22/2016

MyPastest

Previous Question

Tag Question

Feedback

End Review

Difficulty: Average
Peer Responses

Session Progress
Responses Correct:

Responses Incorrect:

298

Responses Total:

298

Responses - % Correct:

0%

Blog (https://www.pastest.com/blog) About Pastest (https://www.pastest.com/about-us)


Contact Us (https://www.pastest.com/contact-us) Help (https://www.pastest.com/help)
Pastest 2016

https://mypastest.pastest.com/Secure/TestMe/Browser/429893#Top

2/2

8/22/2016

MyPastest

Back to Filters (/Secure/TestMe/Filter/429893/QA)

Question 85 of 298

You are asked to review a 53-year-old diabetic man who has diarrhoea. He is currently taking
metformin at total daily dose of 2 g/day. The diarrhoea appears to worsen with increased
metformin dose.
What is the most likely cause of his diarrhoea?
A

Autonomic neuropathy related to diabetes

Osmotic diarrhoea related to diabetic foods

Secretory diarrhoea

Bile acid malabsorption

Steatorrhoea

Explanation
Diarrhoea in diabetics
Biguanides are one cause of bile acid malabsorption. Colchicine, used for treating gout in
patients where non-steroidal anti-inflammatory agents (NSAIDs) are contraindicated, can also
cause bile acid malabsorption. Ileal resection or terminal ileal disease (eg in Crohns disease)
also leads to bile acid malabsorption. Bile salts cause diarrhoea by reducing the absorption of
water and electrolytes in the colon. In higher concentrations they lead to secretion and can
stimulate colonic motility. Treatment in this case would be substitution of another drug for
managing the diabetes; in other cases, cholestyramine is useful for binding and inactivation of
bile acids.
Osmotic diarrhoea occurs in patients with diabetes who ingest too much sorbitol (a common
substitute for glucose in so-called diabetic foods. Secretory diarrhoea commonly occurs in
response to endotoxin-producing bacteria, (eg cholera or Escherichia coli).
3724

Next Question

Previous Question

Tag Question

https://mypastest.pastest.com/Secure/TestMe/Browser/429893#Top

Feedback

End Review
1/2

8/22/2016

MyPastest

Difficulty: Difficult
Peer Responses

Session Progress
Responses Correct:

Responses Incorrect:

298

Responses Total:

298

Responses - % Correct:

0%

Blog (https://www.pastest.com/blog) About Pastest (https://www.pastest.com/about-us)


Contact Us (https://www.pastest.com/contact-us) Help (https://www.pastest.com/help)
Pastest 2016

https://mypastest.pastest.com/Secure/TestMe/Browser/429893#Top

2/2

8/22/2016

MyPastest

Back to Filters (/Secure/TestMe/Filter/429893/QA)

Question 86 of 298

A 62-year-old woman is sent to the Gastroenterology Clinic for review. Her medical history of
note includes ulcerative colitis and prolonged use of oral ketoconazole for recurrent nail-bed
infections. On examination in the clinic she is jaundiced and has two-finger-breadth
hepatomegaly. Biochemistry results show: bilirubin 260 mol/l (normal range 122 mol/),
alanine aminotransferase (ALT) of 410 U/l (535 U/l) and a non-specific increase in globulin
levels. You note that her LFTs were normal 1 year earlier. Autoantibody screen reveals antiLKM2 antibodies.
What diagnosis fits best with this clinical picture?
A

Drug-induced hepatitis

Autoimmune hepatitis

Primary biliary cirrhosis

Primary sclerosing cholangitis

Gallstones

Explanation
Drug-induced hepatitis
Drugs that can result in this clinical picture include:
Methyldopa
Isoniazid
Ketoconazole (an antifungal)
Nitrofurantoin
Drug-induced chronic hepatitis is commoner in women and presents with jaundice and
hepatomegaly, with deranged transaminases and elevated bilirubin. It is associated with antiLKM2 autoantibodies. While autoimmune hepatitis may also be associated with anti-LKM
positivity, the short history and ketoconazole exposure make drug-induced hepatitis more
likely here. Liver function can improve after drug withdrawal but relapses are possible.

https://mypastest.pastest.com/Secure/TestMe/Browser/429893#Top

1/2

8/22/2016

MyPastest

Other causes of chronic hepatitis include:


Hepatitis B
Hepatitis C
Autoimmune hepatitis
Alpha1-antitrypsin deficiency
Wilsons disease
Inflammatory bowel disease
3725

Next Question

Previous Question

Tag Question

Feedback

End Review

Difficulty: Difficult
Peer Responses

Session Progress
Responses Correct:

Responses Incorrect:

298

Responses Total:

298

Responses - % Correct:

0%

Blog (https://www.pastest.com/blog) About Pastest (https://www.pastest.com/about-us)


Contact Us (https://www.pastest.com/contact-us) Help (https://www.pastest.com/help)
Pastest 2016

https://mypastest.pastest.com/Secure/TestMe/Browser/429893#Top

2/2

8/22/2016

MyPastest

Back to Filters (/Secure/TestMe/Filter/429893/QA)

Question 87 of 298

A 42-year-old woman is referred to the Liver Clinic by her GP. She has a raised alanine
aminotransferase (ALT) at 160 U/l (normal range 535 U/l). Her past history of note includes
obesity and gestational diabetes. She had an admission with cholecystitis 2 years ago.
What diagnosis fits best with this clinical picture?
A

Autoimmune hepatitis

Primary biliary cirrhosis

Gallstones

Cirrhosis

Non-alcoholic steatohepatitis

Explanation
Non-alcoholic steatohepatitis (NASH)
Steatohepatitis occurs most commonly in patients with features of insulin resistance, such as
obesity, dyslipidaemia and type 2 diabetes mellitus. The hallmark of the condition on liver
biopsy is the association of inflammation with fatty infiltration of the liver. This can progress
to fibrotic change and eventually to cirrhosis.
Management and prognosis
Weight loss is known to improve the liver biopsy appearance, scored according to the NASH
score. Pioglitazone, which is known to reduce incidence, does have positive data on liver
biopsy appearance from a number of small pilot studies in NASH. The best treatment for
NASH however is weight loss, and prognosis depends on the ability to lose weight and thus
reduce intrahepatic fat stores, but significant numbers of patients progress eventually to
cirrhosis.
3726

Next Question

https://mypastest.pastest.com/Secure/TestMe/Browser/429893#Top

1/2

8/22/2016

MyPastest

Previous Question

Tag Question

Feedback

End Review

Difficulty: Easy
Peer Responses

Session Progress
Responses Correct:

Responses Incorrect:

298

Responses Total:

298

Responses - % Correct:

0%

Blog (https://www.pastest.com/blog) About Pastest (https://www.pastest.com/about-us)


Contact Us (https://www.pastest.com/contact-us) Help (https://www.pastest.com/help)
Pastest 2016

https://mypastest.pastest.com/Secure/TestMe/Browser/429893#Top

2/2

8/22/2016

MyPastest

Back to Filters (/Secure/TestMe/Filter/429893/QA)

Question 88 of 298

A 67-year-old man with a history of atrial fibrillation and cardiovascular disease is brought in
by his relatives with acute abdominal pain and vomiting. On examination he is drowsy and
looks unwell. His blood pressure is 105/60 mmHg, his pulse is 110 bpm and he is in atrial
fibrillation. His abdomen is generally tender. Initial blood tests reveal an amylase of 500 U/l
(normal range 25170 U/l), neutrophilia and renal failure, with a creatinine of 350 mol/l (60
110 mol/l) and a urea of 12.5 mmol/l (2.57.5 mmol/l). Further questioning reveals that he
has complained of intermittent abdominal pain after dinner over the past few months.
What diagnosis fits best with this clinical picture?
A

Acute pancreatitis

Chronic pancreatitis

Chronic mesenteric ischaemia

Acute-on-chronic mesenteric ischaemia

Mesenteric vasculitis

Explanation
Mesenteric ischaemia
Acute mesenteric ischaemia is a cause of elevated amylase that is unrelated to pancreatitis.
The classic presentation is with acute abdominal pain and vomiting. Arterial embolus related
to atrial fibrillation is the commonest cause, with occlusion of the superior mesenteric artery.
Abdominal tenderness is general, with distension and absence of bowel sounds. The mortality
approaches 90% and is related to multiorgan failure. Survivors have a high chance of
developing short bowel syndrome, the severity of which is related to the size of bowel
resection.
There is also evidence of chronic small-bowel ischaemia in this case, indicated by his
abdominal pain after meals over the past few months. Clinical signs of bowel ischaemia might
be absent in the chronic phase due to the development of an adequate collateral circulation.
Bruits can occasionally be heard over the abdomen, but are also found in many normal
individuals.
3727

https://mypastest.pastest.com/Secure/TestMe/Browser/429893#Top

1/2

8/22/2016

MyPastest

Next Question

Previous Question

Tag Question

Feedback

End Review

Difficulty: Easy
Peer Responses

Session Progress
Responses Correct:

Responses Incorrect:

298

Responses Total:

298

Responses - % Correct:

0%

Blog (https://www.pastest.com/blog) About Pastest (https://www.pastest.com/about-us)


Contact Us (https://www.pastest.com/contact-us) Help (https://www.pastest.com/help)
Pastest 2016

https://mypastest.pastest.com/Secure/TestMe/Browser/429893#Top

2/2

8/22/2016

MyPastest

Back to Filters (/Secure/TestMe/Filter/429893/QA)

Question 89 of 298

A 42-year-old woman visits the Inflammatory Bowel Disease Clinic with her two sons, who are
4 years and 7 years old. She was diagnosed with Crohns disease during her early twenties.
Which of the following best describes a feature of the epidemiology of Crohns disease?
A

There is a proven causal link with tuberculous disease

The disease is slightly more common in males

Ashkenazi Jews have a higher risk than Sephardic Jews

20% of sufferers have one or more relatives with the disease

The coefficient of heritability of Crohns disease is low

Explanation
Epidemiology of Crohns disease
Crohns disease has a lower incidence in non-white races; people of Jewish origin are
more prone to inflammatory bowel disease than non-Jews; and Ashkenazi Jews are at
higher risk than Sephardic Jews.
Crohns disease is slightly more common in females (male to female ratio is 1:1.2) and it
tends to present at a younger age in females (mean age at presentation is 26 years in
females and 34 years in males).
About 610% of patients with inflammatory bowel disease have one or more affected
relatives.
Studies of monozygotic twins have shown that the coefficient of heritability of Crohns
disease is high.
An association with Mycobacteria remains unproved, with inconsistent isolation of
mycobacteria from sufferers. The link was first postulated because Johnes disease,
which occurs in cattle and sheep, is caused by Mycobacterium paratuberculosis and is
associated with terminal ileal inflammatory bowel disease.

https://mypastest.pastest.com/Secure/TestMe/Browser/429893#Top

1/2

8/22/2016

MyPastest

In the past, measles virus has also been suggested as a cause, although this has never
been proved either.
3728

Next Question

Previous Question

Tag Question

Feedback

End Review

Difficulty: Difficult
Peer Responses

Session Progress
Responses Correct:

Responses Incorrect:

298

Responses Total:

298

Responses - % Correct:

0%

Blog (https://www.pastest.com/blog) About Pastest (https://www.pastest.com/about-us)


Contact Us (https://www.pastest.com/contact-us) Help (https://www.pastest.com/help)
Pastest 2016

https://mypastest.pastest.com/Secure/TestMe/Browser/429893#Top

2/2

8/22/2016

MyPastest

Back to Filters (/Secure/TestMe/Filter/429893/QA)

Question 90 of 298

A 54-year-old woman with systemic sclerosis is referred to the clinic with chronic diarrhoea.
She has a previous history of chronic oesophageal reflux that has been managed with
conservative measures such as raising the head of the bed. Based on the most likely cause of
this diarrhoea, what would be the best initial treatment option?
A

Metronidazole therapy

Cholestyramine therapy

Codeine phosphate therapy

Neomycin therapy

Imodium therapy

Explanation
Diarrhoea in systemic sclerosis
Patients with systemic sclerosis have areas of stricture, dilatation and diverticulum formation
within the small bowel. These changes, coupled with slow motility, means that they are open
to problems with bacterial overgrowth. The organisms that are usually responsible include
Escherichia coli and Bacteroides spp., which are capable of unconjugating and hydrolysing
bile salts. They are also capable of metabolising vitamin B12 and interfering with intrinsicfactor binding, which can result in vitamin B12 deficiency (although this is rarely severe
enough to result in neurological deficit). Bacterial overgrowth is confirmed by the hydrogen
breath test.
In patients with conditions like systemic sclerosis, rotating antibiotics (eg metronidazole and
ciprofloxacin) might be necessary to prevent the reoccurrence of symptoms.
3729

Next Question

Previous Question

Tag Question

https://mypastest.pastest.com/Secure/TestMe/Browser/429893#Top

Feedback

End Review

1/2

8/22/2016

MyPastest

Difficulty: Difficult
Peer Responses

Session Progress
Responses Correct:

Responses Incorrect:

298

Responses Total:

298

Responses - % Correct:

0%

Blog (https://www.pastest.com/blog) About Pastest (https://www.pastest.com/about-us)


Contact Us (https://www.pastest.com/contact-us) Help (https://www.pastest.com/help)
Pastest 2016

https://mypastest.pastest.com/Secure/TestMe/Browser/429893#Top

2/2

8/22/2016

MyPastest

Back to Filters (/Secure/TestMe/Filter/429893/QA)

Question 91 of 298

A 45-year-old man presents for review at the Gastroenterology Clinic. His past history of note
includes hypertension, which is managed with amlodipine. He has mild ankle oedema and the
amlodipine is thought to be the cause. Over the past few months he has suffered intermittent
epigastric pain. Upper gastrointestinal endoscopy reveals enlarged gastric folds in the body
and fundus of his stomach. Biopsy reveals hyperplasia of the gastric pits, gland atrophy and
an increase in mucosal thickness. A CLO (Campylobacter-like organism) test is negative. The
full blood count is normal and an autoantibody screen is negative; serum albumin is reduced
at 28 g/l (normal range 3749 g/l).
What diagnosis fits best with this clinical picture?
A

Menetriers disease

Helicobacter pylori infection

Autoimmune gastritis

Bacterial overgrowth syndrome

Duodenal reflux

Explanation
Menetriers disease
Menetriers disease is a rare condition associated with giant gastric folds, predominantly in
the fundus and body of the stomach. Histologically there is hyperplasia of the gastric pits,
gland atrophy and an increase in overall mucosal thickness. Hypochlorhydria is usually
present.
Clinical features
Patients often complain of epigastric pain, and protein loss from the gastric mucosa can
result in mild hypoalbuminaemia. The treatment and time-course of the disease is unclear:
some patients improve spontaneously, whereas in others this can be a premalignant state.
Antisecretory drugs such as proton-pump inhibitors can be tried for symptom relief.
The Campylobacter-like organism (CLO) test is used to detect Helicobacter pylori, which
would be a reasonable differential diagnosis with this clinical picture.
3730

https://mypastest.pastest.com/Secure/TestMe/Browser/429893#Top

1/2

8/22/2016

MyPastest

Next Question

Previous Question

Tag Question

Feedback

End Review

Difficulty: Average
Peer Responses

Session Progress
Responses Correct:

Responses Incorrect:

298

Responses Total:

298

Responses - % Correct:

0%

Blog (https://www.pastest.com/blog) About Pastest (https://www.pastest.com/about-us)


Contact Us (https://www.pastest.com/contact-us) Help (https://www.pastest.com/help)
Pastest 2016

https://mypastest.pastest.com/Secure/TestMe/Browser/429893#Top

2/2

8/22/2016

MyPastest

Back to Filters (/Secure/TestMe/Filter/429893/QA)

Question 92 of 298

You are asked to see a 26-year-old nurse who sustained a needlestick injury 8 months ago.
She did not present immediately to Occupational Health, and only attended her GP when she
began to feel tired and lethargic. She has a raised alanine aminotransferase (ALT), anti-HBs
antibodies and anti-HCV antibodies. Low levels of HCV RNA are detected. Liver biopsy
reveals early inflammatory change.
What diagnosis fits best with this clinical picture?
A

Resolving hepatitis C infection

Chronic hepatitis B infection

Chronic hepatitis C infection

Autoimmune hepatitis

Functional symptoms

Explanation
Hepatitis C virus (HCV) infection
This nurse has anti-HBs antibodies, probably as a result of hepatitis B vaccination. Hepatitis C
virus (HCV) RNA has been detected, however, indicating active hepatitis C infection.
Histological changes seen on liver biopsy associated with hepatitis C can vary from minimal
infiltration to severe fibrotic change and cirrhosis; lymphoid follicles are often present in the
portal tracts and fatty change is frequently seen. A falling level of HCV RNA coupled with
high titre anti-HCV antibodies would be most consistent with a diagnosis of resolving
hepatitis C. We are not told that hepatitis C RNA levels are falling here, therefore the
diagnosis is one of chronic hepatitis C infection.
Management of chronic HCV infection
Medical treatment is aimed at stopping the progression of inflammation and eventual fibrosis,
and hopefully preventing the development of hepatocellular carcinoma. Current therapy
involves a combination of interferon alpha-2b and ribavirin, for 612 months. HCV genotype 1
shows the poorest response to treatment (28% sustained response at 12 months), compared
to a 64% response rate with genotypes 2 or 3.
3731

https://mypastest.pastest.com/Secure/TestMe/Browser/429893#Top

1/2

8/22/2016

MyPastest

Next Question

Previous Question

Tag Question

Feedback

End Review

Difficulty: Average
Peer Responses

Session Progress
Responses Correct:

Responses Incorrect:

298

Responses Total:

298

Responses - % Correct:

0%

Blog (https://www.pastest.com/blog) About Pastest (https://www.pastest.com/about-us)


Contact Us (https://www.pastest.com/contact-us) Help (https://www.pastest.com/help)
Pastest 2016

https://mypastest.pastest.com/Secure/TestMe/Browser/429893#Top

2/2

8/22/2016

MyPastest

Back to Filters (/Secure/TestMe/Filter/429893/QA)

Question 93 of 298

A 65-year-old publican with a history of cirrhosis is admitted in a drowsy unkempt state from
home. On examination he appears confused.
In terms of the severity of his liver disease, which of the following additional clinical features
would score 3 points in the ChildPugh classification?
A

Prothrombin time 4 seconds above the normal range

Encephalopathy grade 3

Albumin 29 g/l (normal range 3749 g/l)

Mild ascites

Bilirubin 48 mol/l (122 mol/l)

Explanation
ChildPugh scoring
The ChildPugh classification is a method used for scoring the severity of liver disease in
patients with cirrhosis, although it is not applicable to patients with primary biliary cirrhosis
or sclerosing cholangitis:
Three points are scored for each of: encephalopathy grade 3 or 4, moderatesevere
ascites, bilirubin > 50 mol/l (normal range 122 mol/l), albumin < 28 g/l (3749 g/l),
prothrombin time > 6 seconds above the normal range.
Two points are scored for each of: encephalopathy grade 1 or 2, mild ascites, bilirubin
3450 mol/l, albumin 2835 g/l, prothrombin time 46 seconds above the normal
range.
One point is scored for each of: no encephalopathy, absent ascites, bilirubin
< 34 mol/l, albumin > 35 g/l, prothrombin time 14 seconds above the normal range.
The amalgamated points score then enables patients to be grouped into ChildPugh grade A
(< 6 points), B (79 points) or C (> 10 points). Childs grade C classification is most highly
predictive of a poor prognosis.
3733

https://mypastest.pastest.com/Secure/TestMe/Browser/429893#Top

1/2

8/22/2016

MyPastest

3733

Next Question

Previous Question

Tag Question

Feedback

End Review

Difficulty: Average
Peer Responses

Session Progress
Responses Correct:

Responses Incorrect:

298

Responses Total:

298

Responses - % Correct:

0%

Blog (https://www.pastest.com/blog) About Pastest (https://www.pastest.com/about-us)


Contact Us (https://www.pastest.com/contact-us) Help (https://www.pastest.com/help)
Pastest 2016

https://mypastest.pastest.com/Secure/TestMe/Browser/429893#Top

2/2

8/22/2016

MyPastest

Back to Filters (/Secure/TestMe/Filter/429893/QA)

Question 94 of 298

A 24-year-old woman presents as an emergency to her GP with acute vomiting which began
about 34 hours after attending an afternoon meeting. Cream cakes were served during the
tea break.
Which of the following organisms is the most likely cause of this acute attack of vomiting?
A

Bacillus cereus

Yersinia enterocolitica

Campylobacterspp.

Salmonellaspp.

Staphylococcus aureus

Explanation
Food poisoning
Staphylococcus aureus causes up to 25% of cases of bacterial food poisoning. The
organisms multiply rapidly in foods kept at room temperature that are rich in
carbohydrates and salt (eg dairy products, cold meats or mayonnaise). They produce a
heat-stable endotoxin that causes nausea and vomiting and diarrhoea 16 hours after
the ingestion of contaminated food. Fever is uncommon and supportive treatment only
is usually required.
Bacillus cereus infection is associated with slow-cooked food and re-heated rice (a
common problem in takeaway food). The bacterium produces an emetic toxin that
leads to vomiting in 15 hours or diarrhoea after 816 hours.
Yersinia infection results in diarrhoea about 410 days after contact and presents with
bloody diarrhoea.
Campylobacter infection has an incubation period of 37 days and results in flu-like
symptoms, abdominal pain and diarrhoea.

https://mypastest.pastest.com/Secure/TestMe/Browser/429893#Top

1/2

8/22/2016

MyPastest

Salmonella infections normally lead to an acute gastroenteritis picture 848 hours after
the ingestion of infected food.
3734

Next Question

Previous Question

Tag Question

Feedback

End Review

Difficulty: Average
Peer Responses

Session Progress
Responses Correct:

Responses Incorrect:

298

Responses Total:

298

Responses - % Correct:

0%

Blog (https://www.pastest.com/blog) About Pastest (https://www.pastest.com/about-us)


Contact Us (https://www.pastest.com/contact-us) Help (https://www.pastest.com/help)
Pastest 2016

https://mypastest.pastest.com/Secure/TestMe/Browser/429893#Top

2/2

8/22/2016

MyPastest

Back to Filters (/Secure/TestMe/Filter/429893/QA)

Question 95 of 298

A 21-year-old woman presents to the Emergency Department for review, having been referred
by her GP. She is opening her bowels to bloody diarrhoea some eight times per day, including
at night. She has a resting heart rate of 88 bpm. Her abdomen is bloated but non-tender, her
albumin is 32 g/l (normal range 3749 g/l) and her haemoglobin is 10.4 g/dl (11.516.5 g/dl).
She also has a raised plasma viscosity. X-ray reveals that her transverse colon has a diameter
of 5 cm. Stool culture has proved negative and her symptoms have now been present for a
few weeks. You are considering a diagnosis of ulcerative colitis in this woman.
Which of her clinical features would fit best with severe ulcerative colitis?
A

Heart rate of 95 bpm

Albumin of 32 g/l

Haemoglobin of 10.4 g/dl

Transverse colon diameter 5 cm

Bowels open eight times per day

Explanation
Severe acute ulcerative colitis
Defining features of a severe attack, (according to NICE):
Six or more stools per day
Visible blood in stools and at least one feature of systemic upset:
Temperature above 37.8C
Pulse rate greater than 90/min
Anaemia
ESR above 30 mm/1st hour

Management
Rehydration with intravenous fluids and correction of electrolyte disturbance
https://mypastest.pastest.com/Secure/TestMe/Browser/429893#Top

1/2

8/22/2016

MyPastest

Stool samples should be taken to exclude an infective component to the diarrhoea.


Low-molecular-weight heparin is used as prophylaxis against deep venous thrombosis
(DVT).
Rectal steroid enemas can be useful in patients with ulcerative colitis, but intravenous
corticosteroid therapy is required for severe attacks.
Aminosalicylates should be started at the same time as steroids and then continued as
the steroid dose is tailed off.
Azathioprine or ciclosporin can be used as an alternative in patients with symptoms
that are resistant to corticosteroid treatment.
Close liaison with surgical colleagues is advised in cases of severe ulcerative colitis.
3735

Next Question

Previous Question

Tag Question

Feedback

End Review

Difficulty: Average
Peer Responses

Session Progress
Responses Correct:

Responses Incorrect:

298

Responses Total:

298

Responses - % Correct:

0%

Blog (https://www.pastest.com/blog) About Pastest (https://www.pastest.com/about-us)


Contact Us (https://www.pastest.com/contact-us) Help (https://www.pastest.com/help)
Pastest 2016

https://mypastest.pastest.com/Secure/TestMe/Browser/429893#Top

2/2

8/22/2016

MyPastest

Back to Filters (/Secure/TestMe/Filter/429893/QA)

Question 96 of 298

You are asked to review a 61-year-old man who has been admitted on the surgical take. He
presented with epigastric pain, nausea and vomiting, the pain soon spreading through to his
back. It was noted on admission that he is on the waiting list for cholecystectomy. His serum
amylase on admission was 1235 U/l (normal range 25170 U/l) and he is being managed as for
a diagnosis of acute pancreatitis.
Which of the following additional features on blood testing would fit best with a diagnosis of
severe pancreatitis?
A

Serum lactose dehydrogenase (LDH)of 400 U/l (10250 U/l)

Serum albumin 33 g/l (3749 g/l)

PaO2of 6.5 kPa (11.312.6 kPa)

Serum urea of 14 mmol/l (2.57.5 mmol/l)

White cell count (WCC) of 12 109/l (411 10 9/l)

Explanation
Severe pancreatitis
Prognostic scoring
Poor prognostic features in severe pancreatitis include: age > 55 years, white cell count > 15
109/l (normal range 411 10 9/l), blood glucose > 10 mmol/l (36 mmol/l), serum urea > 16
mmol/l (2.57.5 mmol/l), albumin < 32 g/l (3749 g/l), serum aminotransferases > 200 U/l (5
35 U/l), serum calcium < 2.0 mmol/l (2.22.6 mmol/l), serum lactate dehydrogenase (LDH) >
600 U/l or 350 U/l (10250 U/l) (dependent on whether you are using the Glasgow or
Ranson criteria), and PaO2 of < 8.0 kPa (11.312.6 kPa).
These features of severe pancreatitis have been developed into scoring systems such as the
Ranson and Glasgow scoring systems and these have some utility in predicting prognosis.
However, the APACHE system of prognostic scoring has been more widely adopted because
it is applicable in a wide variety of acute illnesses.
Multiorgan failure

https://mypastest.pastest.com/Secure/TestMe/Browser/429893#Top

1/2

8/22/2016

MyPastest

In a small proportion of patients, multiorgan failure develops in the first few days after
presentation reflecting the extent of pancreatic necrosis. These patients require positivepressure ventilation and often also require additional renal support. Their mortality rate is
extremely high (often > 80%).
3736

Next Question

Previous Question

Tag Question

Feedback

End Review

Difficulty: Average
Peer Responses

Session Progress
Responses Correct:

Responses Incorrect:

298

Responses Total:

298

Responses - % Correct:

0%

Blog (https://www.pastest.com/blog) About Pastest (https://www.pastest.com/about-us)


Contact Us (https://www.pastest.com/contact-us) Help (https://www.pastest.com/help)
Pastest 2016

https://mypastest.pastest.com/Secure/TestMe/Browser/429893#Top

2/2

8/22/2016

MyPastest

Back to Filters (/Secure/TestMe/Filter/429893/QA)

Question 97 of 298

A 54-year-old man with a long history of ulcerative colitis attends the Gastroenterology Clinic
for review. You note that his alanine aminotransferase (ALT) is mildly elevated, but there is a
much greater abnormality in alkaline phosphatase. He has suffered from fluctuating jaundice
and itching over the past months. You suspect that he might have primary sclerosing
cholangitis (PSC).
Which of the following best describes a feature of PSC?
A

There is an association with HLA B27

Cholangiocarcinoma occurs in up to 20% of patients

70% of cases are seen in women

Average age of onset is 60 years

25% of cases are associated with inflammatory bowel disease

Explanation
Primary sclerosing cholangitis Epidemiology Primary sclerosing cholangitis is a chronic liver condition that is characterised by fibrosis and
inflammatory destruction of intra- and extrahepatic bile ducts. Some 75% of cases are
associated with inflammatory bowel disease, although primary sclerosing cholangitis can predate the onset of the bowel symptoms. There is a link to the HLA A1-B8-DR3 haplotype. Men
make up 70% of patients, with 40 years being the average age at onset.
Clinical features
Symptoms include fluctuating jaundice, pruritus and cholangitis.
Diagnosis is by:
endoscopic retrograde cholangiopancreatography (ERCP) or magnetic resonance
cholangiopancreatography (MRCP) reveal both intra- and extrahepatic duct strictures.

https://mypastest.pastest.com/Secure/TestMe/Browser/429893#Top

1/2

8/22/2016

MyPastest

liver biopsy reveals inflammation of intrahepatic biliary radicals with significant scar
tissue, which has been described as having an onion skin appearance.
About 20% of patients eventually progress to develop cholangiocarcinoma.
3737

Next Question

Previous Question

Tag Question

Feedback

End Review

Difficulty: Average
Peer Responses

Session Progress
Responses Correct:

Responses Incorrect:

298

Responses Total:

298

Responses - % Correct:

0%

Blog (https://www.pastest.com/blog) About Pastest (https://www.pastest.com/about-us)


Contact Us (https://www.pastest.com/contact-us) Help (https://www.pastest.com/help)
Pastest 2016

https://mypastest.pastest.com/Secure/TestMe/Browser/429893#Top

2/2

8/22/2016

MyPastest

Back to Filters (/Secure/TestMe/Filter/429893/QA)

Question 98 of 298

You are reviewing a 42-year-old man who has been diagnosed with hepatitis C virus infection.
Liver biopsy shows significant chronic inflammation with fibrotic change. You are considering
him for interferon and ribavirin therapy, and perform a baseline full blood count.
Which of the haematological side-effects of ribavirin would be most likely to occur if he was
on this treatment?
A

Microcytic anaemia

Thrombocythaemia

Neutrophilia

Thrombocytosis

Haemolytic anaemia

Explanation
Ribavirin
Ribavirin inhibits the activity of a wide range of DNA and RNA viruses. It is currently licensed
in inhaled form for the treatment of severe bronchiolitis caused by respiratory syncytial virus
(RSV), and in oral form in combination with interferon alpha for the treatment of hepatitis C
infection.
Specific cautions and contraindications include:
Pregnancy avoid treatment during pregnancy and for 6 months after delivery, and
counsel males to use barrier contraception)
Cardiac disease
Haemoglobinopathies
Severe liver dysfunction
Other cautions autoimmune disease, previous psychiatric history

https://mypastest.pastest.com/Secure/TestMe/Browser/429893#Top

1/2

8/22/2016

MyPastest

Extensive monitoring, with renal function and liver function testing and full blood count is
recommended at weeks 2 and 4 of treatment. Treatment might need to be continued for 612
months.
Side-effects include haemolytic anaemia, nausea, vomiting, dry mouth and stomatitis.
3738

Next Question

Previous Question

Tag Question

Feedback

End Review

Difficulty: Average
Peer Responses

Session Progress
Responses Correct:

Responses Incorrect:

298

Responses Total:

298

Responses - % Correct:

0%

Blog (https://www.pastest.com/blog) About Pastest (https://www.pastest.com/about-us)


Contact Us (https://www.pastest.com/contact-us) Help (https://www.pastest.com/help)
Pastest 2016

https://mypastest.pastest.com/Secure/TestMe/Browser/429893#Top

2/2

8/22/2016

MyPastest

Back to Filters (/Secure/TestMe/Filter/429893/QA)

Question 99 of 298

A 35-year-old patient with epilepsy who is currently managed with phenytoin therapy is
referred by his GP. He has recently been taking oral flucloxacillin as part of treatment for a leg
injury that was precipitated by a recent fit. He occasionally drinks alcohol and has been
taking paracetamol for pain relief after his leg injury.
Clinical examination reveals jaundice, but his abdomen is non-tender. Liver function testing
reveals a markedly raised bilirubin at 280 mol/l (normal range 122 mol/l), with an alkaline
phosphatase of 440 U/l (45105 U/l). The gamma-glutamyltransferase is also raised; the
alanine aminotransferase (ALT) is only just above the normal range.
What is the most likely cause of his jaundice?
A

Paracetamol

Phenytoin

Ethanol

Gallstones

Flucloxacillin

Explanation
Drug-induced jaundice
If this man had jaundice due to cholecystitis he would have had a tender abdomen, but he is
showing signs of cholestatic jaundice. Commonly used drugs that cause a cholestatic picture
include chlorpromazine, azathioprine, captopril, ciclosporin, penicillamine, erythromycin and
the combined oral contraceptive.
Paracetamol, phenytoin and ethanol all produce a predominantly hepatitic picture, with much
larger rises in transaminases, coupled with a smaller rise in alkaline phosphatase. Other
commonly used drugs that cause a hepatitic picture include rifampicin, allopurinol and
isoniazid.
A mixed cholestatic/hepatitic picture can be seen with co-amoxiclav, sulphonamides,
sulfasalazine, flucloxacillin and carbamazepine.

https://mypastest.pastest.com/Secure/TestMe/Browser/429893#Top

1/2

8/22/2016

MyPastest

All drug-induced causes of jaundice should be reported to the Committee on Safety of


Medicines (CSM), so that an accurate log of adverse events can be maintained.
3739

Next Question

Previous Question

Tag Question

Feedback

End Review

Difficulty: Average
Peer Responses

Session Progress
Responses Correct:

Responses Incorrect:

298

Responses Total:

298

Responses - % Correct:

0%

Blog (https://www.pastest.com/blog) About Pastest (https://www.pastest.com/about-us)


Contact Us (https://www.pastest.com/contact-us) Help (https://www.pastest.com/help)
Pastest 2016

https://mypastest.pastest.com/Secure/TestMe/Browser/429893#Top

2/2

8/22/2016

MyPastest

Back to Filters (/Secure/TestMe/Filter/429893/QA)

Question 100 of 298

A 48-year-old man presents to the Gastroenterology Clinic. He has been suffering from
heartburn for some time and has noticed occasional regurgitation of his morning toast.
Endoscopy reveals a gastric-looking mucosa spreading up into the oesophagus, with areas of
columnar metaplasia found on biopsy.
What diagnosis fits best with this clinical picture?
A

Achalasia

Erosive oesophagitis

Gastritis

Hiatus hernia

Barretts oesophagus

Explanation
Barretts oesophagus
Areas of columnar metaplasia within the oesophagus are pathognomonic of this condition,
and this is thought to occur secondary to chronic gastro-oesophageal reflux. The male to
female ratio for Barretts is 4:1, with a mean age at onset of 40 years. The prevalence of
Barretts oesophagus is reported as about 10% in patients undergoing endoscopy.
Clinical features
The condition can be asymptomatic, but patients can present with heartburn or chest pain.
Physical findings range from a normal examination to epigastric tenderness. Therapy with
high-dose proton-pump inhibitors is the mainstay of drug therapy.
Screening is recommended as progression to oesophageal adenocarcinoma occurs in some
patients; screening intervals are set according to local protocol. The risk of adenocarcinoma is
3050 times the risk in patients with Barretts oesophagus versus thos without, which
corresponds to around 500 cases per 100,000 patients with Barretts oesophagus per year.
3740

Next Question

https://mypastest.pastest.com/Secure/TestMe/Browser/429893#Top

1/2

8/22/2016

MyPastest

Previous Question

Tag Question

Feedback

End Review

Difficulty: Easy
Peer Responses

Session Progress
Responses Correct:

Responses Incorrect:

298

Responses Total:

298

Responses - % Correct:

0%

Blog (https://www.pastest.com/blog) About Pastest (https://www.pastest.com/about-us)


Contact Us (https://www.pastest.com/contact-us) Help (https://www.pastest.com/help)
Pastest 2016

https://mypastest.pastest.com/Secure/TestMe/Browser/429893#Top

2/2

8/22/2016

MyPastest

Back to Filters (/Secure/TestMe/Filter/429893/QA)

Question 101 of 298

A 67-year-old man presented to the Emergency Department with epigastric and left upper
quadrant pain for the third time in a year. He has suffered from diarrhoea for around
18 months, and claims that his weight has decreased by 12.7 kg (2 stones). The ambulance
team who visited his accommodation noticed empty whisky bottles by the rubbish bin. The
amylase is within the normal range.
What diagnosis fits best with this clinical picture?
A

Cirrhosis

Acute pancreatitis

Chronic pancreatitis

Peptic ulcer disease

Coeliac disease

Explanation
Chronic pancreatitis
Clinical features
The causes of chronic pancreatitis include chronic alcoholism, duct obstruction, malnutrition,
hyperparathyroidism and cystic fibrosis. The male to female ratio is 5:1. Symptoms include:
Left upper quadrant and epigastric pain
Significant weight loss
Bulky, foul-smelling, fatty stools
Epigastric mass (in 10% of patients)
Jaundice (in 510%)
Investigations
Serum amylase and lipase levels can be elevated, but the amylase can also be normal.
Abnormal glucose tolerance or diabetes can also occur. A 72-hour collection of faeces for
faecal fat estimation is useful in the evaluation of malabsorption and steatorrhoea. Imaging
https://mypastest.pastest.com/Secure/TestMe/Browser/429893#Top

1/2

8/22/2016

MyPastest

studies include plain radiography (may reveal calcification), ultrasound, computed


tomography (CT) scan or endoscopic retrograde cholangiopancreatography (ERCP).
Management
Important in management is the avoidance of alcohol and switching to smaller, more
frequent, low-fat meals. Pancreatic enzyme supplements are useful for malabsorption, and
octreotide might be useful in cases of resistant pancreatic pain. The long-term prognosis is
poor, with 50% of patients expected to die with 10 years due to further pancreatitis or
malignancy.
3741

Next Question

Previous Question

Tag Question

Feedback

End Review

Difficulty: Easy
Peer Responses

Session Progress
Responses Correct:

Responses Incorrect:

298

Responses Total:

298

Responses - % Correct:

0%

Blog (https://www.pastest.com/blog) About Pastest (https://www.pastest.com/about-us)


Contact Us (https://www.pastest.com/contact-us) Help (https://www.pastest.com/help)
Pastest 2016

https://mypastest.pastest.com/Secure/TestMe/Browser/429893#Top

2/2

8/22/2016

MyPastest

Back to Filters (/Secure/TestMe/Filter/429893/QA)

Question 102 of 298

A 28-year-old woman of African ethnic origin is referred to the Gastroenterology Clinic with
symptoms of abdominal tenderness, bloating and intermittent diarrhoea. She eats a European
style diet. Physical examination and sigmoidoscopy are normal.
What is the most appropriate initial management step?
A

Reassurance

Counselling for irritable bowel syndrome

Trial of dairy-free diet

Upper gastrointestinal endoscopy and duodenal biopsy

Faecal fat collection

Explanation
Lactose intolerance
Up to 85% of patients of Far-Eastern origin and over 60% of patients of African origin can
show some degree of lactose intolerance. This woman presents with symptoms of irritable
bowel syndrome, but the symptoms can be exacerbated by lactose intolerance. Lactose that
is not metabolised adequately is metabolised by gut bacteria, which leads to the formation of
gas (causing bloating) and organic acids that can precipitate diarrhoea. Confirmation of
lactose intolerance is by the lactose breath hydrogen test, or patients can be tried on a dairyfree diet. In female patients it is important to stress that they must look for an alternative
form of dietary calcium.
3742

Next Question

Previous Question

Tag Question

Feedback

End Review

Difficulty: Average

https://mypastest.pastest.com/Secure/TestMe/Browser/429893#Top

1/2

8/22/2016

MyPastest

Peer Responses

Session Progress
Responses Correct:

Responses Incorrect:

298

Responses Total:

298

Responses - % Correct:

0%

Blog (https://www.pastest.com/blog) About Pastest (https://www.pastest.com/about-us)


Contact Us (https://www.pastest.com/contact-us) Help (https://www.pastest.com/help)
Pastest 2016

https://mypastest.pastest.com/Secure/TestMe/Browser/429893#Top

2/2

8/22/2016

MyPastest

Back to Filters (/Secure/TestMe/Filter/429893/QA)

Question 103 of 298

A 52-year-old man attends the Gastroenterology Clinic for review. He complains that his
breasts appear to have enlarged slightly over the past few years.
What is the cause of gynaecomastia in cirrhosis?
A

Altered oestrogen metabolism

Furosemide prescription

Excess levels of corticosteroids

Excess energy intake from alcohol

Excess levels of testosterone

Explanation
Gynaecomastia in cirrhosis
The causes of gynaecomastia in cirrhosis are complex. The most likely cause is disordered
metabolism of sex steroids, which leads to excess levels of oestrogens. Often there is
associated testicular atrophy and loss of body hair. Gynaecomastia might also occur in
cirrhosis as a result of spironolactone therapy (an aldosterone antagonist). In cirrhosis related
to alcoholism patients often suffer nutritional deficiency, but although they obtain a large
intake of calories from alcohol they tend to substitute alcohol in place of food, so the overall
energy intake is often increased only marginally.
3866

Next Question

Previous Question

Tag Question

Feedback

End Review

Difficulty: Easy
Peer Responses

https://mypastest.pastest.com/Secure/TestMe/Browser/429893#Top

1/2

8/22/2016

MyPastest

Session Progress
Responses Correct:

Responses Incorrect:

298

Responses Total:

298

Responses - % Correct:

0%

Blog (https://www.pastest.com/blog) About Pastest (https://www.pastest.com/about-us)


Contact Us (https://www.pastest.com/contact-us) Help (https://www.pastest.com/help)
Pastest 2016

https://mypastest.pastest.com/Secure/TestMe/Browser/429893#Top

2/2

8/22/2016

MyPastest

Back to Filters (/Secure/TestMe/Filter/429893/QA)

Question 104 of 298

A 58-year-old man complains of tiredness, fever, weight loss, arthralgia and diarrhoea. Jejunal
biopsy reveals flattened mucosa that contains macrophages positive for periodic acidSchiff
(PAS).
What is the most likely diagnosis?
A

Coeliac disease

Tuberculosis

Tropical sprue

Parasitic infection

Whipples disease

Explanation
Whipples disease
Epidemiology an uncommon condition, usually presenting between 30 and 60 years
of age, occurs more often in men.
Symptoms and signs malabsorption, lymphadenopathy, arthritis, cardiac involvement,
ocular symptoms and neurological sequelae (including dementia, ophthalmoplegia,
myoclonus).
Characteristic histological features of the disease include flattening of villi and the
presence of macrophages positive for periodic acidSchiff (PAS) stain.
The causative organism is Tropheryma whipplei, which is identified by polymerase chain
reaction (PCR).
Standard treatment is co-trimoxazole therapy, which is given for 6 months.
3887

Next Question

https://mypastest.pastest.com/Secure/TestMe/Browser/429893#Top

1/2

8/22/2016

MyPastest

Previous Question

Tag Question

Feedback

End Review

Difficulty: Easy
Peer Responses

Session Progress
Responses Correct:

Responses Incorrect:

298

Responses Total:

298

Responses - % Correct:

0%

Blog (https://www.pastest.com/blog) About Pastest (https://www.pastest.com/about-us)


Contact Us (https://www.pastest.com/contact-us) Help (https://www.pastest.com/help)
Pastest 2016

https://mypastest.pastest.com/Secure/TestMe/Browser/429893#Top

2/2

8/22/2016

MyPastest

Back to Filters (/Secure/TestMe/Filter/429893/QA)

Question 105 of 298

A 70-year-old man is admitted with a 2-week history of pruritus, jaundice and a 2-kg weight
loss. He has not drunk any alcohol for at least 8 years. One month previously he had
completed a course of co-amoxiclav, which had been prescribed by his GP for sinusitis, and
he was also taking ibuprofen for hip osteoarthritis. Investigations reveal: albumin 38 g/l
(normal range 3749 g/l), bilirubin 200 mol/l (122 mol/l), aspartate aminotransferase
(AST) 150 U/l (535 U/l), alkaline phosphatase 200 U/l (50110 U/l). Abdominal ultrasound
reveals gallstones but no biliary duct dilatation.
What is the most likely cause of his jaundice?
A

Co-trimoxazole

Co-amoxiclav

Hepatitis B infection

Hepatitis C infection

Ibuprofen

Explanation
Co-amoxiclav
Cholestatic jaundice can develop during co-amoxiclav therapy or shortly afterwards.
Epidemiological studies put the risk of acute liver toxicity at about six times higher with coamoxiclav compared to amoxicillin therapy alone. Cholestatic jaundice occurs more
commonly in patients aged over 65 years and it is more common in men; these reactions are
rarer in children. The jaundice is usually self-limiting and is rarely fatal.
The duration of co-amoxiclav therapy should be appropriate to the indication and not exceed
14 days on the advice of the Committee on Safety of Medicines. Rarer side-effects of coamoxiclav include erythema multiforme, toxic epidermal necrolysis and exfoliative dermatitis.
3888

Next Question

https://mypastest.pastest.com/Secure/TestMe/Browser/429893#Top

1/2

8/22/2016

MyPastest

Previous Question

Tag Question

Feedback

End Review

Difficulty: Easy
Peer Responses

Session Progress
Responses Correct:

Responses Incorrect:

298

Responses Total:

298

Responses - % Correct:

0%

Blog (https://www.pastest.com/blog) About Pastest (https://www.pastest.com/about-us)


Contact Us (https://www.pastest.com/contact-us) Help (https://www.pastest.com/help)
Pastest 2016

https://mypastest.pastest.com/Secure/TestMe/Browser/429893#Top

2/2

8/22/2016

MyPastest

Back to Filters (/Secure/TestMe/Filter/429893/QA)

Question 106 of 298

A 54-year-old man attends his GP with feelings of lethargy. Routine screening reveals raised
gamma-glutamyltransferase (GGT).
Which of following statements concerning GGT is true?
A

Increased GGT is found in cases of fatty liver

Isolated elevation of GGT in a patient with prostatic carcinoma indicates the


presence of hepatic metastases

It is a better indicator of infectious hepatitis than of cholestasis

It is only present in liver

Serum activity is typically elevated in pregnancy

Explanation
Gamma-glutamyltransferase (GGT)
GGT is a microsomal enzyme that is present in many tissues, including the liver.
GGT can be induced by drugs such as phenytoin and alcohol.
Mild raises in GGT can occur with any alcohol intake, and a rise does not always indicate
liver pathology.
GGT can also be raised in association with fatty liver, a condition associated with type 2
diabetes mellitus, insulin resistance and alcohol intake. Fatty liver is associated with
chronic inflammation and fibrosis some patients progress from fatty liver to a
cirrhotic state.
Alkaline phosphatase activity is typically elevated in pregnancy.
3903

Next Question

https://mypastest.pastest.com/Secure/TestMe/Browser/429893#Top

1/2

8/22/2016

MyPastest

Previous Question

Tag Question

Feedback

End Review

Difficulty: Average
Peer Responses

Session Progress
Responses Correct:

Responses Incorrect:

298

Responses Total:

298

Responses - % Correct:

0%

Blog (https://www.pastest.com/blog) About Pastest (https://www.pastest.com/about-us)


Contact Us (https://www.pastest.com/contact-us) Help (https://www.pastest.com/help)
Pastest 2016

https://mypastest.pastest.com/Secure/TestMe/Browser/429893#Top

2/2

8/22/2016

MyPastest

Back to Filters (/Secure/TestMe/Filter/429893/QA)

Question 107 of 298

A 48-year-old man with haemochromatosis undergoes venesection.


Which of the following features would be most likely to show improvement with this
treatment?
A

Cirrhosis

Cardiomyopathy

Diabetes mellitus

Joint deformity

Testicular atrophy

Explanation
Venesection in haemochromatosis
There is good evidence that cardiac failure associated with haemochromatosis
improves in response to venesection, with improvement in symptoms and a reduced
requirement for diuretic therapy
Where testicular atrophy is established there is rarely any improvement in male sexual
function
Diabetes mellitus related to haemochromatosis does not resolve with venesection,
although requirements for insulin might be reduced
Venesection has been proved to have positive effects on life expectancy, but the risk of
hepatocellular carcinoma is not diminished by venesection if cirrhosis is already
established
Patients can be venesected up to twice weekly during the first 2 years of treatment
5207

Next Question

https://mypastest.pastest.com/Secure/TestMe/Browser/429893#Top

1/2

8/22/2016

MyPastest

Previous Question

Tag Question

Feedback

End Review

Difficulty: Average
Peer Responses

Session Progress
Responses Correct:

Responses Incorrect:

298

Responses Total:

298

Responses - % Correct:

0%

Blog (https://www.pastest.com/blog) About Pastest (https://www.pastest.com/about-us)


Contact Us (https://www.pastest.com/contact-us) Help (https://www.pastest.com/help)
Pastest 2016

https://mypastest.pastest.com/Secure/TestMe/Browser/429893#Top

2/2

8/22/2016

MyPastest

Back to Filters (/Secure/TestMe/Filter/429893/QA)

Question 108 of 298

A 28-year-old woman attends for review after a recent sigmoidoscopy for inflammatory
bowel disease. She has a diagnosis of Crohn's disease, has undergone previous surgery to the
terminal ileum and proximal colon, and despite quiescent inflammatory markers her diarrhoea
has continued. She is currently taking sulfasalazine and a tapering dose of oral steroids.
What would be the most appropriate intervention in this case?
A

Codeine phosphate

Loperamide

Cholestyramine

Increased steroid therapy

Increased sulfasalazine

Explanation
Colestyramine and bile-salt malabsorption
Diarrhoea associated with long-standing Crohns disease might be owing to bile-salt
malabsorption (in the terminal ileum) and cholestryramine would be the most appropriate
therapy in this case. Increased bile acids in the colon reduce absorption of water and
electrolytes and at higher concentrations induce colonic secretion and increased motility.
Diagnosis of bile acid malabsorption is made using the SeHCAT test, where radiolabelled bile
acid analogue is administered and the percentage retention at 7 days is calculated (less than
19% retention is abnormal). The best results are seen with cholestyramine when retention is
less than 5%. Other antidiarrhoeal agents such as loperamide also have a place in treating
chronic diarrhoea but should not be used in active colitis owing to the risk of toxic
megacolon. Other causes of diarrhoea to be considered include bacterial overgrowth, shortbowel syndrome and lactase deficiency.
5208

Next Question

Previous Question

Tag Question

https://mypastest.pastest.com/Secure/TestMe/Browser/429893#Top

Feedback

End Review
1/2

8/22/2016

MyPastest

Difficulty: Average
Peer Responses

Session Progress
Responses Correct:

Responses Incorrect:

298

Responses Total:

298

Responses - % Correct:

0%

Blog (https://www.pastest.com/blog) About Pastest (https://www.pastest.com/about-us)


Contact Us (https://www.pastest.com/contact-us) Help (https://www.pastest.com/help)
Pastest 2016

https://mypastest.pastest.com/Secure/TestMe/Browser/429893#Top

2/2

8/22/2016

MyPastest

Back to Filters (/Secure/TestMe/Filter/429893/QA)

Question 109 of 298

A 52-year-old man presents with an acute upper gastrointestinal haemorrhage, but has no
further bleeding after the initial episode. Upper gastrointestinal endoscopy reveals a
suspicious ulcer, which is biopsied. This reveals the presence of mucosa-associated lymphoid
tissue and Helicobacter pylori.
What is the most appropriate initial treatment in this case?
A

High-dose proton-pump inhibitor therapy

Heliobacter pylori eradication therapy

Chemotherapy for lymphoma

Surveillance endoscopy in 3 months

Referral for surgery

Explanation
MALT lymphoma
Where there is localised mucosa-associated lymphoid tissue (MALT) co-existent with
Helicobacter pylori infection, there is evidence that eradication of H. pylori can result in
resolution of the MALT. However, for larger areas of lymphoid tissue or where the patient is
H. pylori negative, eradication therapy is much less effective.
It is thought that H. pylori infection leads to stimulation of B lymphocytes and that a B-cell
clone can become autonomous after a chromosome 1:14 translocation. Low-grade lymphomas
may then become high-grade lymphomas through the influence of p53 and other factors. It is
now becoming clear that the tyrosine kinase inhibitor imatinib (Glivec) might be useful
treatment for larger tumours.
5209

Next Question

Previous Question

Tag Question

https://mypastest.pastest.com/Secure/TestMe/Browser/429893#Top

Feedback

End Review

1/2

8/22/2016

MyPastest

Difficulty: Easy
Peer Responses

Session Progress
Responses Correct:

Responses Incorrect:

298

Responses Total:

298

Responses - % Correct:

0%

Blog (https://www.pastest.com/blog) About Pastest (https://www.pastest.com/about-us)


Contact Us (https://www.pastest.com/contact-us) Help (https://www.pastest.com/help)
Pastest 2016

https://mypastest.pastest.com/Secure/TestMe/Browser/429893#Top

2/2

8/22/2016

MyPastest

Back to Filters (/Secure/TestMe/Filter/429893/QA)

Question 110 of 298

A 17-year-old girl attends the Emergency Department with her parents. She has had a recent
row with her boyfriend and admits to having ingested 40 500-mg paracetamol tablets
around 24 hours ago.
Which of the following markers is the best indicator of prognosis?
A

Activated partial thromboplastin time (APTT)

Prothrombin time

Alanine aminotransferase (ALT)

Aspartate aminotransferase (AST)

Bilirubin

Explanation
Paracetamol overdose
The best laboratory indicator of prognosis is the prothrombin time or the international
normalised ratio (INR). Large rises in aminotransferase activity can occur in patients
with relatively well-preserved liver function.
Crucial to assessment of the need for N-acetylcysteine therapy is the time elapsed
since the paracetamol overdose:
As long as more than 4 hours have elapsed since ingestion, blood should be drawn for
assessment of paracetamol levels. These levels should then be assessed against the
nomogram for N-acetylcysteine treatment.
If more than 8 hours but less than 15 hours have elapsed since the overdose, then start
N-acetylcysteine treatment immediately, although this can be discontinued if levels are
below the treatment threshold.
In patients where more than 15 hours have elapsed, the nomogram has less predictive
value and treatment with N-acetylcysteine is advised in all patients.

https://mypastest.pastest.com/Secure/TestMe/Browser/429893#Top

1/2

8/22/2016

MyPastest

There is some evidence that late-presenting overdose patients might actually gain
additional benefit from continuous infusion of N-acetylcysteine at the 16-hour rate.
5210

Next Question

Previous Question

Tag Question

Feedback

End Review

Difficulty: Easy
Peer Responses

Session Progress
Responses Correct:

Responses Incorrect:

298

Responses Total:

298

Responses - % Correct:

0%

Blog (https://www.pastest.com/blog) About Pastest (https://www.pastest.com/about-us)


Contact Us (https://www.pastest.com/contact-us) Help (https://www.pastest.com/help)
Pastest 2016

https://mypastest.pastest.com/Secure/TestMe/Browser/429893#Top

2/2

8/22/2016

MyPastest

Back to Filters (/Secure/TestMe/Filter/429893/QA)

Question 111 of 298

You review a 21-year-old woman who presents to her GP with abnormal liver function tests.
On examination he notices that she appears to have a tremor. He arranges a screen for causes
of chronic liver disease.
Which of the following is suggestive of Wilson's disease?
A

Decreased serum caeruloplasmin

Increased serum copper

Reduced urinary copper

Polycythaemia

Reduced liver copper content

Explanation
Wilsons disease
Wilsons disease or hepatolenticular degeneration is caused by a defect within a coppertransporting ATPase encoded by a gene on chromosome 13. This leads to a failure of biliary
excretion of copper by the liver; caeruloplasmin is low in over 80% of patients due to
decreased synthesis.
Clinical features include neurological abnormalities (tremor, dysarthria, involuntary
movements and eventually dementia) and either fulminant Wilsons disease with a
presentation with acute hepatitis or a more chronic liver disease picture. A specific sign is the
KayserFleischer ring, which is caused by copper deposition in Descemets membrane in the
cornea.
Investigations show: serum copper and caeruloplasmin are usually reduced, urinary copper
excretion is increased, liver biopsy reveals increased copper content, and anaemia and
haemolysis can also occur.
5211

Next Question

https://mypastest.pastest.com/Secure/TestMe/Browser/429893#Top

1/2

8/22/2016

MyPastest

Previous Question

Tag Question

Feedback

End Review

Difficulty: Easy
Peer Responses

Session Progress
Responses Correct:

Responses Incorrect:

298

Responses Total:

298

Responses - % Correct:

0%

Blog (https://www.pastest.com/blog) About Pastest (https://www.pastest.com/about-us)


Contact Us (https://www.pastest.com/contact-us) Help (https://www.pastest.com/help)
Pastest 2016

https://mypastest.pastest.com/Secure/TestMe/Browser/429893#Top

2/2

8/22/2016

MyPastest

Back to Filters (/Secure/TestMe/Filter/429893/QA)

Question 112 of 298

A 58-year-old publican presented with haematemesis and malaena. He was transfused.


Gastroscopy showed small oesophageal varices that were not bleeding and multiple bleeding
areas in the stomach.
What is the most appropriate management to reduce long-term risk of bleeding?
A

Variceal banding

Adrenaline (epinephrine) injection

Intravenous ethanolamine

Oral propranolol

Intravenous octreotide

Explanation
Portal hypertensive gastropathy
This patient has only small varices, with no evidence of bleeding from them on this occasion,
but there is evidence of multiple areas of haemorrhage in the stomach. Oral propranolol in a
dose sufficient to reduce the resting pulse rate by 25% has been shown to decrease portal
pressure. It has been shown to decrease the frequency of variceal re-bleeds and reduces
bleeding from portal hypertensive gastropathy. Unfortunately a significant number of patients
either have contraindications to -blockade or are intolerant of the treatment due to sideeffects. Surveillance gastroscopy is advised for the future, with variceal banding in patients
who look likely to bleed.
5212

Next Question

Previous Question

Tag Question

Feedback

End Review

Difficulty: Average

https://mypastest.pastest.com/Secure/TestMe/Browser/429893#Top

1/2

8/22/2016

MyPastest

Peer Responses

Session Progress
Responses Correct:

Responses Incorrect:

298

Responses Total:

298

Responses - % Correct:

0%

Blog (https://www.pastest.com/blog) About Pastest (https://www.pastest.com/about-us)


Contact Us (https://www.pastest.com/contact-us) Help (https://www.pastest.com/help)
Pastest 2016

https://mypastest.pastest.com/Secure/TestMe/Browser/429893#Top

2/2

8/22/2016

MyPastest

Back to Filters (/Secure/TestMe/Filter/429893/QA)

Question 113 of 298

A patient has a history of worsening heartburn and nocturnal cough. Gastroscopy showed
Barrett's oesophagus with mild dysplastic change. There were also dysplastic changes at the
gastrocardiac junction.
What should the next step in the management be?
A

Fundoplication

Oesophagectomy

Acid suppression and repeat gastroscopy

Oesophagogastrectomy

Advice about physical measures such as propping up the bed-head

Explanation
Barretts oesophagus
This patient has Barretts oesophagus with evidence of dysplasia. Barretts oesophagus is
thought to be related to long-standing oesophageal reflux. Histological examination shows
columnar epithelium with intestinal (gastric) metaplasia that extends upwards into the lower
oesophagus. It is a finding in 20% of patients who undergo gastroscopy.
Problems arise because the condition is pre-malignant for adenocarcinoma of the
oesophagus. The recommended therapy is acid suppression with high-dose proton-pump
inhibitors for mild dysplasia; endoscopic mucosal ablation or oesophagectomy is
recommended for patients with severe dysplasia.
5213

Next Question

Previous Question

Tag Question

Feedback

End Review

Difficulty: Average
https://mypastest.pastest.com/Secure/TestMe/Browser/429893#Top

1/2

8/22/2016

MyPastest

Peer Responses

Session Progress
Responses Correct:

Responses Incorrect:

298

Responses Total:

298

Responses - % Correct:

0%

Blog (https://www.pastest.com/blog) About Pastest (https://www.pastest.com/about-us)


Contact Us (https://www.pastest.com/contact-us) Help (https://www.pastest.com/help)
Pastest 2016

https://mypastest.pastest.com/Secure/TestMe/Browser/429893#Top

2/2

8/22/2016

MyPastest

Back to Filters (/Secure/TestMe/Filter/429893/QA)

Question 114 of 298

A 23-year-old woman experienced nausea, vomiting and abdominal cramps 4 hours after
eating a salad and a hamburger in a local restaurant. Watery diarrhoea began a few hours
later.
What is the most likely organism?
A

Vibrio vulnificus

Listeria monocytogenes

Yersinia enterocolitica

Clostridium welchii

Staphylococcus aureus

Explanation
Food poisoning
Staphylococcal food poisoning leads to profuse vomiting that occurs 24 hours after
eating food (often pre-made salads, meat or dairy products), contaminated by an
enterotoxin. Diarrhoea occurs a few hours later and is profuse and watery.
Yersinia is most commonly associated with the ingestion of improperly cooked meat,
but symptoms generally begin more than 1 day after ingestion of the contaminated
food, (average of 4 days incubation period).
Symptoms resulting from Listeria monocytogenes infection also occur more than 24
hours after the ingestion of contaminated foods (milk, ice cream and poultry).
Vibrio vulnificus-associated food poisoning usually presents 2448 hours after the
ingestion of contaminated seafood (usually oysters).
The two clostridia associated with food poisoning are Clostridium perfringens
(Clostridium welchii) and Clostridium botulinum and symptoms typically present from
612 hours after eating the contaminated food.
5256

https://mypastest.pastest.com/Secure/TestMe/Browser/429893#Top

1/2

8/22/2016

MyPastest

Next Question

Previous Question

Tag Question

Feedback

End Review

Difficulty: Average
Peer Responses

Session Progress
Responses Correct:

Responses Incorrect:

298

Responses Total:

298

Responses - % Correct:

0%

Blog (https://www.pastest.com/blog) About Pastest (https://www.pastest.com/about-us)


Contact Us (https://www.pastest.com/contact-us) Help (https://www.pastest.com/help)
Pastest 2016

https://mypastest.pastest.com/Secure/TestMe/Browser/429893#Top

2/2

8/22/2016

MyPastest

Back to Filters (/Secure/TestMe/Filter/429893/QA)

Question 115 of 298

A 35-year-old man presents with a history of diarrhoea for 10 days, characterised by frequent,
low-volume stools with mucus. He also complained of subjective fever and lower abdominal
pain.
The presence of leucocytes in the stool is consistent with which of the following organisms?
A

Clostridium perfringens

Staphylococcus aureus

Giardia lamblia

Enterobius vermicularis

Entamoeba histolytica

Explanation
Entamoeba histolytica
The presence of large numbers of leucocytes in stool suggests colonic mucosal inflammation
and should suggest infection with enteroinvasive organisms such as Shigella, Entamoeba
histolytica, Salmonella, Campylobacter, invasive Escherichia coli or Yersinia enterocolitica.
Entamoeba histolytica is diagnosed via microscopic stool examination or examination of
colonic exudates obtained at sigmoidoscopy.
The diagnosis is confirmed by demonstrating mobile trophozoites containing red blood cells.
Sigmoidoscopy might show colonic ulceration, but is rarely diagnostic of amoebic dysentery.
Treatment is with high-dose oral metronidazole as a short course for colitis, with a more
prolonged treatment course being used for patients with amoebic liver abscess.
5257

Next Question

Previous Question

Tag Question

https://mypastest.pastest.com/Secure/TestMe/Browser/429893#Top

Feedback

End Review

1/2

8/22/2016

MyPastest

Difficulty: Difficult
Peer Responses

Session Progress
Responses Correct:

Responses Incorrect:

298

Responses Total:

298

Responses - % Correct:

0%

Blog (https://www.pastest.com/blog) About Pastest (https://www.pastest.com/about-us)


Contact Us (https://www.pastest.com/contact-us) Help (https://www.pastest.com/help)
Pastest 2016

https://mypastest.pastest.com/Secure/TestMe/Browser/429893#Top

2/2

8/22/2016

MyPastest

Back to Filters (/Secure/TestMe/Filter/429893/QA)

Question 116 of 298

A 42-year-old man presents with intermittent dysphagia to solids and liquids and
regurgitation of food. He has lost 4 pounds in 2 months. His physical examination is normal. A
barium swallow reveals a dilated oesophageal body, with the distal oesophagus terminating in
a narrow end.
Which one of the following options is the most effective long-term therapy?
A

Isosorbide dinitrate

Metoclopramide

Dilation with balloon

Nifedipine

Dilation with a bougie

Explanation
Achalasia
The cause of achalasia is unknown. It is characterised by aperistalsis in the body of the lower
oesophagus and failure of relaxation of the lower oesophageal sphincter.
Mechanical treatment
Achalasia is best treated with mechanical disruption of the lower oesophageal sphincter.
Dilation with a large Hurst bougie might give temporary relief; a few patients have been
maintained with weekly self-dilations, but this treatment is no longer recommended. Dilation
with a pneumatic balloon (bag) under radiographic control is much more effective.
Medical treatment
A successful approach to long-term pharmacological management of achalasia has not been
established. Short-term improvement in clinical symptoms and in scintigraphic oesophageal
emptying may occur with isosorbide mononitrate, a long-acting nitrate or with nifedipine, a
calcium-channel blocker. Promotility agents like metoclopramide increase the lower
oesophageal sphincter pressure and so are contraindicated in achalasia.
5260

https://mypastest.pastest.com/Secure/TestMe/Browser/429893#Top

1/2

8/22/2016

MyPastest

Next Question

Previous Question

Tag Question

Feedback

End Review

Difficulty: Average
Peer Responses

Session Progress
Responses Correct:

Responses Incorrect:

298

Responses Total:

298

Responses - % Correct:

0%

Blog (https://www.pastest.com/blog) About Pastest (https://www.pastest.com/about-us)


Contact Us (https://www.pastest.com/contact-us) Help (https://www.pastest.com/help)
Pastest 2016

https://mypastest.pastest.com/Secure/TestMe/Browser/429893#Top

2/2

8/22/2016

MyPastest

Back to Filters (/Secure/TestMe/Filter/429893/QA)

Question 117 of 298

A 45-year-old man comes to your clinic complaining of having epigastric pain for 2 months.
His GP prescribed him H2-blockers 3 weeks ago, which have produced only partial relief of his
symptoms. His weight is stable. His physical examination is normal. An upper gastrointestinal
endoscopy reveals a 1-cm duodenal ulcer.
Which of the following risk factors is not associated with the development of duodenal ulcer
disease?
A

Daily use of non-steroidal anti-inflammatory drugs (NSAIDs)

Gastric infection with Helicobacter pylori

Emotional stress

Cigarette smoking

Gastrin-secreting tumours

Explanation
Duodenal ulcer
Duodenal ulcers are very common and are two to three times more common than gastric
ulcers. At some time in their lives approximately 1015% of the population will suffer from a
duodenal ulcer. Although considered a risk factor in the past, several studies have shown that
emotional stress is not a risk factor for the development of duodenal ulcer.
Risk factors
Daily non-steroidal anti-inflammatory drug (NSAID) use significantly increases the risk
of ulcer disease (10- to 20-fold).
Gastric infection with Helicobacter pylori increases risk about 57-fold.
Cigarette smoking doubles the risk of duodenal ulcer.
At least 90% of patients with ZollingerEllison syndrome (gastrin-secreting tumours)
have a duodenal ulcer.
5261

https://mypastest.pastest.com/Secure/TestMe/Browser/429893#Top

1/2

8/22/2016

MyPastest

Next Question

Previous Question

Tag Question

Feedback

End Review

Difficulty: Average
Peer Responses

Session Progress
Responses Correct:

Responses Incorrect:

298

Responses Total:

298

Responses - % Correct:

0%

Blog (https://www.pastest.com/blog) About Pastest (https://www.pastest.com/about-us)


Contact Us (https://www.pastest.com/contact-us) Help (https://www.pastest.com/help)
Pastest 2016

https://mypastest.pastest.com/Secure/TestMe/Browser/429893#Top

2/2

8/22/2016

MyPastest

Back to Filters (/Secure/TestMe/Filter/429893/QA)

Question 118 of 298

A 51-year-old woman presents with abdominal pain, weight loss, early satiety and night
sweats. On physical examination she appears cachectic. There are multiple enlarged lymph
nodes in her neck (supraclavicular area) and a mass is palpated in the epigastrium.
Laboratory data reveal: haemoglobin 8 g/dl, normal white blood cell count.
Which of the following investigations is most likely to help reach a diagnosis?
A

Upper gastrointestinal X-ray series

Peripheral blood smear

Computed tomography (CT) of the abdomen

Upper endoscopy with biopsy

Exploratory laparotomy

Explanation
Gastric lymphoma
This patient has lymphoma of the stomach. Lymphoma of the stomach accounts for around
10% of stomach malignancies in the developed world. Development of this tumour is thought
to be linked to Helicobacter pylori infection in many patients and H. pylori eradication can
result in regression of small tumours. The tumours can resemble superficially spreading
carcinoma, linitis plastica or solitary adenocarcinoma.
Diagnosis
Gastroscopy with directed biopsy and brush cytology gives a higher yield than was
previously appreciated, especially in the presence of exophytic lesions. Lymphoma of the
stomach frequently presents radiographically as a bulky mass and less frequently as a
diffusely infiltrating tumour the most common form of secondary lymphoma giving the
appearance of large folds on upper gastrointestinal series, frequently associated with multiple
nodular defects and ulcerations. Although computed tomography can be useful to evaluate
the extent of disease, it will not provide a specific diagnosis.
Management

https://mypastest.pastest.com/Secure/TestMe/Browser/429893#Top

1/2

8/22/2016

MyPastest

Exploratory laparotomy is useful for staging and therapeutic resection where possible.
Glivec, used as a treatment for chronic myeloid leukaemia (CML) is also now proving useful
in managing gastric lymphoma.
5263

Next Question

Previous Question

Tag Question

Feedback

End Review

Difficulty: Average
Peer Responses

Session Progress
Responses Correct:

Responses Incorrect:

298

Responses Total:

298

Responses - % Correct:

0%

Blog (https://www.pastest.com/blog) About Pastest (https://www.pastest.com/about-us)


Contact Us (https://www.pastest.com/contact-us) Help (https://www.pastest.com/help)
Pastest 2016

https://mypastest.pastest.com/Secure/TestMe/Browser/429893#Top

2/2

8/22/2016

MyPastest

Back to Filters (/Secure/TestMe/Filter/429893/QA)

Question 119 of 298

Which of the following features best distinguishes Crohn's disease from ulcerative colitis?
A

Uveitis

Rectal bleeding

Fatty liver

Non-caseating granulomas

Crypt abscesses

Explanation
Inflammatory bowel diseases
The annual incidence of Crohns disease is about 56/100,000; the incidence of ulcerative
colitis is higher at around 615/100,000. There are various differences between the two
conditions in terms of histological findings and extra-gastrointestinal manifestations.
Histological findings
Inflammation is deep (transmural) and patchy in Crohns disease but tends to be
mucosal and continuous in ulcerative colitis.
Non-caseating granulomas are typical of Crohns disease but not ulcerative colitis.
Goblet cells are present in normal numbers in Crohns but depleted in ulcerative colitis.
Crypt abscesses occur in Crohns disease, but are more common in ulcerative colitis.
Extra-gastrointestinal manifestations
Crohns and ulcerative colitis show a similar prevalence of uveitis, episcleritis and
conjunctivitis.
Arthralgia and inflammatory back pain occur with slightly increased frequency in
Crohns disease, as does erythaema nodosum. Pyoderma gangrenosum is seen more
commonly in UC.
https://mypastest.pastest.com/Secure/TestMe/Browser/429893#Top

1/2

8/22/2016

MyPastest

Sclerosing cholangitis is more common in ulcerative colitis.


Nephrolithiasis and gallstones are both more common in Crohns disease.
5264

Next Question

Previous Question

Tag Question

Feedback

End Review

Difficulty: Average
Peer Responses

Session Progress
Responses Correct:

Responses Incorrect:

298

Responses Total:

298

Responses - % Correct:

0%

Blog (https://www.pastest.com/blog) About Pastest (https://www.pastest.com/about-us)


Contact Us (https://www.pastest.com/contact-us) Help (https://www.pastest.com/help)
Pastest 2016

https://mypastest.pastest.com/Secure/TestMe/Browser/429893#Top

2/2

8/22/2016

MyPastest

Back to Filters (/Secure/TestMe/Filter/429893/QA)

Question 120 of 298

A 49-year-old man presents to the Emergency Department because of melaena of 3 days'


duration. He denies having any abdominal pain. Vital signs reveal a resting pulse of 104 bpm
and a 25-mmHg orthostatic drop in blood pressure. Physical findings include bilateral
temporal wasting, pale conjunctivae, spider angiomas on his upper torso, muscle wasting,
hepatosplenomegaly and hyperactive bowel sounds without abdominal tenderness to
palpation. He is passing melaena stool. Nasogastric tube aspiration reveals coffee-grounds
material. His haematocrit is 31%. Aggressive fluid resuscitation is commenced.
What would the appropriate next step in the management of this man's illness be?
A

Pass a SengstakenBlakemore tube

Obtain an upper gastrointestinal barium series

Insert a transjugular intrahepatic portosystemic shunt (TIPS)

Obtain immediate visceral angiography

Perform upper endoscopy

Explanation
Variceal bleeding
This patient has signs of chronic liver disease and an underlying diagnosis of oesophageal
varices secondary to portal hypertension might be suspected here. After this patient has
been haemodynamically stabilised, the next most important step is to perform a
diagnostic/therapeutic upper endoscopy. If the source of his bleeding is from oesophageal
varices, then these can be treated with sclerosing agents or, preferably, endoscopic band
ligation.
The use of a SengstakenBlakemore tube should be reserved for patients in whom upper
endoscopy was unsuccessful in controlling the haemorrhage. A transjugular intrahepatic
portosystemic shunt (TIPS) procedure might be considered in order to relieve portal
hypertension in some specialist centres that offer this surgical technique, but the number of
centres who offer it is small. Barium studies have no role in the evaluation of patients with
suspected variceal haemorrhage.
5265

https://mypastest.pastest.com/Secure/TestMe/Browser/429893#Top

1/2

8/22/2016

MyPastest

Next Question

Previous Question

Tag Question

Feedback

End Review

Difficulty: Easy
Peer Responses

Session Progress
Responses Correct:

Responses Incorrect:

298

Responses Total:

298

Responses - % Correct:

0%

Blog (https://www.pastest.com/blog) About Pastest (https://www.pastest.com/about-us)


Contact Us (https://www.pastest.com/contact-us) Help (https://www.pastest.com/help)
Pastest 2016

https://mypastest.pastest.com/Secure/TestMe/Browser/429893#Top

2/2

8/22/2016

MyPastest

Back to Filters (/Secure/TestMe/Filter/429893/QA)

Question 121 of 298

A 40-year-old sailor presents with gum bleeding. Scurvy is diagnosed. Vitamin C is essential
for which process in collagen synthesis?
A

Hydroxylation of procollagen proline and lysine

Oxidation of elastin

Oxidation of procollagen proline and lysine

Vitamin K activation

Factor X activation

Explanation
Vitamin C
Vitamin C (ascorbic acid) is essential for collagen formation and helps maintain the
integrity of substances of mesenchymal origin, such as connective tissue, osteoid tissue
and dentin.
Vitamin C is essential for wound healing and facilitates recovery from burns.
As a reductant (with oxygen, ferrous iron and a 2-ketoacid), vitamin C activates
enzymes that hydroxylate procollagen proline and lysine to procollagen hydroxyproline
and hydroxylysine in scorbutic animals, elastin becomes increasingly deficient in
hydroxyproline.
Vitamin C facilitates the absorption of iron.
Severe deficiency results in scurvy, an acute or chronic disease characterised by
haemorrhagic manifestations and abnormal osteoid and dentin formation.
5480

Next Question

https://mypastest.pastest.com/Secure/TestMe/Browser/429893#Top

1/2

8/22/2016

MyPastest

Previous Question

Tag Question

Feedback

End Review

Difficulty: Average
Peer Responses

Session Progress
Responses Correct:

Responses Incorrect:

298

Responses Total:

298

Responses - % Correct:

0%

Blog (https://www.pastest.com/blog) About Pastest (https://www.pastest.com/about-us)


Contact Us (https://www.pastest.com/contact-us) Help (https://www.pastest.com/help)
Pastest 2016

https://mypastest.pastest.com/Secure/TestMe/Browser/429893#Top

2/2

8/22/2016

MyPastest

Back to Filters (/Secure/TestMe/Filter/429893/QA)

Question 122 of 298

Which organ is in direct contact with the anterior surface of the left kidney without the
separation of the visceral peritoneum?
A

Duodenum

Jejunum

Pancreas

Spleen

Stomach

Explanation
Anatomical relations of the left kidney
A small area along the upper part of the medial border of the left kidney is in contact with
the left suprarenal gland, and close to the lateral border is a long strip in contact with the
spleen. A somewhat quadrilateral field, about the middle of the anterior surface, marks the
site of contact with the body of the pancreas, on the deep surface of which are the lienal
vessels. Above this is a small triangular portion, between the suprarenal and splenic areas, in
contact with the posteroinferior surface of the stomach. Below the pancreatic area the lateral
part is in contact with the left colic flexure, the medial with the small intestine.
The areas in contact with the stomach and spleen are covered by the peritoneum of the
omental bursa, while that in relation to the small intestine is covered by the peritoneum of the
general cavity; behind the latter are some branches of the left colic vessels. The suprarenal,
pancreatic and colic areas are devoid of peritoneum.
5505

Next Question

Previous Question

Tag Question

Feedback

End Review

Difficulty: Average
https://mypastest.pastest.com/Secure/TestMe/Browser/429893#Top

1/2

8/22/2016

MyPastest

Peer Responses

Session Progress
Responses Correct:

Responses Incorrect:

298

Responses Total:

298

Responses - % Correct:

0%

Blog (https://www.pastest.com/blog) About Pastest (https://www.pastest.com/about-us)


Contact Us (https://www.pastest.com/contact-us) Help (https://www.pastest.com/help)
Pastest 2016

https://mypastest.pastest.com/Secure/TestMe/Browser/429893#Top

2/2

8/22/2016

MyPastest

Back to Filters (/Secure/TestMe/Filter/429893/QA)

Question 123 of 298

A patient with hepatic encephalopathy is given lactulose.


Which of the following statements about lactulose is true?
A

It is absorbed from the gut

It causes hypermagnesaemia

It is contraindicated in diabetes

It reduces proliferation of ammonia producing bacteria

It reduces absorption of spironolactone

Explanation
Lactulose
Lactulose is a synthetic disaccharide; there is no disaccharidase on the microvillus membrane
of enterocytes that hydrolyse lactulose. Its metabolism by colonic bacteria leads to
production of lactic acid and other organic acids, a fall in colonic pH and increased ionisation
of nitrogenous compounds. These changes may lead to a decrease in the absorption of
nitrogenous compounds, including ammonia. Lactulose is a cathartic and is widely believed
to be efficacious in the management of hepatic encephalopathy.
5561

Next Question

Previous Question

Tag Question

Feedback

End Review

Difficulty: Average
Peer Responses

https://mypastest.pastest.com/Secure/TestMe/Browser/429893#Top

1/2

8/22/2016

MyPastest

Session Progress
Responses Correct:

Responses Incorrect:

298

Responses Total:

298

Responses - % Correct:

0%

Blog (https://www.pastest.com/blog) About Pastest (https://www.pastest.com/about-us)


Contact Us (https://www.pastest.com/contact-us) Help (https://www.pastest.com/help)
Pastest 2016

https://mypastest.pastest.com/Secure/TestMe/Browser/429893#Top

2/2

8/22/2016

MyPastest

Back to Filters (/Secure/TestMe/Filter/429893/QA)

Question 124 of 298

A patient underwent endoscopy and a MalloryWeiss tear has been diagnosed.


What is the most likely cause?
A

Oesophageal varices

Barratts oesophagus

Persistent vomiting

Toxic fume inhalation

Bacterial infection

Explanation
MalloryWeiss syndrome
MalloryWeiss syndrome is characterised by upper gastrointestinal bleeding secondary to
longitudinal mucosal lacerations at the gastroesophageal junction or gastric cardia. The
original description by Mallory and Weiss in 1929 was in patients with persistent retching and
vomiting following an alcoholic binge. However, MalloryWeiss syndrome can occur after any
event that provokes a sudden rise in intragastric pressure or gastric prolapse into the
oesophagus.
5576

Next Question

Previous Question

Tag Question

Feedback

End Review

Difficulty: Easy
Peer Responses

https://mypastest.pastest.com/Secure/TestMe/Browser/429893#Top

1/2

8/22/2016

MyPastest

Session Progress
Responses Correct:

Responses Incorrect:

298

Responses Total:

298

Responses - % Correct:

0%

Blog (https://www.pastest.com/blog) About Pastest (https://www.pastest.com/about-us)


Contact Us (https://www.pastest.com/contact-us) Help (https://www.pastest.com/help)
Pastest 2016

https://mypastest.pastest.com/Secure/TestMe/Browser/429893#Top

2/2

8/22/2016

MyPastest

Back to Filters (/Secure/TestMe/Filter/429893/QA)

Question 125 of 298

A 45-year-old man has one episode of haematemesis. His haemoglobin is 11 g/dl. He had
taken 300 mg aspirin just prior to this and admitted to using aspirins regularly for a knee
injury over the past few days.
What would be the most likely cause?
A

Duodenal ulcer

Gastric erosions

MalloryWeiss tear

Oesophageal varices

Helicobacter pylori gastritis

Explanation
Frank haematemesis
Upper gastrointestinal haemorrhage usually presents with haematemesis or coffee-ground
vomiting and melaena. Frank haematemesis indicates a severe bleed. Haematemesis is not
always a feature of an upper GI bleed, but melaena will always follow a significant bleed. In
rapid bleeding, symptoms of hypovolaemia can precede haematemesis or melaena these
include postural hypotension, syncope, shock, and even death.
In most cases the causative lesion will not be known until diagnostic endoscopy is
undertaken. The patient should be asked about ingestion of non-steroidal anti-inflammatory
drugs and whether blood was present in the first vomit (it is usually absent in MalloryWeiss
tear). Given the patient has only taken aspirins for a few days, erosions are much more likely
than a frank gastric ulcer.
5583

Next Question

Previous Question

Tag Question

https://mypastest.pastest.com/Secure/TestMe/Browser/429893#Top

Feedback

End Review

1/2

8/22/2016

MyPastest

Difficulty: Average
Peer Responses

Session Progress
Responses Correct:

Responses Incorrect:

298

Responses Total:

298

Responses - % Correct:

0%

Blog (https://www.pastest.com/blog) About Pastest (https://www.pastest.com/about-us)


Contact Us (https://www.pastest.com/contact-us) Help (https://www.pastest.com/help)
Pastest 2016

https://mypastest.pastest.com/Secure/TestMe/Browser/429893#Top

2/2

8/22/2016

MyPastest

Back to Filters (/Secure/TestMe/Filter/429893/QA)

Question 126 of 298

Which of the following describes the primary mechanism of action of lactulose in the
gastrointestinal tract?
A

Increased intestinal motility

Raised faecal pH

Similar to a bulk-forming laxative

Inhibition of proliferation of ammonia-producing organisms

Osmotic laxative

Explanation
Mechanism of action of lactulose
Lactulose is known as an osmotic laxative. It is made up of sugar molecules that are broken
down by bacteria in the lower part of the gut. This leads to the contents of the gut becoming
more acidic and a consequent reduction in the absorption of ammonia. The presence of
ammonia in the gut causes water to be drawn into the lower bowel, which causes an increase
in the water content and volume of the stools, so relieving constipation.
5585

Next Question

Previous Question

Tag Question

Feedback

End Review

Difficulty: Average
Peer Responses

https://mypastest.pastest.com/Secure/TestMe/Browser/429893#Top

1/2

8/22/2016

MyPastest

Session Progress
Responses Correct:

Responses Incorrect:

298

Responses Total:

298

Responses - % Correct:

0%

Blog (https://www.pastest.com/blog) About Pastest (https://www.pastest.com/about-us)


Contact Us (https://www.pastest.com/contact-us) Help (https://www.pastest.com/help)
Pastest 2016

https://mypastest.pastest.com/Secure/TestMe/Browser/429893#Top

2/2

8/22/2016

MyPastest

Back to Filters (/Secure/TestMe/Filter/429893/QA)

Question 127 of 298

A 56-year-old man presents with weight loss. He has been complaining of dyspepsia for a
long time. Gastric biopsy shows mucosa containing lymphoma cells and there is a
Helicobacter pylori infection present.
What is the most appropriate therapy?
A

Oral chemotherapy

Intravenous chemotherapy

Radiotherapy

H. pylori eradication

Gastrectomy

Explanation
Gastric MALT lymphoma
There are several lines of evidence that implicate Helicobacter pylori in the pathogenesis of
gastric mucosa-associated lymphoid tissue (MALT) lymphoma. One observation is that
normal gastric mucosa is devoid of organised lymphoid tissue but this tissue accumulates as
a consequence of H. pylori infection; in addition, the organism can be detected in most cases
of gastric MALT lymphoma.
The association was strengthened by an epidemiological study that showed that there was a
significantly higher frequency of preceding H. pylori infection in patients with gastric
lymphoma compared with matched controls with non-gastric lymphoma. The evidence
became even more compelling following in-vitro studies which showed that the cells of lowgrade gastric MALT lymphoma respond to H. pylori antigens via a T-cell-mediated
mechanism.
The clinical significance of these findings was first shown by a study that described the
regression of gastric MALT lymphoma in patients following eradication of H. pylori using
appropriate antibiotics. Subsequent studies have shown that eradication of H. pylori results in
striking regression of the lymphoma in approximately 75% of cases.
5622

https://mypastest.pastest.com/Secure/TestMe/Browser/429893#Top

1/2

8/22/2016

MyPastest

Next Question

Previous Question

Tag Question

Feedback

End Review

Difficulty: Easy
Peer Responses

Session Progress
Responses Correct:

Responses Incorrect:

298

Responses Total:

298

Responses - % Correct:

0%

Blog (https://www.pastest.com/blog) About Pastest (https://www.pastest.com/about-us)


Contact Us (https://www.pastest.com/contact-us) Help (https://www.pastest.com/help)
Pastest 2016

https://mypastest.pastest.com/Secure/TestMe/Browser/429893#Top

2/2

8/22/2016

MyPastest

Back to Filters (/Secure/TestMe/Filter/429893/QA)

Question 128 of 298

A 54-year-old man presents to the GP with altered bowel habit. For the past few weeks he
has noticed intermittent bouts of mucous diarrhoea and occasionally this has been
bloodstained. Faecal occult blood testing by his GP has confirmed the presence of blood in
the stool. He undergoes colonoscopy and a suspicious polyp in the descending colon is
removed and classified as a Dukes' A tumour.
Which of the following best describes the appropriate time intervals for follow-up
colonoscopy in this patient?
A

3-monthly

6-monthly

2-yearly

Annually

3-yearly

Explanation
Follow-up of a Dukes' A colon cancer
Dukes' A colonic carcinoma carries an excellent prognosis, with 5-year survival now estimated
at > 80%. It can be followed up using:
Colonoscopy indicated on an annual basis for the first 2 years, then this should be
done 3-yearly
Faecal occult blood should be tested 6-monthly for the first 4 years and then once
yearly
Carcinoembryonic antigen (CEA) can be used to monitor for recurrence if it is
elevated initially, although there is some controversy about the utility of CEA
monitoring and it is not used universally
The Dukes' System

https://mypastest.pastest.com/Secure/TestMe/Browser/429893#Top

1/2

8/22/2016

MyPastest

Dukes' A means the cancer is only in the innermost lining of the colon or rectum or slightly
growing into the muscle layer.
Dukes' B means the cancer has grown through the muscle layer of the colon or rectum.
Dukes' C means the cancer has spread to at least one lymph node in the area close to the
bowel.
Dukes' D means the cancer has spread to somewhere else in the body such as the liver or
lung.
8281

Next Question

Previous Question

Tag Question

Feedback

End Review

Difficulty: Average
Peer Responses

Session Progress
Responses Correct:

Responses Incorrect:

298

Responses Total:

298

Responses - % Correct:

0%

Blog (https://www.pastest.com/blog) About Pastest (https://www.pastest.com/about-us)


Contact Us (https://www.pastest.com/contact-us) Help (https://www.pastest.com/help)
Pastest 2016

https://mypastest.pastest.com/Secure/TestMe/Browser/429893#Top

2/2

8/22/2016

MyPastest

Back to Filters (/Secure/TestMe/Filter/429893/QA)

Question 129 of 298

You review a 24-year-old woman who is noted to be markedly underweight. You suspect that
she might have a protein malabsorption syndrome and contemplate trying her on an
elemental diet.
When thinking about dietary protein, which of the following best describes the site of
polypeptide absorption?
A

The proximal stomach

The distal stomach

The small intestine

The ascending colon

The descending colon

Explanation
Protein digestion
Although dietary protein is digested into polypeptides by pepsin and by exposure to a low
pH in the stomach, this is not the site of polypeptide absorption. Polypeptides pass into the
duodenum where they are further degraded by pancreatic proteases. Further digestion then
occurs at the level of the intestinal brush border and most absorption occurs in the form of
amino acids.
8283

Next Question

Previous Question

Tag Question

Feedback

End Review

Difficulty: Easy
Peer Responses

https://mypastest.pastest.com/Secure/TestMe/Browser/429893#Top

1/2

8/22/2016

MyPastest

Session Progress
Responses Correct:

Responses Incorrect:

298

Responses Total:

298

Responses - % Correct:

0%

Blog (https://www.pastest.com/blog) About Pastest (https://www.pastest.com/about-us)


Contact Us (https://www.pastest.com/contact-us) Help (https://www.pastest.com/help)
Pastest 2016

https://mypastest.pastest.com/Secure/TestMe/Browser/429893#Top

2/2

8/22/2016

MyPastest

Back to Filters (/Secure/TestMe/Filter/429893/QA)

Question 130 of 298

You are asked by a local GP to review a 72-year-old man who complains of lethargy and
tiredness. A recent haemoglobin was low at 10.1 g/dl and he had a macrocytosis. Further
investigation reveals no evidence of haematological malignancy, but screening does reveal
folic acid deficiency.
Which of the following foods contains the largest proportion of folic acid?
A

150 g of liver

1 banana

1 papaya

1 cup of baked beans

1 cup of raw spinach

Explanation
Folic acid deficiency
Folic acid requirements increase during pregnancy and lactation. The following foods contain
folic acid:
150 g liver contains around 180 mg of folic acid
A banana contains 20 mg folic acid
A papaya contains 25 mg folic acid
A cup of spinach contains 60 mg folic acid
A cup of baked beans contains about 60 mg folic acid
Causes of folate deficiency include:
Alcohol abuse
Malabsorption
Kidney dialysis
Liver disease
https://mypastest.pastest.com/Secure/TestMe/Browser/429893#Top

1/2

8/22/2016

MyPastest

Coeliac disease (via a process of villous atrophy)


Small-bowel resection (leads to a decreased area for folic acid absorption)
8285

Next Question

Previous Question

Tag Question

Feedback

End Review

Difficulty: Average
Peer Responses

Session Progress
Responses Correct:

Responses Incorrect:

298

Responses Total:

298

Responses - % Correct:

0%

Blog (https://www.pastest.com/blog) About Pastest (https://www.pastest.com/about-us)


Contact Us (https://www.pastest.com/contact-us) Help (https://www.pastest.com/help)
Pastest 2016

https://mypastest.pastest.com/Secure/TestMe/Browser/429893#Top

2/2

8/22/2016

MyPastest

Back to Filters (/Secure/TestMe/Filter/429893/QA)

Question 131 of 298

You review a 32-year-old woman who is morbidly obese. You are advising her about the
calorie content of commonly used foods.
Which of the following foods contains the greatest number of calories?
A

1 scone (70 g)

1 bowl of cornflakes, not including milk (45 g)

300 g of chicken korma

A sausage and egg triple sandwich pack (256 g)

50 g of salted peanuts

Explanation
Calorie content of snacks
A typical supermarket sausage and egg sandwich pack contains significant levels of fat
(around 53 g per 256-g serving). This is even more than in a chicken korma (around 30 g of
fat per serving). In general, it is the fat component (contributing 9 calories of energy value
per gram) that significantly adds to the total number of calories in a food item. This is the
rationale for following a low-fat diet.

8287

Next Question

Previous Question

Tag Question

Feedback

End Review

Difficulty: Average
Peer Responses
https://mypastest.pastest.com/Secure/TestMe/Browser/429893#Top

1/2

8/22/2016

MyPastest

Session Progress
Responses Correct:

Responses Incorrect:

298

Responses Total:

298

Responses - % Correct:

0%

Blog (https://www.pastest.com/blog) About Pastest (https://www.pastest.com/about-us)


Contact Us (https://www.pastest.com/contact-us) Help (https://www.pastest.com/help)
Pastest 2016

https://mypastest.pastest.com/Secure/TestMe/Browser/429893#Top

2/2

8/22/2016

MyPastest

Back to Filters (/Secure/TestMe/Filter/429893/QA)

Question 132 of 298

A 56-year-old man presents to his GP complaining of lethargy. Routine blood testing reveals
a hypochromic microcytic anaemia with low ferritin. He has had no symptoms of indigestion
or change in bowel habit and is on no medication of note.
Which of the following would be the most appropriate next investigation in this patient?
A

Upper gastrointestinal endoscopy

Rigid sigmoidoscopy

Computed tomography of the abdomen

Barium enema

Flexible colonoscopy

Explanation
Investigation of iron deficiency anaemia
Case series indicate that where there are no symptoms, investigation for iron deficiency
anaemia is quite likely to yield evidence of significant lower gastrointestinal pathology, which
is often malignant. Colonoscopy is therefore recommended as the best choice investigation
from the listed options. While dual pathology with abnormalities such as oesophagitis,
duodenitis or gastritis is often found, the incidence of upper gastrointestinal malignancy is
much lower. Neither the degree of anaemia nor the presence or absence of upper or lower
gastrointestinal symptoms appear to be predictive of whether upper or lower gastrointestinal
endoscopy will yield a diagnosis, however.
10690

Next Question

Previous Question

Tag Question

Feedback

End Review

Difficulty: Average

https://mypastest.pastest.com/Secure/TestMe/Browser/429893#Top

1/2

8/22/2016

MyPastest

Peer Responses

Session Progress
Responses Correct:

Responses Incorrect:

298

Responses Total:

298

Responses - % Correct:

0%

Blog (https://www.pastest.com/blog) About Pastest (https://www.pastest.com/about-us)


Contact Us (https://www.pastest.com/contact-us) Help (https://www.pastest.com/help)
Pastest 2016

https://mypastest.pastest.com/Secure/TestMe/Browser/429893#Top

2/2

8/22/2016

MyPastest

Back to Filters (/Secure/TestMe/Filter/429893/QA)

Question 133 of 298

A 48-year-old man undergoes flexible colonoscopy for the investigation of iron deficiency
anaemia. Three dysplastic polyps are identified and removed, 0.9 cm, 1.4 cm and 1.8 cm in
diameter.
Which of the following represents the most appropriate time period before follow-up
colonoscopy should be carried out?
A

6 months

1 year

3 years

4 years

5 years

Explanation
Dysplastic colonic polyps
The British Society of Gastroenterology (BSG) published guidelines on the follow-up period
for dysplastic colonic polyps in 2002:
5-year interval is indicated for low-risk patients (one to two adenomas that are both
small, ie <1 cm)
3-year follow up is recommended for medium-risk patients (three to four adenomas or
one or two adenomas where one adenoma bigger than or equal to 1 cm)
1-year follow-up is recommended for high-risk patients (five or more small adenomas or
more than three with at least one at or above 1 cm in size).
Where patients have a family history of polyps, unless they have one of the dominant
polyposis syndromes such as familial adenomatous polyposis (FAP), the follow-up periods
should be as above.
10691

Next Question
https://mypastest.pastest.com/Secure/TestMe/Browser/429893#Top

1/2

8/22/2016

MyPastest

Previous Question

Tag Question

Feedback

End Review

Difficulty: Difficult
Peer Responses

Session Progress
Responses Correct:

Responses Incorrect:

298

Responses Total:

298

Responses - % Correct:

0%

Blog (https://www.pastest.com/blog) About Pastest (https://www.pastest.com/about-us)


Contact Us (https://www.pastest.com/contact-us) Help (https://www.pastest.com/help)
Pastest 2016

https://mypastest.pastest.com/Secure/TestMe/Browser/429893#Top

2/2

8/22/2016

MyPastest

Back to Filters (/Secure/TestMe/Filter/429893/QA)

Question 134 of 298

A 19-year-old student returned early from a cricket tour of India. He has recently played in a
cricket match in Chennai, but had to leave the game early owing to sudden onset of painless,
voluminous diarrhoea. There is no associated fever but he is complaining of abdominal
cramps. He was advised to get on a plane home and collapsed as a result of dehydration
soon after leaving the plane at Heathrow. On questioning he reported opening his bowels
some 30 times in 24 hours. A sample collected in your Emergency Department revealed a
rice-water appearance and fishy odour. Vibrio cholerae were identified in the stool sample.
Which of the following is the most appropriate antibiotic in this case?
A

Oral doxycycline

iv Ciprofloxacin

Oral amoxicillin

Oral metronidazole

iv Ceftriaxone

Explanation
Antibiotics and cholera
There is evidence that antibiotic use in cholera infection leads to a shortening of the duration
of diarrhoea and an improved recovery versus oral rehydration alone. Three hundred
milligrams given as a single dose is the recommended regimen. Alternatives include 2 g of
oral tetracycline or oral ciprofloxacin. Erythromycin is an alternative, as is septrin although
generally avoided because of the risk of associated blood dyscrasias. Adequate rehydration is
associated with a good chance of recovery, and long-term sequelae associated with cholera
are extremely rare.
10692

Next Question

Previous Question

Tag Question

https://mypastest.pastest.com/Secure/TestMe/Browser/429893#Top

Feedback

End Review
1/2

8/22/2016

MyPastest

Difficulty: Difficult
Peer Responses

Session Progress
Responses Correct:

Responses Incorrect:

298

Responses Total:

298

Responses - % Correct:

0%

Blog (https://www.pastest.com/blog) About Pastest (https://www.pastest.com/about-us)


Contact Us (https://www.pastest.com/contact-us) Help (https://www.pastest.com/help)
Pastest 2016

https://mypastest.pastest.com/Secure/TestMe/Browser/429893#Top

2/2

8/22/2016

MyPastest

Back to Filters (/Secure/TestMe/Filter/429893/QA)

Question 135 of 298

A 48-year-old man presents with symptoms of indigestion and regurgitation of food. In


addition, he has a painful epigastric mass on palpation of the abdomen. Upper
gastrointestinal endoscopy reveals a gastric mass and biopsy reveals it to be a
gastrointestinal stromal tumour. You begin treatment with imatinib.
Which of the following options fits best with the mode of action of imatinib?
A

Inhibition of p53 production

Stimulation of p53 production

Inhibition of MAP kinase

Inhibition of tyrosine kinase

Stimulation of guanylate cyclase

Explanation
Imatinib
Imatinib (Glivec) is licensed for the treatment of chronic myeloid leukaemia and
gastrointestinal stromal tumours. It is a tyrosine kinase inhibitor, specifically the tyrosine
kinase domain in Abl (the Abelson proto-oncogene), c-kit and the platelet-derived growthfactor receptor. It might have other potential uses in the treatment of systemic mastocytosis
and of some sarcomas. Imatinib is generally well tolerated although severe congestive cardiac
failure is recognised as an uncommon side-effect in some patients.
14947

Next Question

Previous Question

Tag Question

Feedback

End Review

Difficulty: Easy
Peer Responses
https://mypastest.pastest.com/Secure/TestMe/Browser/429893#Top

1/2

8/22/2016

MyPastest

Session Progress
Responses Correct:

Responses Incorrect:

298

Responses Total:

298

Responses - % Correct:

0%

Blog (https://www.pastest.com/blog) About Pastest (https://www.pastest.com/about-us)


Contact Us (https://www.pastest.com/contact-us) Help (https://www.pastest.com/help)
Pastest 2016

https://mypastest.pastest.com/Secure/TestMe/Browser/429893#Top

2/2

8/22/2016

MyPastest

Back to Filters (/Secure/TestMe/Filter/429893/QA)

Question 136 of 298

A 23-year-old woman presents with intermittent diarrhoea, abdominal pain and distension.
She has also suffered increasing tiredness and lethargy for the past 6 months. You arrange
some investigations, which show: haemoglobin 10.5 g/dl, mean corpuscular volume (MCV)
105 fl, white cell count 8.2 109/l, platelets 135 109/l, sodium 140 mmol/l, potassium 3.9
mmol/l, creatinine 100 mol/l. ESR is elevated at 61 mm/1st hour. Colonoscopy with biopsies
shows multiple areas of inflammation and punched-out ulcers. Barium follow-through reveals
evidence of severe small-bowel inflammation.
Given the likely diagnosis, which of the following is the biggest risk factor associated with
the development of the disease?
A

Increasing age

Smoking

Oral contraceptive use

Diet low in soluble fibre

Excess alcohol consumption

Explanation
Crohns disease
This woman has Crohns disease, which has a bimodal distribution, with peak ages of onset at
1530 years and 6080 years. The disease is an inflammatory condition associated with noncaseating granuloma formation and local increases in inflammatory markers. It is thought that
smoking might play a role in increasing the pro-inflammatory state.
See http://www.ncbi.nlm.nih.gov/pubmed/16696783
(http://www.ncbi.nlm.nih.gov/pubmed/16696783)
18592

Next Question

Previous Question

Tag Question

https://mypastest.pastest.com/Secure/TestMe/Browser/429893#Top

Feedback

End Review

1/2

8/22/2016

MyPastest

Difficulty: Average
Peer Responses

Session Progress
Responses Correct:

Responses Incorrect:

298

Responses Total:

298

Responses - % Correct:

0%

Blog (https://www.pastest.com/blog) About Pastest (https://www.pastest.com/about-us)


Contact Us (https://www.pastest.com/contact-us) Help (https://www.pastest.com/help)
Pastest 2016

https://mypastest.pastest.com/Secure/TestMe/Browser/429893#Top

2/2

8/22/2016

MyPastest

Back to Filters (/Secure/TestMe/Filter/429893/QA)

Question 137 of 298

A 45-year-old woman is reviewed in the Gastroenterology Clinic. She has a 12-year history of
asthma. She also has a history of acid reflux, with symptoms of waterbrash and burning in her
throat nearly every night. She has had three lower respiratory tract infections in the past year
despite being on long-term continuous treatment with 40 mg omeprazole. Her body mass
index (BMI) is 28 kg/m2. Upper gastrointestinal endoscopy shows severe oesophagitis.
Which of the following would be the most appropriate next treatment step?
A

Encourage her to raise the head of the bed

Add an antacid to her regime

Refer her for fundoplication

Increase her omeprazole to 60 mg

Insist that she loses at least 15% of her weight

Explanation
Indications for fundoplication
The indications for fundoplication in this patient are her:
Young age
Persistent symptoms of reflux despite maximal proton-pump inhibitor (PPI) therapy
Ongoing respiratory symptoms
The use of laparoscopic fundoplication has expanded the range of patients who are now able
to undergo surgery. Laparoscopic fundoplication is said to lead to resolution of symptoms in
more than 90% of patients who undergo the procedure.
18593

Next Question

Previous Question

Tag Question

https://mypastest.pastest.com/Secure/TestMe/Browser/429893#Top

Feedback

End Review
1/2

8/22/2016

MyPastest

Difficulty: Average
Peer Responses

Session Progress
Responses Correct:

Responses Incorrect:

298

Responses Total:

298

Responses - % Correct:

0%

Blog (https://www.pastest.com/blog) About Pastest (https://www.pastest.com/about-us)


Contact Us (https://www.pastest.com/contact-us) Help (https://www.pastest.com/help)
Pastest 2016

https://mypastest.pastest.com/Secure/TestMe/Browser/429893#Top

2/2

8/22/2016

MyPastest

Back to Filters (/Secure/TestMe/Filter/429893/QA)

Question 138 of 298

A 60-year-old woman is concerned about her risk of osteoporosis and wishes to make
adjustments to her diet to increase her intake of vitamin D. She is already taking calcium
supplements but wants to know which foods are good natural sources of vitamin D.
Which one of the following foodstuffs would you advise her to eat more of?
A

Herring

Eggs

Green vegetables

Red meat

Eels

Explanation
Vitamin D
Causes of vitamin D deficiency include:
Inadequate sun exposure
Fat malabsorption
Liver and kidney failure
Inherited disorders of vitamin D metabolism (rare)
Food sources of vitamin D
3 oz of herring provides over 1300 U of vitamin D.
3.5 oz eel provides only about 200 U vitamin D.
Eggs, red meat and green vegetables contain less vitamin D than herring or eel.
Mushrooms are an efficient source of vitamin D2, particularly if they are exposed to
ultraviolet light after harvesting. Weight for weight they contain about twice as much
vitamin D as herring.
18594

https://mypastest.pastest.com/Secure/TestMe/Browser/429893#Top

1/2

8/22/2016

MyPastest

18594

Next Question

Previous Question

Tag Question

Feedback

End Review

Difficulty: Difficult
Peer Responses

Session Progress
Responses Correct:

Responses Incorrect:

298

Responses Total:

298

Responses - % Correct:

0%

Blog (https://www.pastest.com/blog) About Pastest (https://www.pastest.com/about-us)


Contact Us (https://www.pastest.com/contact-us) Help (https://www.pastest.com/help)
Pastest 2016

https://mypastest.pastest.com/Secure/TestMe/Browser/429893#Top

2/2

8/22/2016

MyPastest

Back to Filters (/Secure/TestMe/Filter/429893/QA)

Question 139 of 298

A 45-year-old woman presents with night sweats, right upper quadrant abdominal pain,
weight loss and anorexia. A scan reveals a liver abscess consistent with bacterial infection.
Past history of note includes Crohns disease, which might be a possible contributing cause.
She is known to be allergic to penicillin, and she tells you she had a rash as a child after taking
amoxicillin.
Which of the following intravenous antibiotic regimens would be most appropriate as an
empirical regimen?
A

Clindamycin + metronidazole

Clindamycin + ciprofloxacin

Vancomycin + meropenem

Ceftriaxone + metronidazole

Azithromycin + clindamycin

Explanation
Antibiotic treatment of liver abscess
Many different organisms can cause liver abscess, but the most commonly implicated are
Gram-negative bacteria, Gram-positive anaerobes and other anaerobic organisms, so the
initial choice of antibiotic treatment should reflect this. Ideally, a penicillin-based -lactamase
antibiotic combined with metronidazole to provide anaerobic cover would be the treatment
of choice.
In penicillin-allergic patients, substituting the penicillin-based antibiotic with a cephalosporin
would be a reasonable choice. Local hospital prescribing policies vary according to resistance
patterns, however, and the choice suggested here would not necessarily be used in every
hospital. In addition, cephalosporins as a group are less used than they were in the past
because of the associated risk of Clostridium difficile colitis. The duration of the antibiotic
course is also the subject of debate, but between 2 weeks and 6 weeks of therapy are
recommended after percutaneous drainage of the abscess.
18595

Next Question
https://mypastest.pastest.com/Secure/TestMe/Browser/429893#Top

1/2

8/22/2016

MyPastest

Previous Question

Tag Question

Feedback

End Review

Difficulty: Average
Peer Responses

Session Progress
Responses Correct:

Responses Incorrect:

298

Responses Total:

298

Responses - % Correct:

0%

Blog (https://www.pastest.com/blog) About Pastest (https://www.pastest.com/about-us)


Contact Us (https://www.pastest.com/contact-us) Help (https://www.pastest.com/help)
Pastest 2016

https://mypastest.pastest.com/Secure/TestMe/Browser/429893#Top

2/2

8/22/2016

MyPastest

Back to Filters (/Secure/TestMe/Filter/429893/QA)

Question 140 of 298

A 53-year-old woman presents with upper gastrointestinal haemorrhage. She has a history of
rheumatoid arthritis, which is managed with low-dose prednisolone, diclofenac and codeine
phosphate. On examination in the Emergency Department her blood pressure is
90/60 mmHg and her pulse 100 bpm. You fluid-resuscitate her and her blood pressure
improves to 115/80 mmHg, with a pulse of 80 bpm.
Investigations show: haemoglobin 10.4 g/dl, white cell count 6.1 109/l, platelets 145 109/l,
sodium 139 mmol/l, potassium 4.9 mmol/l, creatinine 180 mol/l. Electrocardiography (ECG)
shows lateral ST depression and upper gastrointestinal endoscopy reveals a large bleeding
ulcer on the posterior aspect of the duodenum. It cannot be easily reached with the
endoscope and you decide to attempt embolisation.
Which of the following arteries should be targeted for this procedure
A

Splenic artery

Gastroduodenal artery

Gastroepiploic artery

Anterior superior pancreaticoduodenal artery

Posterior superior pancreaticoduodenal artery

Explanation
Embolisation treatment for bleeding duodenal ulcer
This woman is at great risk from her bleeding duodenal ulcer, She has lateral ST depression
that is suggestive of myocardial ischaemia related to her fluid status. The ulcer cannot be
easily reached with the endoscope and her status would preclude surgery. The best option is
therefore embolisation by a skilled interventional radiologist. This portion of the duodenum is
supplied by the posterior superior pancreaticoduodenal artery. (The gastroepiploic artery
primarily supplies the stomach.)
18750

Next Question

https://mypastest.pastest.com/Secure/TestMe/Browser/429893#Top

1/2

8/22/2016

MyPastest

Previous Question

Tag Question

Feedback

End Review

Difficulty: Difficult
Peer Responses

Session Progress
Responses Correct:

Responses Incorrect:

298

Responses Total:

298

Responses - % Correct:

0%

Blog (https://www.pastest.com/blog) About Pastest (https://www.pastest.com/about-us)


Contact Us (https://www.pastest.com/contact-us) Help (https://www.pastest.com/help)
Pastest 2016

https://mypastest.pastest.com/Secure/TestMe/Browser/429893#Top

2/2

8/22/2016

MyPastest

Back to Filters (/Secure/TestMe/Filter/429893/QA)

Question 141 of 298

A 54-year-old woman with a history of cholelithiasis and jaundice undergoes endoscopic


retrograde cholangiopancreatography (ERCP) with sphincterotomy, (2 stones thought to
have been causing obstruction were removed). During the postoperative period she begins to
suffer from severe epigastric pain. Investigations show: haemoglobin 12.1 g/dl, white cell count
14.1 109/l, platelets 295 109/l, sodium 139 mmol/l, potassium 4.0 mmol/l, creatinine 175
mol/l, amylase 985 U/l, alanine aminotransferase (ALT) 120 U/l, alkaline phosphatase 480
U/l, bilirubin 140 mol/l.
Which of the following represents the most appropriate management of this patient?
A

Proceed to cholecystectomy

Insert a percutaneous drain

Intravenous fluids, analgesia and antibiotics

Computed tomography-guided pancreatic aspiration

Proceed to further ERCP

Explanation
Post-ERCP pancreatitis
In the first instance, this woman who has developed pancreatitis following endoscopic
retrograde cholangiopancreatography (post-ERCP pancreatitis) should be managed
medically with intravenous rehydration, analgesia and antibiotics with activity against likely
pathogens (eg ciprofloxacin or a cephalosporin). ERCP is probably the third commonest
cause of pancreatitis after alcohol and biliary tract disease. It might be related to inadequate
hydration at the time of the procedure or to attempts to measure the pressure at the
sphincter of Oddi.
20461

Next Question

Previous Question

Tag Question

https://mypastest.pastest.com/Secure/TestMe/Browser/429893#Top

Feedback

End Review
1/2

8/22/2016

MyPastest

Difficulty: Easy
Peer Responses

Session Progress
Responses Correct:

Responses Incorrect:

298

Responses Total:

298

Responses - % Correct:

0%

Blog (https://www.pastest.com/blog) About Pastest (https://www.pastest.com/about-us)


Contact Us (https://www.pastest.com/contact-us) Help (https://www.pastest.com/help)
Pastest 2016

https://mypastest.pastest.com/Secure/TestMe/Browser/429893#Top

2/2

8/22/2016

MyPastest

Back to Filters (/Secure/TestMe/Filter/429893/QA)

Question 142 of 298

A 34-year-old woman with a history of type 1 diabetes presents to the clinic with increasing
tiredness, mild upper abdominal discomfort, and itching. The GP has arranged some
investigations prior to her clinic visit, given her type 1 diabetes including an autoimmune
profile and an ESR. The results show: haemoglobin 13.1 g/dl, white cell count 5.0 109/l,
platelets 235 109/l, sodium 140 mmol/l, potassium 4.9 mmol/l, creatinine 90 mol/l, HbA1c
67.21mmol/mol (8.3%), ESR 65 mm/1st hour, immunoglobulins increased and anti-smooth
muscle antibody positive.
Which of the following would be the next appropriate investigation?
A

Hepatic ultrasound scan

Liver function testing

Creatine kinase

Short Synacthen test

Thyroid function testing

Explanation
Autoimmune hepatitis
The history is suggestive of autoimmune hepatitis, with anti-smooth muscle antibodies being
a finding in type I disease; elevation in immunoglobulin levels is also seen, as is antinuclear
antibody positivity. More women than men are affected (78% patients are women). About 41%
of people with type I autoimmune hepatitis have co-existent autoimmune pathology. The
disease is highly steroid-responsive but about 45% of patients will progress to cirrhosis.
20462

Next Question

Previous Question

Tag Question

Feedback

End Review

Difficulty: Average
https://mypastest.pastest.com/Secure/TestMe/Browser/429893#Top

1/2

8/22/2016

MyPastest

Peer Responses

Session Progress
Responses Correct:

Responses Incorrect:

298

Responses Total:

298

Responses - % Correct:

0%

Blog (https://www.pastest.com/blog) About Pastest (https://www.pastest.com/about-us)


Contact Us (https://www.pastest.com/contact-us) Help (https://www.pastest.com/help)
Pastest 2016

https://mypastest.pastest.com/Secure/TestMe/Browser/429893#Top

2/2

8/22/2016

MyPastest

Back to Filters (/Secure/TestMe/Filter/429893/QA)

Question 143 of 298

A 22-year-old woman presents to the Gastroenterology Clinic with worsening diarrhoea. She
has a long history of Crohns disease and had an extensive ileal resection 2 years ago. She has
presented on two previous occasions in the past 3 months with renal stones. Investigations
show: haemoglobin 10.5 g/dl, mean corpuscular volume (MCV)109 fl, white cell count
5.4 109/l, platelets 295 109/l, erythrocyte sedimentation rate (ESR) 12 mmin 1 hour, sodium
141 mmol/l, potassium 3.5 mmol/l, creatinine 100 mol/l, albumin 30 g/l. Which of the
following is the most likely diagnosis?
A

Bacterial overgrowth

Worsening small-bowel Crohn's disease

Fat malabsorption

Short-bowel syndrome

Extension of Crohns to involve the colon

Explanation
Short-bowel syndrome
With an erythrocyte sedimentation rate (ESR) of 12 mm in 1 hour, this patient has no signs of
active inflammation. This makes a worsening of the Crohns unlikely. Calcium oxalate renal
stones occur with increased frequency in patients with short-bowel syndrome and, given the
history of extensive ileal resection, this is the most likely diagnosis here.
Management
Maintaining adequate hydration is useful in avoiding renal stones and periods of total
parenteral nutrition (TPN) might be required to maintain nutrition.
Vitamin supplementation is another important component of management.
As the diarrhoea is usually bile acid-related, colestyramine is often used to reduce the
volume.
20463

https://mypastest.pastest.com/Secure/TestMe/Browser/429893#Top

1/2

8/22/2016

MyPastest

Next Question

Previous Question

Tag Question

Feedback

End Review

Difficulty: Average
Peer Responses

Session Progress
Responses Correct:

Responses Incorrect:

298

Responses Total:

298

Responses - % Correct:

0%

Blog (https://www.pastest.com/blog) About Pastest (https://www.pastest.com/about-us)


Contact Us (https://www.pastest.com/contact-us) Help (https://www.pastest.com/help)
Pastest 2016

https://mypastest.pastest.com/Secure/TestMe/Browser/429893#Top

2/2

8/22/2016

MyPastest

Back to Filters (/Secure/TestMe/Filter/429893/QA)

Question 144 of 298

A 49-year-old patient with a history of previous surgery for a bleeding duodenal ulcer and
recurrences on omeprazole therapy comes to the clinic. You review his results, including a
gastrin level. Investigations show: haemoglobin 11.2 g/dl, white cell count 5.3 109/l, platelets
145 109/l, sodium 139 mmol/l, potassium 4.8 mmol/l, creatinine 105 mol/l, alanine
aminotransferase (ALT) 54 U/l, gastrin 128 pg/ml (high). Which of the following statements
about gastrin is true?
A

It is secreted by the parietal cells in the stomach

It is inhibited by pancreatic bicarbonate

It is produced from pancreatic A cells

It is produced from pancreatic B cells

Release is triggered by gastrointestinal luminal peptides

Explanation
Gastrin
Gastirin release is stimulated by:
Gastrointestinal luminal peptides
Stomach distension
Vagal stimulation
Hypercalcaemia
Gastrin is produced by G cells in the duodenum and in the pyloric antrum of the stomach. Its
role is to stimulate parietal cells to secrete gastric acid and to stimulate the secretion of
pepsinogen by chief cells. Gastrin is under negative feedback, and so release is inhibited by
increased acidity. In a patient with recurrent duodenal ulceration, a finding of high levels of
gastrin raises the possibility of ZollingerEllison syndrome (due to a gastrinoma).
20754

Next Question
https://mypastest.pastest.com/Secure/TestMe/Browser/429893#Top

1/2

8/22/2016

MyPastest

Previous Question

Tag Question

Feedback

End Review

Difficulty: Average
Peer Responses

Session Progress
Responses Correct:

Responses Incorrect:

298

Responses Total:

298

Responses - % Correct:

0%

Blog (https://www.pastest.com/blog) About Pastest (https://www.pastest.com/about-us)


Contact Us (https://www.pastest.com/contact-us) Help (https://www.pastest.com/help)
Pastest 2016

https://mypastest.pastest.com/Secure/TestMe/Browser/429893#Top

2/2

8/22/2016

MyPastest

Back to Filters (/Secure/TestMe/Filter/429893/QA)

Question 145 of 298

A 31-year-old woman who is 33 weeks pregnant with her first child comes to the Emergency
Department complaining of a severe headache and easy bruising. On examination she has a
blood pressure of 145/89 mmHg (compared with a booking blood pressure of 128/75 mmHg).
Her only medication is Gaviscon, which she was given a few weeks ago by her GP for
indigestion. Investigations show: haemoglobin 10.0 g/dl, white cell count 8.2 109/l, platelets
52 109/l, sodium 139 mmol/l, potassium 5.6 mmol/l, creatinine 160 mol/l, bilirubin 85
mol/l, ALT 450 U/l. Which of the following is the most appropriate management?
A

Plasma exchange

Prednisolone

Normal human immunoglobulin

Intravenous heparin

Magnesium sulphate

Explanation
HELLP syndrome in pregnancy
Although all of the options listed (apart from magnesium) might all be options for the
treatment of HELLP (haemolysis, elevated liver enzymes and low platelets) syndrome, the key
treatment in pregnancy is to plan for delivery. The immediate risk prior to obstetric review is
one of eclampsia-related seizures, and so magnesium is given as an infusion pre-delivery,
during delivery and in the immediate postpartum period, regardless of blood pressure.
As long as the platelet count remains above 50 109/l, then caesarean section is still
considered an option; for a vaginal delivery, the minimum platelet count is 20 109/l. Two or
three trials have looked at the effect of steroids in managing the condition, but have shown
no positive effect on outcomes.
20755

Next Question

Previous Question

Tag Question

https://mypastest.pastest.com/Secure/TestMe/Browser/429893#Top

Feedback

End Review
1/2

8/22/2016

MyPastest

Difficulty: Average
Peer Responses

Session Progress
Responses Correct:

Responses Incorrect:

298

Responses Total:

298

Responses - % Correct:

0%

Blog (https://www.pastest.com/blog) About Pastest (https://www.pastest.com/about-us)


Contact Us (https://www.pastest.com/contact-us) Help (https://www.pastest.com/help)
Pastest 2016

https://mypastest.pastest.com/Secure/TestMe/Browser/429893#Top

2/2

8/22/2016

MyPastest

Back to Filters (/Secure/TestMe/Filter/429893/QA)

Question 146 of 298

You see a 40-year-old patient with Crohn's disease who has been suffering with diarrhoea >6
times/day, which is unresponsive to steroids and mesalazine (which he has been taking for 3
weeks). Results show haemoglobin 10.4 g/dl, white cell count 12.1 x109/l, platelets 380 x109/l,
sodium 139 mmol/l, potassium 4.0 mmol/l, creatining 150 mol/l, albumin 30 g/l and ESR 65
mm/hr.
What is the most appropriate next treatment?
A

Azathioprine

Infliximab

Methotrexate

Surgery

Cyclophosphamide

Explanation
Treatment of Crohns disease
Azathioprine is recommended as treatment for Crohns disease by the British Society of
Gastroenterology in patients who have failed to achieve control on steroids and 5-ASA
compounds. The Cochrane review cited in the guidance suggests relative odds of 2.32 for
achieving remission versus placebo. Infliximab use is for the time being limited by guidance
described in the NICE technology appraisal, which limits its use to patients who have failed
on 5-ASA, corticosteroids and immunomodulatory therapy. Study evidence suggests that
anti-TNF agents are highly effective in the treatment of Crohns and its use may become
more widespread as experience increases.
20756

Next Question

Previous Question

Tag Question

https://mypastest.pastest.com/Secure/TestMe/Browser/429893#Top

Feedback

End Review

1/2

8/22/2016

MyPastest

Difficulty: Average
Peer Responses

Session Progress
Responses Correct:

Responses Incorrect:

298

Responses Total:

298

Responses - % Correct:

0%

Blog (https://www.pastest.com/blog) About Pastest (https://www.pastest.com/about-us)


Contact Us (https://www.pastest.com/contact-us) Help (https://www.pastest.com/help)
Pastest 2016

https://mypastest.pastest.com/Secure/TestMe/Browser/429893#Top

2/2

8/22/2016

MyPastest

Back to Filters (/Secure/TestMe/Filter/429893/QA)

Question 147 of 298

A 60-year-old man comes to the clinic. He has had worsening dysphagia for the past 6
months. At first this was just for solids such as toast, but he is now having increasing
difficulty even with swallowing soup. He has lost approximately 6 kg in weight over the past 2
months. He smokes 20 cigarettes per day and drinks two glasses of whisky each evening. He
has been treated for a hiatus hernia with omeprazole for 6 years (though he has suffered
from indigestion for nearly 20 years).
Investigations show: haemoglobin 10.9 g/dl, white cell count 5.4 109/l, platelets 180 109/l,
erythrocyte sedimentation rate (ESR) 42 mmin 1 hour, sodium 139 mmol/l, potassium 4.5
mmol/l, creatinine 130 mol/l. A chest X-ray shows a fluid level behind the heart.
What is the most likely diagnosis driving his recent deterioration?
A

Achalasia

Oesophageal carcinoma

Oesophageal diverticulum

Pharyngeal pouch

Rolling hiatus hernia

Explanation
Oesophageal carcinoma
Although this man has had a very long history of symptoms of gastro-oesophageal reflux
disease (GORD) and a hiatus hernia, with x-ray changes, the history of weight loss and
worsening dysphagia should raise significant suspicions of oesophageal carcinoma. Further
suspicion is raised by the fact that he is anaemic and has a raised ESR. Endoscopy with
biopsy is the investigation of choice for this patient. Long-term GORD is the most common
predisposing factor for oesophageal carcinoma.
20757

Next Question

https://mypastest.pastest.com/Secure/TestMe/Browser/429893#Top

1/2

8/22/2016

MyPastest

Previous Question

Tag Question

Feedback

End Review

Difficulty: Average
Peer Responses

Session Progress
Responses Correct:

Responses Incorrect:

298

Responses Total:

298

Responses - % Correct:

0%

Blog (https://www.pastest.com/blog) About Pastest (https://www.pastest.com/about-us)


Contact Us (https://www.pastest.com/contact-us) Help (https://www.pastest.com/help)
Pastest 2016

https://mypastest.pastest.com/Secure/TestMe/Browser/429893#Top

2/2

8/22/2016

MyPastest

Back to Filters (/Secure/TestMe/Filter/429893/QA)

Question 148 of 298

A 61-year-old man presents with a 1-year history of intermittent difficulty with swallowing and
halitosis. He also feels like there is a fullness in his neck. Occasionally he even regurgitates
undigested food. He is diabetic and is currently taking metformin. Other past history of note
includes recurrent bouts of pneumonia (two in the past 3 years) and asthma, which was
recently diagnosed by his GP. On examination, he looks well and his body mass index (BMI) is
32 kg/m2. Investigations show: haemoglobin 12.1 g/dl, white cell count 5.2 109/l, platelets 190
109/l, sodium 139 mmol/l, potassium 4.8 mmol/l, creatinine 135 mol/l. Oesophageal
pressure studies are unremarkable.
Which of the following is the most likely diagnosis?
A

Pharyngeal pouch

Hiatus hernia

Oesophageal carcinoma

Barrett's oesophagus

Oesophageal candidiasis

Explanation
Pharyngeal pouch
We are given every indication that this is a chronic problem. He has suffered for at least 2
years and is overweight. The history of intermittent regurgitation of undigested food and
halitosis raises the possibility of a pharyngeal pouch or severe reflux disease. The presence of
normal manometry studies reduces the likelihood of reflux. Barium swallow to demonstrate
the pouch is the radiological investigation of choice. Where there are significant symptoms,
excision is a possible treatment and this procedure has an operative mortality of around 1% in
case series.
20758

Next Question

Previous Question

Tag Question

https://mypastest.pastest.com/Secure/TestMe/Browser/429893#Top

Feedback

End Review
1/2

8/22/2016

MyPastest

Difficulty: Easy
Peer Responses

Session Progress
Responses Correct:

Responses Incorrect:

298

Responses Total:

298

Responses - % Correct:

0%

Blog (https://www.pastest.com/blog) About Pastest (https://www.pastest.com/about-us)


Contact Us (https://www.pastest.com/contact-us) Help (https://www.pastest.com/help)
Pastest 2016

https://mypastest.pastest.com/Secure/TestMe/Browser/429893#Top

2/2

8/22/2016

MyPastest

Back to Filters (/Secure/TestMe/Filter/429893/QA)

Question 149 of 298

A 62-year-old woman comes to the clinic complaining of problems swallowing both liquids
and solids over the past few months. She says this is associated with retrosternal chest pain
and she tends to regurgitate both liquids and solids. There has been gradual weight loss.
Otherwise she feels well and has no significant past medical history.
On examination, she looks well, her blood pressure is 142/84 mmHg and she has no
lymphadenopathy. Abdominal examination is normal. Her body mass index (BMI) is 27 kg/m2.
Investigations show: haemoglobin 12.9 g/dl, white cell count 5.4 109/l, platelets 210 109/l,
sodium 141 mmol/l, potassium 5.0 mmol/l, creatinine 120 mol/l, albumin 39 g/l, alanine
aminotransferase (ALT) 17 U/l, alkaline phosphatase (ALP) 85 U/l, viscosity 1.7 mPa/s (normal
range 1.51.72 mPa/s). Barium swallow shows a dilated oesophagus with a tapering birds
beak appearance at the distal end.
Which of the following is the most likely diagnosis?
A

Severe oesophageal reflux

Oesophageal carcinoma

Barretts oesophagus

Achalasia

Hiatus hernia

Explanation
Achalasia
The mixed picture of dysphagia to both liquids and solids, rather than progressive solid to
liquid dysphagia over time is most suggestive of achalasia. Further supporting the diagnosis
is the normal blood picture, preserved body mass and typical barium swallow appearance.
Pneumatic dilation and botulinum toxin injection are the mainstays of therapy for achalasia. If
patients are unable to undergo this procedure, then long-term therapy with a dihydropiridine
calcium antagonist might offer some relief.
20922

Next Question

https://mypastest.pastest.com/Secure/TestMe/Browser/429893#Top

1/2

8/22/2016

MyPastest

Previous Question

Tag Question

Feedback

End Review

Difficulty: Easy
Peer Responses

Session Progress
Responses Correct:

Responses Incorrect:

298

Responses Total:

298

Responses - % Correct:

0%

Blog (https://www.pastest.com/blog) About Pastest (https://www.pastest.com/about-us)


Contact Us (https://www.pastest.com/contact-us) Help (https://www.pastest.com/help)
Pastest 2016

https://mypastest.pastest.com/Secure/TestMe/Browser/429893#Top

2/2

8/22/2016

MyPastest

Back to Filters (/Secure/TestMe/Filter/429893/QA)

Question 150 of 298

A 62-year-old man presents with dysphagia. He reports epigastric pain that has worsened
over the past 4 months and is also worried that he is losing weight. There is a history of
20 units of alcohol consumption per week and he smokes 20 cigarettes per day. On
examination, he looks thin, his body mass index (BMI) is 21 kg/m2 and there is some
tenderness on palpation of the epigastrium. Investigations show: haemoglobin 10.4 g/dl,
white cell count 5.1 109/l, platelets 130 109/l, sodium 140 mmol/l, potassium 4.9 mmol/l,
creatinine 131 mol/l.
Which of the following is the most appropriate next step for him?
A

Helicobacter pylori screen

Abdominal ultrasound scan

Barium swallow

Upper gastrointestinal endoscopy

Gastrin levels

Explanation
Investigation of dysphagia and weight loss
This man has symptoms of upper gastrointestinal pathology, with sinister features including
dysphagia, weight loss and anaemia. Given his history of smoking and alcohol consumption,
there is a greatly increased likelihood of his symptoms being due to oesophageal carcinoma
and so gastrointestinal endoscopy would be the most appropriate investigation. Although
barium swallow might demonstrate a typical stricture or might be useful in diagnosing
achalasia, it is not as sensitive as upper gastrointestinal endoscopy for the detection of an
underlying tumour.
20923

Next Question

Previous Question

Tag Question

https://mypastest.pastest.com/Secure/TestMe/Browser/429893#Top

Feedback

End Review
1/2

8/22/2016

MyPastest

Difficulty: Easy
Peer Responses

Session Progress
Responses Correct:

Responses Incorrect:

298

Responses Total:

298

Responses - % Correct:

0%

Blog (https://www.pastest.com/blog) About Pastest (https://www.pastest.com/about-us)


Contact Us (https://www.pastest.com/contact-us) Help (https://www.pastest.com/help)
Pastest 2016

https://mypastest.pastest.com/Secure/TestMe/Browser/429893#Top

2/2

8/22/2016

MyPastest

Back to Filters (/Secure/TestMe/Filter/429893/QA)

Question 151 of 298

A 62-year-old man with known aortic stenosis who has a valve gradient of 40 mmHg
presents to the clinic with worsening tiredness. He currently has no symptoms of heart failure
or syncope but is under active follow-up for his aortic stenosis. He currently takes amlodipine
5 mg for his associated hypertension.
On examination he looks pale, his blood pressure is 155/100 mmHg, his pulse is 80 bpm and
he has an ejection systolic murmur. He is not in cardiac failure. Investigations show:
haemoglobin 9.7 g/dl, mean corpuscular volume (MCV) 78 fl, white cell count 5.4 109/l,
platelets 130 109/l, sodium 141 mmol/l, potassium 4.7 mmol/l, creatinine 100 mol/l. Upper
gastrointestinal endoscopy was normal.
Which of the following investigations is most likely to provide an accurate diagnosis?
A

Colonoscopy

Iron studies

Mesenteric angiography

Small-bowel follow-through

Serum haptoglobins

Explanation
Heyde syndrome
Clinicians have argued for years about whether Heyde syndrome (calcific aortic stenosis and
angiodysplasia of the bowel) actually exists but, in practice, 1040% of patients with aortic
stenosis experience some form of gastrointestinal bleeding. A connective tissue disorder
might be common antecedent linking calcified valvular disease and angiodysplasia.
Investigation
Colonoscopy is effective in picking up the angiodysplastic lesions in the majority of patients.
Mesenteric angiography is most effective when there is very active bleeding. If investigations
are negative and the diagnosis is still suspected, then radionucleotide studies can identify
bleeding over a more prolonged period.
20947

https://mypastest.pastest.com/Secure/TestMe/Browser/429893#Top

1/2

8/22/2016

MyPastest

Next Question

Previous Question

Tag Question

Feedback

End Review

Difficulty: Average
Peer Responses

Session Progress
Responses Correct:

Responses Incorrect:

298

Responses Total:

298

Responses - % Correct:

0%

Blog (https://www.pastest.com/blog) About Pastest (https://www.pastest.com/about-us)


Contact Us (https://www.pastest.com/contact-us) Help (https://www.pastest.com/help)
Pastest 2016

https://mypastest.pastest.com/Secure/TestMe/Browser/429893#Top

2/2

8/22/2016

MyPastest

Back to Filters (/Secure/TestMe/Filter/429893/QA)

Question 152 of 298

An elderly man who underwent an emergency partial gastrectomy 9 months ago is referred
by his GP with symptoms of abdominal bloating, mild abdominal distension, flatulence and
intermittent diarrhoea. On examination, he looks pale, his blood pressure is 135/70 mmHg and
his pulse is 85 bpm and regular. General examination is unremarkable apart from a midline
scar consistent with his partial gastrectomy. Investigations show: haemoglobin 10.0 g/dl,
mean corpuscular volume (MCV) 104 fl, white cell count 5.9 109/l, platelets 145 109/l,
sodium 141 mmol/l, potassium 4.8 mmol/l, creatinine 100 mol/l.
Which of the following is the most appropriate next investigation?
A

Barium follow-through

Hydrogen breath test

Endoscopy of the gastric remnant

Prolonged glucose tolerance test

Schilling test

Explanation
Bacterial overgrowth syndrome
The symptoms seen here are highly suspicious of bacterial overgrowth syndrome. Hydrogen
breath testing is non-invasive and has a specificity of 80% and a sensitivity of up to 75% for
diagnosing the condition. Barium follow-through might be useful to demonstrate strictures,
malrotation or pseudo-obstruction.
Treatment
Co-amoxiclav is a useful first-line choice treatment for bacterial overgrowth syndrome
relating to anatomical abnormalities post-surgery and metronidazole would be a reasonable
option in elderly patients with idiopathic bacterial overgrowth syndrome.
20962

Next Question

https://mypastest.pastest.com/Secure/TestMe/Browser/429893#Top

1/2

8/22/2016

MyPastest

Previous Question

Tag Question

Feedback

End Review

Difficulty: Average
Peer Responses

Session Progress
Responses Correct:

Responses Incorrect:

298

Responses Total:

298

Responses - % Correct:

0%

Blog (https://www.pastest.com/blog) About Pastest (https://www.pastest.com/about-us)


Contact Us (https://www.pastest.com/contact-us) Help (https://www.pastest.com/help)
Pastest 2016

https://mypastest.pastest.com/Secure/TestMe/Browser/429893#Top

2/2

8/22/2016

MyPastest

Back to Filters (/Secure/TestMe/Filter/429893/QA)

Question 153 of 298

A 28-year-old man who has a history of perioral and buccal pigmentation, intermittent
gastrointestinal bleeding and multiple polyposis is diagnosed with PeutzJeghers syndrome.
He has met a partner and wants to start a family and they visit you for genetic counselling.
What is the usual inheritance pattern for PeutzJeghers syndrome?
A

Autosomal dominant

Autosomal recessive

X-linked recessive

X-linked dominant

Mitochondrial

Explanation
PeutzJeghers syndrome
The cause of PeutzJeghers syndrome appears to be a mutation in the STK11/LKB1 gene (the
serine/threonine kinase 11 gene) in most cases, a gene that is located on chromosome 19. The
gene has variable penetrance, so the pattern of polyposis and cancers can vary between
family groups and between patients within families.
Around 50% of people with PeutzJeghers syndrome have developed gastrointestinal
carcinoma by the time they get to the age of 57. Regular screening is required, and upper and
lower gastrointestinal endsocopy, pancreatic ultrasound, testicular ultrasound and smallbowel radiography have been suggested as being indicated.
20963

Next Question

Previous Question

Tag Question

Feedback

End Review

Difficulty: Average
https://mypastest.pastest.com/Secure/TestMe/Browser/429893#Top

1/2

8/22/2016

MyPastest

Peer Responses

Session Progress
Responses Correct:

Responses Incorrect:

298

Responses Total:

298

Responses - % Correct:

0%

Blog (https://www.pastest.com/blog) About Pastest (https://www.pastest.com/about-us)


Contact Us (https://www.pastest.com/contact-us) Help (https://www.pastest.com/help)
Pastest 2016

https://mypastest.pastest.com/Secure/TestMe/Browser/429893#Top

2/2

8/22/2016

MyPastest

Back to Filters (/Secure/TestMe/Filter/429893/QA)

Question 154 of 298

A 29-year-old woman comes to the Gastroenterology Clinic for review. She underwent a
partial bowel resection 2 years ago for Crohns disease and has an ileostomy. On examination
she has a purple-coloured ulcerated lesion on the edge of the stoma at around 9 o'clock. The
lesion itself is 3 cm in diameter and extends into both the muscle layer of the edge of the
stoma and into surrounding skin. Investigations show: haemoglobin 10.5 g/dl, white cell count
7.2 109/l, platelets 180 109/l, C-reactive protein (CRP) 18 mg/l (normal range < 10 mg/l),
sodium 140 mmol/l, potassium 3.7 mmol/l, creatinine 120 mol/l.
Which of the following is the most likely diagnosis?
A

Reactivation of Crohns

Traumatic removal of stoma bag

Contact dermatitis

Pyoderma gangrenosum

Erythema nodosum

Explanation
Pyoderma gangrenosum
Pyoderma gangrenosum is described in patients with inflammatory bowel disease, in whom it
can occur at the stoma site. Histological examination demonstrates neutrophil infiltration,
haemorrhage, epidermal necrosis and, late in the process, granuloma formation. A number of
case reports indicate positive results after treatment with anti-tumour necrosis factor
antibodies (anti-TNF) such as adalimumab; other therapeutic options include systemic or
topical steroids, dapsone and ciclosporin.
Reactivation of Crohn's would be expected to lead to increased flow through the ileostomy as
well as granuloma formation, and we have no evidence of that here. Similarly traumatic
removal of the stoma bag or contact dermatitis are more likely to lead to local erythema /
ulceration rather than the much larger purple coloured lesion seen here. Erythema nodosum
commonly leads to changes on the shins.
20964

Next Question
https://mypastest.pastest.com/Secure/TestMe/Browser/429893#Top

1/2

8/22/2016

MyPastest

Previous Question

Tag Question

Feedback

End Review

Difficulty: Average
Peer Responses

Session Progress
Responses Correct:

Responses Incorrect:

298

Responses Total:

298

Responses - % Correct:

0%

Blog (https://www.pastest.com/blog) About Pastest (https://www.pastest.com/about-us)


Contact Us (https://www.pastest.com/contact-us) Help (https://www.pastest.com/help)
Pastest 2016

https://mypastest.pastest.com/Secure/TestMe/Browser/429893#Top

2/2

8/22/2016

MyPastest

Back to Filters (/Secure/TestMe/Filter/429893/QA)

Question 155 of 298

A 30-year-old woman who works in a pharmacy presents with chronic diarrhoea. She claims
that the problem is very debilitating and is preventing her from completing a normal day at
work. Clinical examination reveals a body mass index (BMI) of 18 kg/m2, but otherwise is
completely normal. Investigations show: haemoglobin 13.1 g/dl, white cell count 5.0 109/l,
platelets 190 109/l, sodium 141 mmol/l, potassium 2.5 mmol/l, creatinine 100 mol/l, Creactive protein (CRP) 8 mg/l (normal range < 10 mg/l. The stool chart (inpatient, fasting)
shows:
Day 1 320 g
Day 1 0 g
Day 3 115 g
Day 4 120 g
Colonoscopy showed evidence of melanosis coli, Which of the following is the most likely
diagnosis?
A

Coeliac disease

Irritable bowel syndrome

Microscopic colitis

VIPoma

Laxative abuse

Explanation
Laxative abuse
The clue here is the fact that this patient works in a pharmacy and will have ready access to
laxatives. The only abnormal finding on clinical examination is that she is underweight and the
only abnormal blood result is profound hypokalaemia. You might consider doing a urine
screen for laxative metabolites, but it is important that you inform her what you are doing.
Patients with a VIPoma (a vasoactive intestinal polypeptide-secreting tumour) tend to have a
much greater throughput of stool and other symptoms such as facial flushing, similar to those
with carcinoid syndrome.
20965

https://mypastest.pastest.com/Secure/TestMe/Browser/429893#Top

1/2

8/22/2016

MyPastest

20965

Next Question

Previous Question

Tag Question

Feedback

End Review

Difficulty: Easy
Peer Responses

Session Progress
Responses Correct:

Responses Incorrect:

298

Responses Total:

298

Responses - % Correct:

0%

Blog (https://www.pastest.com/blog) About Pastest (https://www.pastest.com/about-us)


Contact Us (https://www.pastest.com/contact-us) Help (https://www.pastest.com/help)
Pastest 2016

https://mypastest.pastest.com/Secure/TestMe/Browser/429893#Top

2/2

8/22/2016

MyPastest

Back to Filters (/Secure/TestMe/Filter/429893/QA)

Question 156 of 298

A 54-year-old publican is admitted via the GP surgery with a right-sided pneumonia. On


further questioning he admits to consuming up to 6 pints of strong lager per day and three
glasses of wine. He has been worrying recently about the profits of the pub and has tended
to increase his alcohol consumption. On examination you notice that he has ascites,
distended periumbilical veins (caput medusae) and peripheral oedema. Investigations show:
haemoglobin 10.4 g/dl, white cell count 11.4 109/l, platelets 100 109/l, sodium 141 mmol/l,
potassium 4.0 mmol/l, creatinine 100 mol/l, alanine aminotransferase (ALT) 120 U/l, bilirubin
190 mol/, albumin 29 g/l.
Which of the features below is most indicative of decompensated portal hypertension?
A

Ascites

Peripheral oedema

Raised bilirubin

Decreased albumin

Caput medusae

Explanation
Advanced alcoholic liver disease
The presence of caput medusae indicates that he has severe decompensated portal
hypertension and so he is likely to be at greatly increased risk from the associated
consequences (eg variceal bleeding). His low albumin, ascites, high bilirubin and peripheral
oedema are of course also markers of advanced cirrhosis. Strict avoidance of alcohol, and
treatment with propranolol and spironolactone are the key points of medical management in
this case, but even these measures are unlikely to have a very great effect on his long-term
prognosis.
21073

Next Question

Previous Question

Tag Question

https://mypastest.pastest.com/Secure/TestMe/Browser/429893#Top

Feedback

End Review
1/2

8/22/2016

MyPastest

Difficulty: Average
Peer Responses

Session Progress
Responses Correct:

Responses Incorrect:

298

Responses Total:

298

Responses - % Correct:

0%

Blog (https://www.pastest.com/blog) About Pastest (https://www.pastest.com/about-us)


Contact Us (https://www.pastest.com/contact-us) Help (https://www.pastest.com/help)
Pastest 2016

https://mypastest.pastest.com/Secure/TestMe/Browser/429893#Top

2/2

8/22/2016

MyPastest

Back to Filters (/Secure/TestMe/Filter/429893/QA)

Question 157 of 298

A 54-year-old patient presents to the clinic with indigestion. He denies excessive alcohol
intake and is a non-smoker. Past medical history of note includes mild asthma, which is
controlled with a low-dose seretide inhaler. On examination he looks well, his body mass
index (BMI) is 31 kg/m2 and his blood pressure is 142/80 mmHg. There is mild epigastric
tenderness. Investigations show: haemoglobin 11.0 g/dl, white cell count 6.7 109/l, platelets
185 109/l, sodium 141 mmol/l, potassium 4.4 mmol/l, creatinine 110 mol/l. Biopsy of a
suspicious area in stomach shows a low-grade MALT lymphoma.
Which of the following is the most appropriate management of this patient?
A

Gastrectomy

Partial gastrectomy

Long-term omeprazole therapy

Helicobacter pylori eradication

Oral imatinib

Explanation
MALT lymphoma
More than 90% of gastric MALT lymphomas (MALT = mucosa-associated lymphoid tissue) are
associated with Helicobacter pylori infection and H. pylori eradication is associated with
regression of the lymphoma in up to 80% of cases. Genotyping to look for the t (11;18)
(q21;q21) translocation can help predict response to therapy, with the presence of this
translocation indicating a poorer response to H. pylori eradication. Gastrectomy is rarely
required because combination chemotherapy, together with the addition of biological agents
such as imatinib or rituximab, with or without limited radiotherapy, is often sufficient to
achieve remission.
21111

Next Question

Previous Question

Tag Question

https://mypastest.pastest.com/Secure/TestMe/Browser/429893#Top

Feedback

End Review
1/2

8/22/2016

MyPastest

Difficulty: Easy
Peer Responses

Session Progress
Responses Correct:

Responses Incorrect:

298

Responses Total:

298

Responses - % Correct:

0%

Blog (https://www.pastest.com/blog) About Pastest (https://www.pastest.com/about-us)


Contact Us (https://www.pastest.com/contact-us) Help (https://www.pastest.com/help)
Pastest 2016

https://mypastest.pastest.com/Secure/TestMe/Browser/429893#Top

2/2

8/22/2016

MyPastest

Back to Filters (/Secure/TestMe/Filter/429893/QA)

Question 158 of 298

A 32-year-old man has a follow-up endoscopy 6 months after undergoing emergency surgery
for a bleeding duodenal ulcer. He drinks no alcohol and is a non-smoker. The repeat
endoscopy reveals a large new ulcer in the first part of the duodenum and two or three
smaller ulcers. He has been taking long-term omeprazole therapy during the intervening few
months.
On examination he looks slim and his body mass index (BMI) is 21 kg/m2. Investigations show:
haemoglobin 10.2 g/dl, mean corpuscular volume (MCV) 89 fl, white cell count 5.0 109/l,
platelets 183 109/l, sodium 141 mmol/l, potassium 4.9 mmol/l, creatinine 110 mol/l.
Which of the following is the most appropriate next step?
A

Measure gastrin levels

Start iron tablets

Repeat oesophagogastroduodenoscopy in 6 months

Screen for Helicobacter pylori

Arrange pH studies

Explanation
Gastrinoma
The suspicion of gastrinoma is raised by the presence of recurrent duodenal ulceration
despite long-term omeprazole therapy, especially as this patient appears to have no other
precipitating factors for ulceration (eg excess alcohol consumption or smoking). Fasting
serum gastrin measurement is the most sensitive investigation, coupled with detection of
increased basal acid output and a positive secretion stimulation test. Around 50% of
gastrinomas are malignant and hepatic metastases at the point of diagnosis are associated
with a very poor prognosis.
21255

Next Question

https://mypastest.pastest.com/Secure/TestMe/Browser/429893#Top

1/2

8/22/2016

MyPastest

Previous Question

Tag Question

Feedback

End Review

Difficulty: Average
Peer Responses

Session Progress
Responses Correct:

Responses Incorrect:

298

Responses Total:

298

Responses - % Correct:

0%

Blog (https://www.pastest.com/blog) About Pastest (https://www.pastest.com/about-us)


Contact Us (https://www.pastest.com/contact-us) Help (https://www.pastest.com/help)
Pastest 2016

https://mypastest.pastest.com/Secure/TestMe/Browser/429893#Top

2/2

8/22/2016

MyPastest

Back to Filters (/Secure/TestMe/Filter/429893/QA)

Question 159 of 298

A 29-year-old man returns from a holiday in India complaining of fever, diarrhoea and
dizziness on standing. He has eaten widely from a number of places during his holiday, and
has eaten local meat and fish dishes from street food sellers. On examination he is pyrexial
(38 C) and looks dehydrated. His blood pressure is 120/70 mmHg with a significant postural
drop and his pulse is 98 bpm and regular. He has abdominal tenderness, especially in the
right iliac fossa. You also notice erythema nodosum. He has a second bag of 1 litre of normal
saline running. Investigations show: haemoglobin 14.3 g/dl, white cell count 12.3 109/l,
platelets 200 109/l, sodium 145 mmol/l, potassium 3.2 mmol/l, creatinine 184 mol/l.
Given the suspected diagnosis, which of the following would be the most appropriate
treatment for his underlying condition?
A

Metronidazole

Ciprofloxacin

Intravenous hydrocortisone

Co-amoxiclav

Erythromycin

Explanation
Yersinia infection
The history of severe diarrhoea including abdominal and right iliac fossa pain is suggestive of
possible Yersinia enterocolitica infection. The most appropriate therapy is therefore
ciprofloxacin, although doxycycline is a reasonable alternative. Uncomplicated cases of
Yersinia infection might not require treatment, but the presence of pyrexia, renal impairment
and a significant postural drop suggests antibiotics would be worthwhile in this case. The
condition is usually self-limiting, but Yersinia bacteraemia with spread to distant organs
results in significant morbidity and mortality. IV normal saline should also of course be
continued as he is significantly dehydrated.
21338

Next Question

https://mypastest.pastest.com/Secure/TestMe/Browser/429893#Top

1/2

8/22/2016

MyPastest

Previous Question

Tag Question

Feedback

End Review

Difficulty: Difficult
Peer Responses

Session Progress
Responses Correct:

Responses Incorrect:

298

Responses Total:

298

Responses - % Correct:

0%

Blog (https://www.pastest.com/blog) About Pastest (https://www.pastest.com/about-us)


Contact Us (https://www.pastest.com/contact-us) Help (https://www.pastest.com/help)
Pastest 2016

https://mypastest.pastest.com/Secure/TestMe/Browser/429893#Top

2/2

8/22/2016

MyPastest

Back to Filters (/Secure/TestMe/Filter/429893/QA)

Question 160 of 298

A 60-year-old man presents with epigastric pain which is radiating to his back, as well as
nausea and vomiting for the past few weeks. He has lost 4 kg in weight over the past 3
months. He drinks 4 pints of beer and a bottle of wine per day and smokes 20 cigarettes per
day. On examination he looks thin, his body mass index (BMI) is 19 kg/m2 and he has mild
epigastric tenderness on palpation. Investigations show: haemoglobin 10.9 g/dl, mean
corpuscular volume 102 fl, white cell count 8.1 109/l, platelets 210 109/l, sodium 141 mmol/l,
potassium 4.0 mmol/l, creatinine 90 mol/l. Upper gastrointestinal endoscopy shows mild
oesophagitis.
Which of the following is the next most appropriate investigation?
A

Colonoscopy

Computed tomography of the abdomen

24-hour pH monitoring

Repeat endoscopy following acid suppression

Endoscopic retrograde cholangiopancreatography (ERCP)

Explanation
<h2>Oesophagitis with significant weight loss</h2>
It is unlikely that moderate oesophagitis would account for the weight loss seen here, so 24hour pH monitoring or repeat endoscopy, while providing information about the oesophagitis,
will probably not provide an explanation of his weight loss. Two differential diagnoses high on
the list would be chronic pancreatitis related to his alcoholism and pancreatic carcinoma.
Taking these two possibilities into account, computed tomography of the abdomen would
therefore be the next most logical investigation.
21373

Next Question

Previous Question

Tag Question

https://mypastest.pastest.com/Secure/TestMe/Browser/429893#Top

Feedback

End Review

1/2

8/22/2016

MyPastest

Difficulty: Average
Peer Responses

Session Progress
Responses Correct:

Responses Incorrect:

298

Responses Total:

298

Responses - % Correct:

0%

Blog (https://www.pastest.com/blog) About Pastest (https://www.pastest.com/about-us)


Contact Us (https://www.pastest.com/contact-us) Help (https://www.pastest.com/help)
Pastest 2016

https://mypastest.pastest.com/Secure/TestMe/Browser/429893#Top

2/2

8/22/2016

MyPastest

Back to Filters (/Secure/TestMe/Filter/429893/QA)

Question 161 of 298

A 55-year-old white man presents with a 2-year history of arthritis, fever, recurrent cough and
pleuritic chest pain. He has spent the past few years working on a farm in the Netherlands
and has just returned home to the UK. He has been feeling generally unwell and most recently
he has developed diarrhoea and weight loss.
On examination there is mild skin pigmentation and finger clubbing. A pansystolic murmur is
heard on auscultation of the heart. Investigations show: haemoglobin 12.1 g/dl, white cell
count 10.5 109/l, platelets 183 109/l, sodium 140 mmol/l, potassium 4.0 mmol/l, creatinine
130 mol/l, erythrocyte sedimentation rate (ESR) 45 mmin 1 hour.
Which of the following investigations would be most likely to confirm your clinical diagnosis?
A

Echocardiography

Blood cultures

Serology testing for Coxiella

Small-bowel biopsy

Mesenteric angiography

Explanation
Q fever
This man has worked on a farm and has symptoms that fit with chronic Q fever, with arthritis,
pleuritic chest pain and endocarditis. Exposure to farm animals and small mammals such as
cats increases the risk of contracting Q fever, and two recent outbreaks in Europe occurred in
the Netherlands. In the presence of culture-negative endocarditis, serology testing for
Coxiella is the test most likely to confirm the diagnosis.
Differential diagnoses that might be considered include coeliac disease and Whipples
disease, but these are not usually associated with endocarditis. Treatment is preferably with
doxycycline, usually combined with another agent such as quinolone.
21374

Next Question

https://mypastest.pastest.com/Secure/TestMe/Browser/429893#Top

1/2

8/22/2016

MyPastest

Previous Question

Tag Question

Feedback

End Review

Difficulty: Average
Peer Responses

Session Progress
Responses Correct:

Responses Incorrect:

298

Responses Total:

298

Responses - % Correct:

0%

Blog (https://www.pastest.com/blog) About Pastest (https://www.pastest.com/about-us)


Contact Us (https://www.pastest.com/contact-us) Help (https://www.pastest.com/help)
Pastest 2016

https://mypastest.pastest.com/Secure/TestMe/Browser/429893#Top

2/2

8/22/2016

MyPastest

Back to Filters (/Secure/TestMe/Filter/429893/QA)

Question 162 of 298

A 54-year-old man presents with joint pains, anorexia, diarrhoea and intermittent fevers. He
has lost 5 kg in weight over the past 6 months and feels washed out. He has hypertension,
which is managed with amlodipine 5 mg but no other significant medical history. On
examination, he looks very thin, his body mass index (BMI) is 18 kg/m2, his blood pressure is
138/72 mmHg and he has inguinal lymphadenopathy. His abdomen appears distended and he
has bilateral pitting oedema, but there are no other abnormal findings.
Investigations show: haemoglobin 10.0 g/dl, white cell count 9.2 109/l, platelets 191 109/l,
sodium 139 mmol/l, potassium 3.8 mmol/l, creatinine 125 mol/l, albumin 24 g/l, alanine
aminotransferase (ALT) 186 U/l. A small-bowel biopsy shows expanded villi and periodic
acidSchiff- (PAS-) positive macrophages.
Which of the following is the most likely diagnosis?
A

Intestinal lymphoma

Whipples disease

Tropical sprue

Coeliac disease

Hepatitis

Explanation
Whipples disease
The presence of malabsorption with chronic diarrhoea, joint pains and intermittent fevers,
accompanied by macrophages positive for periodic acidSchiff (PAS) is typical of Whipples
disease. Hypoalbuminaemia fits with this picture, as does the anaemia; increases in
transaminases are also seen.
Management
Antibiotic therapy is the mainstay of therapy, with a prolonged duration of treatment of up to
1 year recommended. Pencillin V, amoxicillin or co-trimoxazole are typical regimes used.
Polymerase chain reaction (PCR) is suggested as an effective way to monitor response to
therapy, with adequate treatment reflected by negative PCR for Tropheryma whipplei.
21375

https://mypastest.pastest.com/Secure/TestMe/Browser/429893#Top

1/2

8/22/2016

MyPastest

Next Question

Previous Question

Tag Question

Feedback

End Review

Difficulty: Easy
Peer Responses

Session Progress
Responses Correct:

Responses Incorrect:

298

Responses Total:

298

Responses - % Correct:

0%

Blog (https://www.pastest.com/blog) About Pastest (https://www.pastest.com/about-us)


Contact Us (https://www.pastest.com/contact-us) Help (https://www.pastest.com/help)
Pastest 2016

https://mypastest.pastest.com/Secure/TestMe/Browser/429893#Top

2/2

8/22/2016

MyPastest

Back to Filters (/Secure/TestMe/Filter/429893/QA)

Question 163 of 298

A 72-year-old man presents to the clinic with dyspepsia, early satiety, a feeling of bloating
after meals and gradual weight loss of 5kg over the past 6 months. He had surgery some
years ago for a duodenal ulcer. Other past medical history of note includes emphysema, for
which he takes a combination high-dose steroid inhaler. On examination, his blood pressure is
142/88 mmHg, his heart sounds are normal and respiratory examination is consistent with
emphysema. Abdominal examination reveals a midline laparotomy scar and a small, nontender smooth liver edge.
Investigations show: haemoglobin 10.2 g/dl, mean corpuscular volume (MCV) 103 fl, platelets
167 109/l, vitamin B12 100 ng/l (normal range 150675 ng/l) sodium 139 mmol/l, potassium
4.4 mmol/l, creatinine 142 mol/l, alanine aminotransferase (ALT) 34 U/l.
Which of the following is the investigation most likely to reveal the underlying diagnosis?
A

Upper gastrointestinal endoscopy

Colonoscopy

Computed tomography of the abdomen

Ultrasound scan of the liver

Hydrogen breath test

Explanation
Bacterial overgrowth syndrome
The suspicion here is that he has bacterial overgrowth syndrome, related to the previous
upper gastrointestinal surgery. The palpable smooth liver edge could well be normal and
related to his underlying emphysema.
The hydrogen breath test is non-invasive and so would be the default next investigation. A
dose of 12 g/kg, to a maximum of around 50 g of glucose is recommended, with a rise in
hydrogen to more than 20 parts per million being diagnostic of the condition. Metronidazole
is the usual first-line treatment in the elderly.
22441

Next Question
https://mypastest.pastest.com/Secure/TestMe/Browser/429893#Top

1/2

8/22/2016

MyPastest

Previous Question

Tag Question

Feedback

End Review

Difficulty: Difficult
Peer Responses

Session Progress
Responses Correct:

Responses Incorrect:

298

Responses Total:

298

Responses - % Correct:

0%

Blog (https://www.pastest.com/blog) About Pastest (https://www.pastest.com/about-us)


Contact Us (https://www.pastest.com/contact-us) Help (https://www.pastest.com/help)
Pastest 2016

https://mypastest.pastest.com/Secure/TestMe/Browser/429893#Top

2/2

8/22/2016

MyPastest

Back to Filters (/Secure/TestMe/Filter/429893/QA)

Question 164 of 298

A 54-year-old man presents to the GP complaining of shortness of breath and dizziness. He


has no symptoms of gastrointestinal disturbance. Apart from a past history of hypertension,
which is managed with amlodipine, he has no significant known medical illnesses. On
examination, he looks pale, his blood pressure is 141/82 mmHg and his pulse is 84 bpm.
Investigations show: haemoglobin 8.9 g/dl, mean corpuscular volume (MCV) 74 fl, white cell
count 5.6 109/l, platelets 192 109/l, sodium 139 mmol/l, potassium 4.4 mmol/l, creatinine
129 micromol/l. Ferritin is low at 12 microg/l. Urine testing is negative for both red cells and
protein.
Which of the following is the most appropriate next investigation to determine the underlying
cause of his anaemia?
A

Computed tomography of the abdomen

Barium swallow

Colonoscopy

Technetium scan

Sigmoidoscopy

Explanation
Investigation of microcytic anaemia
Without any history of gastrointestinal disturbance, the highest index of suspicion here would
be for a right-sided colonic lesion, so colonoscopy would be the investigation of choice. If this
is normal, then upper gastrointestinal endoscopy would be the next most appropriate
investigation. A red cell-labelled technetium scan might be useful to detect a possible
Meckels diverticulum because it detects ectopic gastric mucosa, showing up on the scan far
distant from the stomach. Computed tomography of the abdomen might play a role in
staging any underlying neoplasm once a diagnosis has been made.
22442

Next Question

https://mypastest.pastest.com/Secure/TestMe/Browser/429893#Top

1/2

8/22/2016

MyPastest

Previous Question

Tag Question

Feedback

End Review

Difficulty: Easy
Peer Responses

Session Progress
Responses Correct:

Responses Incorrect:

298

Responses Total:

298

Responses - % Correct:

0%

Blog (https://www.pastest.com/blog) About Pastest (https://www.pastest.com/about-us)


Contact Us (https://www.pastest.com/contact-us) Help (https://www.pastest.com/help)
Pastest 2016

https://mypastest.pastest.com/Secure/TestMe/Browser/429893#Top

2/2

8/22/2016

MyPastest

Back to Filters (/Secure/TestMe/Filter/429893/QA)

Question 165 of 298

A 44-year-old man presents to the clinic with tiredness, lethargy and night sweats. He has a
history of hypertension and chronic renal failure, and has a catheter in place to begin
peritoneal dialysis in a few weeks. On examination, he is pyrexial (37.9 C) and has a mitral
murmur. He is not in cardiac failure and his blood pressure is well controlled at 142/82 mmHg.
Investigations show: haemoglobin 9.9 g/dl, mean corpuscular volume (MCV) 79 fl, white cell
count 13.1 109/l, platelets 201 109/l, sodium 141 mmol/l, potassium 4.9 mmol/l, creatinine
415 mol/l. Blood cultures are positive for Streptococcus bovis and vegetations are visualised
on the mitral valve on echocardiography.
Which of the following is the most likely source of infection?
A

The continuous ambulatory peritoneal dialysis (CAPD) line

Dental abscess

Large bowel

Biliary tree

Small bowel

Explanation
Streptococcus bovis infection
Streptococcus bovis endocarditis should raise alarm bells about possibe underlying colonic
carcinoma. Colonoscopy is recommended in all patients who present with S. bovis disease. If
colonoscopy is normal, then upper gastrointestinal endoscopy and imaging of the biliary tree
is recommended to rule out these areas as sources of the organism. Continuous ambulatory
peritoneal dialysis (CAPD) line infections are normally staphylococcal in origin.
22443

Next Question

Previous Question

Tag Question

https://mypastest.pastest.com/Secure/TestMe/Browser/429893#Top

Feedback

End Review

1/2

8/22/2016

MyPastest

Difficulty: Average
Peer Responses

Session Progress
Responses Correct:

Responses Incorrect:

298

Responses Total:

298

Responses - % Correct:

0%

Blog (https://www.pastest.com/blog) About Pastest (https://www.pastest.com/about-us)


Contact Us (https://www.pastest.com/contact-us) Help (https://www.pastest.com/help)
Pastest 2016

https://mypastest.pastest.com/Secure/TestMe/Browser/429893#Top

2/2

8/22/2016

MyPastest

Back to Filters (/Secure/TestMe/Filter/429893/QA)

Question 166 of 298

A 70-year-old man presents with tiredness and lethargy. He says that over the past few
months he has been suffering from fevers and gradual weight loss. He has a history of
hypertension, for which he takes ramipril, and he has had some teeth removed because of
dental decay. On examination he has a temperature of 37.9 C, a pulse of 90 bpm and a blood
pressure of 125/82 mmHg. Auscultation of the heart reveals pansystolic and early diastolic
murmurs.
Investigations show: haemoglobin 10.0 g/dl, white cell count 11.2 109/l, platelets 210 109/l,
sodium 141 mmol/l, potassium 4.9 mmol/l, creatinine 135 mol/l, C-reactive protein (CRP) 120
mg/l (normal range < 10 mg/l). Streptococcus bovis is isolated from one bottle of blood
cultures and echocardiography shows vegetations on the mitral valve.
Which of the following is the most appropriate next investigation?
A

Upper gastrointestinal endoscopy

Barium swallow

Barium follow-through

Dental examination

Colonoscopy

Explanation
Streptococcus bovis infection
S. bovis endocarditis strongly raises the possibility of an underlying colonic carcinoma, so
colonoscopy is mandatory, (studies suggest up to 60% of patients with S bovis endocarditis
may have an underlying colonic neoplasm). If the colonoscopy is normal, then an upper
gastrointestinal endoscopy would be the next most appropriate investigation. Liver
ultrasonography is also indicated to rule out associated hepatobiliary disease.
22444

Next Question

https://mypastest.pastest.com/Secure/TestMe/Browser/429893#Top

1/2

8/22/2016

MyPastest

Previous Question

Tag Question

Feedback

End Review

Difficulty: Easy
Peer Responses

Session Progress
Responses Correct:

Responses Incorrect:

298

Responses Total:

298

Responses - % Correct:

0%

Blog (https://www.pastest.com/blog) About Pastest (https://www.pastest.com/about-us)


Contact Us (https://www.pastest.com/contact-us) Help (https://www.pastest.com/help)
Pastest 2016

https://mypastest.pastest.com/Secure/TestMe/Browser/429893#Top

2/2

8/22/2016

MyPastest

Back to Filters (/Secure/TestMe/Filter/429893/QA)

Question 167 of 298

A 56-year-old woman who has a history of systemic sclerosis comes to the clinic. She is
currently managed with amlodipine for symptoms of Raynauds disease and omeprazole for
gastro-oesophageal reflux disease (GORD), but over the past few months she has suffered
from increasing symptoms of abdominal bloating, intermittent diarrhoea and constipation.
She is also increasingly nauseous despite taking her omeprazole regularly.
On examination there is clear evidence of peripheral calcinosis. She has a slightly distended
abdomen, which is soft with only mild tenderness. Investigations show: haemoglobin 11.9 g/dl,
white cell count 6.4 109/l, platelets 209 109/l, sodium 139 mmol/l, potassium 4.9 mmol/l,
creatinine 134 mol/l. A barium swallow shows evidence of severe oesophageal dysmotility.
Given the likely diagnosis, which of the following would be the most appropriate intervention?
A

Gluten-free diet

Metronidazole

Doxycycline

Prednisolone

Metoclopramide

Explanation
Bacterial overgrowth syndrome
The history described here is suggestive of bacterial overgrowth syndrome, which is known to
be associated with systemic sclerosis, particularly where there is demonstrable dysmotility, as
seen here. It occurs because the dysmotility associated with fibrotic change within the small
bowel increases the risk of bacterial colonisation. The best management strategy is unclear,
but initial attempts at eradication of bacterial overgrowth with metronidazole, ciprofloxacin
or co-amoxiclav would seem appropriate. Of course, these patients have long-term gut
motility problems, so a rotating programme of antibiotics to avoid problems with resistance
might be appropriate.
22467

Next Question

https://mypastest.pastest.com/Secure/TestMe/Browser/429893#Top

1/2

8/22/2016

MyPastest

Previous Question

Tag Question

Feedback

End Review

Difficulty: Difficult
Peer Responses

Session Progress
Responses Correct:

Responses Incorrect:

298

Responses Total:

298

Responses - % Correct:

0%

Blog (https://www.pastest.com/blog) About Pastest (https://www.pastest.com/about-us)


Contact Us (https://www.pastest.com/contact-us) Help (https://www.pastest.com/help)
Pastest 2016

https://mypastest.pastest.com/Secure/TestMe/Browser/429893#Top

2/2

8/22/2016

MyPastest

Back to Filters (/Secure/TestMe/Filter/429893/QA)

Question 168 of 298

A patient with known systemic sclerosis comes to the clinic. She has a history of symptoms of
oesophageal reflux, which are managed with omeprazole, and Raynauds disease, which is
managed with nifedipine. Her main complaint is worsening symptoms of bloating and acid
reflux. On examination, her body mass index (BMI) is 22 kg/m2 and her blood pressure is
155/90 mmHg. She has soft-tissue calcinosis on examination of her fingers. Examination of
the chest reveals scattered crackles over both lung fields.
Investigations show: haemoglobin 12.3 g/dl, white cell count 7.9 109/l, platelets 182 109/l,
sodium 141 mmol/l, potassium 4.3 mmol/l, creatinine 132 mol/l. The result of a hydrogen
breath test off PPI is 10 parts per million (normal range < 20 ppm).
Which of the following is the most appropriate intervention?
A

Domperidone

Metronidazole

Ciprofloxacin

Ranitidine

Fundoplication

Explanation
Oesophageal reflux in a patient with systemic sclerosis
The hydrogen breath test is negative, which effectively excludes bacterial overgrowth
syndrome, which is a recognised problem in patients with systemic sclerosis because of the
associated dysmotility. Given that her symptoms are predominantly those of worsening
reflux, addition of domperidone (a prokinetic agent) would seem most appropriate. In
patients who fail to respond, a small dose of erythromycin might also be of value.
22468

Next Question

Previous Question

Tag Question

https://mypastest.pastest.com/Secure/TestMe/Browser/429893#Top

Feedback

End Review
1/2

8/22/2016

MyPastest

Difficulty: Difficult
Peer Responses

Session Progress
Responses Correct:

Responses Incorrect:

298

Responses Total:

298

Responses - % Correct:

0%

Blog (https://www.pastest.com/blog) About Pastest (https://www.pastest.com/about-us)


Contact Us (https://www.pastest.com/contact-us) Help (https://www.pastest.com/help)
Pastest 2016

https://mypastest.pastest.com/Secure/TestMe/Browser/429893#Top

2/2

8/22/2016

MyPastest

Back to Filters (/Secure/TestMe/Filter/429893/QA)

Question 169 of 298

A 62-year-old man presents with jaundice to his GP. He also complains of itching and
unexplained weight loss of 3 kg over the past 6 months. On further questioning he admits to
having had vague abdominal pain on a few occasions. He drinks 12 units of alcohol per day.
Other history of note includes a distant history of smoking of 5 cigarettes per day, but he
gave them up many years ago because he developed emphysema at a very young age. He
has also had intermittent diarrhoea.
On examination, his blood pressure is 135/70 mmHg, his pulse is 75 bpm and regular. He has
jaundiced sclerae. Abdominal examination is relatively normal apart from some vague right
upper quadrant tenderness to palpation. Investigations show: haemoglobin 12.1 g/dl, white cell
count 7.8 109/l, platelets 201 109/l, sodium 139 mmol/l, potassium 4.9 mmol/l, creatinine
130 mol/l, alanine aminotransferase (ALT) 195 U/l, alkaline phosphatase 280 U/l. An
ultrasound scan shows intra- and extrahepatic bile duct dilatation.
Which of the following is the most appropriate next investigation?
A

Magnetic resonance cholangiopancreatography (MRCP)

Endoscopic retrograde cholangiopancreatography (ERCP)

Liver biopsy

Autoimmune profile

Alpha1-antitrypsin genetic status

Explanation
Cholangiocarcinoma
This man has relatively painless jaundice, of insidious onset, only complaining of relatively
minor weight loss and other vague symptoms. There are some useful pointers to the
underlying diagnosis in the history, however alcohol excess and early-onset emphysema. A
finding of intra- and extrahepatic duct dilatation is a pointer towards possible primary
sclerosing cholangitis and subsequent cholangiocarcinoma.
Magnetic resonance cholangiopancreatography (MRCP) is superior to endoscopic retrograde
cholangiopancreatography (ERCP) for investigating this condition because it is non-invasive
and permits tumour staging with assessment of hepatic involvement.
https://mypastest.pastest.com/Secure/TestMe/Browser/429893#Top

1/2

8/22/2016

MyPastest

The diarrhoea might indicate a component of underlying inflammatory bowel disease.


Because they key problem is the liver dysfunction as such we need a step which defines the
underlying liver condition. Given the ultrasound has uncovered duct dilatation, the most
urgent thing is to determine the cause of the obstruction.
22477

Next Question

Previous Question

Tag Question

Feedback

End Review

Difficulty: Difficult
Peer Responses

Session Progress
Responses Correct:

Responses Incorrect:

298

Responses Total:

298

Responses - % Correct:

0%

Blog (https://www.pastest.com/blog) About Pastest (https://www.pastest.com/about-us)


Contact Us (https://www.pastest.com/contact-us) Help (https://www.pastest.com/help)
Pastest 2016

https://mypastest.pastest.com/Secure/TestMe/Browser/429893#Top

2/2

8/22/2016

MyPastest

Back to Filters (/Secure/TestMe/Filter/429893/QA)

Question 170 of 298

A 35-year-old patient presents to the Transplant Clinic for review a few weeks after a renal
transplant. She is taking a combination of prednisolone, mycophenolate mofetil and
ciclosporin for immunosupression. Her main complaint is of non-bloody diarrhoea and mucus,
with six to eight loose motions per day, unrelated to any particular foods.
On examination she looks dehydrated. Her blood pressure is 130/80 mmHg lying and drops to
110/70 mmHg when she stands up. Physical examination is otherwise unremarkable, apart
from a soft, mildly tender abdomen. Investigations show: haemoglobin 11.5 g/dl, white cell
count 4.2 109/l, platelets 282 109/l, sodium 139 mmol/l, potassium 4.7 mmol/l, creatinine
167 mol/l.
Which of the following is the most likely cause of her diarrhoea?
A

Irritable bowel syndrome

New-onset ulcerative colitis

Ciclosporin

Mycophenolate mofetil

Prednisolone

Explanation
Crohns-like enterocolitis with mycophenolate mofetil
A Crohns-like enterocolitis has been reported in a number of renal transplant patients who
have received mycophenolate mofetil. Investigations will reveal mucosal ulceration and skip
lesions ordinarily seen in Crohns. Withdrawal of mycophenolate is associated with resolution
of the symptoms over the course of a few weeks.
22488

Next Question

Previous Question

Tag Question

https://mypastest.pastest.com/Secure/TestMe/Browser/429893#Top

Feedback

End Review

1/2

8/22/2016

MyPastest

Difficulty: Average
Peer Responses

Session Progress
Responses Correct:

Responses Incorrect:

298

Responses Total:

298

Responses - % Correct:

0%

Blog (https://www.pastest.com/blog) About Pastest (https://www.pastest.com/about-us)


Contact Us (https://www.pastest.com/contact-us) Help (https://www.pastest.com/help)
Pastest 2016

https://mypastest.pastest.com/Secure/TestMe/Browser/429893#Top

2/2

8/22/2016

MyPastest

Back to Filters (/Secure/TestMe/Filter/429893/QA)

Question 171 of 298

A 45-year-old man comes to the Emergency Department with acute severe epigastric pain.
He works as a pub manager and on further questioning he admits to drinking a bottle of wine
a day and up to 510 measures of spirits. He has been seen previously for intoxication but he
discharged himself from hospital on that occasion. On examination, his blood pressure is
100/60 mmHg, his pulse is 105 bpm, he looks unwell and he has severe epigastric tenderness.
He has a raised respiratory rate and you can hear some crackles on auscultation of both lung
fields.
Investigations show: haemoglobin 10.5 g/dl, mean corpuscular volume (MCV) raised, white
cell count 14.2 109/l, platelets 150 109/l, sodium 138 mmol/l, potassium 4.5 mmol/l,
creatinine 167 mol/l, amylase 1490 U/l, Po2 9.1 kPa, Pco2 4.0 kPa. A chest X-ray shows
bilateral pulmonary infiltrates. He becomes more short of breath and his hypoxia worsens.
Given the likely diagnosis underlying his hypoxia, which of the following findings is most
characteristic?
A

Raised capillary wedge pressure

Protein-rich oedema fluid

Increased lung compliance

Macrophage-rich oedema fluid

Excess production of surfactant

Explanation
Acute respiratory distress syndrome (ARDS)
ARDS is a recognised consequence of acute pancreatitis
Patients have decreased or ineffective surfactant and decreased lung compliance
Oedema fluid that shows marked haemorrhage or very large numbers of white cells
might suggest an alternative diagnosis, and these features are not therefore
characteristic of the diagnosis, although protein-rich oedema fluid is typical of the
condition
The capillary wedge pressure is not elevated in association with ARDS
https://mypastest.pastest.com/Secure/TestMe/Browser/429893#Top

1/2

8/22/2016

MyPastest
22527

Next Question

Previous Question

Tag Question

Feedback

End Review

Difficulty: Average
Peer Responses

Session Progress
Responses Correct:

Responses Incorrect:

298

Responses Total:

298

Responses - % Correct:

0%

Blog (https://www.pastest.com/blog) About Pastest (https://www.pastest.com/about-us)


Contact Us (https://www.pastest.com/contact-us) Help (https://www.pastest.com/help)
Pastest 2016

https://mypastest.pastest.com/Secure/TestMe/Browser/429893#Top

2/2

8/22/2016

MyPastest

Back to Filters (/Secure/TestMe/Filter/429893/QA)

Question 172 of 298

A 29-year-old woman comes to the Gastroenterology Clinic for review. She has Crohns
disease which has been resistant to therapy with oral Budesonide and you are considering
starting Azathioprine therapy. She tells you that her mother was treated with Azathioprine for
arthritis and had problems with a low white cell count.
Which of the following is appropriate with respect to considerations before starting therapy?
A

CYP2C9 inhibitors should be discontinued if possible

TMPT enzyme activity should be assessed

CYP2D6 activity should be assessed

CYP3A4 enzyme inhibitors should be discontinued if possible

Influenza vaccination should be avoided whilst on therapy

Explanation
The answer is TMPT enzyme activity should be assessed
The fact that her mother had problems with a low white count on Azathioprine raises the
question of inherited low TMPT activity. As such it seems prudent to either assess TMPT
enzyme activity in this patient before starting Azathioprine or whether she carries the gene
for low activity. The results may impact on choice of dose of Azathioprine or even whether
you decide to start treatment at all. Acetylation status does not impact on Azathioprine
metabolism significantly. 2D6 activity impacts on metabolism of Codeine and beta blockers,
2C9 on Warfarin metabolism. Influenza vaccination may take place in patients on
Azathioprine.
36435

Next Question

Previous Question

Tag Question

Feedback

End Review

Difficulty: Average
https://mypastest.pastest.com/Secure/TestMe/Browser/429893#Top

1/2

8/22/2016

MyPastest

Peer Responses

Session Progress
Responses Correct:

Responses Incorrect:

298

Responses Total:

298

Responses - % Correct:

0%

Blog (https://www.pastest.com/blog) About Pastest (https://www.pastest.com/about-us)


Contact Us (https://www.pastest.com/contact-us) Help (https://www.pastest.com/help)
Pastest 2016

https://mypastest.pastest.com/Secure/TestMe/Browser/429893#Top

2/2

8/22/2016

MyPastest

Back to Filters (/Secure/TestMe/Filter/429893/QA)

Question 173 of 298

A 62-year-old patient with a history of Type 2 diabetes is referred to the Gastroenterology


Clinic with significant unintended weight loss and epigastric pain radiating to her back.
Normally taking Gliclazide and Metformin for diabetes, she has lost 8kg in weight over the
past 3 months and has stopped the Gliclazide completely due to hypoglycaemia. The
epigastric pain is becoming more severe and radiates to her back. There is no history of
waterbrash, food sticking or regurgitation. Cardiovascular and respiratory system examination
is unremarkable. She is tender to palpation in epigastrium.
Investigations;
Hb

11.1 g/dl

WCC

9.2 x109/l

PLT

198 x109/l

Visc

2.3 mPa/s (1.50-1.72)

Na+

138 mmol/l

K+

4.1 mmol/l

Creatinine 112 micromol/l


ALT

56 U/l

ALP

421 U/l

Gamma

GT 240 U/l (4-35)

Which of the following is the most likely diagnosis?


A

Gastric carcinoma

Gastro-oesophageal reflux disease

Hepatocellular carcinoma

Pancreatic carcinoma

Peptic ulcer disease

https://mypastest.pastest.com/Secure/TestMe/Browser/429893#Top

1/2

8/22/2016

MyPastest

Explanation
The answer is Pancreatic carcinoma
With such profound weight loss over a very short time, coupled with the history of Type 2
diabetes and epigastric pain, pancreatic carcinoma is the most likely diagnosis. Compared to
a non-diabetic cohort, those with Type 2 diabetes over the age of 50 years are 8 times more
likely to develop pancreatic carcinoma. The lack of symptoms of waterbrash, food
regurgitation or vomiting, make GORD, peptic ulcer disease and gastric carcinoma less likely.
The profound obstructive picture in her LFTs also supports pancreatic carcinoma as the
likeliest diagnosis versus hepatocellular Ca.
Ultrasound is usually the primary radiological investigation, followed by CT to assess staging
of the underlying carcinoma. CA-19-9 is a tumour marker for pancreatic carcinoma, but it is
usually used to monitor response to treatment and possible recurrance, rather than for
diagnosis, it has a sensitivity of 80% and specificity of 73% for pancreatic carcinoma.
37277

Next Question

Previous Question

Tag Question

Feedback

End Review

Difficulty: Easy
Peer Responses

Session Progress
Responses Correct:

Responses Incorrect:

298

Responses Total:

298

Responses - % Correct:

0%

Blog (https://www.pastest.com/blog) About Pastest (https://www.pastest.com/about-us)


Contact Us (https://www.pastest.com/contact-us) Help (https://www.pastest.com/help)
Pastest 2016

https://mypastest.pastest.com/Secure/TestMe/Browser/429893#Top

2/2

8/22/2016

MyPastest

Back to Filters (/Secure/TestMe/Filter/429893/QA)

Question 174 of 298

A 74-year-old man presents to the clinic for review. He has a mobile, non tender mass in the
right iliac fossa. Past medical history of note includes hypertension for which he is treated
with Amlodipine and Indapamide, and asthma for which he is treated with low dose Seretide.
On examination his BP is 142/72 mmHg, pulse is 72/min and regular. He has a 4cm, firm, non
tender, mobile mass in the right iliac fossa.
Investigations;
Hb

10.0 g/dl

MCV

78 fl

WCC

5.9 x109/l

PLT

200 x109/l

Na+

137 mmol/l

K+

4.3 mmol/l

Creatinine 122 micromol/l


ESR

78 mm/1st hour

Which of the following is the most likely diagnosis?


A

Caecal carcinoma

Dermoid cyst

Femoral hernia

Inguinal hernia

Lipoma

Explanation
The answer is Caecal carcinoma -

https://mypastest.pastest.com/Secure/TestMe/Browser/429893#Top

1/2

8/22/2016

MyPastest

With possible occult GI blood loss, and the position of the mass, a caecal carcinoma is the
most likely diagnosis. Dermoid cysts may present in a similar way, but are extremely rare. The
position as described is wrong for a femoral or inguinal hernia. A lipoma would not be
associated with probable iron deficiency anaemia.
Ultrasound is non-invasive and will confirm that the mass originates from large bowel. This
makes it the initial radiological investigation of choice. It can be followed either with
colonoscopy to gain a tissue diagnosis, or pre-operative staging CT scan. Caecal carcinomas
are subject only to late obstruction because faeces are predominantly liquid when they pass
through the caecum.
37278

Next Question

Previous Question

Tag Question

Feedback

End Review

Difficulty: Average
Peer Responses

Session Progress
Responses Correct:

Responses Incorrect:

298

Responses Total:

298

Responses - % Correct:

0%

Blog (https://www.pastest.com/blog) About Pastest (https://www.pastest.com/about-us)


Contact Us (https://www.pastest.com/contact-us) Help (https://www.pastest.com/help)
Pastest 2016

https://mypastest.pastest.com/Secure/TestMe/Browser/429893#Top

2/2

8/22/2016

MyPastest

Back to Filters (/Secure/TestMe/Filter/429893/QA)

Question 175 of 298

A 45-year-old man is referred by the GP to the Endoscopy Clinic. He has no weight loss but
has been suffering from severe reflux symptoms, refractory to high dose PPI, stopping
smoking and reducing alcohol consumption over the past 6 months. He is otherwise
completely well. The GP is concerned about possible malignancy. Endoscopy reveals Barretts
changes with high grade dysplasia.
Which of the following is the most appropriate next step?
A

Add Metoclopramide

Continue high dose PPI

Photodynamic therapy

Referral for oesophagectomy

Repeat endoscopy in 2 years

Explanation
The answer is Referral for oesophagectomy
High-grade dysplasia is associated with one or more areas of invasive adenocarcinoma in 3040% of patients. Photodynamic therapy appears to be effective in downgrading the dysplasia
when used for high-grade dysplasia, but its efficacy in preventing the progression of Barrett's
oesophagus to invasive cancer is not clear. As such in younger, fitter patients with Barretts
and high grade dysplasia, oesophagectomy is considered. Surveillance endoscopy is
considered for those patients with low grade dysplasia. High dose PPI has not proved
effective so far, and therefore is not the optimal intervention. Metoclopramide is no longer
recommended in the EU for chronic use.
37279

Next Question

Previous Question

Tag Question

https://mypastest.pastest.com/Secure/TestMe/Browser/429893#Top

Feedback

End Review

1/2

8/22/2016

MyPastest

Difficulty: Average
Peer Responses

Session Progress
Responses Correct:

Responses Incorrect:

298

Responses Total:

298

Responses - % Correct:

0%

Blog (https://www.pastest.com/blog) About Pastest (https://www.pastest.com/about-us)


Contact Us (https://www.pastest.com/contact-us) Help (https://www.pastest.com/help)
Pastest 2016

https://mypastest.pastest.com/Secure/TestMe/Browser/429893#Top

2/2

8/22/2016

MyPastest

Back to Filters (/Secure/TestMe/Filter/429893/QA)

Question 176 of 298

A 67-year-old woman presents to the clinic for review. She has extensive osteoarthritis. The
pain is not controlled despite maximal Paracetamol and a low dose of Codeine phosphate.
You are considering adding a non-steroidal anti-inflammatory drug (NSAID), but she has a
previous history of peptic ulcer disease and is worried about this recurring.
Which of the following NSAIDs is associated with the lowest risk of peptic ulcer disease?
A

Diclofenac

Ibuprofen

Indometacin

Naproxen

Piroxicam

Explanation
The answer is Ibuprofen
A number of systematic reviews have estimated the risk of peptic ulcer disease associated
with NSAID prescription. For Ibuprofen the relative risk (RR), vs those not prescribed a
NSAID is 2.69 [95% CI 2.173.33]. For the other agents listed the RR is higher. Estimated RRs
were 5.63 (95% CI 3.838.28) for Naproxen, 5.40 (95% CI 4.167.00) for Indometacin, 3.98
(95% CI 3.364.72) for Diclofenac and 9.94 [95% CI 5.9916.50] for Piroxicam. As such in this
patient it would seem most sensible to start with ibuprofen from the point of view of peptic
ulcer risk.
http://onlinelibrary.wiley.com/doi/10.1002/art.27412/full
(http://onlinelibrary.wiley.com/doi/10.1002/art.27412/full)
37310

Next Question

Previous Question

Tag Question

https://mypastest.pastest.com/Secure/TestMe/Browser/429893#Top

Feedback

End Review

1/2

8/22/2016

MyPastest

Difficulty: Difficult
Peer Responses

Session Progress
Responses Correct:

Responses Incorrect:

298

Responses Total:

298

Responses - % Correct:

0%

Blog (https://www.pastest.com/blog) About Pastest (https://www.pastest.com/about-us)


Contact Us (https://www.pastest.com/contact-us) Help (https://www.pastest.com/help)
Pastest 2016

https://mypastest.pastest.com/Secure/TestMe/Browser/429893#Top

2/2

8/22/2016

MyPastest

Back to Filters (/Secure/TestMe/Filter/429893/QA)

Question 177 of 298

A 54-year-old man presents in the early morning with acute severe pain in his upper
abdomen. He is a smoker of 20 cigarettes per day, drinks a bottle of whisky every 3 days, and
admits to indigestion which has increased over the past few weeks. He says this episode of
pain awoke him from sleep and he has been in agony since. On examination his BP is 135/75
mmHg, pulse is 104/min and regular. His abdomen is extremely tender, rigid, and you can not
hear any bowel sounds.
Investigations;
Hb

10.3 g/dl

WCC

15.9 x109/l

PLT

203 x109/l

Na+

137 mmol/l

K+

5.0 mmol/l

Creatinine 132 micromol/l


Amylase

322 U/l

ALT

110 U/l

Albumin

38 g/l

Bilirubin

13 micromol/l

Which of the following is the most likely diagnosis?


A

Acute pancreatitis

Cholecystitis

Mesenteric angina

Oesophageal perforation

Perforated duodenal ulcer

https://mypastest.pastest.com/Secure/TestMe/Browser/429893#Top

1/2

8/22/2016

MyPastest

Explanation
The answer is Perforated duodenal ulcer
Whilst the amylase is outside the normal range, elevations are commonly seen in patients
who suffer a perforated abdominal viscus. Acute pancreatitis and cholecystitis are more
usually associated with epigastric pain and vomiting, whereas vomiting is an unusual feature
in duodenal ulcer perforation and is not seen here. Oesophageal perforation is associated
with retrosternal pain radiating to the back, and mesenteric angina usually with paroxysms of
abdominal pain and diarrhoea associated with ischaemia. The overall clinical picture suggests
that ulcer perforation is the most likely diagnosis.
Erect chest X-ray has traditionally been seen as the initial investigation of choice, but it is
important to remember that up to 30% of patients with a perforation may not have visible air
under the diaphragm on radiography. As such supine and erect abdominal films may in fact
be more useful initially, although CT abdomen is the definitive investigation.
37311

Next Question

Previous Question

Tag Question

Feedback

End Review

Difficulty: Average
Peer Responses

Session Progress
Responses Correct:

Responses Incorrect:

298

Responses Total:

298

Responses - % Correct:

0%

Blog (https://www.pastest.com/blog) About Pastest (https://www.pastest.com/about-us)


Contact Us (https://www.pastest.com/contact-us) Help (https://www.pastest.com/help)
Pastest 2016

https://mypastest.pastest.com/Secure/TestMe/Browser/429893#Top

2/2

8/22/2016

MyPastest

Back to Filters (/Secure/TestMe/Filter/429893/QA)

Question 178 of 298

A 48-year-old man comes to the Hepatology Clinic for review. He presented with features of
early cirrhosis and was found to be hepatitis C positive on further investigation with
genotype 1. On examination his BP is 122/72 mmHg, pulse is 73/min and regular. He has a
number of spider naevi over his upper body and is mildly tender in the right upper quadrant.
There is no ascites.
Investigations;

Hb

12.5 g/dl

WCC

7.6 x109/l

PLT

189 x109/l

Na+

137 mmol/l

K+

4.3 mmol/l

Creatinine 100 micromol/l


ALT

122 U/l

Bilirubin

22 micromol/l

ALP

195 U/l

He is treated with Interferon, Ribavirin and Boceprevir.


Which of the following is the recommended duration of therapy?
A

16 weeks

24 weeks

32 weeks

48 weeks

60 weeks

https://mypastest.pastest.com/Secure/TestMe/Browser/429893#Top

1/2

8/22/2016

MyPastest

Explanation
The answer is 48 weeks Genotype 1 hepatitis C is recognised to have low rates of viral clearance with dual interferon
and ribavirin therapy alone. As such a longer duration of anti-viral therapy, up to 48 weeks of
therapy is recommended. NICE has also recommended Boceprevir and Telaprevir as options
for the treatment of people with genotype 1 chronic hepatitis C. Both Boceprevir and
Telaprevir work by inhibiting the activity of the NS3/4A serine protease. This protease is
essential for viral replication and may be partially responsible for the ability of HCV to evade
clearance by the host immune system. Sofosbuvir is a nucleotide analogue recommended by
NICE in triple Hepatitis C therapy, primarily for genotype 1 disease.
Common side effects of Interferon therapy include leukopaenia, symptoms of URTI, thyroid
disorders, and changes in mood. Anaemia and leukopaenia are particularly reported in
patients taking Interferon in the triple combination regimen with Ribavirin and Boceprevir.
For this reason close monitoring of FBC is recommended, with initial review after 4 weeks of
therapy.
37324

Next Question

Previous Question

Tag Question

Feedback

End Review

Difficulty: Difficult
Peer Responses

Session Progress
Responses Correct:

Responses Incorrect:

298

Responses Total:

298

Responses - % Correct:

0%

Blog (https://www.pastest.com/blog) About Pastest (https://www.pastest.com/about-us)


Contact Us (https://www.pastest.com/contact-us) Help (https://www.pastest.com/help)
Pastest 2016

https://mypastest.pastest.com/Secure/TestMe/Browser/429893#Top

2/2

8/22/2016

MyPastest

Back to Filters (/Secure/TestMe/Filter/429893/QA)

Question 179 of 298

A 45-year-old alcoholic man who is known to have oesophageal varices comes to the
Emergency Department by ambulance having suffered an acute upper GI bleed estimated at
100-200ml by the paramedics. He missed his last surveillance endoscopy some 3 months
earlier and is known to be poorly compliant with Propranolol. Examination reveals a BP of
90/60 mmHg, his pulse is 110/min and regular. He is distresssed and complaining of upper
abdominal/epigastric pain and there is evidence of dried blood around his mouth. PR reveals
an empty rectum. IV fluid rehydration is commenced.
Investigations;
Hb

8.4 g/dl

WCC

10.1 x109/l

PLT

112 x109/l

Na+

134 mmol/l

K+

4.5 mmol/l

Creatinine 85 micromol/l
Urea

10.4 mmol/l

Which of the following is the most appropriate next step?


A

IV Octreotide

IV Omeprazole

IV Terlipressin

Oral Omeprazole

O negative blood transfusion

Explanation
The answer is IV Terlipressin https://mypastest.pastest.com/Secure/TestMe/Browser/429893#Top

1/2

8/22/2016

MyPastest

NICE guidance states that Terlipressin should be offered to patients with suspected variceal
bleeding at presentation. Treatment should be stopped after definitive haemostasis has been
achieved, or after five days, unless there is another indication for its use. It is most important
action in this context is in reducing blood flow through the splanchnic circulation. Proton
pump inhibitors do not impact on outcomes when used acutely in this situation. Whilst blood
transfusion is important, cross matched blood is preferable to use of o negative unless there
is significant, life threatening circulatory compromise. Octreotide is thought to only have a
very short-term effect on the portal circulation, and for this reason terlipressin is more widely
used.
38088

Next Question

Previous Question

Tag Question

Feedback

End Review

Difficulty: Average
Peer Responses

Session Progress
Responses Correct:

Responses Incorrect:

298

Responses Total:

298

Responses - % Correct:

0%

Blog (https://www.pastest.com/blog) About Pastest (https://www.pastest.com/about-us)


Contact Us (https://www.pastest.com/contact-us) Help (https://www.pastest.com/help)
Pastest 2016

https://mypastest.pastest.com/Secure/TestMe/Browser/429893#Top

2/2

8/22/2016

MyPastest

Back to Filters (/Secure/TestMe/Filter/429893/QA)

Question 180 of 298

You are asked to see a 74-year-old woman on the Orthopaedic Ward who has had multiple
courses of antibiotics following an infection of her left hip prosthesis. For the past 48hrs she
has suffered from increasing diarrhoea and is now opening her bowels up to 10 times per day,
passing brown water flecked with blood. On examination her BP is 95/60 mmHg, pulse is
92/min and regular. Her abdomen is generally tender, particularly on the left side. PR reveals
an empty rectum.
Investigations;
Hb

10.4 g/dl

WCC

12.3 x109/l

PLT

201 x109/l

Na+

137 mmol/l

K+

5.2 mmol/l

Creatinine

167 micromol/l (112 micromol/l on admission)

Albumin

28 g/l

Which of the following is the most appropriate way to confirm the underlying diagnosis?
A

Clostridium difficile toxin testing

Colonoscopy

Flexible sigmoidoscopy

Stool culture

Trial of Metronidazole

Explanation
The answer is Clostridium difficile toxin testing -

https://mypastest.pastest.com/Secure/TestMe/Browser/429893#Top

1/2

8/22/2016

MyPastest

The stool cytotoxin test, has high sensitivity (94-100%) and specificity (99%) for
pseudomembranous colitis and has become the standard screening test for C. difficile. In
patients who are usually debilitated with diarrhoea, and often post-op or following a serious
medical illness, this non-invasive option is preferred to sigmoidoscopy or colonoscopy. Both
Metronidazole and Vancomycin are potential treatment options, evidence suggests that
Vancomycin may be more effective because of changes in the luminal concentration of
Metronidazole associated with recovery.
39055

Next Question

Previous Question

Tag Question

Feedback

End Review

Difficulty: Easy
Peer Responses

Session Progress
Responses Correct:

Responses Incorrect:

298

Responses Total:

298

Responses - % Correct:

0%

Blog (https://www.pastest.com/blog) About Pastest (https://www.pastest.com/about-us)


Contact Us (https://www.pastest.com/contact-us) Help (https://www.pastest.com/help)
Pastest 2016

https://mypastest.pastest.com/Secure/TestMe/Browser/429893#Top

2/2

8/22/2016

MyPastest

Back to Filters (/Secure/TestMe/Filter/429893/QA)

Question 181 of 298

A 17-year-old woman comes to the Gastroenterology Clinic. She feels tired all the time, has no
energy, and has had to pull out of taking her end of year exams. Over the past few months
she has seen her GP on two occasions with symptoms of intermittent diarrhoea which she
says is sometimes hard to flush away, and abdominal bloating. Examination reveals a BP of
95/60 mmHg, pulse is 75/min and regular. Abdomen is soft and non-tender, her BMI is
reduced at 19.5.
Investigations;
Hb

9.8 g/dl

MCV

104 fl

WCC

10.4 x109/l

PLT

181 x109/l

Na+

137 mmol/l

K+

4.3 mmol/l

Creatinine

90 micromol/l

Albumin

30 g/l

ALT

43 U/l

ALP

162 U/l

Calcium

2.10 mmol/l

Which of the following is the most appropriate next step?


A

Anti TTG antibodies

Psychological assessment

Serum vitamin D

Small bowel follow through

https://mypastest.pastest.com/Secure/TestMe/Browser/429893#Top

1/2

8/22/2016

MyPastest

Upper GI endoscopy and biopsy

Explanation
The answer is Anti TTG antibodies Symptoms of malabsorption, coupled with anaemia and probable folate deficiency raise the
possibility in this age group of coeliac disease. In patients where the titre for anti-TTG
antibodies is >10 times the upper limit of normal who also have symptoms, the likelihood of
underlying coeliac disease is very high. The calcium is also at the lower limit of the normal
range, raising the possibility of low levels of vitamin D. There is no reason to suspect an
underlying psychological disorder.
39062

Next Question

Previous Question

Tag Question

Feedback

End Review

Difficulty: Average
Peer Responses

Session Progress
Responses Correct:

Responses Incorrect:

298

Responses Total:

298

Responses - % Correct:

0%

Blog (https://www.pastest.com/blog) About Pastest (https://www.pastest.com/about-us)


Contact Us (https://www.pastest.com/contact-us) Help (https://www.pastest.com/help)
Pastest 2016

https://mypastest.pastest.com/Secure/TestMe/Browser/429893#Top

2/2

8/22/2016

MyPastest

Back to Filters (/Secure/TestMe/Filter/429893/QA)

Question 182 of 298

A 54-year-old man who is known to drink alcohol to excess comes to the Emergency
Department for review. He has severe burning epigastric pain radiating to his back that has
increased in severity over the past 6 weeks. He has lost 3 stones in weight over the past 4
months and over the past 3 days has begun to suffer from severe itching. A past medical
history of hypertension managed with Bendroflumethiazide is noted. On examination his BP
is 105/70 mmHg, pulse is 85/min and regular. He has jaundiced sclerae and there are scratch
marks across the upper body. Abdominal examination reveals a fullness in the epigastrium
and he is in obvious pain.
Investigations;
Hb

10.2 g/dl

WCC

11.4 x109/l

PLT

167 x109/l

Na+

137 mmol/l

K+

3.9 mmol/l

Creatinine 122 micromol/l


Glucose

5.4 mmol/l

ALP

381 U/l

ALT

112 U/l

Bilirubin

71 micromol/l

GGT

312 U/l

Which of the following is the most useful next investigation?


A

CEA

CA 19-9

CT abdomen

ERCP

https://mypastest.pastest.com/Secure/TestMe/Browser/429893#Top

1/2

8/22/2016

MyPastest

USS abdomen

Explanation
The answer is CT abdomen The symptoms, signs and blood results seen here are suggestive of biliary obstruction due to
possible underlying pancreatic carcinoma. Non-invasive imaging is the appropriate next step,
Between the two choices, CT and USS abdomen, there is a greater possibility that ultrasound
may miss a pancreatic mass; as such CT is the best next step. ERCP can then be considered
to further evaluate any underlying tumour, and give the opportunity for brush or forceps
biopsy. CA 19-9 is a tumour marker associated pancreatic carcinoma, although it used to
monitor for recurrance post surgical resection, rather than to make the initial diagnosis. CEA
is elevated in a range of GI tumours and in medullary thyroid carcinoma.
40126

Next Question

Previous Question

Tag Question

Feedback

End Review

Difficulty: Average
Peer Responses

Session Progress
Responses Correct:

Responses Incorrect:

298

Responses Total:

298

Responses - % Correct:

0%

Blog (https://www.pastest.com/blog) About Pastest (https://www.pastest.com/about-us)


Contact Us (https://www.pastest.com/contact-us) Help (https://www.pastest.com/help)
Pastest 2016

https://mypastest.pastest.com/Secure/TestMe/Browser/429893#Top

2/2

8/22/2016

MyPastest

Back to Filters (/Secure/TestMe/Filter/429893/QA)

Question 183 of 298

A 62-year-old woman is reviewed in the Oncology Clinic following resection of a Dukes B


carcinoma from her transverse colon some 3 months earlier. She has completely recovered
from the operation and is looking to the future.
Which of the following strategies for monitoring for recurrence is best supported by
evidence?
A

CEA monitoring

CEA and CT scanning

CT scanning

CA19-9 and CT scanning

Alpha-fetoprotein and CT scanning

Explanation
The answer is CEA monitoring An RCT conducted in the UK looked at the value of CT scanning, CEA measurement and the
combination of the two in monitoring for colon cancer recurrence and subsequent survival.
They found no difference between CEA alone, CT alone and the combination of the two on
outcomes. As such it is most appropriate to avoid the significant radiation load associated
with CT scanning, given it does not confer an additive survival benefit. CA 19-9 is commonly
used to monitor for pancreatic cancer recurrence, and alpha-fetoprotein is used to monitor
for hepatocellular carcinoma.
http://jama.jamanetwork.com/article.aspx?articleID=1814213
(http://jama.jamanetwork.com/article.aspx?articleID=1814213)
40156

Next Question

Previous Question

Tag Question

https://mypastest.pastest.com/Secure/TestMe/Browser/429893#Top

Feedback

End Review

1/2

8/22/2016

MyPastest

Difficulty: Average
Peer Responses

Session Progress
Responses Correct:

Responses Incorrect:

298

Responses Total:

298

Responses - % Correct:

0%

Blog (https://www.pastest.com/blog) About Pastest (https://www.pastest.com/about-us)


Contact Us (https://www.pastest.com/contact-us) Help (https://www.pastest.com/help)
Pastest 2016

https://mypastest.pastest.com/Secure/TestMe/Browser/429893#Top

2/2

8/22/2016

MyPastest

Back to Filters (/Secure/TestMe/Filter/429893/QA)

Question 184 of 298

A 24-year-old man is admitted to the Emergency department following a coffee ground


vomit. Apparently he was out with friends celebrating his birthday when he drank 8 pints of
strong lager and a number of shots of spirits. He began to vomit and retch uncontrollably.
When he vomited for the third time, after a number of retches, he produced a coffee ground
vomit. He has no past medical history of note and is a corporal in the army. On examination
he is clearly intoxicated, his BP is 115/72 mmHg, pulse is 82/min and regular. With the help of
his friends, you stand him up, and there is no postural drop in BP. He is mildly tender in the
epigastrium.
Investigations;
Hb

14.2 g/dl

WCC

6.3 x109/l

PLT

191 x109/l

Na+

137 mmol/l

K+

4.5 mmol/l

Creatinine 90 micromol/l
ALT

42 U/l

ALP

91 U/l

Albumin

40 g/l

Bilirubin

10 micromol/l

Which of the following is the most likely diagnosis?


A

Alcoholic gastritis

Duodenal ulcer

Gastro-oesophageal reflux disease (GORD)

Mallory Weiss tear

https://mypastest.pastest.com/Secure/TestMe/Browser/429893#Top

1/2

8/22/2016

MyPastest

Oesophageal varices

Explanation
The answer is Mallory Weiss tear The history of excessive retching followed by a coffee ground vomit is very typical of a
Mallory Weiss tear, due to mucosal lacerations in the region of the gastro-oesophageal
junction. This history also steers us away from alternative diagnoses such as duodenal ulcer
or GORD. It is clear that there is no haemodynamic compromise; as long as vomiting settles
overnight and there is no deterioration, there is no need to proceed to endoscopy. It is highly
unlikely this man could function as a serving soldier with liver disease significant enough to
cause oesophageal varices, and the LFT picture is benign on admission bloods.
40157

Next Question

Previous Question

Tag Question

Feedback

End Review

Difficulty: Easy
Peer Responses

Session Progress
Responses Correct:

Responses Incorrect:

298

Responses Total:

298

Responses - % Correct:

0%

Blog (https://www.pastest.com/blog) About Pastest (https://www.pastest.com/about-us)


Contact Us (https://www.pastest.com/contact-us) Help (https://www.pastest.com/help)
Pastest 2016

https://mypastest.pastest.com/Secure/TestMe/Browser/429893#Top

2/2

8/22/2016

MyPastest

Back to Filters (/Secure/TestMe/Filter/429893/QA)

Question 185 of 298

A 25-year-old woman presents to the Emergency Department with anorexia, vomiting, and
deep jaundice. She lives alone and according to a neighbour has recently lost her job due to
problems with alcoholism and poor performance. According to paramedics the house was
filthy with a number of rats droppings around the premises and rotting food. On examination
her BP is 95/70 mmHg; pulse is 90/min and regular. She is deeply jaundiced, drowsy and
confused. Abdominal examination reveals tenderness in the right upper quadrant.
Investigations;
Hb

10.9 g/dl

WCC

11.8 x109/l

PLT

72 x109/l

PT

61.4 s

APTT

41.0 s

Na+

134 mmol/l

K+

5.4 mmol/l

Creatinine

149 micromol/l

Glucose

4.0 mmol/l

ALT

3100 U/l

AST

2950 U/l

Bilirubin

132 micromol/l

Albumin

28 g/l

ALP

395 U/l

Which of the following is the most likely diagnosis?


A

Alcoholic hepatitis

Hepatitis A

https://mypastest.pastest.com/Secure/TestMe/Browser/429893#Top

1/3

8/22/2016

MyPastest

Hepatitis B

Paracetamol overdose

Weils disease

Explanation
The answer is Paracetamol overdose There is a suspicion this patient is depressed and has a significant alcohol problem, putting
her at risk of paracetamol overdose. The similar values for AST and ALT and values above
3000 count against alcoholic hepatitis, where transaminases rarely exceed 500U/l, and the
AST: ALT ratio is often>2.0. It is likely that she took the overdose a few days earlier, and is
presenting late to the Emergency Department. Whilst Weils disease is a possibility, it is much
less likely than paracetamol overdose.
40185

Next Question

Previous Question

Tag Question

Feedback

End Review

Difficulty: Difficult
Peer Responses

Session Progress
Responses Correct:

Responses Incorrect:

298

Responses Total:

298

Responses - % Correct:

0%

Blog (https://www.pastest.com/blog) About Pastest (https://www.pastest.com/about-us)


Contact Us (https://www.pastest.com/contact-us) Help (https://www.pastest.com/help)
Pastest 2016
https://mypastest.pastest.com/Secure/TestMe/Browser/429893#Top

2/3

8/22/2016

https://mypastest.pastest.com/Secure/TestMe/Browser/429893#Top

MyPastest

3/3

8/22/2016

MyPastest

Back to Filters (/Secure/TestMe/Filter/429893/QA)

Question 186 of 298

A 42-year-old woman is admitted to the Gastroenterology Ward following an upper GI bleed.


She is found to have changes consistent with chronic liver disease and oesophageal varices
on endoscopy. On further questioning it transpires that she used IV heroin during her
twenties, although she has not used for the past 15 years. Examination reveals a BP of 122/82
mmHg, pulse is 70/min and regular. There are spider naevi over the upper body and there is
minor pitting oedema affecting both ankles.
Investigations:
Hb

10.9 g/dl

WCC

7.0 x109/l

PLT

99 x109/l

Na+

135 mmol/l

K+

3.5 mmol/l

Creatinine

85 micromol/l

Glucose

4.5 mmol/l

ALT

112 U/l

ALP

195 U/l

Bilrubin

18 micromol/l

Anti-HCV antibody positive


Hep C RNA

positive

Which of the following is the most important factor with respect to duration of anti-viral
therapy?
A

ALT

ALP

Hepatitis C genotype

https://mypastest.pastest.com/Secure/TestMe/Browser/429893#Top

1/3

8/22/2016

MyPastest

History of varices

Liver biopsy

Explanation
The answer is Hepatitis C genotype Geontypes 1,4,5 and 6 require 48 weeks of treatment with conventional therapy for hepatitis
C (48 weeks of pegylated Inteferon and Ribavirin), Genotypes 2 and 3 require 24 weeks of
anti-viral therapy. Modern anti-Hepatitis C agents Boceprevir and Telaprevir act by blocking
the activity of NS3/4A. When Boceprevir was added to Interferon and Ribavirin, response
rates over 48 weeks of therapy for genotype 1 disease increased from 38% to greater than
60%. Significant liver decompensation impacts more on the choice of anti-viral agent rather
than the duration of therapy. Sofosbuvir, a nucleotide analogue is also recommended by NICE
as a component of triple therapy for hepatitis C.
http://www.nice.org.uk/guidance/ta253/documents/hepatitis-c-genotype-1-boceprevir-finalappraisal-determination-document2
(http://www.nice.org.uk/guidance/ta253/documents/hepatitis-c-genotype-1-boceprevir-finalappraisal-determination-document2)
40233

Next Question

Previous Question

Tag Question

Feedback

End Review

Difficulty: Average
Peer Responses

Session Progress
Responses Correct:

Responses Incorrect:

298

Responses Total:

298

Responses - % Correct:

https://mypastest.pastest.com/Secure/TestMe/Browser/429893#Top

0%

2/3

8/22/2016

MyPastest

Blog (https://www.pastest.com/blog) About Pastest (https://www.pastest.com/about-us)


Contact Us (https://www.pastest.com/contact-us) Help (https://www.pastest.com/help)
Pastest 2016

https://mypastest.pastest.com/Secure/TestMe/Browser/429893#Top

3/3

8/22/2016

MyPastest

Back to Filters (/Secure/TestMe/Filter/429893/QA)

Question 187 of 298

A 45-year-old alcoholic man is admitted to the Emergency Department with severe epigastric
pain and vomiting. He has suffered from two previous episodes of pancreatitis and continues
to drink 0.5l of whisky per day. Medication includes indapamide for hypertension and a
salbutamol inhaler for mild asthma. On examination he has a BP of 100/60 mmHg; pulse is
95/min and regular. His temperature is 38.2C. Abdominal palpation reveals severe tenderness
in the epigastrium. His BMI is 22.
Investigations;
Hb

10.9 g/dl

WCC

13.1 x109/l

PLT

120 x109/l

Na+

138 mmol/l

K+

3.7 mmol/l

Creatinine 100 micromol/l


Urea

12.3 mmol/l

Ca++

1.95 mmol/l

ALT

85 U/l

LDH

490 U/l

Albumin

34 g/l

Amylase

740 U/l

Which of the following is a marker of severity according to the modified Glasgow criteria?
A

Albumin 34 g/l

ALT 85 U/l

Calcium 1.95 mmol/l

LDH 490 U/l

https://mypastest.pastest.com/Secure/TestMe/Browser/429893#Top

1/3

8/22/2016

MyPastest

WCC 13.1 x109/l

Explanation
The answer is Calcium 1.95mmol/l Significant hypocalcaemia, as indicated by uncorrected calcium <2.0mmol/l is a marker of
severity according to the modified Glasgow criteria.
Other criteria are listed below:
age >55 years
pO2 <8.0 kPa
WCC >15x109/litre
ALT >100 IU
LDH >600 IU
glucose >10 mmol/L
urea >16 mmol/L
albumin <32g/L
A score of 3 or more predicts a severe episode of pancreatitis.
40234

Next Question

Previous Question

Tag Question

Feedback

End Review

Difficulty: Average
Peer Responses

Session Progress
Responses Correct:

Responses Incorrect:

298

Responses Total:

298

Responses - % Correct:

https://mypastest.pastest.com/Secure/TestMe/Browser/429893#Top

0%

2/3

8/22/2016

MyPastest

Blog (https://www.pastest.com/blog) About Pastest (https://www.pastest.com/about-us)


Contact Us (https://www.pastest.com/contact-us) Help (https://www.pastest.com/help)
Pastest 2016

https://mypastest.pastest.com/Secure/TestMe/Browser/429893#Top

3/3

8/22/2016

MyPastest

Back to Filters (/Secure/TestMe/Filter/429893/QA)

Question 188 of 298

A 56 year old woman presents to the Combined Surgical and Oncology Clinic for review. She
presents with a 1-year history of change in bowel habit, and has a locally advanced rectal
carcinoma which is abutting the bladder anteriorly. She has a good functional status, is still
working part time as a solicitor, and walks up to 2 miles per day. CT of the abdomen with
contrast does not reveal any evidence of hepatic metastases. On examination her BP is
135/70 mmHg; pulse is 65/min and regular. Heart sounds are normal and her chest is clear.
Her abdomen is soft and non-tender and she has a BMI of 24.
Investigations;
Hb

11.0 g/l

WCC

7.4 x109/l

PLT

189 x109/l

Na+

137 mmol/l

K+

4.3 mmol/l

Creatinine

100 micromol/l

ALT

54 U/l

ALP

95 U/l

Bilirubin

12 micromol/l

Which of the following is the most appropriate next step?


A

Neoadjuvant chemoradiotherapy

Palliative radiotherapy

Proceed to surgery

Proceed to surgery with adjuvant chemotherapy

Refer for end of life care

https://mypastest.pastest.com/Secure/TestMe/Browser/429893#Top

1/2

8/22/2016

MyPastest

Explanation
The answer is Neoadjuvant chemoradiotherapy In this situation clinical trials suggest that pre-operative chemoradiotherapy for locally
advanced rectal carcinoma is associated with a reduced rate of pelvic relapse, although
survival rates are similar for pre- versus post-operative chemoradiotherapy at the 10 year
time point. Surgery without other intervention is definitely associated with poorer overall
outcomes and is therefore not the correct option here. Equally, given her age, good
functional status and lack of distant metastases, referral for palliation is inappropriate.
40235

Next Question

Previous Question

Tag Question

Feedback

End Review

Difficulty: Difficult
Peer Responses

Session Progress
Responses Correct:

Responses Incorrect:

298

Responses Total:

298

Responses - % Correct:

0%

Blog (https://www.pastest.com/blog) About Pastest (https://www.pastest.com/about-us)


Contact Us (https://www.pastest.com/contact-us) Help (https://www.pastest.com/help)
Pastest 2016

https://mypastest.pastest.com/Secure/TestMe/Browser/429893#Top

2/2

8/22/2016

MyPastest

Back to Filters (/Secure/TestMe/Filter/429893/QA)

Question 189 of 298

A 54-year-old man is admitted with an episode of acute pancreatitis. He presents with severe
vomiting, hypocalcaemia and an elevated urea and is admitted to the intensive care unit for
further management. You are reviewing his nutrition and need to complete a referral to the
on-call dietetics service.
Which of the following is the most appropriate plan for nutrition?
A

Encourage oral intake

IV fluids then feed in 24hrs

Nasogastric feeding

Nasojejunal feeding

Total parenteral nutrition

Explanation
The answer is Nasogastric feeding In this situation two important factors are taken into account with respect to nutrition: firstly,
maintaining an adequate calorie intake at a time of significant catabolism; and secondly,
maintaining the integrity of the GI tract against bacterial incursion. At a time of significant
nausea and vomiting, encouraging oral intake is unlikely to be sufficient. Nasogastric feeding
is however likely to achieve sufficient calorie intake and maintain the integrity of the upper GI
tract. As an alternative, if NG feeding is not tolerated because of vomiting, nasojejunal
feeding is a potential alternative. TPN should be considered only if NG or NJ feeding are
inappropriate.
40236

Next Question

Previous Question

Tag Question

Feedback

End Review

Difficulty: Difficult
https://mypastest.pastest.com/Secure/TestMe/Browser/429893#Top

1/2

8/22/2016

MyPastest

Peer Responses

Session Progress
Responses Correct:

Responses Incorrect:

298

Responses Total:

298

Responses - % Correct:

0%

Blog (https://www.pastest.com/blog) About Pastest (https://www.pastest.com/about-us)


Contact Us (https://www.pastest.com/contact-us) Help (https://www.pastest.com/help)
Pastest 2016

https://mypastest.pastest.com/Secure/TestMe/Browser/429893#Top

2/2

8/22/2016

MyPastest

Back to Filters (/Secure/TestMe/Filter/429893/QA)

Question 190 of 298

A 26-year-old woman complains of tiredness, abdominal pain and intermittent diarrhoea with
a 10-kg weight loss over a period of 6 months. She is pale and has an itchy vesicular rash on
her elbows and knees. She has a microcytic, hypochromic anaemia and low ferritin and folate
levels, a low serum albumin and normal free T4, but a slightly raised thyroid-stimulating
hormone (TSH) level.
Which of the following investigations is most appropriate?
A

Bone marrow examination

Colonoscopy

Duodenal biopsy

Schilling test

Small-bowel follow-through

Explanation

The answer is Duodenal biopsy


This woman has symptoms, signs and a biochemical picture of malabsorption. The
description of an itchy vesicular rash over extensor areas suggests dermatitis
herpetiformis and this, combined with her malabsorption, are together highly suggestive
of coeliac disease. Coeliac serology should be performed, although a duodenal biopsy is
required to confirm the diagnosis definitively. She should be managed with a gluten-free
diet and her iron and folate stores replenished. Coeliac disease is associated with a
number of other conditions. This woman also has subclinical hypothyroidism and her
thyroid antibodies should be checked. Insulin-dependent diabetes mellitus,
hypothyroidism, chronic liver disease and fibrosing alveolitis are all more commonly seen
in patients with coeliac disease.

Bone marrow examination (Option A) is incorrect. Although bone marrow examination may
be a useful investigation for anaemia of uncertain aetiology, the clinical scenario given is
much more suggestive of her anaemia representing iron malabsorption rather than primary
bone marrow failure, making option B incorrect.
https://mypastest.pastest.com/Secure/TestMe/Browser/429893#Top

1/3

8/22/2016

MyPastest

Colonoscopy (Option B) is incorrect. Although this womans chronic diarrhoea might suggest
that colonoscopy is appropriate, her blood tests are very suggestive of malabsorption.
Malabsorption does not occur through primary large bowel disease, so a colonoscopy is
unlikely to be helpful.
The Schilling test (Option D) is incorrect. The Schilling test has been one of the classical tests
used to evaluate whether vitamin B12 deficiency is caused by pernicious anaemia. However, it
is performed in only very few centres now that antibody testing for anti-intrinsic factor
antibodies has become so widely available. No mention is given of B12 deficiency here, so the
test is unlikely to be useful.
A small bowel follow-through (Option E) is incorrect. Small bowel follow-through remains a
useful means of looking for small bowel disease (including narrowing of the lumen in
inflammation), and would be particularly useful if this woman had suspected small bowel
Crohns disease as the cause of her diarrhoea and malabsorption. However, the typical
description of dermatitis herpetiformis given here makes coeliac disease much more likely to
be the diagnosis, and duodenal biopsy would therefore be the better first-line test.
44879

Next Question

Previous Question

Tag Question

Feedback

End Review

Difficulty: Average
Peer Responses

Session Progress
Responses Correct:

Responses Incorrect:

298

Responses Total:

298

Responses - % Correct:

0%

Blog (https://www.pastest.com/blog) About Pastest (https://www.pastest.com/about-us)


Contact Us (https://www.pastest.com/contact-us) Help (https://www.pastest.com/help)
Pastest 2016

https://mypastest.pastest.com/Secure/TestMe/Browser/429893#Top

2/3

8/22/2016

https://mypastest.pastest.com/Secure/TestMe/Browser/429893#Top

MyPastest

3/3

8/22/2016

MyPastest

Back to Filters (/Secure/TestMe/Filter/429893/QA)

Question 191 of 298

A 56-year-old man complains of diarrhoea, abdominal pain, weight loss and joint pains, with
two or three pale, bulky stools daily. A duodenal biopsy shows stunted villi and stains
macrophages positively with PAS stain, and electron microscopy shows bacilli within the
macrophages.
What is the best treatment?
A

Amoxicillin

Anti-tuberculosis treatment

Gluten-free diet

Low-fat diet

Metronidazole

Explanation

The answer is Amoxicillin


This patient has Whipples disease. The causative organism is the Gram-positive bacillus
Tropheryma whipplei. Diarrhoea and arthropathy are typical presenting features of the
condition. The best test for the condition is a duodenal biopsy, with subtotal villous
atrophy, PAS staining of macrophages and the presence of bacilli within macrophages
confirming the diagnosis. Although Whipples disease most typically involves the
gastrointestinal tract, involvement of the heart, brain and lungs is well recognised too.
Possible choices for antibiotic therapy include amoxicillin, co-trimoxazole and
chloramphenicol.

Anti-tuberculosis treatment (Option B) is incorrect. Whipples disease is caused by bacteria


rather than mycobacteria, so anti-tuberculosis treatment will be of no benefit.
Gluten-free diet (Option C) is incorrect. Although the combination of diarrhoea and stunted
villi on duodenal biopsy may suggest coeliac disease, coeliac disease would not explain the
staining of macrophages with PAS stain or the presence of bacilli within macrophages. A

https://mypastest.pastest.com/Secure/TestMe/Browser/429893#Top

1/2

8/22/2016

MyPastest

gluten-free diet would be the right choice of treatment for coeliac disease, but will not be
effective here.
A low-fat diet (Option D) is incorrect. A low-fat diet may reduce some of this mans
diarrhoea, but it does not provide any specific treatment for his condition, so is not the
correct choice here.
Metronidazole (Option E) is incorrect. Metronidazole is not recognised as an antibiotic that is
effective against the condition.
44880

Next Question

Previous Question

Tag Question

Feedback

End Review

Difficulty: Average
Peer Responses

Session Progress
Responses Correct:

Responses Incorrect:

298

Responses Total:

298

Responses - % Correct:

0%

Blog (https://www.pastest.com/blog) About Pastest (https://www.pastest.com/about-us)


Contact Us (https://www.pastest.com/contact-us) Help (https://www.pastest.com/help)
Pastest 2016

https://mypastest.pastest.com/Secure/TestMe/Browser/429893#Top

2/2

8/22/2016

MyPastest

Back to Filters (/Secure/TestMe/Filter/429893/QA)

Question 192 of 298

A 55-year-old man complains of a recurrent bluish-red rash on his neck, wheeze, abdominal
pain and watery diarrhoea. He has facial telangiectasia, mild pedal oedema, an elevated
jugular venous pressure (JVP), a pansystolic murmur and moderate hepatomegaly.
What is the most likely diagnosis?
A

Carcinoid syndrome

Carcinoma of the pancreas with liver metastases

Intestinal tuberculosis

Lymphoma

Tropical sprue

Explanation

The answer is Carcinoid syndrome


Carcinoid syndrome occurs in only 5% of patients with carcinoid tumours, when there are
liver metastases. Although the serotonin (and other vasoactive molecules) produced by
these neuroendocrine tumours are normally degraded by the liver, infiltration of the liver
by metastases means that there is a lack of normal liver parenchyma to perform this role,
and these molecules enter the systemic circulation where they cause their effects.
Carcinoid syndrome manifests as bluish-red flushing (mainly on the neck and face),
bronchospasm, abdominal pain and recurrent watery diarrhoea. Cardiac abnormalities
develop in 50% of patients and consist of tricuspid regurgitation or pulmonary stenosis.

Carcinoma of the pancreas with liver metastases (Option B) is incorrect. Carcinoid tumours
may occur throughout the gastrointestinal tract (including the pancreas), although the vast
majority of pancreatic cancers are non-secretory adenocarcinomas that would not give the
sort of symptoms described here.
Intestinal tuberculosis (Option C) is incorrect. Intestinal tuberculosis may present with any of
a wide range of gastrointestinal symptoms, including diarrhoea and abdominal pain; however,
this option is incorrect as rash, wheeze and cardiac failure would not be explained by TB.
https://mypastest.pastest.com/Secure/TestMe/Browser/429893#Top

1/2

8/22/2016

MyPastest

Lymphoma (Option D) is incorrect. Although lymphoma may present with a wide range of
symptoms (including diarrhoea and abdominal pain, in the case of intestinal lymphoma), the
constellation of rash, wheeze and heart failure would not be explained by lymphoma.
Tropical sprue (Option E) is incorrect. Tropical sprue is a condition of presumed infectious
aetiology that affects the small intestine; although it may present with
diarrhoea/malabsorption, none of the other symptoms described here would be typical.
44881

Next Question

Previous Question

Tag Question

Feedback

End Review

Difficulty: Average
Peer Responses

Session Progress
Responses Correct:

Responses Incorrect:

298

Responses Total:

298

Responses - % Correct:

0%

Blog (https://www.pastest.com/blog) About Pastest (https://www.pastest.com/about-us)


Contact Us (https://www.pastest.com/contact-us) Help (https://www.pastest.com/help)
Pastest 2016

https://mypastest.pastest.com/Secure/TestMe/Browser/429893#Top

2/2

8/22/2016

MyPastest

Back to Filters (/Secure/TestMe/Filter/429893/QA)

Question 193 of 298

A 40-year-old man with coeliac disease who has good concordance with a gluten-free diet
complains of a recurrence of his symptoms.
What is the most likely diagnosis?
A

Bacterial overgrowth

Crohns disease

Giardia infection

Intestinal lymphangiectasia

Intestinal lymphoma

Explanation

The answer is Intestinal lymphoma


The major cause of people with coeliac disease having ongoing gastrointestinal
symptoms is because of incomplete concordance with a gluten-free diet. If this has been
excluded, then intestinal lymphoma should be considered, since this is a well-recognised
potential complication of coeliac disease. The diagnosis of intestinal lymphoma may be
hard to make, and a combination of GI endoscopy, imaging and biopsy is often required.
Coeliac disease also has a rare (but well-recognised) association with small bowel
adenocarcinoma.

Bacterial overgrowth (Option A) is incorrect. Small bowel bacterial overgrowth is associated


with a number of GI conditions (Crohns disease, previous abdominal surgery, etc), along with
other conditions that affect the immunology and motility of the gut (including diabetes
mellitus, hypogammaglobulinaemia, etc). Diarrhoea and abdominal discomfort are common
symptoms. However, it does not have a particular association with coeliac disease, so it is not
the most likely diagnosis here.
Crohns disease (Option B) is incorrect. Although certainly not impossible, it is somewhat
unusual for Crohns disease to first present in a man of this age. Crohns disease has no close
association with coeliac disease. As such, it is the incorrect answer here.
https://mypastest.pastest.com/Secure/TestMe/Browser/429893#Top

1/2

8/22/2016

MyPastest

Giardia infection (Option C) is incorrect. Giardia infection can certainly cause symptoms
similar to coeliac disease (including loose stool, abdominal discomfort and bloating), but
again is not directly associated with it. As such, it is not the right answer here.
Intestinal lymphangiectasia (Option D) is incorrect. Intestinal lymphangiectasia is a condition
defined by dilated lymphatic vessels. It often results in a protein-losing enteropathy
manifesting clinically with symptoms including diarrhoea. However, it is a rare disorder that
has no well-recognised association with coeliac disease, so it is unlikely to be the explanation
here.
44882

Next Question

Previous Question

Tag Question

Feedback

End Review

Difficulty: Average
Peer Responses

Session Progress
Responses Correct:

Responses Incorrect:

298

Responses Total:

298

Responses - % Correct:

0%

Blog (https://www.pastest.com/blog) About Pastest (https://www.pastest.com/about-us)


Contact Us (https://www.pastest.com/contact-us) Help (https://www.pastest.com/help)
Pastest 2016

https://mypastest.pastest.com/Secure/TestMe/Browser/429893#Top

2/2

8/22/2016

MyPastest

Back to Filters (/Secure/TestMe/Filter/429893/QA)

Question 194 of 298

A 40-year-old woman with pernicious anaemia is admitted with jaundice. She drinks 10 units
of alcohol per week. Examination shows bruises on her arms and legs, palmar erythema,
spider naevi and hepatosplenomegaly. She also complains of joint pains and amenorrhoea of
recent onset. She is on long-term nitrofurantoin for recurrent urinary tract infection.
Investigations show a mild, normochromic, normocytic anaemia, bilirubin 89 mol/l (normal
range < 17 mol/l) and aspartate transaminase (AST) 450 U/l (1040 U/l).
What is the most likely diagnosis?
A

Alcoholic liver disease

Idiopathic cirrhosis

Nitrofurantoin-induced chronic active hepatitis

Primary biliary cholangitis

Viral hepatitis

Explanation

The answer is Nitrofurantoin-induced chronic active hepatitis


A huge range of different drugs have been recognised to potentially cause drug-induced
liver injury (DILI). The clinical spectrum of this may range from a small perturbation of
liver enzymes through to more dramatic presentations, including acute liver failure.
Nitrofurantoin is one of the more common causes of DILI, and may cause a chronic active
hepatitis syndrome, which results in symptoms and signs of decompensated chronic liver
disease. Improvement is likely to occur following the withdrawal of nitrofurantoin. This
chronic active hepatitis may be accompanied by features of autoimmune disease;
common clinical features include symptoms such as the joint pain described here, and
autoantibodies (eg anti-liver/kidney microsomal antibodies) may be positive. The
normocytic anaemia may also be consistent with anaemia of chronic disease, whereas
amenorrhoea may reflect chronic liver disease.

Alcoholic liver disease (Option A) is incorrect. This woman has features of chronic liver
disease, but drinks alcohol within recommended limits, meaning this is unlikely to be the
https://mypastest.pastest.com/Secure/TestMe/Browser/429893#Top

1/2

8/22/2016

MyPastest

diagnosis.
Idiopathic cirrhosis (Option B) is incorrect. Although a small number of cases of cirrhosis are
idiopathic, the constellation of features of chronic liver disease, features suggestive of
immune overactivity (pernicious anaemia, joint pains, etc) and the use of nitrofurantoin
collectively mean that Option C is far more likely than Option B.
Primary biliary cholangitis (Option D) is incorrect. Primary biliary cholangitis (PBC; until
recently known as primary biliary cirrhosis) may present with non-specific symptoms,
although fatigue and pruritus are common. PBC is a cholestatic liver disease, so typically
manifests with a raised alkaline phosphatase but normal transaminases; an AST of 450 U/l
would be inconsistent with the diagnosis of PBC, and much more consistent with a form of
hepatitis, making Option A incorrect. Anti-mitochondrial antibody (AMA) would be the test
of choice to assess for suspected PBC.
Viral hepatitis (Option E) is incorrect. Her raised bilirubin and AST indicate a hepatitic
process, but no risk factors for viral hepatitis are described.
44883

Next Question

Previous Question

Tag Question

Feedback

End Review

Difficulty: Average
Peer Responses

Session Progress
Responses Correct:

Responses Incorrect:

298

Responses Total:

298

Responses - % Correct:

0%

Blog (https://www.pastest.com/blog) About Pastest (https://www.pastest.com/about-us)


Contact Us (https://www.pastest.com/contact-us) Help (https://www.pastest.com/help)
Pastest 2016

https://mypastest.pastest.com/Secure/TestMe/Browser/429893#Top

2/2

8/22/2016

MyPastest

Back to Filters (/Secure/TestMe/Filter/429893/QA)

Question 195 of 298

A 56-year-old man with polycythaemia rubra vera is admitted with acute abdominal pain,
nausea, vomiting and abdominal distension. He is apyrexial, with tender hepatomegaly and
ascites. An ascitic tap reveals fluid with a high protein content and no organisms.
What is the most likely diagnosis?
A

BuddChiari syndrome

Haemochromatosis

Malignant liver disease

Mesenteric artery occlusion

Spontaneous bacterial peritonitis

Explanation

The answer is BuddChiari syndrome


BuddChiari syndrome occurs following obstruction to the venous outflow system of the
liver due to occlusion of the hepatic vein. In approximately one-third of patients with the
condition, the cause is unknown. Patients with hypercoagulable states (such as
polycythaemia or leukaemia) and women taking the oral contraceptive pill are at
increased risk. The syndrome may present as either an acute or chronic form. Typical
symptoms in the acute form include abdominal pain, nausea, vomiting, tender
hepatomegaly and ascites. Liver histology shows centrilobular congestion with
hepatocyte atrophy. The chronic form presents with hepatomegaly, mild jaundice, ascites
and splenomegaly with portal hypertension.

Haemochromatosis (Option B) is incorrect. Haemochromatosis (as well as other infiltrative


diseases) may explain hepatomegaly, although this would not typically be associated with
the other symptoms described. It is recognised that haemochromatosis may cause chronic
liver disease, but this tends to cause ascites with a low protein count, making it unlikely here.
Malignant liver disease (Option C) is incorrect. Malignant liver disease may explain ascites
with a high protein count, but it is unlikely to present as such an acute syndrome until very
https://mypastest.pastest.com/Secure/TestMe/Browser/429893#Top

1/2

8/22/2016

MyPastest

late in the disease process. In addition, no information is given regarding a cancer that may
have metastasised to the liver, or of risk factors for chronic liver disease that may cause a
hepatocellular carcinoma.
Mesenteric artery occlusion (Option D) is incorrect. Mesenteric ischaemia can present with
acute abdominal pain and vomiting, but the constellation of tender hepatomegaly and
ascites is much more suggestive of venous outflow obstruction than arterial insufficiency.
Spontaneous bacterial peritonitis (Option E) is incorrect. Although the symptoms described
and the presence of ascites with a high protein count are consistent with spontaneous
bacterial peritonitis (SBP), other features in the scenario do not fit with this diagnosis.
Specifically, a high neutrophil count (> 250 neutrophils/mm3) and organisms on ascitic fluid
culture would be expected (and sometimes a fever), but none of these is described here. In
addition, tender hepatomegaly is not a typical feature of SBP.
44884

Next Question

Previous Question

Tag Question

Feedback

End Review

Difficulty: Average
Peer Responses

Session Progress
Responses Correct:

Responses Incorrect:

298

Responses Total:

298

Responses - % Correct:

0%

Blog (https://www.pastest.com/blog) About Pastest (https://www.pastest.com/about-us)


Contact Us (https://www.pastest.com/contact-us) Help (https://www.pastest.com/help)
Pastest 2016

https://mypastest.pastest.com/Secure/TestMe/Browser/429893#Top

2/2

8/22/2016

MyPastest

Back to Filters (/Secure/TestMe/Filter/429893/QA)

Question 196 of 298

A 45-year-old woman with ulcerative colitis is admitted with a history of jaundice, pruritus
and intermittent abdominal pain. Examination shows hepatosplenomegaly and mild ascites.
Blood tests confirm an obstructive jaundice, and anti-mitochondrial antibodies are not
detected.
What is the most likely diagnosis?
A

Chronic active hepatitis

Liver cirrhosis

Metastatic carcinoma

Pancreatic carcinoma

Sclerosing cholangitis

Explanation

The answer is Sclerosing cholangitis


This patient has inflammatory bowel disease and is therefore at risk of developing
sclerosing cholangitis. This results from inflammation and fibrosis of the bile ducts, which
leads to the formation of multiple areas of narrowing throughout the biliary system.
Patients with HIV with low CD4 counts may also develop sclerosing cholangitis. Affected
patients can be asymptomatic or can present with jaundice, pruritus and intermittent
abdominal pain. The alkaline phosphatase level will be raised and anti-mitochondrial
antibody is not detectable (in contrast to PSC). Liver biopsy will show fibrous obliterative
cholangitis with loss of interlobular and adjacent septal bile ducts. Endoscopic
retrograde cholangiopancreatography (ERCP) will demonstrate the multiple strictures.
Affected patients require close surveillance for the development of cholangiocarcinoma
and colonic carcinoma, along with surveillance for hepatocellular carcinoma if chronic
liver disease develops. There are no specific medical therapies; liver transplantation may
be required in the most severe cases.

Chronic active hepatitis (Option A) is incorrect. This patients blood tests show an
obstructive picture with no mention of raised transaminases, so chronic active hepatitis is not
https://mypastest.pastest.com/Secure/TestMe/Browser/429893#Top

1/2

8/22/2016

MyPastest

the most likely diagnosis.


Liver cirrhosis (Option B) is incorrect. Although early chronic liver disease may be associated
with hepatomegaly, cirrhosis is often associated with a shrunken liver, meaning it is unlikely
to be the diagnosis here.
Metastatic carcinoma (Option C) is incorrect. Metastatic carcinoma with malignant infiltration
of the liver may cause hepatomegaly, cholestatic liver enzymes, abdominal pain and ascites,
but the description of splenomegaly would not be typically consistent with this diagnosis.
Pancreatic carcinoma (Option D) is incorrect. On a similar note, although a cancer of the
head of the pancreas may present with obstructive jaundice (often painlessly, with tumours
more distally in the pancreas often first presenting with abdominal pain), pancreatic
carcinomas are not a typical cause of hepatosplenomegaly.
44885

Next Question

Previous Question

Tag Question

Feedback

End Review

Difficulty: Average
Peer Responses

Session Progress
Responses Correct:

Responses Incorrect:

298

Responses Total:

298

Responses - % Correct:

0%

Blog (https://www.pastest.com/blog) About Pastest (https://www.pastest.com/about-us)


Contact Us (https://www.pastest.com/contact-us) Help (https://www.pastest.com/help)
Pastest 2016

https://mypastest.pastest.com/Secure/TestMe/Browser/429893#Top

2/2

8/22/2016

MyPastest

Back to Filters (/Secure/TestMe/Filter/429893/QA)

Question 197 of 298

A 47-year-old diabetic man is referred from the Fertility Clinic. He has occasional palpitations
and takes diuretics for dyspnoea. He is tanned, has hepatomegaly and a normal full blood
count and liver biochemistry.
What is the most appropriate next investigation?
A

Cardiac echocardiography

Liver biopsy

Magnetic resonance imaging of the liver

Serum -fetoprotein level

Serum iron studies

Explanation

The answer is Serum iron studies


The combination of diabetes mellitus, probable hypogonadism, hepatomegaly and skin
pigmentation suggests haemochromatosis. This is an autosomal recessive disorder and is
more commonly seen in middle-aged men. Excess iron is deposited in the endocrine
glands, heart and skin. Gonadal function is impaired. Patients develop arrhythmias and
heart failure. Serum iron and serum ferritin levels are raised. The total iron-binding
capacity is reduced and transferrin saturation is complete (> 60%); genetic testing is
used to confirm the diagnosis. Patients can develop cirrhosis and 30% of patients who
develop cirrhosis can go on to develop hepatocellular carcinoma.

Cardiac echocardiography (Option A) is incorrect. Cardiac echocardiography will help define


any cardiomyopathy associated with iron deposition in haemochromatosis, but is not useful
in making the diagnosis.
Liver biopsy (Option B) is incorrect. Although liver biopsy would demonstrate iron deposition
within the liver in people with haemochromatosis, it is an invasive test with well-recognised
potential complications, and iron studies and genetic studies have the advantages of being
non-invasive as well as sensitive and specific for the condition, ruling liver biopsy out.
https://mypastest.pastest.com/Secure/TestMe/Browser/429893#Top

1/2

8/22/2016

MyPastest

Magnetic resonance imaging (MRI) of the liver (Option C) is incorrect. MRI can identify iron
deposition within the liver, but cannot be used to confirm or refute a suspected diagnosis of
haemochromatosis, making this incorrect.
Serum -fetoprotein level (Option D) is incorrect. Serum -fetoprotein level forms part of the
surveillance regimen for hepatocellular carcinoma (which may occur in people with chronic
liver disease, including that which may occur in association with haemochromatosis), but
again is not specific for haemochromatosis itself, ruling this option out.
44886

Next Question

Previous Question

Tag Question

Feedback

End Review

Difficulty: Average
Peer Responses

Session Progress
Responses Correct:

Responses Incorrect:

298

Responses Total:

298

Responses - % Correct:

0%

Blog (https://www.pastest.com/blog) About Pastest (https://www.pastest.com/about-us)


Contact Us (https://www.pastest.com/contact-us) Help (https://www.pastest.com/help)
Pastest 2016

https://mypastest.pastest.com/Secure/TestMe/Browser/429893#Top

2/2

8/22/2016

MyPastest

Back to Filters (/Secure/TestMe/Filter/429893/QA)

Question 198 of 298

A 56-year-old man with severe exertional dyspnoea is admitted with jaundice and ascites. He
has recently been diagnosed with COPD by his GP, although he insists that he only smokes
occasional cigars. His father died of respiratory illness at 54 years of age. Bilirubin, aspartate
transaminase (AST) and alkaline phosphatase levels are elevated and liver biopsy reveals the
presence of periodic acidSchiff- (PAS-) positive, diastase-resistant globules in periportal
hepatocytes.
What is the most likely diagnosis?
A

1-antitrypsin deficiency

Alcohol-related liver disease

BuddChiari syndrome

Cor pulmonale

Haemochromatosis

Explanation

The answer is 1-antitrypsin deficiency


The combination of liver disease, a COPD-like illness (despite a history of very light
smoking), and a family history of lung disease together suggests a diagnosis of 1antitrypsin deficiency. The liver biopsy appearance is also supportive of the diagnosis,
with 1-antitrypsin deficiency pre-disposing to cirrhosis. Absolute avoidance of alcohol
and cigarettes is crucial, and recombinant 1-antitrypsin may be considered for some
patients.

Alcohol-related liver disease (Option B) is incorrect. No history of alcohol consumption is


described, and the liver histology is also inconsistent with alcohol-related liver disease.
BuddChiari syndrome (Option C) is incorrect. The absence of any description of hepatic
venous outflow makes BuddChiari syndrome unlikely, and the personal and family history of
lung disease would also not fit in with this diagnosis.

https://mypastest.pastest.com/Secure/TestMe/Browser/429893#Top

1/2

8/22/2016

MyPastest

Cor pulmonale (Option D) is incorrect. Cor pulmonale may be associated with congestive
liver disease and ascites, but the other details of this clinical scenario (along with liver
histology results) do not fit with this as the diagnosis.
Haemochromatosis (Option E) is incorrect. Haemochromatosis similarly has no association
with lung disease and is associated with liver histology demonstrating iron deposition.
44887

Next Question

Previous Question

Tag Question

Feedback

End Review

Difficulty: Average
Peer Responses

Session Progress
Responses Correct:

Responses Incorrect:

298

Responses Total:

298

Responses - % Correct:

0%

Blog (https://www.pastest.com/blog) About Pastest (https://www.pastest.com/about-us)


Contact Us (https://www.pastest.com/contact-us) Help (https://www.pastest.com/help)
Pastest 2016

https://mypastest.pastest.com/Secure/TestMe/Browser/429893#Top

2/2

8/22/2016

MyPastest

Back to Filters (/Secure/TestMe/Filter/429893/QA)

Question 199 of 298

A 19-year-old man is referred with tremors, difficulty speaking and forgetfulness. He is pale,
mildly jaundiced, and has palmar erythema and telangiectasia on his anterior chest wall. He
also has a postural tremor and dysarthria.
What investigation is most likely to confirm the suspected diagnosis?
A

Computed tomography scan of the brain

Magnetic resonance imaging of the posterior cranial fossa

Serum -fetoprotein assay

Serum iron studies

Urinary copper level

Explanation

The answer is Urinary copper level


This patient has a combination of liver disease and neurological signs. The diagnosis is
Wilsons disease. It is an autosomal recessive disorder and occurs worldwide. Wilsons
disease is characterised by abnormal copper metabolism, with the underlying metabolic
problem being excessive tissue retention of copper. The urinary copper level is raised and
the serum copper and caeruloplasmin levels are usually reduced. Diagnosis depends on
measuring the amount of copper in the liver. A characteristic feature is the presence of
corneal KayserFleischer rings, which are due to copper deposition in Descemets
membrane. Long-term treatment with penicillamine is effective in chelating copper.

Computed tomography (CT) scan of the brain (Option A) is incorrect. Abnormal brain
imaging may be detected in those with Wilsons disease, but none of these changes is
sufficiently specific to help make the diagnosis of the condition.
Magnetic resonance imaging (MRI) of the posterior cranial fossa (Option B) is incorrect.
Abnormal brain imaging may be detected in those with Wilsons disease, but none of these
changes is sufficiently specific to help make the diagnosis of the condition.

https://mypastest.pastest.com/Secure/TestMe/Browser/429893#Top

1/2

8/22/2016

MyPastest

Serum -fetoprotein assay (Option C) is incorrect. Alpha-fetoprotein is a screen for


hepatocellular carcinoma, an unlikely diagnosis in this 19-year-old man.
Serum iron studies (Option D) is incorrect. Serum iron studies are an appropriate initial
screen for people with suspected haemochromatosis, but are not helpful in the diagnosis of
Wilsons disease.
44888

Next Question

Previous Question

Tag Question

Feedback

End Review

Difficulty: Average
Peer Responses

Session Progress
Responses Correct:

Responses Incorrect:

298

Responses Total:

298

Responses - % Correct:

0%

Blog (https://www.pastest.com/blog) About Pastest (https://www.pastest.com/about-us)


Contact Us (https://www.pastest.com/contact-us) Help (https://www.pastest.com/help)
Pastest 2016

https://mypastest.pastest.com/Secure/TestMe/Browser/429893#Top

2/2

8/22/2016

MyPastest

Back to Filters (/Secure/TestMe/Filter/429893/QA)

Question 200 of 298

A 76-year-old man presents with worsening jaundice, intermittent abdominal pain and weight
loss. He is jaundiced, cachectic and has a non-tender mass in the right upper quadrant.
Which of the following investigations is most likely to establish the diagnosis?
A

Alpha-fetoprotein level

Computed tomography scan of the abdomen

Liver biopsy

Serum gastrin level

Sweat test

Explanation

The answer is Computed tomography scan of the abdomen


The combination of jaundice, a non-tender right upper abdominal mass and weight loss
in a man of this age is most likely to be explained by carcinoma of the pancreas. Other
differential diagnoses would include biliary tumours (such as cholangiocarcinoma, or
ampullary cancer) and liver masses (including primary tumours and liver metastasis).
Computed tomography of the abdomen is the most appropriate test for confirming the
diagnosis, and will also help in staging disease if cancer is present too.

Alpha-fetoprotein level (Option A) is incorrect. Alpha-fetoprotein (-FP) is a biomarker that


forms part of the assessment for hepatocellular carcinoma; however, it is not a biomarker for
pancreatic cancer, so is unlikely to be of any benefit in this scenario.
Liver biopsy (Option C) is incorrect. A liver biopsy may eventually be required to definitively
confirm the diagnosis (eg where a CT scan is suggestive of a tumour with liver metastasis,
and where histology is required before considering chemotherapy), but would not be
performed without imaging first, so is not the appropriate next investigation here.
Serum gastrin level (Option D) is incorrect. Serum gastrin levels are a useful test in suspected
cases of gastrinoma/ZollingerEllison syndrome (a neuroendocrine tumour associated with
overproduction of gastrin, often manifesting clinically with recurrent peptic ulceration and
https://mypastest.pastest.com/Secure/TestMe/Browser/429893#Top

1/2

8/22/2016

MyPastest

diarrhoea). However, there is nothing from the scenario given to strongly suggest that this is
the underlying diagnosis.
Sweat test (Option E) is incorrect. A sweat test is useful to perform in cases of suspected
cystic fibrosis, but this is unlikely to be the diagnosis here.
44889

Next Question

Previous Question

Tag Question

Feedback

End Review

Difficulty: Average
Peer Responses

Session Progress
Responses Correct:

Responses Incorrect:

298

Responses Total:

298

Responses - % Correct:

0%

Blog (https://www.pastest.com/blog) About Pastest (https://www.pastest.com/about-us)


Contact Us (https://www.pastest.com/contact-us) Help (https://www.pastest.com/help)
Pastest 2016

https://mypastest.pastest.com/Secure/TestMe/Browser/429893#Top

2/2

8/22/2016

MyPastest

Back to Filters (/Secure/TestMe/Filter/429893/QA)

Question 201 of 298

A 65-year-old woman has recent-onset diarrhoea, a temperature of 37.6 C and intermittent


abdominal cramps. She had a chest infection 3 weeks ago and was prescribed antibiotics.
What is the most likely cause of her diarrhoea?
A

Clostridium difficile infection

Enterotoxigenic Escherichia co-infection

Salmonella infection

Staphylococcus aureus infection

Viral infection

Explanation

The answer is Clostridium difficile infection


This woman most probably has Clostridium difficile infection. The clue is in the
information that she has recently had antibiotics antibiotics disrupt the normal healthy
gut flora (which helps to protect against GI infection), leaving the gut vulnerable to
infection with spore-forming bacteria such as C. difficile. The conventional means of
diagnosis include ELISA of the toxins (A and B) produced by the organism, or by PCR.
Culture of C. difficile is not diagnostic because it is carried by approximately 5% of
normal healthy adults. The main treatments for a first episode of disease include
vancomycin or metronidazole. For recurrent C. difficile infection, there is an increasingly
recognised role for faecal microbiota transplantation, ie generation of a liquidised
bacterial suspension from the stool of healthy people, and delivery of this suspension
into the gut of affected people in order to restore the normal gut flora.
Although any of the other organisms listed may cause acute diarrhoeal disease, none of
them has a particular association with recent antibiotic use, in contrast to C. difficile
infection.

Enterotoxigenic Escherichia co-infection (Option B) is incorrect for the reasons already


described.
https://mypastest.pastest.com/Secure/TestMe/Browser/429893#Top

1/2

8/22/2016

MyPastest

Salmonella infection (Option C) is incorrect for the reasons already described.


Staphylococcus aureus infection (Option D) is incorrect for the reasons already described
Viral infection (Option E) is incorrect for the reasons already described.
44890

Next Question

Previous Question

Tag Question

Feedback

End Review

Difficulty: Average
Peer Responses

Session Progress
Responses Correct:

Responses Incorrect:

298

Responses Total:

298

Responses - % Correct:

0%

Blog (https://www.pastest.com/blog) About Pastest (https://www.pastest.com/about-us)


Contact Us (https://www.pastest.com/contact-us) Help (https://www.pastest.com/help)
Pastest 2016

https://mypastest.pastest.com/Secure/TestMe/Browser/429893#Top

2/2

8/22/2016

MyPastest

Back to Filters (/Secure/TestMe/Filter/429893/QA)

Question 202 of 298

A 55-year-old man complains of dysphagia for both solids and liquids. He says this began
first with liquids around 6 months ago, and has progressively worsened, although he has only
lost 2 kg in weight. On examination his BP is 147/87 mmHg, pulse is 75/min and regular, and
his BMI is 32.
What is the most likely diagnosis?
A

Achalasia

Barretts oesophagus

Benign oesophageal stricture

Carcinoma of the oesophagus

Schatzkis rings

Explanation

The answer is Achalasia


The presence of dysphagia to both liquids and solids in a patient with preserved weight
is together highly suggestive of an oesophageal motility disorder. Of the options given,
achalasia only is an oesophageal motility disorder, and therefore is the correct answer
here.

Barretts oesophagus (Option B) is incorrect. Barretts oesophagus would not in itself


typically be expected to cause dysphagia.
Benign oesophageal stricture (Option C) is incorrect. Obstructive oesophageal disorders
(including oesophageal carcinoma and benign oesophageal strictures) typically first present
with dysphagia to solids, and progress with time to dysphagia to liquids.
Carcinoma of the oesophagus (Option D) is incorrect. Obstructive oesophageal disorders
(including oesophageal carcinoma and benign oesophageal strictures) typically first present
with dysphagia to solids, and progress with time onto dysphagia to liquids.

https://mypastest.pastest.com/Secure/TestMe/Browser/429893#Top

1/2

8/22/2016

MyPastest

Schatzkis rings (Option E) is incorrect. The dysphagia associated with Schatzkis rings is
intermittent.
44891

Next Question

Previous Question

Tag Question

Feedback

End Review

Difficulty: Average
Peer Responses

Session Progress
Responses Correct:

Responses Incorrect:

298

Responses Total:

298

Responses - % Correct:

0%

Blog (https://www.pastest.com/blog) About Pastest (https://www.pastest.com/about-us)


Contact Us (https://www.pastest.com/contact-us) Help (https://www.pastest.com/help)
Pastest 2016

https://mypastest.pastest.com/Secure/TestMe/Browser/429893#Top

2/2

8/22/2016

MyPastest

Back to Filters (/Secure/TestMe/Filter/429893/QA)

Question 203 of 298

A 67-year-old woman is admitted with iron deficiency anaemia. She has an ejection systolic
murmur radiating to both carotids. Upper gastrointestinal endoscopy and colonoscopy are
normal.
Which of the following is the most appropriate next investigation?
A

Barium enema

Bone marrow examination

Repeat colonoscopy

Repeat upper gastrointestinal endoscopy

Selective mesenteric angiography

Explanation

The answer is Repeat colonoscopy


This woman presents with an iron deficiency anaemia and also has features of aortic
stenosis. Given that her upper gastrointestinal endoscopy and colonoscopy are normal
(and, therefore, a mass lesion is excluded), the most likely diagnosis is angiodysplasia.
Angiodysplasia is the most common vascular lesion of the gastrointestinal tract; the
condition may be asymptomatic, or it may cause gastrointestinal bleeding (estimated
incidence of active bleeding being about 10% of affected cases). The most commonly
affected site is the large bowel: 77% of angiodysplasia cases are located in the caecum
and ascending colon, 15% in the jejunum and ileum, and the remainder distributed
throughout the alimentary tract. Angiodysplasia is associated with aortic stenosis,
making it even more likely that this is the diagnosis here.

Barium enema (Option A) is incorrect. Barium enema is a test that is helpful when looking for
structural colonic abnormalities, but is not good at identifying subtle mucosal abnormalities
such as angiodysplasia. Given that this womans colonoscopy has already excluded
significant structural colonic disease, there is no additional merit in a barium enema.

https://mypastest.pastest.com/Secure/TestMe/Browser/429893#Top

1/2

8/22/2016

MyPastest

Bone marrow examination (Option B) is incorrect. Although bone marrow examination may
be an appropriate test to investigate unexplained cases of anaemia, iron deficiency anaemia
in a woman of this age is much more likely to reflect blood loss than marrow failure.
Repeat upper GI endoscopy (Option D) is incorrect. If this womans previous upper
gastrointestinal endoscopy had good views of the whole mucosa, it is likely to have been
sufficient to confidently rule out an upper gastrointestinal tract lesion as the source of her
anaemia.
Selective mesenteric angiography (Option E) is incorrect. It is less successful in the diagnosis
of chronic bleeding.
44892

Next Question

Previous Question

Tag Question

Feedback

End Review

Difficulty: Average
Peer Responses

Session Progress
Responses Correct:

Responses Incorrect:

298

Responses Total:

298

Responses - % Correct:

0%

Blog (https://www.pastest.com/blog) About Pastest (https://www.pastest.com/about-us)


Contact Us (https://www.pastest.com/contact-us) Help (https://www.pastest.com/help)
Pastest 2016

https://mypastest.pastest.com/Secure/TestMe/Browser/429893#Top

2/2

8/22/2016

MyPastest

Back to Filters (/Secure/TestMe/Filter/429893/QA)

Question 204 of 298

A 77-year-old woman complains of abdominal pain, nausea, anorexia and weight loss of 4
months duration. The pain is dull, boring and radiates through to her back. Examination
reveals mild epigastric tenderness but there are no palpable masses. Gamma GT and alkaline
phosphatase are both above the upper limit of normal.
What is the most likely diagnosis?
A

Carcinoma of the pancreas

Cholangiocarcinoma

Gastric carcinoma

Peptic ulcer disease

ZollingerEllison syndrome

Explanation

The answer is Carcinoma of the pancreas


In a woman of this age with this history and constellation of physical findings, carcinoma
of the body or tail of the pancreas would seem to be the most likely diagnosis. Unlike
carcinoma of the head of the pancreas, obstructive jaundice does not typically occur
with tumours in the body or tail (unless tumours are very large/advanced). However, an
increase in cholestatic liver enzymes (gamma GT and alkaline phosphatase) is frequently
seen.

Cholangiocarcinoma (Option B) is incorrect. The commonest presentation of


cholangiocarcinoma (cancer of the biliary tree) is with obstructive jaundice, often without
significant abdominal discomfort; the absence of overt jaundice here makes it less likely to
be the underlying diagnosis.
Gastric carcinoma (Option C) is incorrect. The description of a dull, boring pain that radiates
through to the back is highly suggestive of pancreatic disease, making gastric carcinoma less
likely. Furthermore, it would be unusual for these gastric carcinomas to cause abnormal liver

https://mypastest.pastest.com/Secure/TestMe/Browser/429893#Top

1/2

8/22/2016

MyPastest

enzymes (although this may be found in the case of gastric carcinoma that has metastasised
to the stomach).
Peptic ulcer disease (Option D) is incorrect. The description of a dull, boring pain that
radiates through to the back is highly suggestive of pancreatic disease, making peptic ulcer
disease less likely.
ZollingerEllison syndrome (Option E) is incorrect. ZollingerEllison syndrome occurs in
association with pancreatic gastrinomas; these are very rare tumours (most pancreatic
tumours are adenocarcinomas), and tend to present with recurrent peptic ulceration and
diarrhoea, and this is unlikely to be the case here.
44894

Next Question

Previous Question

Tag Question

Feedback

End Review

Difficulty: Average
Peer Responses

Session Progress
Responses Correct:

Responses Incorrect:

298

Responses Total:

298

Responses - % Correct:

0%

Blog (https://www.pastest.com/blog) About Pastest (https://www.pastest.com/about-us)


Contact Us (https://www.pastest.com/contact-us) Help (https://www.pastest.com/help)
Pastest 2016

https://mypastest.pastest.com/Secure/TestMe/Browser/429893#Top

2/2

8/22/2016

MyPastest

Back to Filters (/Secure/TestMe/Filter/429893/QA)

Question 205 of 298

A 60-year-old woman with type 2 diabetes complains of blood-stained diarrhoea and


cramping abdominal pain of several months duration. She underwent a hysterectomy and
radiotherapy 8 months ago for endometrial carcinoma and her diarrhoea began a short
period after this.
What is the most likely diagnosis?
A

Chronic radiation enteritis

Irritable bowel syndrome

Lymphoma

Pseudomembranous colitis

Ulcerative colitis

Explanation

The answer is Chronic radation enteritis


The clear relationship time-wise between this woman receiving radiotherapy and
development of abdominal pain and blood-stained diarrhoea is very suggestive of
radiation enteritis.

Irritable bowel syndrome (Option B) is incorrect. Irritable bowel syndrome may cause chronic
abdominal pain and diarrhoea, but certainly cannot account for bleeding from the
gastrointestinal tract, and therefore cannot be the diagnosis here.
Lymphoma (Option C) is incorrect. Lymphoma may present with a wide range of
gastrointestinal and systemic symptoms, but the absence of either features suggestive of B
symptoms or the presence of lymphadenopathy makes this unlikely.
Pseudomembranous colitis (Option D) is incorrect. Pseudomembranous colitis is colonic
inflammation occurring in association with Clostridium difficile infection. The major risk
factor for C. difficile infection is recent use of antibiotics; no history of recent antibiotic use is
given, making this diagnosis unlikely. In addition, pseudomembranous colitis tends to give
https://mypastest.pastest.com/Secure/TestMe/Browser/429893#Top

1/2

8/22/2016

MyPastest

such marked symptoms that affected patients would typically present to medical services
and be diagnosed much earlier than after several months of symptoms.
Ulcerative colitis (Option E) is incorrect. Ulcerative colitis (UC) may occur at any age, but
tends to first present at a somewhat younger age than this woman is. In addition, UC often
gives symptoms relating to rectal involvement (including urgency and tenesmus), and
untreated ulcerative colitis tends to give symptoms that relapse and remit over the course of
months to years; this does not fit with the scenario described here.
44895

Next Question

Previous Question

Tag Question

Feedback

End Review

Difficulty: Average
Peer Responses

Session Progress
Responses Correct:

Responses Incorrect:

298

Responses Total:

298

Responses - % Correct:

0%

Blog (https://www.pastest.com/blog) About Pastest (https://www.pastest.com/about-us)


Contact Us (https://www.pastest.com/contact-us) Help (https://www.pastest.com/help)
Pastest 2016

https://mypastest.pastest.com/Secure/TestMe/Browser/429893#Top

2/2

8/22/2016

MyPastest

Back to Filters (/Secure/TestMe/Filter/429893/QA)

Question 206 of 298

A 23-year-old woman complains of intermittent diarrhoea containing blood and mucus,


tiredness and anorexia going back over the past 2 to 3 months. Her abdomen is soft and nontender. Cultures of stool samples are negative. Examination is unremarkable.
What is the most appropriate investigation?
A

Barium enema

Colonoscopy and biopsies

Hydrogen breath test

Plain X-ray of the abdomen

Upper gastrointestinal endoscopy with duodenal biopsy

Explanation

The answer is Colonoscopy and biopsies


A history of a young person with chronic rectal bleeding and systemic symptoms that
may represent anaemia is very suggestive of ulcerative colitis (UC). Colonoscopy will
allow endoscopic views of the mucosa and biopsies that will be definitive in confirming
the diagnosis, and will also allow staging of the extent and severity of disease.

Barium enema (Option A) is incorrect. Barium enema is helpful for looking for significant
structural lesions of the colon (eg tumours, diverticular disease etc), but does not allow
assessment of the colonic mucosa or give the ability to take biopsies, limiting its role in cases
of suspected UC.
Hydrogen breath test (Option C) is incorrect. A glucose-hydrogen breath test is useful in
cases of suspected small bowel bacterial overgrowth, but again the presence of rectal
bleeding is inconsistent with this as the underlying diagnosis.
Plain X-ray of the abdomen (Option D) is incorrect. X-ray of the abdomen is helpful when
looking for dilated bowel loops in cases of suspected intestinal obstruction, but is unlikely to
be of any benefit here.
https://mypastest.pastest.com/Secure/TestMe/Browser/429893#Top

1/2

8/22/2016

MyPastest

Upper gastrointestinal (GI) endoscopy with duodenal biopsy (Option E) is incorrect. Upper
GI endoscopy with duodenal biopsy is the reference standard for diagnosis of coeliac disease
and other enteric causes of malabsorption. However, rectal bleeding clearly suggests a lower
gastrointestinal source of her symptoms, so this investigation is unlikely to be of any benefit
here.
44896

Next Question

Previous Question

Tag Question

Feedback

End Review

Difficulty: Average
Peer Responses

Session Progress
Responses Correct:

Responses Incorrect:

298

Responses Total:

298

Responses - % Correct:

0%

Blog (https://www.pastest.com/blog) About Pastest (https://www.pastest.com/about-us)


Contact Us (https://www.pastest.com/contact-us) Help (https://www.pastest.com/help)
Pastest 2016

https://mypastest.pastest.com/Secure/TestMe/Browser/429893#Top

2/2

8/22/2016

MyPastest

Back to Filters (/Secure/TestMe/Filter/429893/QA)

Question 207 of 298

A 62-year-old woman complains of diarrhoea, weight loss and abdominal pain, with malaise
and fever. She has oral ulcers, red, itchy eyes and tender nodules on her shins. She has
tenderness in the right iliac fossa and a vague right iliac fossa mass.
What is the most likely diagnosis?
A

Appendicular abscess

Crohns disease

Ileocaecal tuberculosis

Ovarian mass

Ulcerative colitis

Explanation

The answer is Crohns disease


The most likely diagnosis is Crohns disease. Although this condition often first presents
in people somewhat younger than this patient, it is not unusual for the condition to first
present in women over 60 years of age. The intestinal features of the condition are the
most well recognised, but Crohns disease also has a huge number of extra-intestinal
manifestations, including episcleritis, uveitis, erythema nodosum, pyoderma
gangrenosum, vasculitis, gallstones, kidney stones or abnormal liver enzymes.

Appendicular abscess (Option A) is incorrect. Appendicular abscess could certainly explain


some of the abdominal symptoms described, but would not explain the eye and skin features
of this womans illness.
Ileocaecal tuberculosis (Option C) is incorrect. Ileocaecal tuberculosis may present with very
similar symptoms to ileocaecal Crohns disease; both conditions may also look similar
endoscopically and demonstrate granulomas on histology, and the differentiation of the two
conditions can therefore be difficult. However, no risk factors for tuberculosis are given in the
question, making it less likely to be the diagnosis.

https://mypastest.pastest.com/Secure/TestMe/Browser/429893#Top

1/2

8/22/2016

MyPastest

Ovarian mass (Option D) is incorrect. Ovarian mass could certainly explain some of the
abdominal symptoms described, but would not explain the eye and skin features of this
womans illness.
Ulcerative colitis (Option E) is incorrect. Given that the intestinal symptoms described are so
suggestive of terminal ileitis (with no description given of the urgency and tenesmus that
may characterise proctitis), Crohns disease is much more likely than ulcerative colitis.
44897

Next Question

Previous Question

Tag Question

Feedback

End Review

Difficulty: Average
Peer Responses

Session Progress
Responses Correct:

Responses Incorrect:

298

Responses Total:

298

Responses - % Correct:

0%

Blog (https://www.pastest.com/blog) About Pastest (https://www.pastest.com/about-us)


Contact Us (https://www.pastest.com/contact-us) Help (https://www.pastest.com/help)
Pastest 2016

https://mypastest.pastest.com/Secure/TestMe/Browser/429893#Top

2/2

8/22/2016

MyPastest

Back to Filters (/Secure/TestMe/Filter/429893/QA)

Question 208 of 298

A 55-year-old man complains of intermittent epigastric pain, diarrhoea and steatorrhoea. He


has mild iron deficiency anaemia and raised calcium and parathyroid hormone levels. An
upper gastrointestinal endoscopy has shown peptic ulcers in the stomach, duodenum and
jejunum.
Which of the following tests is most likely to help in diagnosis?
A

Duodenal biopsy

Plain X-ray of the abdomen

Serum gastrin level

Small-bowel follow-through

Stool fat content

Explanation

The answer is serum gastrin level


The presence of multiple ulcers and hyperparathyroidism suggests ZollingerEllison
syndrome. This syndrome is caused by a pancreatic tumour that secretes gastrin, and for
this reason serum gastrin levels are a helpful test in trying to make the diagnosis.
Gastrinoma may occur as part of the multiple endocrine neoplasia-1 (MEN-1) syndrome,
which is associated with parathyroid and pituitary adenomas; the presence of raised
parathyroid and calcium levels here is clearly suggestive of parathyroid disease.
If this mans diarrhoea was thought to be attributable to malabsorption, there would be a
potential role for small-bowel follow-through (to look for features of small intestinal
Crohns disease), stool fat (to assess for fat malabsorption) and for duodenal biopsy (to
examine for coeliac disease); however, his diarrhoea seems much more likely to be
caused by a gastrinoma, and none of these tests is therefore likely to be of use.

Duodenal biopsy (Option A) is incorrect for the reasons described.


Plain X-ray of the abdomen (Option B) is incorrect. Plain X-ray of the abdomen is useful to
look for dilated loops of bowel in suspected intestinal obstruction, but appears unlikely to be
https://mypastest.pastest.com/Secure/TestMe/Browser/429893#Top

1/2

8/22/2016

MyPastest

helpful in this scenario.


Small-bowel follow-through (Option D) is incorrect for the reasons described.
Stool fat content (Option E) is incorrect for the reasons described.
44898

Next Question

Previous Question

Tag Question

Feedback

End Review

Difficulty: Average
Peer Responses

Session Progress
Responses Correct:

Responses Incorrect:

298

Responses Total:

298

Responses - % Correct:

0%

Blog (https://www.pastest.com/blog) About Pastest (https://www.pastest.com/about-us)


Contact Us (https://www.pastest.com/contact-us) Help (https://www.pastest.com/help)
Pastest 2016

https://mypastest.pastest.com/Secure/TestMe/Browser/429893#Top

2/2

8/22/2016

MyPastest

Back to Filters (/Secure/TestMe/Filter/429893/QA)

Question 209 of 298

A 71-year-old is found to have a mobile, non-tender mass in the right iliac fossa. His bowel
habit is normal. He is apyrexial and has a haemoglobin of 14.9 g/dl with a normal white cell
count.
What is the most likely diagnosis?
A

Actinomycosis

Caecal volvulus

Femoral hernia

Varicocele

Villous adenoma in the caecum

Explanation

The answer is Actinomycosis


Actinomycosis is caused by a Gram-positive, filamentous bacterium which survives on
little or no oxygen and which may cause multiple abscesses. These most commonly
occur in the abdomen and at the angle of the jaw.

Caecal volvulus (Option B) is incorrect. A clinically significant caecal volvulus would produce
signs of intestinal obstruction.
Femoral hernia (Option C) is incorrect. A femoral hernia would produce an inguinal swelling
but would not give any abdominal findings.
Varicocele (Option D) is incorrect. A varicocele would be palpable, but in the scrotum rather
than the abdomen.
Villous adenoma in the caecum (Option E) is incorrect. A flat villous adenoma is unlikely to
be palpable, even in a thin patient.
44899

Next Question
https://mypastest.pastest.com/Secure/TestMe/Browser/429893#Top

1/2

8/22/2016

MyPastest

Previous Question

Tag Question

Feedback

End Review

Difficulty: Average
Peer Responses

Session Progress
Responses Correct:

Responses Incorrect:

298

Responses Total:

298

Responses - % Correct:

0%

Blog (https://www.pastest.com/blog) About Pastest (https://www.pastest.com/about-us)


Contact Us (https://www.pastest.com/contact-us) Help (https://www.pastest.com/help)
Pastest 2016

https://mypastest.pastest.com/Secure/TestMe/Browser/429893#Top

2/2

8/22/2016

MyPastest

Back to Filters (/Secure/TestMe/Filter/429893/QA)

Question 210 of 298

A 32-year-old woman presents with a history of an 8 kg weight loss, frothy stools and general
malaise. Her haemoglobin is 10.2 g/dl with a mean corpuscular volume (MCV) of 98 fl.
Which of the following best supports a diagnosis of coeliac disease?
A

Dermatitis herpetiformis

Negative family history

Neutrophil infiltration of a rectal mucosal biopsy

Reduced hydrogen excretion on hydrogen breath testing

Remission following metronidazole therapy

Explanation

The answer is dermatitis herpetiformis


Understanding of coeliac disease has progressed considerably over recent years. In terms
of diagnosis, although positive coeliac serology may be highly suggestive of the
condition, histopathology (as obtained via small bowel biopsy) is the definitive
investigation. Villous atrophy and lymphocytic duodenosis ( 25 intraepithelial
lymphocytes/100 enterocytes) are the classical histological features of the condition.
Dermatitis herpetiformis is the skin manifestation of gluten sensitivity, and 7080% of
patients with dermatitis herpetiformis will have coexisting coeliac disease.

Negative family history (Option B) is incorrect. Family members of people with coeliac
disease who are HLA-DQ2 or HLA-DQ8-positive are at increased risk of coeliac disease.
Neutrophil infiltration of a rectal mucosal biopsy (Option C) is incorrect. Neutrophil
infiltration is not a feature of coeliac disease histology (and there would not be any
histological changes expected in the rectum).
Reduced hydrogen excretion on hydrogen breath testing (Option D) is incorrect. The
glucose-hydrogen breath test may in fact show increased excretion of hydrogen in people
with coeliac disease, as this is a product of bacterial fermentation of unabsorbed lactose;
however, this is not the same as having true small bowel bacterial overgrowth.
https://mypastest.pastest.com/Secure/TestMe/Browser/429893#Top

1/2

8/22/2016

MyPastest

Remission following metronidazole therapy (Option E) is incorrect. The glucose-hydrogen


breath test may in fact show increased excretion of hydrogen in people with coeliac disease,
as this is a product of bacterial fermentation of unabsorbed lactose; however, this is not the
same as having true small bowel bacterial overgrowth, hence metronidazole would not be
expected to help.
44900

Next Question

Previous Question

Tag Question

Feedback

End Review

Difficulty: Average
Peer Responses

Session Progress
Responses Correct:

Responses Incorrect:

298

Responses Total:

298

Responses - % Correct:

0%

Blog (https://www.pastest.com/blog) About Pastest (https://www.pastest.com/about-us)


Contact Us (https://www.pastest.com/contact-us) Help (https://www.pastest.com/help)
Pastest 2016

https://mypastest.pastest.com/Secure/TestMe/Browser/429893#Top

2/2

8/22/2016

MyPastest

Back to Filters (/Secure/TestMe/Filter/429893/QA)

Question 211 of 298

A 59-year-old, otherwise fit man undergoes endoscopic follow-up for Barretts disease of the
oesophagus. His latest biopsy shows poorly differentiated cells.
Which of the following is the best management option?
A

Continue the maintenance dose of a proton-pump inhibitor and follow up every 3


months with focused biopsies

Give an increased dose of a proton-pump inhibitor and annual Barretts surveillance

Refer for oesophagectomy

Refer for photodynamic therapy

Repeat endoscopy and biopsies immediately

Explanation

The answer is Repeat endoscopies and biopsies immediately


The most appropriate next step would be an immediate repeat endoscopy and biopsies.
If biopsies again show poorly differentiated cells, then an evaluation for oesophagectomy
would be required. This is because the presence of poorly differentiated cells is very
suggestive of the presence of an oesophageal malignancy.

Continue the maintenance dose of a PPI and follow up every 3 months with focused biopsies
(Option A) is incorrect. Although continuing the dose of PPI and close endoscopic
surveillance may be appropriate if the histology from a repeat endoscopy/biopsy is negative,
the presence of poorly differentiated cells should be sufficiently suspicious of possible
malignancy to merit immediately repeating the endoscopy.
Give an increased dose of a proton-pump inhibitor (PPI) and annual Barretts surveillance
(Option B) is incorrect. Although increasing the dose of PPI and close endoscopic
surveillance may be appropriate if the histology from a repeat endoscopy/biopsy is negative,
the presence of poorly differentiated cells should be sufficiently suspicious of possible
malignancy to merit immediately repeating the endoscopy.

https://mypastest.pastest.com/Secure/TestMe/Browser/429893#Top

1/2

8/22/2016

MyPastest

Refer for oesophagectomy (Option C) is incorrect. Oesophagectomy is a major operation


with significant associated morbidity and mortality, so would only be considered in people
who unequivocally have histologically confirmed oesophageal cancer; thus it is an
inappropriate option at this stage.
Refer for photodynamic therapy (Option D) is incorrect. Photodynamic therapy has a limited
role in treating flat high-grade dysplasia of the oesophagus, but would require confirmatory
histology before even being considered.
44901

Next Question

Previous Question

Tag Question

Feedback

End Review

Difficulty: Average
Peer Responses

Session Progress
Responses Correct:

Responses Incorrect:

298

Responses Total:

298

Responses - % Correct:

0%

Blog (https://www.pastest.com/blog) About Pastest (https://www.pastest.com/about-us)


Contact Us (https://www.pastest.com/contact-us) Help (https://www.pastest.com/help)
Pastest 2016

https://mypastest.pastest.com/Secure/TestMe/Browser/429893#Top

2/2

8/22/2016

MyPastest

Back to Filters (/Secure/TestMe/Filter/429893/QA)

Question 212 of 298

A 35-year-old man presents with recent-onset dysphagia. This had been initially to liquids,
but then, 2 months later, to solids. His weight has been stable. In the past week he has woken
up coughing during the night. An upper gastrointestinal endoscopy performed at the onset of
his symptoms was reported as being normal.
What is the most useful diagnostic test?
A

13

24-hour oesophageal pH study

Barium follow-through

Oesophageal pull-through manometry

Repeat upper gastrointestinal endoscopy

C urea breath test

Explanation

The answer is Oesophageal pull-through manometry


The history described is the classic one for achalasia, ie an abrupt-onset dysphagia, often
without weight loss. Liquids often cause more of a problem than solids. Achalasia is a
condition associated with failure of relaxation of the lower oesophageal sphincter, and
where the sphincter actually is often hypertensive. The oesophagus dilates and loses the
normal pattern of peristaltic waves; these are often replaced by simultaneous contraction
of the oesophageal body or loss of all activity. It is important to recognise symptoms of
regurgitation and aspiration a night-time cough or recurrent sore throats in the
morning could be suggestive of this. Investigation consists of upper gastrointestinal
endoscopy (to rule out pseudo-achalasia due to a carcinoma), barium swallow (which
classically shows the rats tail appearance of the lower sphincter), and pull-through
manometry. Pull-through manometry is the most specific diagnostic test, and will
demonstrate the hypertensive sphincter and motility disorder.

13

C urea breath test (Option A) is incorrect. A 13C urea breath test may be used to assess for
the presence of Helicobacter pylori in the upper gastrointestinal tract, but would not add any
useful information here.
https://mypastest.pastest.com/Secure/TestMe/Browser/429893#Top

1/2

8/22/2016

MyPastest

24-hour oesophageal pH study (Option B) is incorrect. A 24-hour oesophageal pH study is


useful in the diagnosis of gastro-oesophageal reflux disease, but is not a useful test for
suspected achalasia.
Barium follow-through (Option C) is incorrect. As already described, a barium swallow may
be suggestive of the condition, but extending a contrast study as far as a follow-through is
unlikely to be of any additional benefit.
Repeat upper gastrointestinal endoscopy (Option E) is incorrect. Assuming the first
endoscopy had good views, it is unlikely that a repeat gastroscopy would provide any further
useful information (furthermore, upper gastrointestinal malignancy is clearly very unlikely in
a man of this age).
44902

Next Question

Previous Question

Tag Question

Feedback

End Review

Difficulty: Average
Peer Responses

Session Progress
Responses Correct:

Responses Incorrect:

298

Responses Total:

298

Responses - % Correct:

0%

Blog (https://www.pastest.com/blog) About Pastest (https://www.pastest.com/about-us)


Contact Us (https://www.pastest.com/contact-us) Help (https://www.pastest.com/help)
Pastest 2016

https://mypastest.pastest.com/Secure/TestMe/Browser/429893#Top

2/2

8/22/2016

MyPastest

Back to Filters (/Secure/TestMe/Filter/429893/QA)

Question 213 of 298

A 56-year-old woman had an upper gastrointestinal endoscopy for long-standing reflux


symptoms. This showed the squamo-columnar junction at 32 cm, the gastro-oesophageal
junction at 39 cm and the diaphragmatic hiatus at 40 cm. Biopsies from the distal
oesophagus revealed a columnar mucosa with goblet cells.
What is the most likely explanation for these findings?
A

Barretts oesophagus

Gastric metaplasia

Oesophageal atresia

Rolling hiatus hernia

Sliding hiatus hernia

Explanation

The answer is Barretts oesophagus


Barretts oesophagus is the presence of a specialised intestinal-type mucosa (columnar
epithelium with mucin-containing goblet cells) replacing the normal squamous mucosa
of the distal oesophagus.

Gastric metaplasia (Option B) is incorrect. In the absence of goblet cells, this is called a
columnar-lined oesophagus (probably relating to gastric metaplasia rather than to small- or
large-intestinal types), and this might have a lower risk of progression to adenocarcinoma.
Oesophageal atresia (Option C) is incorrect. Oesophageal atresia is a congenital failure of the
formation of a segment of oesophagus that results in a blind-ended oesophagus.
Rolling hiatus hernia (Option D) is incorrect. A hiatus hernia would result in a distance
between the diaphragmatic hiatus and gastro-oesophageal junction of more than 2 cm.
Sliding hiatus hernia (Option E) is incorrect. A hiatus hernia would result in a distance
between the diaphragmatic hiatus and gastro-oesophageal junction of more than 2 cm.
44904

https://mypastest.pastest.com/Secure/TestMe/Browser/429893#Top

1/2

8/22/2016

MyPastest

Next Question

Previous Question

Tag Question

Feedback

End Review

Difficulty: Average
Peer Responses

Session Progress
Responses Correct:

Responses Incorrect:

298

Responses Total:

298

Responses - % Correct:

0%

Blog (https://www.pastest.com/blog) About Pastest (https://www.pastest.com/about-us)


Contact Us (https://www.pastest.com/contact-us) Help (https://www.pastest.com/help)
Pastest 2016

https://mypastest.pastest.com/Secure/TestMe/Browser/429893#Top

2/2

8/22/2016

MyPastest

Back to Filters (/Secure/TestMe/Filter/429893/QA)

Question 214 of 298

A 67-year-old man has just been admitted with haematemesis and melaena. He is in a
shocked state. He has a history of ischaemic heart disease. On examination, he has a postural
drop in blood pressure of 100/60 mmHg to 70/40 mmHg.
What is the most appropriate immediate management step?
A

Blood transfusion

Intravenous crystalloid

Intravenous omeprazole

Intravenous ranitidine

Intravenous terlipressin

Explanation

The answer is Intravenous crystalloid


In the absence of any more specific history, it is impossible to gauge the cause of the
bleeding. Even in patients with known varices, bleeding is from a non-variceal source in
2030% of cases.
Fluid resuscitation is the only manoeuvre that has been shown to improve outcome and
is the mainstay of treatment, together with endoscopy within 24 hours.

Blood transfusion (Option A) is incorrect. Blood transfusion may be appropriate at some


point, but the immediate intervention required is treating his shock.
Intravenous omeprazole (Option C) is incorrect. Acid suppression medications have not
consistently been shown to improve outcome measures prior to endoscopy; however, they
are of benefit in patients with a high risk of re-bleeding as determined by endoscopy.
Intravenous ranitidine (Option D) is incorrect. Acid suppression medications have not
consistently been shown to improve outcome measures prior to endoscopy; however, they
are of benefit in patients with a high risk of re-bleeding as determined by endoscopy.

https://mypastest.pastest.com/Secure/TestMe/Browser/429893#Top

1/2

8/22/2016

MyPastest

Intravenous terlipressin (Option E) is incorrect. Terlipressin effectively reduces portal and


therefore variceal blood flow; however, its vasoconstricting action may exacerbate cardiac
ischaemia (which clearly could be a problem for this man, given his history of ischaemic
heart disease), and is therefore generally only used when a variceal bleed is strongly
suspected.
44905

Next Question

Previous Question

Tag Question

Feedback

End Review

Difficulty: Average
Peer Responses

Session Progress
Responses Correct:

Responses Incorrect:

298

Responses Total:

298

Responses - % Correct:

0%

Blog (https://www.pastest.com/blog) About Pastest (https://www.pastest.com/about-us)


Contact Us (https://www.pastest.com/contact-us) Help (https://www.pastest.com/help)
Pastest 2016

https://mypastest.pastest.com/Secure/TestMe/Browser/429893#Top

2/2

8/22/2016

MyPastest

Back to Filters (/Secure/TestMe/Filter/429893/QA)

Question 215 of 298

A 24-year-old woman was referred with abdominal pain, diarrhoea and weight loss. On
examination, she had an itchy blistering rash on both elbows. Investigations revealed
haemoglobin of 10.3 g/dl and mean corpuscular volume (MCV) 71 fl.
What is the most appropriate diagnostic test?
A

Barium meal and follow-through

Crosbie capsule jejunal biopsy

Endoscopy and duodenal biopsy

Skin biopsy of the rash

Ultrasound of the pancreas

Explanation

The answer is Endoscopy and duodenal biopsy


The history is that of coeliac disease and the description given of the rash is typical for
dermatitis herpetiformis. Dermatitis herpetiformis is the skin manifestation of gluten
sensitivity, and 7080% of patients with dermatitis herpetiformis will have coexisting
coeliac disease. In the UK, 1 in 300 patients have clinical evidence of the disease, with 1 in
200 having positive coeliac serology. Some 50% of patients are iron-deficient at
presentation. Anti-endomysial IgA, anti-gliadin IgG, anti-reticulin IgM (or IgG) and antitissue transglutaminase IgG antibodies are used in the detection of coeliac disease. Of
these, anti-endomysial IgA and tissue transglutamase are the most commonly used, as
they are both sensitive and specific. However, 10% of patients with coeliac disease are
IgA-deficient and therefore give a false-negative result. The definitive test is a duodenal
biopsy; healing would be seen on biopsies taken after instigation of a gluten-free diet.

Barium meal and follow-through (Option A) is incorrect. A barium meal and follow-through
would be a helpful test in looking for features of suspected small bowel Crohns disease (eg
strictures), but is unlikely to be useful here.

https://mypastest.pastest.com/Secure/TestMe/Browser/429893#Top

1/2

8/22/2016

MyPastest

Crosbie capsule jejunal biopsy (Option B) is incorrect. Crosbie capsules are no longer used
so would be an inappropriate choice here.
Skin biopsy of the rash (Option D) is incorrect. A skin biopsy may help confirm the diagnosis
of dermatitis herpetiformis, but confirmation of a diagnosis of coeliac disease requires a
duodenal biopsy.
Ultrasound of the pancreas (Option E) is incorrect. Ultrasound of the pancreas may be useful
if it was suspected that this womans diarrhoea and weight loss was caused by malabsorption
secondary to pancreatic disease. However, no risk factors for pancreatic disease are given,
and the description provided of dermatitis herpetiformis means that the diagnosis is much
more likely to be coeliac disease than pancreatic insufficiency.
44909

Next Question

Previous Question

Tag Question

Feedback

End Review

Difficulty: Average
Peer Responses

Session Progress
Responses Correct:

Responses Incorrect:

298

Responses Total:

298

Responses - % Correct:

0%

Blog (https://www.pastest.com/blog) About Pastest (https://www.pastest.com/about-us)


Contact Us (https://www.pastest.com/contact-us) Help (https://www.pastest.com/help)
Pastest 2016

https://mypastest.pastest.com/Secure/TestMe/Browser/429893#Top

2/2

8/22/2016

MyPastest

Back to Filters (/Secure/TestMe/Filter/429893/QA)

Question 216 of 298

A 32-year-old man presents with abdominal pain, bloating and nausea after meals. He says
that he has lost 10 kg in weight over the last 3 months and also has had some non-bloodstained diarrhoea. He smokes 30 cigarettes per day but does not drink alcohol. His plain
abdominal X-ray demonstrates dilated loops of small bowel.
What is the most likely diagnosis?
A

Coeliac disease

Crohns disease

Giardiasis

Pancreatic insufficiency

Small-bowel adenocarcinoma

Explanation

The answer is Crohns disease


The features described are suggestive of a small-bowel stricture. This, together with the
diarrhoea, supports a diagnosis of inflammatory bowel disease; ulcerative colitis only
affects the large bowel, but this scenario would fit well with small-bowel Crohns disease.
In addition, Crohns disease is more common and severe in smokers (in contrast to
ulcerative colitis, which is less common in smokers).

Coeliac disease (Option A) is incorrect. Uncomplicated coeliac disease does not cause
strictures, but may be associated with ulcerating jejunitis or small bowel lymphoma, which
could present in this way; however, these are uncommon complications of coeliac disease,
and are not the most likely cause of the symptoms described here.
Giardiasis (Option C) is incorrect. Giardiasis may cause diarrhoea, some weight loss, and
occasional bloating and abdominal pain, but could not explain dilated loops of small bowel.
Pancreatic insufficiency (Option D) is incorrect. Chronic pancreatitis and pancreatic
insufficiency could explain some of the symptoms outlined here, but would not be a cause of
small-bowel obstruction.
https://mypastest.pastest.com/Secure/TestMe/Browser/429893#Top

1/2

8/22/2016

MyPastest

Small-bowel adenocarcinoma (Option E) is incorrect. Small-bowel adenocarcinoma would be


a differential diagnosis in this scenario, but is much rarer than Crohns disease.
44910

Next Question

Previous Question

Tag Question

Feedback

End Review

Difficulty: Average
Peer Responses

Session Progress
Responses Correct:

Responses Incorrect:

298

Responses Total:

298

Responses - % Correct:

0%

Blog (https://www.pastest.com/blog) About Pastest (https://www.pastest.com/about-us)


Contact Us (https://www.pastest.com/contact-us) Help (https://www.pastest.com/help)
Pastest 2016

https://mypastest.pastest.com/Secure/TestMe/Browser/429893#Top

2/2

8/22/2016

MyPastest

Back to Filters (/Secure/TestMe/Filter/429893/QA)

Question 217 of 298

A 77-year-old man presents with facial flushing, diarrhoea, weight loss and leg oedema. He
reports a recent gradual worsening in his breathing. On examination, he has a bilateral
wheeze and a palpable liver. Ultrasound shows multiple lesions within the liver.
What test is most likely to reveal the cause of his symptoms?
A

Colonoscopy

Computed tomography scan of the abdomen

Fasting serum gastrin

Urinary catecholamines

Urinary 5-hydroxyindoleacetic acid

Explanation

The answer is Urinary 5-hydroxyindoleacetic acid


The features described within this scenario are very suggestive of this man having
carcinoid syndrome. Carcinoid tumours are found in 12% of resected appendixes and in
~25% of individuals at postmortem. However, they typically only present clinically in the
following scenarios:
Metastasis to the liver replacement of the normal liver parenchyma with
metastatic lesions means that there is no first-pass metabolism of 5hydroxytryptamine (5-HT, serotonin), bradykinins and histamine, the chemicals
responsible for causing the features of the carcinoid syndrome as they enter the
systemic circulation.
Acute appendicitis due to obstruction of the lumen by the carcinoid tumour.
Pellagra syndrome (the three Ds diarrhoea, dermatitis and dementia) due to
nicotinic acid deficiency (carcinoid tumours utilise tryptophan, a nicotinamide
precursor).

https://mypastest.pastest.com/Secure/TestMe/Browser/429893#Top

1/3

8/22/2016

MyPastest

Carcinoid tumours may also cause right-sided cardiac valve lesions, which lead to
peripheral oedema. Other symptoms associated with carcinoids include wheeze, flushing,
diarrhoea and weight loss.
Urinary 5-hyroxyindoleacetic acid (5-HIAA) levels are typically very elevated in carcinoid
syndrome, so this is the best test here.

Colonoscopy (Option A) is incorrect. Most gastrointestinal tract carcinoid tumours are within
the small bowel, so a normal colonoscopy would not be helpful for ruling out the presence of
a GI carcinoid tumour.
Computed tomography scan of the abdomen (Option B) is incorrect. Carcinoid tumours may
be variable in size and location (with approximately 30% of them occurring within the lungs),
so a CT scan of the abdomen would not necessarily demonstrate the tumour.
Fasting serum gastrin (Option C) is incorrect. Fasting serum gastrin levels may be very
elevated in people with gastrinoma (another form of neuroendocrine tumour), but not in
carcinoid syndrome.
Urinary catecholamines (Option D) is incorrect. Urinary catecholamines are a useful test in
people with suspected phaeochromocytoma and paraganglionoma, but levels would not be
expected to be raised in those with carcinoid syndrome.
44911

Next Question

Previous Question

Tag Question

Feedback

End Review

Difficulty: Average
Peer Responses

Session Progress
Responses Correct:

Responses Incorrect:

298

Responses Total:

298

Responses - % Correct:

https://mypastest.pastest.com/Secure/TestMe/Browser/429893#Top

0%

2/3

8/22/2016

MyPastest

Blog (https://www.pastest.com/blog) About Pastest (https://www.pastest.com/about-us)


Contact Us (https://www.pastest.com/contact-us) Help (https://www.pastest.com/help)
Pastest 2016

https://mypastest.pastest.com/Secure/TestMe/Browser/429893#Top

3/3

8/22/2016

MyPastest

Back to Filters (/Secure/TestMe/Filter/429893/QA)

Question 218 of 298

A 65-year-old man with long-standing insulin-dependent diabetes mellitus was referred with
nausea and recurrent vomiting. At endoscopy, a large gastric food residue was noted despite
a 6-hour fast.
What would the most useful diagnostic test be?
A

Barium meal and follow-through

Gamma scintigraphy, gastric emptying study.

Lactose hydrogen breath test

Lying and standing blood pressure

Oesophageal manometry

Explanation

The answer is Gamma scintigraphy, gastric emptying study


Diabetes-associated gastroparesis occurs in ~10% of diabetics after 10 years. It appears
to be very underdiagnosed, and probably represents the most common gastrointestinal
complication of the condition. Other causes of gastroparesis include post-operative
(particularly related to gastric and thoracic surgery), medications (such as antimuscarinic medications and calcium channel blockers) and neurological conditions,
although a significant proportion of cases are apparently idiopathic. Scintigraphic gastric
emptying studies are the most reliable diagnostic test. Management is with prokinetics
(domperidone and erythromycin). Gastric pacing is also possible.

Barium meal and follow-through (Option A) is incorrect. A barium meal and follow-through
may be useful for excluding obstruction of the upper gastrointestinal tract (which may
present with similar symptoms), but is not the best test for diagnosing the dysmotility of
gastroparesis.
Lactose hydrogen breath test (Option C) is incorrect. Diabetes is a risk factor for small bowel
bacterial overgrowth, but this is a much less common cause of these symptoms than

https://mypastest.pastest.com/Secure/TestMe/Browser/429893#Top

1/2

8/22/2016

MyPastest

gastroparesis in a patient such as this, and the lactose hydrogen breath test is therefore likely
to be normal.
Lying and standing blood pressure (Option D) is incorrect. A proportion of patients will have
an autonomic neuropathy but not all, so a postural drop in blood pressure is unlikely to be
found.
Oesophageal manometry (Option E) is incorrect. Oesophageal manometry is helpful in
investigating oesophageal symptoms (such as reflux and dysphagia), but is unlikely to be of
any benefit here.
44984

Next Question

Previous Question

Tag Question

Feedback

End Review

Difficulty: Average
Peer Responses

Session Progress
Responses Correct:

Responses Incorrect:

298

Responses Total:

298

Responses - % Correct:

0%

Blog (https://www.pastest.com/blog) About Pastest (https://www.pastest.com/about-us)


Contact Us (https://www.pastest.com/contact-us) Help (https://www.pastest.com/help)
Pastest 2016

https://mypastest.pastest.com/Secure/TestMe/Browser/429893#Top

2/2

8/22/2016

MyPastest

Back to Filters (/Secure/TestMe/Filter/429893/QA)

Question 219 of 298

A 44-year-old man with a long history of alcohol excess is admitted with abdominal pain,
weight loss of 10 kg in the previous 6 months and diarrhoea. Investigations show speckled
pancreatic calcification on plain abdominal X-ray and an albumin level of 23 g/l.
What is the diagnostic test most likely to establish the underlying cause of his symptoms?
A

Computed tomography scan of the pancreas

Faecal elastase

Lactulose hydrogen breath test

Oral glucose tolerance test

Serum folate

Explanation

The answer is Faecal elastase


This man has evidence of alcohol-related chronic pancreatitis and malabsorption, with
alcohol being the most common aetiology of the condition. Other causes include:
Chronic obstruction of the pancreatic and/or distal common bile duct (eg recurrent
gallstone disease, pancreas divisum, etc)
Hereditary causes (eg mutations in the CFTR gene)
Tropical pancreatitis
Autoimmune pancreatitis
Idiopathic.
The faecal elastase test is the most straightforward and sensitive of the pancreatic
function tests and will demonstrate pancreatic exocrine insufficiency. Other options
include a pancreolauryl test and absorption of a test meal (eg a Lundh meal), but these
tests are now rarely performed. Treatment includes vitamin and pancreatic enzyme
supplementation, and the avoidance of alcohol.

https://mypastest.pastest.com/Secure/TestMe/Browser/429893#Top

1/2

8/22/2016

MyPastest

Computed tomography scan of the pancreas (Option A) is incorrect. A CT scan of the


pancreas may outline structural abnormalities of the pancreas that may predispose to
chronic pancreatitis, or may directly demonstrate chronic pancreatitis itself; however, a
normal CT scan would not rule out chronic pancreatitis, so it is not the best diagnostic test.
Furthermore, an MRCP/MRI scan of the pancreas often gives more detail regarding structural
abnormalities of the pancreas and biliary tree than CT.
Lactulose hydrogen breath test (Option C) is incorrect. A lactulose hydrogen breath test is
useful for diagnosing small bowel bacterial overgrowth, but is not helpful in demonstrating
pancreatic insufficiency.
Oral glucose tolerance test (Option D) is incorrect. An oral glucose tolerance test may be
used to detect endocrine insufficiency occurring in association with chronic pancreatitis, but
is not helpful for diagnosing the condition per se.
Serum folate (Option E) is incorrect. Folate metabolism may be affected by having chronic
pancreatitis, but it has no use in itself as a diagnostic test for the condition.
44985

Next Question

Previous Question

Tag Question

Feedback

End Review

Difficulty: Average
Peer Responses

Session Progress
Responses Correct:

Responses Incorrect:

298

Responses Total:

298

Responses - % Correct:

0%

Blog (https://www.pastest.com/blog) About Pastest (https://www.pastest.com/about-us)


Contact Us (https://www.pastest.com/contact-us) Help (https://www.pastest.com/help)
Pastest 2016

https://mypastest.pastest.com/Secure/TestMe/Browser/429893#Top

2/2

8/22/2016

MyPastest

Back to Filters (/Secure/TestMe/Filter/429893/QA)

Question 220 of 298

A 65-year-old woman was admitted with acute severe abdominal pain, vomiting and fever.
Investigations showed an amylase level of 1250 IU/l and a corrected calcium concentration of
1.78 mmol/l.
Which one of the following suggests a poor prognosis?
A

Amylase1250 IU/1

Glucose 9.1 mmol/l

Pao2 8.7 kPa

pH 7.30

White cell count 18 x 109/l

Explanation

The answer is White cell count 18 109/l


There are a number of scoring systems that have been validated for assessing severity in
acute pancreatitis, with ongoing debate about which is the most sensitive and specific.
One scoring system that has been validated for use in people with both alcohol- and
gallstone-related pancreatitis is the Glasgow criteria.

PANCREAS:
P Pao2 < 8 kPa
A Age > 55 years
N Neutrophilia, ie white cell count (WCC) > 15 109/l
C Calcium < 2 mmol/l
R Renal dysfunction, i.e. urea > 16 mmol/l
E Enzymes, ie aspartate aminotransferase (AST) > 200 IU/l lactate dehydrogenase (LDH)
> 600 IU/l
A Albumin < 30 g/l
S Sugar, ie glucose > 10 mmol/l

https://mypastest.pastest.com/Secure/TestMe/Browser/429893#Top

1/2

8/22/2016

MyPastest

Although all of these criteria are individually associated with a worse prognosis, three of
the criteria together define severe acute pancreatitis, and a patient likely to require highdependency/intensive care support.

Amylase1250 IU/l (Option A) is incorrect. This would not be considered as a poor prognostic
factor.
Glucose 9.1 mmol/l (Option B) is incorrect. This would not be considered as a poor
prognostic factor.
Pao2 28.7 kPa (Option C) is incorrect. This would not be a poor prognostic factor as
described.
pH 7.30 (Option D) is incorrect. This would not be considered as a poor prognostic factor.
44986

Next Question

Previous Question

Tag Question

Feedback

End Review

Difficulty: Average
Peer Responses

Session Progress
Responses Correct:

Responses Incorrect:

298

Responses Total:

298

Responses - % Correct:

0%

Blog (https://www.pastest.com/blog) About Pastest (https://www.pastest.com/about-us)


Contact Us (https://www.pastest.com/contact-us) Help (https://www.pastest.com/help)
Pastest 2016

https://mypastest.pastest.com/Secure/TestMe/Browser/429893#Top

2/2

8/22/2016

MyPastest

Back to Filters (/Secure/TestMe/Filter/429893/QA)

Question 221 of 298

A 34-year-old man was admitted after a 32-day period of starvation. His body mass index
was 17 kg/m2 and he estimated that he had lost 18 kg in weight. He was commenced on
enteral feeding.
Which of the following is the most serious consequence of refeeding syndrome?
A

Albumin 28 g/l (normal range 37-49 g/l)

Calcium 2.74 mmol/l (2.2-2.6 mmol/l)

Ferritin 434 ug/l (15-300 ug/l)

Magnesium 0.69 mmol/l (0.75-1.05 mmol/l)

Phosphate 0.49 mmol/l (0.81-1.4 mmol/l)

Explanation

The answer is Phosphate 0.49 mmol/l (0.811.4 mmol/l)


The refeeding syndrome is driven by insulin in response to the intake of calories after a
prolonged fast. The insulin drives potassium and phosphate into the cells, leading to
acute hypophosphataemia. Although all of the other biochemical parameters given as
options in the question may also be found in a person with refeeding syndrome,
hypophosphataemia is the one with the most profound implications for tissue
dysfunction, making it the correct answer here. This is because the increased insulin
levels that accompany refeeding syndrome also trigger cells to produce a variety of
molecules that require phosphate (such as ATP), but these cannot be produced in the
presence of hypophosphataemia; this results in progressive tissue anaerobic respiration,
and eventually in cardiac dysfunction.
The major clinical complications of refeeding syndrome are cardiac arrhythmias and
sudden death, but rhabdomyolysis, pancreatitis and hepatitis may also occur.
Management involves a reduction in the calorie intake for the first 2448 hours (to half
the calculated requirement), aggressive phosphate replacement, vitamin
supplementation (particularly B vitamins) and careful electrolyte monitoring.

https://mypastest.pastest.com/Secure/TestMe/Browser/429893#Top

1/2

8/22/2016

MyPastest

Albumin 28 g/l (normal range 3749 g/l) (Option A) is incorrect. Although this is a possible
finding, it would not be as potentially serious as hypophosphataemia for the reasons
described.
Calcium 2.74 mmol/l (2.22.6 mmol/l) (Option B) is incorrect. Although this is a possible
finding, it would not be as potentially serious as hypophosphataemia for the reasons
described.
Ferritin 434 g/l (15300 g/l) (Option C) is incorrect. Although this is a possible finding, it
would not be as potentially serious as hypophosphataemia for the reasons described.
Magnesium 0.69 mmol/l (0.751.05 mmol/l) (Option D) is incorrect. Although this is a
possible finding, it would not be as potentially serious as hypophosphataemia for the reasons
described.
44987

Next Question

Previous Question

Tag Question

Feedback

End Review

Difficulty: Average
Peer Responses

Session Progress
Responses Correct:

Responses Incorrect:

298

Responses Total:

298

Responses - % Correct:

0%

Blog (https://www.pastest.com/blog) About Pastest (https://www.pastest.com/about-us)


Contact Us (https://www.pastest.com/contact-us) Help (https://www.pastest.com/help)
Pastest 2016

https://mypastest.pastest.com/Secure/TestMe/Browser/429893#Top

2/2

8/22/2016

MyPastest

Back to Filters (/Secure/TestMe/Filter/429893/QA)

Question 222 of 298

A woman with known abetalipoproteinaemia was reviewed in the clinic.


Which of the following would be most in keeping with her disease?
A

Albumin 28 g/l (normal range 3749 g/l)

Calcium 2.29 mmol/l (2.22.6 mmol/l)

Ferritin 6 g/l (15300 g/l)

Folate 0.8 g/l (211 g/l)

Prothrombin time 18 s (< 12 s)

Explanation

The answer is Prothrombin time 18 s (< 12 s)


Abetalipoproteinaemia is the failure of synthesis of chylomicrons in the villi resulting
from an inability to make -lipoproteins because of a mutation inherited in an autosomal
recessive manner. This results in malabsorption of dietary fat, as well as malabsorption of
the fat-soluble vitamins A, D, E and K. Loss of vitamin D may result in rickets, whereas
vitamin K deficiency results in the failure of vitamin K-dependent clotting-factor
synthesis, thus prolonging the international normalised ratio (INR) or prothrombin time.

Albumin 28 g/l (normal range 3749 g/l) (Option A) is incorrect. Albumin levels would not be
expected to be directly altered by the condition.
Calcium 2.29 mmol/l (2.22.6 mmol/l) (Option B) is incorrect. Calcium levels would be
expected to be low, if anything, because of the associated vitamin D deficiency. The mainstay
of treatment is replacement of the vitamins.
Ferritin 6 g/l (15300 g/l) (Option C) is incorrect. Ferritin levels would not be expected to
be directly altered by the condition.
Folate 0.8 g/l (211 g/l) (Option D) is incorrect. Folate levels would not be expected to be
directly altered by the condition.
44989

https://mypastest.pastest.com/Secure/TestMe/Browser/429893#Top

1/2

8/22/2016

MyPastest

Next Question

Previous Question

Tag Question

Feedback

End Review

Difficulty: Average
Peer Responses

Session Progress
Responses Correct:

Responses Incorrect:

298

Responses Total:

298

Responses - % Correct:

0%

Blog (https://www.pastest.com/blog) About Pastest (https://www.pastest.com/about-us)


Contact Us (https://www.pastest.com/contact-us) Help (https://www.pastest.com/help)
Pastest 2016

https://mypastest.pastest.com/Secure/TestMe/Browser/429893#Top

2/2

8/22/2016

MyPastest

Back to Filters (/Secure/TestMe/Filter/429893/QA)

Question 223 of 298

A 56-year-old woman with type 1 diabetes mellitus and hypothyroidism was referred for
investigation of a megaloblastic anaemia.
Which of the following is most likely to be the cause?
A

Crohn's disease

Metaformin therapy

Pancreatic insufficiency

Pernicious anaemia

Small-bowel bacterial overgrowth

Explanation

The answer is Pernicious anaemia


The list of causes of vitamin B12 deficiency is long, and includes:

Gastric causes pernicious anaemia, chronic severe atrophic gastritis


Pancreatic any cause of pancreatic insufficiency
Small-bowel bacterial overgrowth (since bacteria utilise vitamin B12), terminal ileal
resection, severe terminal ileal disease (ie Crohns disease)
Tuberculosis
Metformin therapy
ZollingerEllison syndrome.

Although all of the options listed can cause vitamin B12 malabsorption, the information
provided here that this woman has other autoimmune disease makes pernicious anaemia
the most likely explanation. Pernicious anaemia results from anti-parietal and antiintrinsic factor antibodies, together resulting in reduced intrinsic factor production,
reduced intrinsic factor activity, and consequently reduced B12 absorption. Salivary Rhttps://mypastest.pastest.com/Secure/TestMe/Browser/429893#Top

1/3

8/22/2016

MyPastest

binders bind to vitamin B12, and these protect the vitamin from gastric acid
denaturation. Intrinsic factor, synthesised in the stomach, cannot bind to vitamin B12 until
the R-binder has been split off by pancreatic trypsin. Once the vitamin B12intrinsic
factor complex is formed, it is actively absorbed in the terminal ileum.

Crohns disease (Option A) is incorrect. No history is given that would particularly suggest
Crohns disease (such as diarrhoea, abdominal pain and/or weight loss).
Metformin therapy (Option B) is incorrect. This woman may be using metformin if she had
type 2 diabetes mellitus, but this is not a treatment used for type 1.
Pancreatic insufficiency (Option C) is incorrect. This woman has pancreatic endocrine failure
because of diabetes, but there is no information to suggest her having pancreatic exocrine
failure too.
Small-bowel bacterial overgrowth (Option E) is incorrect. Although small-bowel bacterial
overgrowth occurs at higher rates in diabetics than in people without the condition, there is
no description of diarrhoea, abdominal bloating, or any other features that may suggest this
condition.
44990

Next Question

Previous Question

Tag Question

Feedback

End Review

Difficulty: Average
Peer Responses

Session Progress
Responses Correct:

Responses Incorrect:

298

Responses Total:

298

Responses - % Correct:

0%

Blog (https://www.pastest.com/blog) About Pastest (https://www.pastest.com/about-us)


Contact Us (https://www.pastest.com/contact-us) Help (https://www.pastest.com/help)
https://mypastest.pastest.com/Secure/TestMe/Browser/429893#Top

2/3

8/22/2016

MyPastest

Pastest 2016

https://mypastest.pastest.com/Secure/TestMe/Browser/429893#Top

3/3

8/22/2016

MyPastest

Back to Filters (/Secure/TestMe/Filter/429893/QA)

Question 224 of 298

A 46-year-old woman was referred with profuse watery diarrhoea and dehydration.
Investigations showed an average daily stool weight of 4353 g/24 hours and a serum
vasoactive intestinal polypeptide (VIP) level of > 400 pg/ml (normal range < 20 pg/ml).
What is the most likely mechanism of her diarrhoea?
A

Infective due to small-bowel overgrowth

Inflammatory due to intercurrent malignancy

Malabsorptive due to pancreatic insufficiency

Osmotic secondary to high oral water intake

Secretory due to enterocyte stimulation

Explanation

The answer is Secretory due to enterocyte stimulation


The diagnosis is VIPoma, a gut hormone-secreting tumour that is typically found in the
pancreas or small bowel (VIP stands for vasoactive intestinal polypeptide). The diarrhoea
is secretory (due to VIP causing general cellular secretion), with the result being massive,
watery diarrhoea and electrolytes deficiencies (particularly hypokalaemia). The normal
daily stool weight is 250300 g, so the figures given here are significantly elevated.

Infective due to small-bowel overgrowth (Option A) is incorrect. Small-bowel bacterial


overgrowth causes diarrhoea through different routes (including secretory diarrhoea related
to bacterial toxins, malabsorption of food components and osmotic drive from these
malabsorbed components), but does not in itself cause such profound diarrhoea as found
here.
Inflammatory due to intercurrent malignancy (Option B) is incorrect. There may be gut
inflammation related to a neuro-endocrine tumour such as this, but it generally is mild and
contributes only modestly to the diarrhoea.

https://mypastest.pastest.com/Secure/TestMe/Browser/429893#Top

1/2

8/22/2016

MyPastest

Malabsorptive due to pancreatic insufficiency (Option C) is incorrect. Large pancreatic


tumours may result in pancreatic exocrine insufficiency, but this would only occur if a tumour
was large enough to replace the majority of the normal pancreatic parenchyma.
Osmotic secondary to high oral water intake (Option D) is incorrect. For oral water intake to
act osmotically to cause such significant diarrhoea, the intake would have to be hugely
increased compared to normal levels, making this very unlikely to be contributory.
44991

Next Question

Previous Question

Tag Question

Feedback

End Review

Difficulty: Average
Peer Responses

Session Progress
Responses Correct:

Responses Incorrect:

298

Responses Total:

298

Responses - % Correct:

0%

Blog (https://www.pastest.com/blog) About Pastest (https://www.pastest.com/about-us)


Contact Us (https://www.pastest.com/contact-us) Help (https://www.pastest.com/help)
Pastest 2016

https://mypastest.pastest.com/Secure/TestMe/Browser/429893#Top

2/2

8/22/2016

MyPastest

Back to Filters (/Secure/TestMe/Filter/429893/QA)

Question 225 of 298

A 27-year-old woman who had previously undergone a terminal ileal and limited right-colon
resection for Crohns disease was seen in the clinic. She reported increased diarrhoea but was
otherwise well. Investigations showed: C-reactive protein (CRP) < 5 g/dl, haemoglobin 13.2
g/dl, white cell count (WCC) 8.6 109/l, platelets 244 109/l. Repeat colonoscopy was
normal to the neoterminal ileum; a barium follow-through showed a normal mucosa; and a
lactose hydrogen breath test was normal.
What is the most likely cause of her diarrhoea?
A

Collagenous colitis

Bile-salt malabsorption

Mesalazine

Primary sclerosing cholangitis

Small-bowel bacterial overgrowth

Explanation

The answer is Bile-salt malabsorption


This is a classic scenario for bile-salt malabsorption, which causes a secretory diarrhoea.
This is because bile-salt metabolism is usually via the enterohepatic circulation, being
reabsorbed in the terminal ileum. After resection of the terminal ileum, these bile salt
salts will no longer be re-absorbed, so enter the colon where they irritate the mucosal
lining (causing water secretion into the lumen) and affect gut motility, together resulting
in diarrhoea. There is a theoretical risk of failure of absorption of fat-soluble vitamins, but
this rarely becomes clinically significant. Treatment is with bile-salt sequestrants
(binders), such as cholestyramine.

Collagenous colitis (Option A) is incorrect. Collagenous colitis is a cause of diarrhoea that is


pathologically defined by a thickening of the subepithelial collagen band, and may occur in
association with coeliac disease; this is diagnosed histologically, typically in people being
investigated for diarrhoea with a macroscopically normal colonoscopy but who have had
biopsies taken.
https://mypastest.pastest.com/Secure/TestMe/Browser/429893#Top

1/2

8/22/2016

MyPastest

Mesalazine (Option C) is incorrect. Mesalazine may cause an increase in bowel frequency, but
rarely causes true diarrhoea.
Primary sclerosing cholangitis (Option D) is incorrect. No information is given on this
womans liver blood tests or biliary tree imaging, so there is nothing to support her having
primary sclerosing cholangitis (PSC); furthermore, PSC is not associated with diarrhoea.
Small-bowel bacterial overgrowth (Option E) is incorrect. Small-bowel bacterial overgrowth
is rare but may occur after intestinal surgery (although the lack of an early peak on the
breath test counts against this).
44992

Next Question

Previous Question

Tag Question

Feedback

End Review

Difficulty: Average
Peer Responses

Session Progress
Responses Correct:

Responses Incorrect:

298

Responses Total:

298

Responses - % Correct:

0%

Blog (https://www.pastest.com/blog) About Pastest (https://www.pastest.com/about-us)


Contact Us (https://www.pastest.com/contact-us) Help (https://www.pastest.com/help)
Pastest 2016

https://mypastest.pastest.com/Secure/TestMe/Browser/429893#Top

2/2

8/22/2016

MyPastest

Back to Filters (/Secure/TestMe/Filter/429893/QA)

Question 226 of 298

A 45-year-old man has been referred. His father died at the age of 56 from a sigmoid colon
adenocarcinoma, and his uncle underwent a colectomy at the age of 61 for a caecal
carcinoma.
What is his lifetime risk of colorectal carcinoma?
A

1 in 2

1 in 12

1 in 25

1 in 50

1 in 200

Explanation

The answer is 1 in 12
This man has one first-degree relative who had colorectal carcinoma at a young age, as
well as a second-degree relative. The risks for colorectal carcinoma are:
Population risk 1 in 40
One first-degree relative more than 45 years old
1 in 17 One first-degree plus one second-degree relative
1 in 12 Two first-degree relatives
1 in 6 Familial polyposis
1 in 2
Screening is by colonoscopy, with the age at which screening starts influenced by the
age at which other family members were diagnosed with the condition.

1 in 2 (Option A) is incorrect. This is the risk for familial polyposis.


1 in 25 (Option C) is incorrect.
1 in 50 (Option D) is incorrect.
https://mypastest.pastest.com/Secure/TestMe/Browser/429893#Top

1/2

8/22/2016

MyPastest

1 in 200 (Option E) is incorrect


44993

Next Question

Previous Question

Tag Question

Feedback

End Review

Difficulty: Average
Peer Responses

Session Progress
Responses Correct:

Responses Incorrect:

298

Responses Total:

298

Responses - % Correct:

0%

Blog (https://www.pastest.com/blog) About Pastest (https://www.pastest.com/about-us)


Contact Us (https://www.pastest.com/contact-us) Help (https://www.pastest.com/help)
Pastest 2016

https://mypastest.pastest.com/Secure/TestMe/Browser/429893#Top

2/2

8/22/2016

MyPastest

Back to Filters (/Secure/TestMe/Filter/429893/QA)

Question 227 of 298

A 58-year-old woman with dyspepsia was referred for endoscopy.


Which of the following is not a risk factor for gastric adenocarcinoma?
A

Aspirin use

Helicobacter pylori

Low dietary Vitamin C

Partial Gastrectomy

Pernicious anaemia

Explanation

The answer is Aspirin use Aspirin in fact appears to reduce the risk of oesophageal, gastric and colorectal
carcinoma, probably by inhibition of cyclooxygenase 2 (COX-2) and several other
malignancy-related pathways.
The risk factors for gastric cancer include:
Helicobacter pylori
Low dietary vitamin C
Family history
High dietary salt
Racial origin (Japan > UK > Sweden)
Gastric surgery (with the risk particularly increased with biliary diversion to the
stomach)
Pernicious anaemia
Smoking.

https://mypastest.pastest.com/Secure/TestMe/Browser/429893#Top

1/2

8/22/2016

MyPastest

Helicobacter pylori (Option B) is incorrect. H. pylori is a risk factor for gastric


adenocarcinoma.
Low dietary vitamin C (Option C) is incorrect. Low dietary vitamin C is a risk factor for
gastric adenocarcinoma.
Partial gastrectomy (Option D) is incorrect. Partial gastrectomy is a risk factor for gastric
adenocarcinoma.
Pernicious anaemia (Option E) is incorrect. Pernicious anaemia is a risk factor for gastric
adenocarcinoma.
44994

Next Question

Previous Question

Tag Question

Feedback

End Review

Difficulty: Average
Peer Responses

Session Progress
Responses Correct:

Responses Incorrect:

298

Responses Total:

298

Responses - % Correct:

0%

Blog (https://www.pastest.com/blog) About Pastest (https://www.pastest.com/about-us)


Contact Us (https://www.pastest.com/contact-us) Help (https://www.pastest.com/help)
Pastest 2016

https://mypastest.pastest.com/Secure/TestMe/Browser/429893#Top

2/2

8/22/2016

MyPastest

Back to Filters (/Secure/TestMe/Filter/429893/QA)

Question 228 of 298

A 78-year-old woman was found to have a gastric ulcer at endoscopy.


Which of the following has the lowest risk of causing this finding?
A

Aspirin

Celecoxib

Diclofenac

Ibuprofen

Piroxicam

Explanation

The answer is Celecoxib


Cyclo-oxygenase-2 (COX-2) selective inhibitors (coxibs) are safer than NSAIDs, but still
carry a risk of peptic ulceration that is greater than placebo. The other key complications
of NSAIDs and coxibs include hypertension, peripheral oedema and interstitial nephritis.

Ibuprofen is the safest of the non-steroidal anti-inflammatory drugs (NSAIDs) with


regards to peptic ulcer risk, with piroxicam one of the least safe (with diclofenac and
aspirin carrying an intermittent degree of risk).
Coxibs lack the benefits of aspirin in terms of cardiovascular risk. In the case of low-dose
aspirin, for the majority of patients with controlled blood pressure, the cardioprotective
benefits outweigh the risks of bleeding. Some data suggest that the concomitant use of
aspirin and another NSAID reduces the cardiovascular protection, and this is therefore
not recommended.

Aspirin (Option A) is incorrect. Aspirin has a higher risk of causing gastric ulcer than
celecoxib.

https://mypastest.pastest.com/Secure/TestMe/Browser/429893#Top

1/2

8/22/2016

MyPastest

Diclofenac (Option C) is incorrect. Diclofenac has a higher risk of causing gastric ulcer than
celecoxib.
Ibuprofen (Option D) is incorrect. Ibuprofen has a higher risk of causing gastric ulcer than
celecoxib.
Piroxicam (Option E) is incorrect. Piroxicam has a higher risk of causing gastric ulcer than
celecoxib.
44995

Next Question

Previous Question

Tag Question

Feedback

End Review

Difficulty: Average
Peer Responses

Session Progress
Responses Correct:

Responses Incorrect:

298

Responses Total:

298

Responses - % Correct:

0%

Blog (https://www.pastest.com/blog) About Pastest (https://www.pastest.com/about-us)


Contact Us (https://www.pastest.com/contact-us) Help (https://www.pastest.com/help)
Pastest 2016

https://mypastest.pastest.com/Secure/TestMe/Browser/429893#Top

2/2

8/22/2016

MyPastest

Back to Filters (/Secure/TestMe/Filter/429893/QA)

Question 229 of 298

A 44-year-old woman was referred with a change in bowel habit.


Which of the following would most strongly suggest an organic rather than a functional
cause?
A

Abdominal Bloating

Abdominal Pain

Alternating diarrhoea and constipation

Mucus per rectum

Nocturnal diarrhoea

Explanation

The answer is Nocturnal diarrhoea


The presence of nocturnal diarrhoea should prompt careful investigation for a structural
cause of bowel disease, as this is not a recognised feature of functional bowel disorders
such as irritable bowel syndrome. Other red flag symptoms that should prompt
investigation for structural disease include rectal bleeding, weight loss and the
development of a new change of bowel habit towards diarrhoea.

The other options are all common features of functional bowel disorders. Functional
bowel disorders are often diagnosed on the basis of a constellation of symptoms,
without any specific investigation being required. However, these symptoms could still
signify underlying structural bowel disease (particularly abdominal pain), and an
evaluation is required on a case-by-case basis as to the appropriate degree of
investigation that is required.

Abdominal bloating (Option A) is incorrect. This would not be the most strongly suggestive
of an organic cause.

https://mypastest.pastest.com/Secure/TestMe/Browser/429893#Top

1/2

8/22/2016

MyPastest

Abdominal pain (Option B) is incorrect. This would not be the most strongly suggestive of an
organic cause.
Alternating diarrhoea and constipation (Option C) is incorrect. This would not be the most
strongly suggestive of an organic cause.
Mucus per rectum (Option D) is incorrect. This would not be the most strongly suggestive of
an organic cause.
44996

Next Question

Previous Question

Tag Question

Feedback

End Review

Difficulty: Average
Peer Responses

Session Progress
Responses Correct:

Responses Incorrect:

298

Responses Total:

298

Responses - % Correct:

0%

Blog (https://www.pastest.com/blog) About Pastest (https://www.pastest.com/about-us)


Contact Us (https://www.pastest.com/contact-us) Help (https://www.pastest.com/help)
Pastest 2016

https://mypastest.pastest.com/Secure/TestMe/Browser/429893#Top

2/2

8/22/2016

MyPastest

Back to Filters (/Secure/TestMe/Filter/429893/QA)

Question 230 of 298

A 27-year-old man was referred with a 10-week history of worsening diarrhoea. On review, he
was opening his bowels 11 times per day and had blood mixed in with the stools. Full blood
count showed Hb of 10.9 g/dl. The problem had worsened over the past few days, and he was
now suffering from severe abdominal pain and distension. He had a negative stool sample
sent one month earlier.
What is the first investigation that you would request?
A

Abdominal radiograph

Barium Enema

Computed Tomography (CT) scan of the abdomen

Flexible sigmoidoscopy

Stool microscopy

Explanation

The answer is Abdominal radiograph


This man has acute severe colitis, which is characterised in the Truelove and Witts criteria
by the following features:
Fever of > 37.8 C
Bowels open more than six times per day, with blood
Erythrocyte sedimentation rate (ESR) > 30 mm in 1 hour
Pulse > 90 bpm.
The key diagnosis to exclude is a toxic megacolon; presence of toxic megacolon puts the
patient at higher risk of perforation, which could be life-threatening. A plain abdominal
radiograph is therefore the most important next test.
Causes of toxic megacolon include:
Ulcerative colitis
https://mypastest.pastest.com/Secure/TestMe/Browser/429893#Top

1/3

8/22/2016

MyPastest

Crohns colitis
Pseudomembranous colitis (Clostridium difficile infection).
Dysenteric illnesses (ie enteroinvasive and enterohaemorrhagic Escherichia coli,
Shigella spp.).

Barium enema (Option B) is incorrect.


When barium enemas are used in people with chronic ulcerative colitis, a lead pipe
appearance is typically found (ie complete loss of haustral markings in the wall of the
affected colon); however, it would not be safe to perform a barium enema with someone as
acutely unwell as this man because of the risk of perforation.
Computed tomography (CT) scan of the abdomen (Option C) is incorrect.
A CT scan may be useful for assessing the extent of colitis and for any complications of his
condition (eg collection); however, this requires a much higher radiation dose than an
abdominal radiograph, and should not therefore be the first choice in assessing for toxic
megacolon.
Flexible sigmoidoscopy (Option D) is incorrect.
Flexible sigmoidoscopy is useful for evaluating the degree of colitis and in obtaining
histology to confirm the diagnosis. However, it must be performed with considerable care in
severe colitis, as the bowel wall will be inflamed and friable and therefore at increased risk of
perforation.
Stool microscopy (Option E) is incorrect.
Although stool microscopy is clearly warranted to assess for this man having an infective
component to his colitis, the first-line intervention should still be to assess toxic megacolon,
given the seriousness of this finding, if present.
44998

Next Question

Previous Question

Tag Question

Feedback

End Review

Difficulty: Average
Peer Responses

Session Progress
https://mypastest.pastest.com/Secure/TestMe/Browser/429893#Top

2/3

8/22/2016

MyPastest

Responses Correct:

Responses Incorrect:

298

Responses Total:

298

Responses - % Correct:

0%

Blog (https://www.pastest.com/blog) About Pastest (https://www.pastest.com/about-us)


Contact Us (https://www.pastest.com/contact-us) Help (https://www.pastest.com/help)
Pastest 2016

https://mypastest.pastest.com/Secure/TestMe/Browser/429893#Top

3/3

8/22/2016

MyPastest

Back to Filters (/Secure/TestMe/Filter/429893/QA)

Question 231 of 298

A 21-year-old man with a family history of familial adenomatous polyposis (FAP) was referred
for post-operative follow-up.
Which of the following statements about FAP is correct?
A

Duodenal malignancy is an important cause of death

Gastric adenomas are rare

Rectal surveillance is not required post-surgery

Selective COX-2 inhibition increases the risk of malignant transformation

Small-bowel hamartomas can result in chronic abdominal pain

Explanation

The answer is Duodenal malignancy is an important cause of death


Germline mutations of the APC gene result in the syndrome of familial adenomatous
polyposis (FAP). This results in multiple small- and large-bowel adenomas. The risk of
colorectal carcinoma is reported to be 90100%, with a median age of developing such
tumours of 30 years. The earliest cancers have been detected in people in their early
teens. Prophylactic colectomy is therefore advised at the age of 16.
The second most common malignant cause of death in people with FAP is duodenal
adenoma with malignant transformation. As such, endoscopic surveillance is mandatory.
Fundic gland polyps are very common in people with FAP. Low-grade dysplasia may
occur in up to 50% of these, although malignant changes tend to be rare.
Gardner syndrome, caused by another APC gene mutation, is the FAP genotype with
osteomas of the jaw, skull and long bones. Other soft-tissue tumours may also occur.

Gastric adenomas are rare (Option B) is incorrect.


As described, FAP results in multiple small- and large-bowel adenomas.
Rectal surveillance is not required post-surgery (Option C) is incorrect.
https://mypastest.pastest.com/Secure/TestMe/Browser/429893#Top

1/2

8/22/2016

MyPastest

The post-surgical rectal remnant remains at risk of adenomas and carcinoma, so rectal
surveillance is definitely indicated.
Selective COX-2 inhibition increases the risk of malignant transformation (Option D) is
incorrect.
Aspirin, sulindac and selective cyclooxygenase 2 (COX-2) inhibitors reduce the risk of further
polyp formation, rather than increase it.
Small-bowel hamartomas can result in chronic abdominal pain (Option E) is incorrect.
Hamartomas are well described in PeutzJeghers syndrome (another inherited cancer
syndrome), but are not associated with FAP.
45000

Next Question

Previous Question

Tag Question

Feedback

End Review

Difficulty: Average
Peer Responses

Session Progress
Responses Correct:

Responses Incorrect:

298

Responses Total:

298

Responses - % Correct:

0%

Blog (https://www.pastest.com/blog) About Pastest (https://www.pastest.com/about-us)


Contact Us (https://www.pastest.com/contact-us) Help (https://www.pastest.com/help)
Pastest 2016

https://mypastest.pastest.com/Secure/TestMe/Browser/429893#Top

2/2

8/22/2016

MyPastest

Back to Filters (/Secure/TestMe/Filter/429893/QA)

Question 232 of 298

A 33-year-old man with ulcerative pancolitis was seen. His symptoms were worsening despite
intravenous hydrocortisone.
Which of the following statements about treatment options is correct?
A

Ciclosporin is indicated to induce remission

Cytomegalovirus is a common cause of non-responsive colitis

Methotrexate is indicated to induce remission

Non-steroidal anti-inflammatory drugs (NSAIDs) are a useful adjunct to therapy.

Surgery is contraindicated

Explanation

The answer is Ciclosporin is indicated to induce remission


The options for treating acute severe ulcerative colitis that is refractory to intravenous
hydrocortisone are medical or surgical. The major options for medical therapy are either
intravenous ciclosporin or infliximab; the surgical option is a subtotal colectomy.

Other immunosuppressive medications that have a role in treating chronic ulcerative


colitis including the thiopurines, azathioprine and 6-mercaptopurine take several
weeks before they start to produce benefit. As such, unlike ciclosporin or infliximab,
these may not be used in people with refractory acute severe colitis to induce remission
quickly. However, it is not an unusual scenario for a patient with refractory acute severe
colitis to start ciclosporin and a thiopurine together, with the hope that the ciclosporin
will quickly induce remission, and serve as a bridge until the effect of the thiopurine
starts to appear over the next few weeks.

Cytomegalovirus is a common cause of non-responsive colitis (Option B) is incorrect.


Cytomegalovirus is in fact an uncommon cause of non-responsive colitis, but is important to
consider because anti-viral treatment is available.
https://mypastest.pastest.com/Secure/TestMe/Browser/429893#Top

1/2

8/22/2016

MyPastest

Methotrexate is indicated to induce remission (Option C) is incorrect.


Methotrexate has a limited role in the treatment of chronic Crohns disease, but no generally
recognised role in the treatment of ulcerative colitis, either in acute or chronic disease.
Non-steroidal anti-inflammatory drugs (NSAIDs) are a useful adjunct to therapy (Option D) is
incorrect.
Non-steroidal anti-inflammatory drugs (NSAIDs) cause flares of inflammatory bowel disease,
rather than helping to induce remission.
Surgery is contraindicated (Option E) is incorrect.
Although these patients may be very unwell and potentially are risky surgical candidates, it is
also possible that colectomy will be life-saving, and surgery is therefore not contraindicated.
45001

Next Question

Previous Question

Tag Question

Feedback

End Review

Difficulty: Average
Peer Responses

Session Progress
Responses Correct:

Responses Incorrect:

298

Responses Total:

298

Responses - % Correct:

0%

Blog (https://www.pastest.com/blog) About Pastest (https://www.pastest.com/about-us)


Contact Us (https://www.pastest.com/contact-us) Help (https://www.pastest.com/help)
Pastest 2016

https://mypastest.pastest.com/Secure/TestMe/Browser/429893#Top

2/2

8/22/2016

MyPastest

Back to Filters (/Secure/TestMe/Filter/429893/QA)

Question 233 of 298

A 45-year-old man with a 20-year history of ulcerative colitis (but who had been lost to
follow-up) was reviewed. He was found to have had a change in bowel habit for 4 months,
with increasing diarrhoea.
What is the most appropriate next management step?
A

Plain abdominal X-ray

Oral mesalazine

Oral prednisolone

Stool microscopy and culture

Urgent colonoscopy

Explanation

The answer is Urgent colonoscopy


There is a stepwise increased risk of colonic adenocarcinoma that starts approximately
710 years after the onset of ulcerative colitis; as such, colonoscopy is the right answer
here. The risk of cancer is proportional to the severity of the disease and its extent.
Surveillance colonoscopy is offered to patients, starting at 10 years, with biopsies taken
to assess for disease activity and for dysplasia. The 5-aminosalicylic acid (5-ASA) drugs
(eg mesalazine) probably reduce this risk. A new change in bowel habit should always be
investigated in a patient with long-standing disease.

Plain abdominal X-ray (Option A) is incorrect.


Although a plain abdominal X-ray is useful in acutely ill patients with ulcerative colitis to
assess for complications (such as toxic megacolon), it is unlikely to be of any benefit here.
Oral mesalazine (Option B) is incorrect.
It is possible that his diarrhoea represents an increase in activity of his ulcerative colitis, and
mesalazine would be an appropriate therapy were this the case; however, he clearly requires
a colonoscopy to assess if his diarrhoea is caused by a malignancy.
https://mypastest.pastest.com/Secure/TestMe/Browser/429893#Top

1/2

8/22/2016

MyPastest

Oral prednisolone (Option C) is incorrect.


It is possible that his diarrhoea represents an increase in activity of his ulcerative colitis, and
prednisolone would be appropriate therapy were this the case; however, he clearly requires a
colonoscopy to assess whether his diarrhoea is caused by a malignancy.
Stool microscopy and culture (Option D) is incorrect.
Although his symptoms could represent GI infection, they have been present longer than
would typically be expected were this the case, and no other symptoms of infection (eg
fever) are described; as such, although it is worth considering stool cultures, a colonoscopy
would be the key investigation to arrange.
45002

Next Question

Previous Question

Tag Question

Feedback

End Review

Difficulty: Average
Peer Responses

Session Progress
Responses Correct:

Responses Incorrect:

298

Responses Total:

298

Responses - % Correct:

0%

Blog (https://www.pastest.com/blog) About Pastest (https://www.pastest.com/about-us)


Contact Us (https://www.pastest.com/contact-us) Help (https://www.pastest.com/help)
Pastest 2016

https://mypastest.pastest.com/Secure/TestMe/Browser/429893#Top

2/2

8/22/2016

MyPastest

Back to Filters (/Secure/TestMe/Filter/429893/QA)

Question 234 of 298

A 35-year-old man with ulcerative colitis was found to have abnormal liver biochemistry.
Which of the following is most likely to be the cause?
A

Budd-Chiari syndrome

Chronic autoimmune hepatitis

Chronic hepatitis C

Primary biliary cholangitis

Primary sclerosing cholangitis

Explanation

The answer is Primary sclerosing cholangitis (PSC)


PSC is strongly associated with inflammatory bowel disease (70% have ulcerative colitis).
Men are more commonly affected than women (7:1), and the abnormality can pre-date
the onset of colitis. It is characterised by abnormal liver biochemistry, with multiple
beaded biliary strictures seen on magnetic resonance cholangiopancreatography (MRCP)
or endoscopic retrograde cholangiopancreatography (ERCP). These strictures lead to
episodic cholangitis and obstruction. There is an increased risk of cholangiocarcinoma.
Positive anti-smooth muscle and antinuclear antibodies and myeloperoxidase
antineutrophil cytoplasmic antibody (ANCA) are also features. There is no specific
medical treatment at present; liver transplantation may be indicated in cases
complicated by chronic liver disease and/or hepatobiliary malignancies.

BuddChiari syndrome (Option A) is incorrect.


Lack of any risk factors for or symptoms of BuddChiari makes it an unlikely diagnosis.
Additionally there is no specific association with ulcerative colitis.
Chronic autoimmune hepatitis (Option B) is incorrect.
Lack of any risk factors for or symptoms of chronic autoimmune hepatitis makes it an
unlikely diagnosis. Additionally there is no specific association with ulcerative colitis.
https://mypastest.pastest.com/Secure/TestMe/Browser/429893#Top

1/2

8/22/2016

MyPastest

Chronic hepatitis C (Option C) is incorrect.


Lack of any risk factors for or symptoms of chronic hepatitis C makes it an unlikely diagnosis.
Additionally there is no specific association with ulcerative colitis.
Primary biliary cholangitis (Option D) is incorrect.
Lack of any risk factors for or symptoms of primary biliary cholangitis makes it an unlikely
diagnosis. Additionally there is no specific association with ulcerative colitis.
45003

Next Question

Previous Question

Tag Question

Feedback

End Review

Difficulty: Average
Peer Responses

Session Progress
Responses Correct:

Responses Incorrect:

298

Responses Total:

298

Responses - % Correct:

0%

Blog (https://www.pastest.com/blog) About Pastest (https://www.pastest.com/about-us)


Contact Us (https://www.pastest.com/contact-us) Help (https://www.pastest.com/help)
Pastest 2016

https://mypastest.pastest.com/Secure/TestMe/Browser/429893#Top

2/2

8/22/2016

MyPastest

Back to Filters (/Secure/TestMe/Filter/429893/QA)

Question 235 of 298

A 56-year-old man referred with abnormal blood biochemistry was found to be positive for
hepatitis C virus (HCV) antibody.
Which of the following treatment options is most likely to be beneficial?
A

Aciclovir and foscarnet

Emtricitabine and tenofovir

Entecavir

Oseltamivir

Simepravir and sofobuvir

Explanation

The answer is Simepravir and sofosbuvir


The next test would be required for this man would be a hepatitis C RNA level; this will
help distinguish if he is one of the small number of people who spontaneously clear
hepatitis C (who will have undetectable hepatitis C RNA levels), as opposed to the larger
number who develop chronic infection. Before consideration of treatment, he will need
further tests to establish which genotype of infection he has, and what degree of liver
disease he has. Staging of liver disease was conventionally done with a liver biopsy, but
non-invasive modalities are becoming increasingly available.

Hepatitis C treatment has undergone an explosion in the past few years, with a huge
range of very effective new direct-acting anti-virals arriving on the market. These are
replacing/supplementing pegylated interferon and ribavirin, which were until recently the
reference standard of therapy. Simepravir and sofosbuvir are examples of direct-acting
anti-virals with activity against hepatitis C that may be used together as therapy for the
condition.

Aciclovir and foscarnet (Option A) is incorrect.


https://mypastest.pastest.com/Secure/TestMe/Browser/429893#Top

1/2

8/22/2016

MyPastest

Aciclovir and foscarnet are used to treat herpesviruses, including CMV.


Emtricitabine and tenofovir (Option B) is incorrect.
Emtricitabine and tenofovir are used together as a component of HIV therapy, with tenofovir
also used as monotherapy, as a treatment of chronic hepatitis B.
Entecavir (Option C) is incorrect.
Similarly, entecavir may also be used as monotherapy for chronic hepatitis B.
Oseltamivir (Option D) is incorrect. Oseltamivir is used to treat influenza.
45004

Next Question

Previous Question

Tag Question

Feedback

End Review

Difficulty: Difficult
Peer Responses

Session Progress
Responses Correct:

Responses Incorrect:

298

Responses Total:

298

Responses - % Correct:

0%

Blog (https://www.pastest.com/blog) About Pastest (https://www.pastest.com/about-us)


Contact Us (https://www.pastest.com/contact-us) Help (https://www.pastest.com/help)
Pastest 2016

https://mypastest.pastest.com/Secure/TestMe/Browser/429893#Top

2/2

8/22/2016

MyPastest

Back to Filters (/Secure/TestMe/Filter/429893/QA)

Question 236 of 298

A 21-year-old woman was admitted following a paracetamol overdose above the level known
to cause toxicity according to the nomogram.
Which of the following statements is correct?
A

A normal bilirubin at 48 hours indicates successful treatment

Chronic alcohol use reduces the risk of hepatitis

Glutathione conjugation is the first phase of liver metabolism

N-acetylcysteine is contraindicated 48 hours after the overdose

The prothrombin time is a good indicator of prognosis

Explanation

The answer is The prothrombin time is a good indicator of prognosis


Any paracetamol level above the treatment line should be treated with N-acetylcysteine,
irrespective of the time since the overdose, and should certainly be continued if liver
failure develops. The international normalised ratio (INR) at 48 hours is the best early
indicator, and a doubling of the INR in a 24-hour period should lead to referral to a
specialist liver unit, making Option E correct here. Other useful biochemical markers to
measure include pH, bilirubin and creatinine; paracetamol levels or liver enzymes do not
give any useful indicator about the possible degree of liver failure/prognosis. It is the
pattern taken in these tests in combination, rather than one particular blood test result at
a particular point that is important; as such, a normal bilirubin at 48 hours certainly in
itself does not indicate successful treatment.

The risk of severity of a paracetamol overdose increases with:


Older age or very young age
Presence of cytochrome P450 inducers, ie anticonvulsants, rifampicin, chronic
alcohol misuse
Malnutrition
https://mypastest.pastest.com/Secure/TestMe/Browser/429893#Top

1/3

8/22/2016

MyPastest

HIV positive (may relate to zidovudine use rather than HIV per se)
Very high doses of paracetamol consumed (>48 g).

A normal bilirubin at 48 hours indicates successful treatment (Option A) is incorrect.


As described, normal bilirubin at 48 hours in itself does not indicate successful treatment.
Chronic alcohol use reduces the risk of hepatitis (Option B) is incorrect.
Alcohol increases the risk of severity of a paracetamol overdose.
Glutathione conjugation is the first phase of liver metabolism (Option C) is incorrect.
The hepatic metabolism of paracetamol is to a quinoneimine (the toxic metabolite), and only
then subsequent glutathione conjugation (making this option incorrect). Once the
glutathione stores are depleted (ie when it reaches a toxic dose), the quinoneimine binds to
sulfhydryl groups on the hepatocyte membrane, causing necrosis.
N-acetylcysteine is contraindicated 48 hours after the overdose (Option D) is incorrect.
N-acetylcysteine frees glutathione groups, increasing their availability.
45005

Next Question

Previous Question

Tag Question

Feedback

End Review

Difficulty: Average
Peer Responses

Session Progress
Responses Correct:

Responses Incorrect:

298

Responses Total:

298

Responses - % Correct:

https://mypastest.pastest.com/Secure/TestMe/Browser/429893#Top

0%

2/3

8/22/2016

MyPastest

Blog (https://www.pastest.com/blog) About Pastest (https://www.pastest.com/about-us)


Contact Us (https://www.pastest.com/contact-us) Help (https://www.pastest.com/help)
Pastest 2016

https://mypastest.pastest.com/Secure/TestMe/Browser/429893#Top

3/3

8/22/2016

MyPastest

Back to Filters (/Secure/TestMe/Filter/429893/QA)

Question 237 of 298

A 26-year-old man presented having recently returned from Bangladesh. He reported


jaundice and itching. His viral hepatitis serology showed active hepatitis E infection.
Which of the following correctly describes the pathogenesis of hepatitis E?
A

Cholestasis is not a feature

Chronic hepatitis commonly occurs

Co-infection with hepatitis B is required for pathogenicity

The risk of fulminant hepatitis in pregnancy is 1%

Transmission is by the faeco-oral route

Explanation

The answer is Transmission is by the faeco-oral route


Hepatitis E follows a similar course to hepatitis A. It is faeco-orally transmitted and
causes an acute hepatitis that is often accompanied or followed by cholestasis. The main
risk is in pregnancy, since as many as 1020% of affected women (particularly those
infected in the third trimester) are reported to develop fulminant hepatitis although this
may be an overestimate.

Cholestasis is not a feature (Option A) is incorrect.


Hepatitis E is often accompanied or followed by cholestasis.
Chronic hepatitis commonly occurs (Option B) is incorrect.
Chronic infection is rare, although it has been observed in immunosuppressed patients.
Management is generally supportive, although there may be a role for ribavirin in treating
chronically infected patients.
Co-infection with hepatitis B is required for pathogenicity (Option C) is incorrect.
It is hepatitis D virus that requires co-infection with hepatitis B virus to cause inflammation,
rather than hepatitis E.
https://mypastest.pastest.com/Secure/TestMe/Browser/429893#Top

1/2

8/22/2016

MyPastest

The risk of fulminant hepatitis in pregnancy is 1% (Option D) is incorrect.


The risk in pregnancy of fulminant hepatitis as described is higher than 1%.
45006

Next Question

Previous Question

Tag Question

Feedback

End Review

Difficulty: Average
Peer Responses

Session Progress
Responses Correct:

Responses Incorrect:

298

Responses Total:

298

Responses - % Correct:

0%

Blog (https://www.pastest.com/blog) About Pastest (https://www.pastest.com/about-us)


Contact Us (https://www.pastest.com/contact-us) Help (https://www.pastest.com/help)
Pastest 2016

https://mypastest.pastest.com/Secure/TestMe/Browser/429893#Top

2/2

8/22/2016

MyPastest

Back to Filters (/Secure/TestMe/Filter/429893/QA)

Question 238 of 298

A 76-year-old man with a long-standing history of alcohol excess was admitted because of
increasing confusion and drowsiness. He has been admitted previously with both upper GI
bleeds and spontaneous bacterial peritonitis.
Which feature would be most suggestive of encephalopathy as the cause?
A

Bilateral 5-Hz hand tremor

Dysdiadochokinesis

Elevated serum ammonia

Normal computed tomography scan of the brain

Normal electroencephalography (EEG)

Explanation

The answer is Elevated serum ammonia


Serum ammonia can be a useful marker of hepatic encephalopathy, although high levels
do not necessarily confirm encephalopathy, just as normal levels do not completely rule
it out.

Bilateral 5-Hz hand tremor (Option A) is incorrect.


The liver flap of hepatic disease typically occurs once every few seconds, not as a bilateral 5Hz hand tremor (which is more consistent with withdrawal tremor).
Dydiadochokinesis (Option B) is incorrect.
Dysdiadochokinesis is a cerebellar sign that is seen in Wernicke and Korsakoff syndromes,
rather than being suggestive of hepatic encephalopathy.
Normal computed tomography (CT) scan of the brain (Option D) is incorrect.
CT of the head is often unremarkable in patients with hepatic encephalopathy, but cannot in
itself be used teither to confirm or refute the diagnosis.
Normal electroencephalography (EEG) (Option E) is incorrect.
https://mypastest.pastest.com/Secure/TestMe/Browser/429893#Top

1/2

8/22/2016

MyPastest

EEG in hepatic encephalopathy typically shows slow-wave activity that reverses.


45152

Next Question

Previous Question

Tag Question

Feedback

End Review

Difficulty: Average
Peer Responses

Session Progress
Responses Correct:

Responses Incorrect:

298

Responses Total:

298

Responses - % Correct:

0%

Blog (https://www.pastest.com/blog) About Pastest (https://www.pastest.com/about-us)


Contact Us (https://www.pastest.com/contact-us) Help (https://www.pastest.com/help)
Pastest 2016

https://mypastest.pastest.com/Secure/TestMe/Browser/429893#Top

2/2

8/22/2016

MyPastest

Back to Filters (/Secure/TestMe/Filter/429893/QA)

Question 239 of 298

A 56-year-old woman with established alcohol-related cirrhosis was admitted in a confused


and drowsy state.
What is the most important immediate investigation?
A

Blood culture

Blood glucose

Computed tomography scan of the brain

Red cell folate

Upper gastrointestinal endoscopy

Explanation

The answer is Blood glucose


This woman is particularly at risk of hypoglycaemia; hepatic gluconeogenesis can be
significantly down-regulated both by cirrhosis and by heavy alcohol consumption. Blood
glucose is a simple, straightforward test here, and may be life-saving.

Blood culture (Option A) is incorrect.


Sepsis can certainly cause people with cirrhosis to become confused and drowsy by various
mechanisms (including exacerbation of hepatic encephalopathy), but blood culture results
will take some time to come back, and should not be performed before checking blood
glucose.
Computed tomography (CT) scan of the brain (Option C) is incorrect.
A CT of the brain is appropriate to consider (eg to look for subdural haematoma, a pathology
that is common in this patient group), but again would not supersede checking blood
glucose.
Red cell folate (Option D) is incorrect.

https://mypastest.pastest.com/Secure/TestMe/Browser/429893#Top

1/2

8/22/2016

MyPastest

Folate deficiency may manifest as confusion and drowsiness, but again would not be the first
test performed.
Upper GI endoscopy (Option E) is incorrect.
Gastroscopy would be appropriate to consider if there were features of gastrointestinal
haemorrhage, but none is described here.
45153

Next Question

Previous Question

Tag Question

Feedback

End Review

Difficulty: Average
Peer Responses

Session Progress
Responses Correct:

Responses Incorrect:

298

Responses Total:

298

Responses - % Correct:

0%

Blog (https://www.pastest.com/blog) About Pastest (https://www.pastest.com/about-us)


Contact Us (https://www.pastest.com/contact-us) Help (https://www.pastest.com/help)
Pastest 2016

https://mypastest.pastest.com/Secure/TestMe/Browser/429893#Top

2/2

8/22/2016

MyPastest

Back to Filters (/Secure/TestMe/Filter/429893/QA)

Question 240 of 298

A 34-year-old man, originally from Pakistan, was admitted with ascites and weight loss. The
protein level on ascitic tap was 9 g/l.
What is the most likely cause of the ascites?
A

Intra-abdominal malignancy

Hepatic cirrhosis

Liver metastases

Peritoneal lymphoma

Tuberculous peritonitis

Explanation

The answer is Hepatic cirrhosis


Of the options given, only cirrhosis is associated with transudative ascites. All of the
other options given typically cause exudates. Although the exact cut-off of ascitic fluid
protein level used to define an exudate or transudate is influenced by the patients
plasma albumin level, this mans ascitic fluid protein level is so low that a transudate
seems extremely likely.

Causes of a transudate (protein < 30 g/l, assuming a normal albumin level):


Hepatic cirrhosis
Right-sided cardiac failure
Hypoalbuminaemia (nephrotic syndrome)
Acute nephritis
BuddChiari syndrome
Causes of an exudate (protein > 30 g/l):

https://mypastest.pastest.com/Secure/TestMe/Browser/429893#Top

1/3

8/22/2016

MyPastest

Infection (tuberculosis, spontaneous bacterial peritonitis, etc)


Inflammation (vasculitis, pancreatitis, etc)
Malignancy

Intra-abdominal malignancy (Option A) is incorrect.


As described this would typically cause exudates.
Liver metastases (Option C) is incorrect.
As described this would typically cause exudates.
Peritoneal lymphoma (Option D) is incorrect.
As described this would typically cause exudates.
Tuberculous peritonitis (Option E) is incorrect.
As described this would typically cause exudates.
45154

Next Question

Previous Question

Tag Question

Feedback

End Review

Difficulty: Average
Peer Responses

Session Progress
Responses Correct:

Responses Incorrect:

298

Responses Total:

298

Responses - % Correct:

0%

Blog (https://www.pastest.com/blog) About Pastest (https://www.pastest.com/about-us)


Contact Us (https://www.pastest.com/contact-us) Help (https://www.pastest.com/help)
https://mypastest.pastest.com/Secure/TestMe/Browser/429893#Top

2/3

8/22/2016

MyPastest

Pastest 2016

https://mypastest.pastest.com/Secure/TestMe/Browser/429893#Top

3/3

8/22/2016

MyPastest

Back to Filters (/Secure/TestMe/Filter/429893/QA)

Question 241 of 298

A 45-year-old woman was referred after an occupational health screen. Investigations


showed the following hepatitis B serology: surface Ag positive, surface Ab negative, IgG core
antibody positive, IgM core antibody negative, e antigen positive, e antibody negative.
What term best describes this womans hepatitis B status?
A

Persistent carrier, high infectivity

Persistent carrier, low infectivity

Pre-core mutant carrier

Recent primary infection

Spontaneously cleared infection

Explanation

The answer is option A, peristent carrier, high infectivity.

Being hepatitis B surface antigen positive for at least 6 months defines chronic carrier
status; given this womans age and that this hepatitis B serology was discovered
incidentally, it seems very likely that she is a chronic carrier. The loss of hepatitis B
surface antigen and its replacement with the surface antibody marks seroconversion, and
is the marker used to detect immunity. Core antibody will be positive in anyone who has
been exposed to hepatitis B virus in the past (but not in those who have been vaccinated
against the virus). e antigen is a useful marker of the degree of infectivity; e antigen
positive confers higher infectivity, whereas e antibody positive implies lower infectivity.
However, the e antigen will be negative in people in people with hepatitis B with precore mutations, giving a falsely negative result.
The presence of both hepatitis B surface antigen and e antigen in this woman implies a
persistent carrier with high infectivity.

Persistent carrier, low infectivity (Option B) is incorrect.


https://mypastest.pastest.com/Secure/TestMe/Browser/429893#Top

1/2

8/22/2016

MyPastest

This would only be the case were e antigen positive.


Pre-core mutant carrier (Option C) is incorrect.
The presence of a positive e antigen means that this option is not correct.
Recent primary infection (Option D) is incorrect.
This womans negative IgM core antibody means that recent infection is unlikely, with her
positive core IgG strongly suggesting that she has chronic infection.
Spontaneously cleared infection (Option E) is incorrect.
The presence of a positive surface antigen but negative surface antibody means that she has
not cleared the infection, making this option incorrect.
45155

Next Question

Previous Question

Tag Question

Feedback

End Review

Difficulty: Average
Peer Responses

Session Progress
Responses Correct:

Responses Incorrect:

298

Responses Total:

298

Responses - % Correct:

0%

Blog (https://www.pastest.com/blog) About Pastest (https://www.pastest.com/about-us)


Contact Us (https://www.pastest.com/contact-us) Help (https://www.pastest.com/help)
Pastest 2016

https://mypastest.pastest.com/Secure/TestMe/Browser/429893#Top

2/2

8/22/2016

MyPastest

Back to Filters (/Secure/TestMe/Filter/429893/QA)

Question 242 of 298

A 78-year-old woman was referred with abnormal liver biochemistry. Investigations showed:
bilirubin 54 mol/l, alanine aminotransferase (ALT) 43 U/l, gamma-glutamyltransferase (GGT)
299 U/l, alkaline phosphatase 323 U/l. She was diagnosed with drug-induced liver injury.
Which of the following drugs is most likely to be responsible?
A

Digoxin

Erythromycin

Furosemide

Nifedipine

Paracetamol

Explanation

The answer is Erythromycin


A huge range of different drugs may affect the liver and cause a drug-induced liver injury
(DILI). Some drugs are particularly well recognised as potentially causing this, and tend
to be associated with a characteristic pattern of liver enzyme changes.

The following drugs are recognised as potentially causing acute hepatitis: isoniazid,
rifampicin, methyldopa, atenolol, enalapril, verapamil, nifedipine, amiodarone,
ketoconazole, cytotoxics and halothane.

The drugs which may cause acute cholestasis include: oestrogens, ciclosporin,
azathioprine, chlorpromazine, haloperidol, cimetidine, ranitidine, erythromycin,
nitrofurantoin, imipramine, ibuprofen and hypoglycaemics.

Digoxin (Option A) is incorrect.


Digoxin may very rarely cause a DILI associated with hepatitis, but is not a recognised cause
of cholestatic liver injury.
https://mypastest.pastest.com/Secure/TestMe/Browser/429893#Top

1/2

8/22/2016

MyPastest

Furosemide (Option C) is incorrect.


Furosemide may very rarely cause a DILI associated with hepatitis, but is not a recognised
cause of cholestatic liver injury.
Nifedipine (Option D) is incorrect.
Nifedipine may very rarely cause a DILI associated with hepatitis, but is not a recognised
cause of cholestatic liver injury.
Paracetamol (Option E) is incorrect.
Although paracetamol may cause acute hepatitis if taken in excess (and is one of the leading
causes of acute liver failure worldwide), it is not associated with a cholestatic liver injury.
45156

Next Question

Previous Question

Tag Question

Feedback

End Review

Difficulty: Average
Peer Responses

Session Progress
Responses Correct:

Responses Incorrect:

298

Responses Total:

298

Responses - % Correct:

0%

Blog (https://www.pastest.com/blog) About Pastest (https://www.pastest.com/about-us)


Contact Us (https://www.pastest.com/contact-us) Help (https://www.pastest.com/help)
Pastest 2016

https://mypastest.pastest.com/Secure/TestMe/Browser/429893#Top

2/2

8/22/2016

MyPastest

Back to Filters (/Secure/TestMe/Filter/429893/QA)

Question 243 of 298

A 26-year-old man was referred by his GP with episodic jaundice, but was otherwise well. He
had no risk factors for viral hepatitis and no family history of liver disease. He drank 58 units
of alcohol per week.
Which feature would favour Gilbert syndrome as the cause?
A

Abdominal pain

Clay-coloured stools

Concomitant diabetes mellitus

Resolution of symptoms on fasting

Unconjugated hyperbilirubinaemia

Explanation

The answer is Unconjugated hyperbilirubinaemia

Gilbert syndrome is a mild loss of glucuronosyltransferase activity, with an incidence of


57%. It shows an autosomal recessive inheritance pattern. The failure of conjugation
results in an unconjugated hyperbilirubinaemia. However, the bilirubin concentration is
always less than 102 mol/l. In itself, it has no significant sequelae. Episodes of jaundice
are induced by fasting or by infection and are self-limiting.

CriglerNajjar syndrome is the second form of glucuronosyltransferase deficiency:


Type I (autosomal recessive) results from the loss of both copies of the gene and
causes death in early childhood if a transplant is not carried out.
Type II (autosomal dominant) is the inherited loss of one gene, which causes a
much milder illness with an unconjugated hyperbilirubinaemia.

https://mypastest.pastest.com/Secure/TestMe/Browser/429893#Top

1/2

8/22/2016

MyPastest

Abdominal pain (Option A) is incorrect.


Abdominal pain is not associated with Gilbert syndrome.
Clay-coloured stools (Option B) is incorrect.
None of options Ato C is associated with Gilbert syndrome; abdominal pain and claycoloured stools in combination with jaundice are suggestive of biliary obstruction, such as by
gallstones.
Concomitant diabetes mellitus (Option C) is incorrect.
Concomitant diabetes mellitus is not associated with Gilbert syndrome.
Resolution of symptoms on fasting (Option D) is incorrect.
Fasting will precipitate jaundice rather than resolve it.
45157

Next Question

Previous Question

Tag Question

Feedback

End Review

Difficulty: Average
Peer Responses

Session Progress
Responses Correct:

Responses Incorrect:

298

Responses Total:

298

Responses - % Correct:

0%

Blog (https://www.pastest.com/blog) About Pastest (https://www.pastest.com/about-us)


Contact Us (https://www.pastest.com/contact-us) Help (https://www.pastest.com/help)
Pastest 2016

https://mypastest.pastest.com/Secure/TestMe/Browser/429893#Top

2/2

8/22/2016

MyPastest

Back to Filters (/Secure/TestMe/Filter/429893/QA)

Question 244 of 298

A 23-year-old woman was referred with abnormal liver biochemistry in the third trimester of
pregnancy.
Which feature would be most consistent with pregnancy-related cholestasis as the
underlying diagnosis?
A

Elevated serum bile acids

Elevated urate

Hypoalbuminaemia

Macrocytosis

Thrombocytopenia

Explanation

A number of pregnancy-related liver disorders are well recognised. These include:


Pregnancy-related cholestasis results in elevated serum bile acids, normal urate
and fat-soluble vitamin malabsorption (prolonging the prothrombin time).
Acute fatty liver of pregnancy is much more serious and is associated with preeclampsia (hypertension, proteinuria) and a high urate level. Immediate delivery is
mandatory if other features of toxaemia exist.
Hyperemesis gravidarum can result in abnormal liver enzymes.
HELLP (Haemolysis, Elevated Liver enzymes and Low Platelets) is a potentially lifethreatening condition (a microangiopathic process associated with hypertension in
some cases, with risk of haemorrhage and stroke), and again merits consideration
for immediate delivery.
Elevated serum bile acids are a typical feature of pregnancy-related cholestasis. The
other options given may be associated with other pregnancy-related liver diseases (eg
thrombocytopenia and HELLP) but not cholestasis, making elevated serum bile acids the
correct answer here.
https://mypastest.pastest.com/Secure/TestMe/Browser/429893#Top

1/2

8/22/2016

MyPastest

Elevated urate (Option B) is incorrect.


As described, elevated urate is not associated with cholestasis.
Hypoalbuminaemia (Option C) is incorrect.
As described, hypoalbuminaemia is not associated with cholestasis.
Macrocytosis (Option D) is incorrect.
As described, macrocytosis is not associated with cholestasis.
Thrombocytopenia (Option E) is incorrect.
As described, thrombocytopenia is not associated with cholestasis.
45160

Next Question

Previous Question

Tag Question

Feedback

End Review

Difficulty: Average
Peer Responses

Session Progress
Responses Correct:

Responses Incorrect:

298

Responses Total:

298

Responses - % Correct:

0%

Blog (https://www.pastest.com/blog) About Pastest (https://www.pastest.com/about-us)


Contact Us (https://www.pastest.com/contact-us) Help (https://www.pastest.com/help)
Pastest 2016

https://mypastest.pastest.com/Secure/TestMe/Browser/429893#Top

2/2

8/22/2016

MyPastest

Back to Filters (/Secure/TestMe/Filter/429893/QA)

Question 245 of 298

A 34-year-old man returns from India with abdominal pain, fever, nausea and sweats.
Examination reveals an enlarged tender liver. On ultrasound, several abscesses are visualised.
An aspirate is taken from one of the abscesses.
Which bacteria are most likely to be grown from the aspirate?
A

Clostridium perfringens

Klebsiella histolytica

Pseudomonas aeruginosa

Staphylococcus aureus

Streptococcus pneumoniae

Explanation

The answer is Staphylococcus aureus


The most common causes of a pyogenic abscess are enterococci, Staphylococcus aureus
and Escherichia coli, making S. aureus the correct answer here. Klebsiella histolytica is
not a true bacterial organism; the other options given may all cause pyogenic abscess,
but only very rarely.
Patients with pyogenic abscesses typically present with pain, fever, shock and weight
loss. Management is by ultrasound-guided drainage and antibiotics targeted to the
cultured organism. Assessment for other gastrointestinal pathology sometimes reveals
an underlying cause for loss of the normal bowel wall barrier to infections.
Amoebic abscess tends to present later, unless the abscesses burst. These are caused by
infection with Entamoeba histolytica. Again, they are managed with aspiration and
appropriate antibiotics.
Hydatid liver abscesses have a typical ultrasound appearance and should not be
aspirated because of the risk of seeding and peritoneal spread.

Clostridium perfringens (Option A) is incorrect.


https://mypastest.pastest.com/Secure/TestMe/Browser/429893#Top

1/2

8/22/2016

MyPastest

As outlined, C. perfringens only rarely causes pyogenic abscess.


Klebsiella histolytica (Option B) is incorrect.
As described, K. histolytica is not a true bacterial organism.
Pseudomonas aeruginosa (Option D) is incorrect.
As outlined, P. aeruginosa only rarely causes pyogenic abscess.
Streptococcus pneumoniae (Option E) is incorrect.
As outlined, S. pneumonia only rarely causes pyogenic abscess.
45162

Next Question

Previous Question

Tag Question

Feedback

End Review

Difficulty: Average
Peer Responses

Session Progress
Responses Correct:

Responses Incorrect:

298

Responses Total:

298

Responses - % Correct:

0%

Blog (https://www.pastest.com/blog) About Pastest (https://www.pastest.com/about-us)


Contact Us (https://www.pastest.com/contact-us) Help (https://www.pastest.com/help)
Pastest 2016

https://mypastest.pastest.com/Secure/TestMe/Browser/429893#Top

2/2

8/22/2016

MyPastest

Back to Filters (/Secure/TestMe/Filter/429893/QA)

Question 246 of 298

A 36-year-old man with ulcerative colitis was admitted with abdominal pain and an amylase
level of 1433 U/l.
Which medication is most likely to be a cause of his symptoms?
A

Aspirin

Azathioprine

Budesonide

Paracetamol

Sulfasalazine

Explanation

The answer is Azathioprine


In the maintenance treatments used for ulcerative colitis, azathioprine is effective but
associated with a number of potentially serious side effects. These include:
Pancreatitis acute pancreatitis is well recognised as a potential complication of
azathioprine use. The presence of acute abdominal pain and a raised amylase
clearly makes this the likely diagnosis here.
Hypersensitivity reactions a variety of symptoms are possible, including malaise,
dizziness, vomiting, diarrhoea, fever, rigors, myalgia, arthralgia and rash.
Interstitial nephritis rare, but calls for immediate withdrawal.
Bone marrow suppression this effect is dose related. Leucopenia can result in
increased susceptibility to infection, particularly in people also receiving
corticosteroids.
Liver disease a liver injury that may present with cholestatic liver enzymes.

Causes of acute pancreatitis include: GET SMASHED:


https://mypastest.pastest.com/Secure/TestMe/Browser/429893#Top

1/3

8/22/2016

MyPastest

Gallstone disease
Ethanol, ie alcohol excess
Trauma/ surgery
Steroids
Mumps, and other infections, including Coxsackievirus, Salmonella, Campylobacter,
Mycoplasma
Autoimmune, eg SLE, polyarteritis nodosa
Scorpion venom
Hypothermia, hypercalcaemia, hyperlipidaemia, hereditary
ERCP
Drugs including oestrogens
(plus other causes pregnancy, idiopathic, pancreatitc tumours, etc).

Aspirin (Option A) is incorrect. Aspirin is not a recognised cause of pancreatitis.


Budesonide (Option B) is incorrect. Although budesonide is a steroid, it is an unlikely cause
of pancreatitis as it is used in dose forms in inflammatory bowel disease that allow local
delivery of steroid and reduce systemic exposure.
Paracetamol (Option D) is incorrect. Paracetamol is not recognised as causing pancreatitis.
Sulfasalazine (Option E) is incorrect. Mesalazine may rarely cause pancreatitis, but this is
much less likely with sulfasalazine.
45164

Next Question

Previous Question

Tag Question

Feedback

End Review

Difficulty: Average
Peer Responses

Session Progress
Responses Correct:
Responses Incorrect:

https://mypastest.pastest.com/Secure/TestMe/Browser/429893#Top

0
298

2/3

8/22/2016

Responses Total:
Responses - % Correct:

MyPastest

298
0%

Blog (https://www.pastest.com/blog) About Pastest (https://www.pastest.com/about-us)


Contact Us (https://www.pastest.com/contact-us) Help (https://www.pastest.com/help)
Pastest 2016

https://mypastest.pastest.com/Secure/TestMe/Browser/429893#Top

3/3

8/22/2016

MyPastest

Back to Filters (/Secure/TestMe/Filter/429893/QA)

Question 247 of 298

A 56-year-old woman was referred with right upper quadrant pain and nausea. Ultrasound
revealed multiple gallstones.
Which of these conditions is associated with an increased risk of gallstone disease?
A

Coeliac disease

Crohns disease

Diverticulosis

High HDL cholesterol

Ulcerative colitis

Explanation

The answer is Crohns disease


Gallstone disease appears to be increasing in frequency, although this may just be owing
to the increasing numbers of abdominal ultrasound examinations being carried out. Risk
factors for the condition include:
Female sex
Multiparity
Oral contraceptive use
Terminal ileal Crohns disease presumed as a result of the production of
lithogenic bile
High dietary animal fat intake
Rapid weight loss
Diabetes mellitus.

Coeliac disease (Option A) is incorrect.


Coeliac disease is not associated with an increased risk of gallstone disease.
https://mypastest.pastest.com/Secure/TestMe/Browser/429893#Top

1/2

8/22/2016

MyPastest

Diverticulosis (Option C) is incorrect.


Diverticulosis is not associated with an increased risk of gallstone disease.
High HDL cholesterol (Option D) is incorrect.
High HDL cholesterol is not associated with an increased risk of gallstone disease.
Ulcerative colitis (Option E) is incorrect.
Ulcerative colitis is not associated with an increased risk of gallstone disease.
45166

Next Question

Previous Question

Tag Question

Feedback

End Review

Difficulty: Average
Peer Responses

Session Progress
Responses Correct:

Responses Incorrect:

298

Responses Total:

298

Responses - % Correct:

0%

Blog (https://www.pastest.com/blog) About Pastest (https://www.pastest.com/about-us)


Contact Us (https://www.pastest.com/contact-us) Help (https://www.pastest.com/help)
Pastest 2016

https://mypastest.pastest.com/Secure/TestMe/Browser/429893#Top

2/2

8/22/2016

MyPastest

Back to Filters (/Secure/TestMe/Filter/429893/QA)

Question 248 of 298

A 56-year-old man underwent an upper gastrointestinal endoscopy for iron deficiency


anaemia. Biopsies of a mass in the stomach are reported as showing a mucosa-associated
lymphoid tissue (MALT) lymphoma.
Which of the following is true of MALT lymphomas?
A

40% regress with Helicobacter pylori eradication

Metastatic spread to the liver is common

Paraproteins may be a feature

The stomach is the only site of origin

They are a T-lymphocyte clone

Explanation

The answer is Paraproteins may be a feature


Gastric mucosa-associated lymphoid tissue (MALT) lymphomas are understood to arise
principally as a result of Helicobacter pylori infection in the stomach. Paraprotein
formation and pseudohyponatraemia are recognised complications, making option C the
correct answer here.

40% regress with Helicobacter pylori eradication (Option A) is incorrect.


In non-metastatic tumours, H. pylori eradication results in complete remission in ~80% of
cases (rather than 40%, hence option A is incorrect). Where metastasis has occurred,
standard B-cell lymphoma regimens are usually effective.
Metastatic spread to the liver is common (Option B) is incorrect.
Metastasis of MALTomas is rare (and so option B can be rejected), but if it does occur then it
may make treatment much more difficult.
The stomach is the only site of origin (Option D) is incorrect.
They may arise from any mucosal surface (rather than just the stomach).
https://mypastest.pastest.com/Secure/TestMe/Browser/429893#Top

1/2

8/22/2016

MyPastest

They are a T-lymphocyte clone (Option E) is incorrect.


These lymphomas originate from B cells rather than T cells.
45167

Next Question

Previous Question

Tag Question

Feedback

End Review

Difficulty: Average
Peer Responses

Session Progress
Responses Correct:

Responses Incorrect:

298

Responses Total:

298

Responses - % Correct:

0%

Blog (https://www.pastest.com/blog) About Pastest (https://www.pastest.com/about-us)


Contact Us (https://www.pastest.com/contact-us) Help (https://www.pastest.com/help)
Pastest 2016

https://mypastest.pastest.com/Secure/TestMe/Browser/429893#Top

2/2

8/22/2016

MyPastest

Back to Filters (/Secure/TestMe/Filter/429893/QA)

Question 249 of 298

A 76-year-old man is admitted with jaundice and weight loss. He has no history of abdominal
pain or fevers.
What is the most likely diagnosis?
A

Choledocho-duodenal fistula

Chronic pancreatitis

Gallstone obstruction

Mirizzi syndrome

Pancreatic adenocarcinoma

Explanation

The answer is Pancreatic adenocarcinoma


The combination of painless jaundice and weight loss in a person of this age is very
suggestive of malignant obstruction of the biliary tree. Adenocarcinoma of the head of
the pancreas would be the most likely cause of this, although other malignancies
(including cholangiocarcinoma and ampullary tumours) would also be possibilities.

Choledocho-duodenal fistula (Option A) is incorrect.


Choledocho-duodenal fistulae typically cause abdominal pain and peritonism and often an
ileus so this is also unlikely to be the diagnosis.
Chronic pancreatitis (Option B) is incorrect.
Chronic pancreatitis typically presents with a long history of typical epigastric pancreatitic
pain and weight loss; the lack of any prior abdominal pain here (or any risk factors for
chronic pancreatitis, such as alcohol excess) means that this is unlikely to be the diagnosis
here.
Gallstone obstruction (Option C) is incorrect.

https://mypastest.pastest.com/Secure/TestMe/Browser/429893#Top

1/2

8/22/2016

MyPastest

Gallstone impaction within the common bile duct may result in jaundice, but weight loss
would not be explained by this. Similarly, it would be unusual for a man of this age to have
gallstone disease without other symptoms suggestive of biliary colic or cholecystitis in the
past.
Mirizzi syndrome (option D) is incorrect.
Mirizzi syndrome is impaction of a gallstone in the cystic duct, so is unlikely to result in a
presentation similar to that described here.
45168

Next Question

Previous Question

Tag Question

Feedback

End Review

Difficulty: Average
Peer Responses

Session Progress
Responses Correct:

Responses Incorrect:

298

Responses Total:

298

Responses - % Correct:

0%

Blog (https://www.pastest.com/blog) About Pastest (https://www.pastest.com/about-us)


Contact Us (https://www.pastest.com/contact-us) Help (https://www.pastest.com/help)
Pastest 2016

https://mypastest.pastest.com/Secure/TestMe/Browser/429893#Top

2/2

8/22/2016

MyPastest

Back to Filters (/Secure/TestMe/Filter/429893/QA)

Question 250 of 298

A 36-year-old man with diabetes is referred with abnormal liver biochemistry.


Which blood test result is most suggestive of haemochromatosis?
A

Alkaline phosphatase 178 U/l (45105)

Ferritin 324 g/l (15300)

Serum iron 25 mol/l (1230)

Total iron-binding capacity (TIBC) 43 mol/l (4575)

Transferrin saturation 78% (2050)

Explanation

The answer is Transferrin saturation 78% (2050)


Haemochromatosis is caused by a mutation in the gene HFE, which results in iron
accumulation in different organs. The disease has an autosomal recessive inheritance
pattern, with a gene frequency of 1:400 in the population. Iron accumulation within the
liver leads to hepatitis and cirrhosis; in the pancreas, to diabetes; in the heart, to
cardiomyopathy; in the pituitary, to hypogonadism; and in the skin, to discoloration.
Women are relatively protected by menstrual blood loss.

The following biochemical findings are suggestive of iron accumulation, and consistent
with a diagnosis of haemochromatosis:
Ferritin > 500 g/l
Serum iron > 30 mol/l
Transferrin saturation > 60%
Total iron-binding capacity < 20 mol/l.
As such, Transferrin saturation 78% is the correct answer here, and the other options can
be ruled out.
https://mypastest.pastest.com/Secure/TestMe/Browser/429893#Top

1/3

8/22/2016

MyPastest

A liver biopsy for iron accumulation and genotype testing are necessary for absolute
confirmation of the diagnosis.

Alkaline phosphatase 178 U/l (45105) (Option A) is incorrect.


Liver enzymes may be variable in people with the condition, but rises in transaminases are
more typical than increases in alkaline phosphatase, ruling this option out.
Ferritin 324 g/l (15300) (Option B) is incorrect.
This result would not be suggestive of haemocrhomatosis.
Serum iron 25 mol/l (1230) (Option C) is incorrect.
This result would not be suggestive of haemochromatosis.
Total iron-binding capacity (TIBC) 43 mol/l (4575) (Option D) is incorrect.
This result would not be suggestive of haemochromatosis.
45169

Next Question

Previous Question

Tag Question

Feedback

End Review

Difficulty: Average
Peer Responses

Session Progress
Responses Correct:

Responses Incorrect:

298

Responses Total:

298

Responses - % Correct:

0%

Blog (https://www.pastest.com/blog) About Pastest (https://www.pastest.com/about-us)


Contact Us (https://www.pastest.com/contact-us) Help (https://www.pastest.com/help)
Pastest 2016
https://mypastest.pastest.com/Secure/TestMe/Browser/429893#Top

2/3

8/22/2016

https://mypastest.pastest.com/Secure/TestMe/Browser/429893#Top

MyPastest

3/3

8/22/2016

MyPastest

Back to Filters (/Secure/TestMe/Filter/429893/QA)

Question 251 of 298

A 43-year-old man, who was seeking asylum from Somalia, was admitted to hospital after
arrival in the UK. His body mass index was 15 kg/m2.
Which of the following is a recognised feature in protein-energy malnutrition?
A

An elevated serum globulin

An elevated serum IgE concentration

An exaggerated response to intradermal tuberculin

An increased concentration of reversed T3

Steatohepatitis

Explanation

The answer is Steatohepatitis


Malnutrition is classically divided into marasmus (with a normal serum albumin level) and
kwashiorkor (much rarer, with hypoalbuminaemia and oedema). In clinical practice, a low
albumin level is linked to any underlying metabolic process.
Steatohepatitis and hypercholesterolaemia are common in severe malnutrition (making
option E the correct answer here), and result from the anabolic processes that occur in
the liver in this state as an attempt to maintain albumin levels.

An elevated serum globulin (Option A) is incorrect.


Immune responses and function are generally blunted in malnutrition.
An elevated serum IgE concentration (Option B) is incorrect.
Immune responses and function are generally blunted in malnutrition.
An exaggerated response to intradermal tuberculin (Option C) is incorrect.
Tuberculosis (TB) is endemic in Somalia, and considering this as a contributory factor to his
low weight is reasonable. However, no symptoms are described (eg cough and haemoptysis)
that specifically suggest TB; furthermore, once again, since immune responses are blunted in
https://mypastest.pastest.com/Secure/TestMe/Browser/429893#Top

1/2

8/22/2016

MyPastest

malnutrition, it is possible that this test would be falsely negative even if he had TB.
An increased concentration of reversed T3 (Option D) is incorrect.
Thyroid function tests are generally suppressed.
45171

Next Question

Previous Question

Tag Question

Feedback

End Review

Difficulty: Average
Peer Responses

Session Progress
Responses Correct:

Responses Incorrect:

298

Responses Total:

298

Responses - % Correct:

0%

Blog (https://www.pastest.com/blog) About Pastest (https://www.pastest.com/about-us)


Contact Us (https://www.pastest.com/contact-us) Help (https://www.pastest.com/help)
Pastest 2016

https://mypastest.pastest.com/Secure/TestMe/Browser/429893#Top

2/2

8/22/2016

MyPastest

Back to Filters (/Secure/TestMe/Filter/429893/QA)

Question 252 of 298

A 65-year-old woman with scleroderma and Raynauds phenomenon complained of weight


loss and was referred for further investigation.
Which of the following conditions is most strongly associated with progressive systemic
sclerosis?
A

Abnormal exocrine pancreatic function

Diverticulitis

Oesophageal stricture

Primary sclerosing cholangitis

Small-bowel lymphoma

Explanation

The answer is Oesophageal stricture. Systemic sclerosis results in vascular damage,


fibrosis, alterations in gastrointestinal tract motility and an activated immune system.
Gastrointestinal tract involvement includes:
Oesophageal strictures
Malabsorption including small-bowel bacterial overgrowth
Intestinal pseudo-obstruction
Anal incontinence.
As such, oesophageal stricture is the correct choice here.

Abnormal exocrine pancreatic function (Option A) is incorrect.


Although systemic sclerosis is associated with certain causes of malabsorption (including
small-bowel bacterial overgrowth), it is not strongly associated with pancreatic exocrine
failure.
Diverticulitis (Option B) is incorrect.
https://mypastest.pastest.com/Secure/TestMe/Browser/429893#Top

1/2

8/22/2016

MyPastest

Diverticulitis has no consistent association with systemic sclerosis.


Primary sclerosing cholangitis (Option D) is incorrect.
The CREST variant of systemic sclerosis (ie Calcinosis cutis, Raynauds phenomenon,
oEsophageal hypomobility, Sclerodactyly and Telangiectasia) is linked with primary biliary
cholangitis but not with sclerosing cholangitis.
Small-bowel lymphoma (Option E) is incorrect.
Small-bowel lymphoma has no consistent association with systemic sclerosis.
45174

Next Question

Previous Question

Tag Question

Feedback

End Review

Difficulty: Average
Peer Responses

Session Progress
Responses Correct:

Responses Incorrect:

298

Responses Total:

298

Responses - % Correct:

0%

Blog (https://www.pastest.com/blog) About Pastest (https://www.pastest.com/about-us)


Contact Us (https://www.pastest.com/contact-us) Help (https://www.pastest.com/help)
Pastest 2016

https://mypastest.pastest.com/Secure/TestMe/Browser/429893#Top

2/2

8/22/2016

MyPastest

Back to Filters (/Secure/TestMe/Filter/429893/QA)

Question 253 of 298

A 34-year-old man with profuse watery diarrhoea was referred for assessment of a possible
neuroendocrine tumour of the gastrointestinal tract.
Which statement correctly describes the action of gastrointestinal hormones?
A

Enteroglucagon increases the small-bowel transit rate

Gastrin increases gastric motor activity

Pancreatic polypeptide stimulates pancreatic bicarbonate secretion

Secretin maintains mucosal growth

Somatostatin increases gastrin secretion

Explanation

The answer is Gastrin increases gastric motor activity


Gastrin (secreted by gastric antral G cells) causes acid secretion, mucosal growth and
smooth muscle contraction.

Enteroglucagon increases the small-bowel transit rate (Option A) is incorrect.


Enteroglucagon is released in response to food, promoting mucosal growth and slowing
transit of the gastrointestinal tract (rather than increasing it, making this option incorrect),
and therefore enhances food absorption.
Pancreatic polypeptide stimulates pancreatic bicarbonate secretion (Option C) is incorrect.
Pancreatic polypeptide inhibits pancreatic enzyme secretion (rather than increases it, making
this option incorrect).
Secretin maintains mucosal growth (Option D) is incorrect.
Secretin stimulates pancreatic secretions (as well as reduces acid secretion from gastric
parietal cells) in response to the presence of duodenal acid. It has no direct effect on
mucosal growth.
Somatostatin increases gastrin secretion (Option E) is incorrect.
https://mypastest.pastest.com/Secure/TestMe/Browser/429893#Top

1/2

8/22/2016

MyPastest

Somatostatin reduces gastrointestinal motility, and also reduces secretion of gastrin (rather
than increases it, making this option incorrect). It also down-regulates portal blood flow and
all gastrointestinal tract secretions.
45175

Next Question

Previous Question

Tag Question

Feedback

End Review

Difficulty: Average
Peer Responses

Session Progress
Responses Correct:

Responses Incorrect:

298

Responses Total:

298

Responses - % Correct:

0%

Blog (https://www.pastest.com/blog) About Pastest (https://www.pastest.com/about-us)


Contact Us (https://www.pastest.com/contact-us) Help (https://www.pastest.com/help)
Pastest 2016

https://mypastest.pastest.com/Secure/TestMe/Browser/429893#Top

2/2

8/22/2016

MyPastest

Back to Filters (/Secure/TestMe/Filter/429893/QA)

Question 254 of 298

A 62-year-old man was admitted after an episode of haematemesis thought to be related to


varices.
Which of the following statements about acute upper gastrointestinal haemorrhage is
correct?
A

Intravenous nitrates reduce the risk of re-bleeding

Intravenous ranitidine reduces the risk of re-bleeding

Intravenous somatostatin analogues reduce the risk of re-bleeding

Oral omeprazole reduces the risk of re-bleeding

Oral tranexamic acid reduces the risk of re-bleeding

Explanation

The answer is Intravenous somatostatin analogues reduce the risk of re-bleeding


The modern treatment for upper gastrointestinal (GI) bleeding is:
Active fluid resuscitation via adequate venous access.
Endoscopic identification of the lesion within 24 hours, and therapeutic
intervention if appropriate. This includes direct intervention (with heat, clips, drugs
(adrenaline) or haemostatic powder), or specific therapy for oesophageal varices
(typically variceal banding).
Radiological intervention (ie embolisation of bleeding vessel) and/or surgical
intervention have an important role in cases of upper gastrointestinal bleeding that
cannot be controlled by endoscopy.
Acid suppression prior to endoscopy has not consistently been shown to have a role in
reducing the risk of re-bleeding. However, intravenous proton pump inhibitor does
appear to reduce re-bleeding after endoscopy if a patient is at high risk of a re-bleed. As
such, the intravenous nitrates and intravenous ranitidine options are incorrect.
https://mypastest.pastest.com/Secure/TestMe/Browser/429893#Top

1/3

8/22/2016

MyPastest

Somatostatin, a naturally occurring peptide, displays a wide range of biological actions,


mainly inhibitory ones, leading to its use in the treatment of a variety of digestive
diseases. Octreotide is the most extensively investigated of these. The marked effect of
somatostatin on splanchnic haemodynamics together with its inhibitory action on acidpeptic and pancreatic exocrine secretions represents the rationale for its use in upper
gastrointestinal bleeding. As well as its haemodynamic effects, the peptide also increases
lower oesophageal sphincter pressure, thereby reducing the inflow of blood into the
submucous venous plexus of the oesophagus and hence into oesophageal varices.
Through its inhibitory action on acid-peptic secretion, somatostatin may also inhibit
peptic digestion of any blood clots at the site of haemostasis on a varix itself. In addition,
the natural peptide has been shown to enhance human platelet aggregation in vitro,
whose stimulation can activate the haemostatic process. The drug can be given by
intravenous infusion.

Intravenous nitrates reduce the risk of re-bleeding (Option A) is incorrect.


Intravenous nitrates have no role in the management of acute GI bleeding.
Intravenous ranitidine reduces the risk of re-bleeding (Option B) is incorrect.
Acid suppression prior to endoscopy has not consistently been shown to have a role in
reducing the risk of re-bleeding.
Oral omeprazole reduces the risk of re-bleeding (Option D) is incorrect.
Acid suppression prior to endoscopy has not consistently been shown to have a role in
reducing the risk of re-bleeding.
Oral tranexamic acid reduces the risk of re-bleeding (Option E) is incorrect.
Further studies are ongoing about the possible role for intravenous tranexamic acid in
reducing re-bleed risk in GI bleeds, but there is no consistent evidence for oral tranexamic
acid being of any benefit in this situation.
45177

Next Question

Previous Question

Tag Question

Feedback

End Review

Difficulty: Average
Peer Responses

Session Progress
https://mypastest.pastest.com/Secure/TestMe/Browser/429893#Top

2/3

8/22/2016

MyPastest

Responses Correct:

Responses Incorrect:

298

Responses Total:

298

Responses - % Correct:

0%

Blog (https://www.pastest.com/blog) About Pastest (https://www.pastest.com/about-us)


Contact Us (https://www.pastest.com/contact-us) Help (https://www.pastest.com/help)
Pastest 2016

https://mypastest.pastest.com/Secure/TestMe/Browser/429893#Top

3/3

8/22/2016

MyPastest

Back to Filters (/Secure/TestMe/Filter/429893/QA)

Question 255 of 298

A 61-year-old man was referred for further investigation of malabsorption and villous atrophy
found on duodenal biopsy. His coeliac serology was negative and his symptoms failed to
improve on a gluten-free diet. He was investigated for possible Whipples disease.
Which of the following statements about Whipples disease is correct?
A

Cerebral involvement responds to antibiotic treatment

It typically affects middle-aged women

Periodic acidSchiff-staining granules are seen in the macrophages

Recurrence after treatment is rare

The causative organism is a Gram-negative coccus

Explanation

The answer is Periodic acidSchiff (PAS)-staining granules are seen in the macrophages
Whipples disease is an infective disease that may involve multiple organ systems, but
which particularly affects the gastrointestinal tract. It is caused by Tropheryma whipplei;
this bacterium has an equivocal Gram stain result on staining and has a bacillus-like
morphology.
The clinical features that may occur are diverse, and include depression,
ophthalmoplegia, arthropathy, encephalitis, arthritis, and cardiac, cerebral and lung
involvement. Most lesions (except cerebral lesions) are reversible on prolonged antibiotic
therapy, but lifelong follow-up is mandatory as relapses do occur.
The typical gastrointestinal features of the condition include bloating, abdominal
discomfort, weight loss and diarrhoea. These are caused by small-bowel malabsorption
related to infection of the small bowel by the causative bacterium. Diagnosis may be
made by endoscopy and biopsy from the duodenum or jejunum; villous atrophy is a
typical appearance, along with macrophages full of PAS-positive (periodic acidSchiff)
material.

Cerebral involvement responds to antibiotic treatment (Option A) is incorrect.


https://mypastest.pastest.com/Secure/TestMe/Browser/429893#Top

1/2

8/22/2016

MyPastest

Most lesions (except cerebral lesions) are reversible on prolonged antibiotic therapy.
It typically affects middle-aged women (Option B) is incorrect.
It most typically affects middle-aged white men.
Recurrence after treatment is rare (Option D) is incorrect.
Lifelong follow-up is mandatory as relapses do occur as outlined.
The causative organism is a Gram-negative coccus (Option E) is incorrect.
As described T. whipplei has a bacillus-like morphology.
45178

Next Question

Previous Question

Tag Question

Feedback

End Review

Difficulty: Average
Peer Responses

Session Progress
Responses Correct:

Responses Incorrect:

298

Responses Total:

298

Responses - % Correct:

0%

Blog (https://www.pastest.com/blog) About Pastest (https://www.pastest.com/about-us)


Contact Us (https://www.pastest.com/contact-us) Help (https://www.pastest.com/help)
Pastest 2016

https://mypastest.pastest.com/Secure/TestMe/Browser/429893#Top

2/2

8/22/2016

MyPastest

Back to Filters (/Secure/TestMe/Filter/429893/QA)

Question 256 of 298

A 45-year-old patient was referred with itching and abnormal liver biochemistry.
Which of the following is a feature of primary biliary cholangitis?
A

Anti-mitochondrial antibodies

Histological piecemeal necrosis

Increased serum copper level

Increased serum IgA

Middle-aged male patients

Explanation

The answer is Anti-mitochondrial antibodies


Primary biliary cholangitis (PBC; previously known as primary biliary cirrhosis) is a
condition associated with positive anti-mitochondrial (M2) antibodies and elevated
serum IgM levels.

PBC is also associated with an increased level of copper in the liver; however, this
increased copper occurs as a complication of cholestasis and is not necessarily related to
any difference in plasma levels, meaning that the increased serum copper level option is
incorrect.

Given how sensitive and specific anti-mitochondrial antibodies are for confirming the
diagnosis, a liver biopsy is not a necessity to confirm the diagnosis; however, a biopsy
may still be helpful in assessing for hepatic fibrosis/ severity of liver disease. Typical
biopsy findings include inflammatory duct destruction and patchy fibrosis, with cirrhosis
occurring in those most greatly affected.

https://mypastest.pastest.com/Secure/TestMe/Browser/429893#Top

1/3

8/22/2016

MyPastest

PBC usually affects middle-aged women, therefore the middle-aged male option is not
correct. The progression of the condition is variable, with a small number of affected
people developing sufficiently severe liver disease to merit liver transplantation.
Treatment with ursodeoxycholic acid improves liver biochemistry, but probably does not
improve the prognosis. Colestyramine may be used as a symptomatic treatment for
pruritus, one of the most common symptoms in those affected with the condition.

Histological piecemeal necrosis (Option B) is incorrect.


Piecemeal necrosis is well described in the histology of chronic active hepatitis, but is not
part of the histology of PBC.
Increased serum copper level (Option C) is incorrect.
As described, PBC is associated with increased levels of copper in the liver.
Increased serum IgA (Option D) is incorrect.
Serum IgA is not typically raised in the condition, ruling out option D.
Middle-aged male patients (Option E) is incorrect.
PBC usually affects middle-aged women, as outlined.
45179

Next Question

Previous Question

Tag Question

Feedback

End Review

Difficulty: Average
Peer Responses

Session Progress
Responses Correct:

Responses Incorrect:

298

Responses Total:

298

Responses - % Correct:

https://mypastest.pastest.com/Secure/TestMe/Browser/429893#Top

0%

2/3

8/22/2016

MyPastest

Blog (https://www.pastest.com/blog) About Pastest (https://www.pastest.com/about-us)


Contact Us (https://www.pastest.com/contact-us) Help (https://www.pastest.com/help)
Pastest 2016

https://mypastest.pastest.com/Secure/TestMe/Browser/429893#Top

3/3

8/22/2016

MyPastest

Back to Filters (/Secure/TestMe/Filter/429893/QA)

Question 257 of 298

A 43-year-old man was referred after he was found to have abnormal liver biochemistry.
Investigations showed he had an alanine aminotransferase (ALT) of 98 U/l and he was
hepatitis B surface antigen positive.
Which of the following statements about chronic active hepatitis owing to the hepatitis B
virus is correct?
A

It carries an increased risk of subsequent hepatocellular carcinoma

It causes marked elevation of serum transaminases

It is associated with positive hepatitis D serology

It is more common in women than in men

It responds well to corticosteroids

Explanation

The answer is It carries an increased risk of subsequent hepatocellular carcinoma


Chronic hepatitis B is well recognised as being a carcinogen, with hepatocellular
carcinoma (HCC) being the result. Although many cases of HCC in those with chronic
hepatitis B occur in those who have developed cirrhosis, there are particular groups of
people with chronic hepatitis B even without cirrhosis who merit surveillance for HCC
because they are at higher risk (ie based on ethnicity and/or family history).

It causes marked elevation of serum transaminases (Option B) is incorrect.


Liver enzymes may be almost completely normal in the condition, ruling out this option.
It is associated with positive hepatitis D serology (Option C) is incorrect.
Hepatitis D infection can only occur in people who already have hepatitis B infection, but
hepatitis B is not directly associated with positive hepatitis D serology, ruling this option out.
It is more common in women than in men (Option D) is incorrect.
Chronic hepatitis B is just as common in men as in women.
https://mypastest.pastest.com/Secure/TestMe/Browser/429893#Top

1/2

8/22/2016

MyPastest

It responds well to corticosteroids (Option E) is incorrect.


Several different treatments are available to treat chronic hepatitis B, but corticosteroids are
certainly not an effective treatment.
45181

Next Question

Previous Question

Tag Question

Feedback

End Review

Difficulty: Average
Peer Responses

Session Progress
Responses Correct:

Responses Incorrect:

298

Responses Total:

298

Responses - % Correct:

0%

Blog (https://www.pastest.com/blog) About Pastest (https://www.pastest.com/about-us)


Contact Us (https://www.pastest.com/contact-us) Help (https://www.pastest.com/help)
Pastest 2016

https://mypastest.pastest.com/Secure/TestMe/Browser/429893#Top

2/2

8/22/2016

MyPastest

Back to Filters (/Secure/TestMe/Filter/429893/QA)

Question 258 of 298

A 25-year-old man is admitted with cerebellar ataxia and abnormal liver biochemistry. He is
suspected to have Wilsons disease.
Which finding is most typical of Wilsons disease?
A

A poor prognosis with treatment

A raised serum copper level

Bilateral upper motor neurone signs

Onset of symptoms usually between 10 and 25 years

The absence of a KayserFleischer ring

Explanation

The answer is Onset of symptoms usually between 10 and 25 years


Wilsons disease has a genetic origin, occurring because of a mutation in the ATP7B gene
(inherited in an autosomal recessive manner). The result is defective cellular transport of
copper, and consequent copper deposition in organs throughout the body, including the
liver and nervous system. The onset of symptoms is typically between 10 and 25 years,
although clinical manifestations may first occur at any point between early childhood
and late adulthood.

A poor prognosis with treatment (Option A) is incorrect.


Removal of copper from the body by a chelating agent such as penicillamine has greatly
improved the prognosis of the condition, so patients now lead a normal life and expect a
normal life expectancy. Alternatives to penicillamine include trientene and zinc acetate.
A raised serum copper level (Option B) is incorrect.
The intracellular accumulation of copper in Wilsons disease is associated with low serum
concentrations of copper and of the copper-containing protein, caeruloplasmin; urinary
excretion of the metal is high in most patients with the condition.
Bilateral upper motor neurone signs (Option C) is incorrect.
https://mypastest.pastest.com/Secure/TestMe/Browser/429893#Top

1/2

8/22/2016

MyPastest

Neurological signs in Wilsons disease are most often owing to involvement of the cerebellar
and basal ganglia. As such, movement disorders (eg tremor/Parkinsonism, ataxia) are
common. Neurobehavioural manifestations (such as cognitive impairment) are also well
recognised. However, pyramidal tract involvement is not typical, and bilateral upper motor
neurone signs are not suggestive of Wilsons.
The absence of a KayserFleischer ring (Option E) is incorrect.
KayserFleischer rings are brown-/green-coloured depositions of copper within Descemets
membrane in the cornea, and may be identified via slit lamp examination. These are found in
> 90% of people with Wilsons disease who have neurological involvement; given that this
man has cerebellar ataxia, this makes the absence of a KayserFleischer ring incorrect here.
The liver manifestations of Wilsons disease are very widely variable, ranging from
asymptomatic abnormalities in liver enzmyes, to more severe manifestations including acute
liver failure or advanced cirrhosis.
45431

Next Question

Previous Question

Tag Question

Feedback

End Review

Difficulty: Average
Peer Responses

Session Progress
Responses Correct:

Responses Incorrect:

298

Responses Total:

298

Responses - % Correct:

0%

Blog (https://www.pastest.com/blog) About Pastest (https://www.pastest.com/about-us)


Contact Us (https://www.pastest.com/contact-us) Help (https://www.pastest.com/help)
Pastest 2016

https://mypastest.pastest.com/Secure/TestMe/Browser/429893#Top

2/2

8/22/2016

MyPastest

Back to Filters (/Secure/TestMe/Filter/429893/QA)

Question 259 of 298

A 54-year-old man with long-standing ulcerative colitis is seen in the Gastroenterology Clinic.
His general practitioner has noticed that he has abnormal liver enzymes, and also that he has
lost weight.
What is the most likely diagnosis?
A

Biliary tract carcinoma

Non-specific urethritis

Primary biliary cholangitis

Seropositive arthritis

Small-bowel lymphoma

Explanation

The answer is Biliary tract carcinoma


Ulcerative colitis (UC) is associated with a number of different conditions, including
erythema nodosum, pyoderma gangrenosum, uveitis, scleritis, episcleritis and primary
sclerosing cholangitis (PSC). PSC is a chronic disease of unclear aetiology that is
characterised by inflammation, scarring and stricturing of the biliary tree. The diagnosis
is usually first suspected from the finding of cholestatic liver enzymes in people with
inflammatory bowel disease, with the identification of biliary stricturing on
cholangiography (such as by MRCP or ERCP) helping to confirm the diagnosis. It is
usually the medium and large biliary ducts of both the intra- and extrahepatic biliary tree
that are affected in PSC, although a small number of people with the condition have
involvement of the intrahepatic biliary tree only (small duct PSC); such patients require
a liver biopsy to confirm the diagnosis. PSC has a number of well-recognised
complications, particularly an increased risk of colonic cancer (meaning that an annual
colonoscopy is required as surveillance). PSC with intrahepatic duct involvement is also a
risk factor for chronic liver disease and its complications, including hepatocellular
carcinoma. One other well-recognised complication of PSC is cholangiocarcinoma
(biliary tract malignancy), and regular imaging of the biliary tree is indicated as
surveillance. The description in this question of a man with UC who has abnormalities in
liver enzymes and weight loss makes biliary tract carcinoma the correct answer here.
https://mypastest.pastest.com/Secure/TestMe/Browser/429893#Top

1/3

8/22/2016

MyPastest

Non-specific urethritis (Option B) is incorrect.


Non-specific urethritis would not explain this mans symptoms, and is also not a condition
related to inflammatory bowel disease.
Primary biliary cholangitis (PBC) (Option C) is incorrect.
PBC is unlike PSC not associated with inflammatory bowel disease.
Seropositive arthritis (Option D) is incorrect.
A seronegative arthritis may occur in association with inflammatory bowel disease (so-called
enteropathic arthritis), but there is no strong association with seropositive disease.
Small-bowel lymphoma (Option E) is incorrect.
The risk of small bowel lymphoma is increased in people with inflammatory bowel disease,
although they are still rare malignancies. They are typically associated with abnormal liver
enzymes.
45432

Next Question

Previous Question

Tag Question

Feedback

End Review

Difficulty: Average
Peer Responses

Session Progress
Responses Correct:

Responses Incorrect:

298

Responses Total:

298

Responses - % Correct:

0%

Blog (https://www.pastest.com/blog) About Pastest (https://www.pastest.com/about-us)


Contact Us (https://www.pastest.com/contact-us) Help (https://www.pastest.com/help)
Pastest 2016
https://mypastest.pastest.com/Secure/TestMe/Browser/429893#Top

2/3

8/22/2016

https://mypastest.pastest.com/Secure/TestMe/Browser/429893#Top

MyPastest

3/3

8/22/2016

MyPastest

Back to Filters (/Secure/TestMe/Filter/429893/QA)

Question 260 of 298

A 23-year-old man was referred with a bilirubin concentration of 55 mol/l. The rest of his
liver biochemistry was normal. He has been diagnosed with Gilbert syndrome.
Which finding is most consistent with Gilbert syndrome?
A

Abnormal liver histology

Conjugated hyperbilirubinaemia

Decreased bilirubin on fasting

Gamma-glutamyltransferase in the normal range

Kernicterus

Explanation

The answer is Gamma-glutamyltransferase in the normal range


Gilbert syndrome is a benign, familial, mild unconjugated hyperbilirubinaemia. It occurs
in 5% of Caucasian adults in Western Europe, and usually first manifests during the
second or third decades of life. It results from a decreased hepatic clearance of
unconjugated bilirubin, probably due to reduced activity of UDP glucuronosyltransferase
(the hepatic enzymes responsible for conjugation of bilirubin).
Although hyperbilirubinaemia occurs in the condition, there is no abnormality of the liver
enzymes, meaning that gamma-glutamyltransferase in the normal range is the right
answer here.

Abnormal liver histology (Option A) is incorrect.


Liver histology is normal, and reassurance only is required. Life expectancy is normal.
Conjugated hyperbilirubinaemia (Option B) is incorrect.
The increase in plasma bilirubin in Gilbert syndrome is all unconjugated, making this option
incorrect. Clinically, there might be non-specific symptoms such as anorexia and malaise.
Decreased bilirubin on fasting (Option C) is incorrect.
https://mypastest.pastest.com/Secure/TestMe/Browser/429893#Top

1/2

8/22/2016

MyPastest

Jaundice is usually mild, but increases two- to threefold on fasting, fatigue, infection,
dehydration and administration of intravenous nicotinic acid. Jaundice decreases with the
administration of phenobarbital, which induces glucuronosyltransferase activity.
Kernicterus (Option E) is incorrect.
Conditions associated with severe unconjugated hyperbilirubinaemia may result in
kernicterus (ie neurological impairment secondary to bilirubin-induced neurotoxicity). Such
diseases include the paediatric condition of CriglerNajjar syndrome; the type I version of
this condition is associated with no UDP glucuronosyltransferease expression at all, and
affected babies development very marked unconjugated hyperbilirubinaemia and a high risk
of kernicterus. However, the unconjungated hyperbilirubinaemia of Gilbert syndrome is much
more modest than this, and kernicterus does not occur, making this option incorrect.
45433

Next Question

Previous Question

Tag Question

Feedback

End Review

Difficulty: Average
Peer Responses

Session Progress
Responses Correct:

Responses Incorrect:

298

Responses Total:

298

Responses - % Correct:

0%

Blog (https://www.pastest.com/blog) About Pastest (https://www.pastest.com/about-us)


Contact Us (https://www.pastest.com/contact-us) Help (https://www.pastest.com/help)
Pastest 2016

https://mypastest.pastest.com/Secure/TestMe/Browser/429893#Top

2/2

8/22/2016

MyPastest

Back to Filters (/Secure/TestMe/Filter/429893/QA)

Question 261 of 298

A 54-year-old man presented with dysphagia and a normal upper gastrointestinal endoscopy.
A barium swallow demonstrated achalasia.
Which of the following statements about achalasia is correct?
A

Decreased risk of oesophageal adenocarcinoma

Difficulty in swallowing liquids but not solids

Failure of relaxation of lower oesophageal sphincter

Increased incidence of celiac disease

Small-bowel dysmotolity

Explanation

The answer is Failure of relaxation of lower oesophageal sphincter


Achalasia is characterised by failure of relaxation of the lower oesophageal sphincter and
abnormal oesophageal body motility (simultaneous contractions).

Decreased risk of oesophageal adenocarcinoma (Option A) is incorrect.


Oesophageal squamous-cell carcinoma develops in 12% of patients with achalasia, and there
also appears to be a small increased risk of adenocarcinoma.
Difficulty in swallowing liquids but not solids (Option B) is incorrect.
Dysphagia is usually intermittent, of long duration, and for both liquids and solids.
Increased incidence of celiac disease (Option D) is incorrect.
Achalasia has no association with coeliac disease.
Small-bowel dysmotolity (Option E) is incorrect.
Achalasia is not associated with dysmotility in the gastrointestinal tract anywhere other than
the oesophagus/gastro-oesophageal junction, making this option incorrect.
45434

https://mypastest.pastest.com/Secure/TestMe/Browser/429893#Top

1/2

8/22/2016

MyPastest

Next Question

Previous Question

Tag Question

Feedback

End Review

Difficulty: Average
Peer Responses

Session Progress
Responses Correct:

Responses Incorrect:

298

Responses Total:

298

Responses - % Correct:

0%

Blog (https://www.pastest.com/blog) About Pastest (https://www.pastest.com/about-us)


Contact Us (https://www.pastest.com/contact-us) Help (https://www.pastest.com/help)
Pastest 2016

https://mypastest.pastest.com/Secure/TestMe/Browser/429893#Top

2/2

8/22/2016

MyPastest

Back to Filters (/Secure/TestMe/Filter/429893/QA)

Question 262 of 298

Which of the following conditions is not associated with acute painful scrotal swelling?
A

Fourniers gangrene

Idiopathic scrotal oedema

Leukaemia

Strangulated hernia

Torsion of appendix of testis

Explanation

The answer is Idiophatic scrotal oedema


Idiopathic scrotal oedema is an unexplained condition (probably allergic) that occurs
particularly in children, and leads to a swollen, erythematous scrotum; this can extend to
the groin and perianal regions. The affected person is usually well in themselves (without
significant pain). No treatment is needed as it usually resolves in 23 days.

Fourniers gangrene (Option A) is incorrect.


Fourniers gangrene a variant of necrotising fasciitis of the scrotum causes gross oedema,
subcutaneous emphysema and patchy necrosis, and is therefore typically painful.
Leukaemia (Option C) is incorrect.
Leukaemia can lead to acute painful enlargement of the testis and scrotum (particularly in
children), but this is rare.
Strangulated hernia (Option D) is incorrect.
Strangulated hernia refers to a painful irreducible hernia with a compromised blood supply.
Torsion of appendix of testis (Option E) is incorrect.
The appendix of the testis (hydatid of Morgagni) is a pedunculated remnant of the Mullerian
duct that sits on top of the epididymis. Torsion of the testicular appendix can lead to bluish,
tender swellings of the skin. The pain can simulate testicular torsion, but tends to be less
https://mypastest.pastest.com/Secure/TestMe/Browser/429893#Top

1/2

8/22/2016

MyPastest

severe.
45435

Next Question

Previous Question

Tag Question

Feedback

End Review

Difficulty: Average
Peer Responses

Session Progress
Responses Correct:

Responses Incorrect:

298

Responses Total:

298

Responses - % Correct:

0%

Blog (https://www.pastest.com/blog) About Pastest (https://www.pastest.com/about-us)


Contact Us (https://www.pastest.com/contact-us) Help (https://www.pastest.com/help)
Pastest 2016

https://mypastest.pastest.com/Secure/TestMe/Browser/429893#Top

2/2

8/22/2016

MyPastest

Back to Filters (/Secure/TestMe/Filter/429893/QA)

Question 263 of 298

A 65-year-old woman was referred with dysphagia.


Which of the following findings would be most useful in supporting a diagnosis of a
connective tissue disorder?
A

Cerebellar ataxia

Cogwheel tremor

Iron-deficiency anaemia

Left hemiparesis

Raynauds phenomenon

Explanation

The answer is Raynauds phenomenon


The occurrence of dysphagia together with Raynauds phenomenon is very suggestive of
an underlying connective tissue disorder, and particularly CREST syndrome (Calcinosis
cutis, Raynauds phenomenon, oEsophageal hypomotility, Sclerodactyly and
Telangiectasia) or progressive systemic sclerosis.
None of the other options given would strongly suggest underlying connective tissue
disorders.

Cerebellar ataxia (Option A) is incorrect.


Cerebellar disease may be associated with dysarthria, but not with dysphagia.
Cogwheel tremor (Option B) is incorrect.
Cogwheel tremor would be consistent with Parkinsons disease. Parkinsons is often
associated with impaired bulbar function causing speech and respiratory dysfunction
(dysphonia, stridor, sleep apnoea etc), but not typically dysphagia.
Iron-deficiency anaemia (Option C) is incorrect.

https://mypastest.pastest.com/Secure/TestMe/Browser/429893#Top

1/2

8/22/2016

MyPastest

Iron-deficiency anaemia together with dysphagia often merits gastroscopy to assess for
serious structural causes of dysphagia (eg oesophageal carcinoma). One rare explanation
could be PlummerVinson syndrome, a condition of unclear aetiology that is characterised
by the the combination of iron-deficiency anaemia and oesophageal web; this occurs
particularly in middle-aged women.
Left hemiparesis (Option D) is incorrect.
Hemiparesis may represent previous stroke and is often associated with impaired bulbar
function causing speech and respiratory dysfunction (dysphonia, stridor, sleep apnoea, etc),
but not typically dysphagia.
45436

Next Question

Previous Question

Tag Question

Feedback

End Review

Difficulty: Average
Peer Responses

Session Progress
Responses Correct:

Responses Incorrect:

298

Responses Total:

298

Responses - % Correct:

0%

Blog (https://www.pastest.com/blog) About Pastest (https://www.pastest.com/about-us)


Contact Us (https://www.pastest.com/contact-us) Help (https://www.pastest.com/help)
Pastest 2016

https://mypastest.pastest.com/Secure/TestMe/Browser/429893#Top

2/2

8/22/2016

MyPastest

Back to Filters (/Secure/TestMe/Filter/429893/QA)

Question 264 of 298

A 76-year-old man with primary biliary cholangitis was reviewed in the Liver Clinic.
Which of the following is a common feature of primary biliary cholangitis?
A

Back pain

Increased levels of serum IgA

Mesangiocapillary glomerulonephritis

Psoriatic arthritis

Pyoderma gangrenosum

Explanation

The answer is Back pain


Back pain may be prominent in patients with advanced primary biliary cholangitis (PBC;
known as primary biliary cirrhosis until recently). It is usually attributed to osteomalacia
resulting from malabsorption or osteoporosis, sometimes called hepatic osteodystrophy.
A number of autoimmune/connective tissue diseases are associated with primary biliary
cholangitis, include the CREST syndrome (Calcinosis cutis, Raynauds phenomenon,
oEsophageal hypomobility, Sclerodactyly, Telangiectasia), systemic lupus erythematosus
(SLE), hypothyroidism, rheumatoid arthritis and sicca syndrome.

Increased levels of serum IgA (Option B) is incorrect.


PBC is also associated with increased IgM levels (but not with IgA levels).
Mesangiocapillary glomerulonephritis (Option C) is incorrect.
Renal associations with PBC include renal tubular acidosis and membranous
glomerulonephritis; however, there is no association with mesangiocapillary
glomerulonephritis.
Psoriatic arthritis (Option D) is incorrect.

https://mypastest.pastest.com/Secure/TestMe/Browser/429893#Top

1/2

8/22/2016

MyPastest

PBC is associated with certain dermatological conditions (including dermatomyositis and


lichen planus); however, it is not associated with psoriatic arthritis.
Pyoderma gangrenosum (Option E) is incorrect.
PBC is associated with certain dermatological conditions (including dermatomyositis and
lichen planus); however, it is not associated with pyoderma gangrenosum.
45437

Next Question

Previous Question

Tag Question

Feedback

End Review

Difficulty: Average
Peer Responses

Session Progress
Responses Correct:

Responses Incorrect:

298

Responses Total:

298

Responses - % Correct:

0%

Blog (https://www.pastest.com/blog) About Pastest (https://www.pastest.com/about-us)


Contact Us (https://www.pastest.com/contact-us) Help (https://www.pastest.com/help)
Pastest 2016

https://mypastest.pastest.com/Secure/TestMe/Browser/429893#Top

2/2

8/22/2016

MyPastest

Back to Filters (/Secure/TestMe/Filter/429893/QA)

Question 265 of 298

A 25-year-old woman presented with weight loss, abdominal pain and diarrhoea. Her antiendomysial antibody was positive, and a duodenal biopsy confirmed subtotal villous atrophy
and intraepithelial lymphocytes.
Which of the following is a feature of coeliac disease?
A

Colonic ulceration

Constipation

Erythema nodosum

Hypersplenism

Splenic atrophy

Explanation

The answer is Splenic atrophy


Splenic atrophy is well recognised as a potential complication of coeliac disease, being
identified in approximately 50% of affected patients. The identification of HowellJolly
bodies on blood film microscopy may be a useful indicator of underlying hyposplenism.
Splenic atrophy responds poorly to gluten withdrawal.

Colonic ulceration (Option A) is incorrect.


Although ulcers in the colon are clearly associated with ulcerative colitis, there is no strong
association with coeliac disease.
Constipation (Option B) is incorrect.
Coeliac disease often clinically presents with diarrhoea (reflecting it being a malabsorptive
disease), but not constipation.
Erythema nodosum (Option C) is incorrect.
Erythema nodosum has a number of associations, which include inflammatory bowel disease;
however, it is not associated with coeliac disease.
https://mypastest.pastest.com/Secure/TestMe/Browser/429893#Top

1/2

8/22/2016

MyPastest

Hypersplenism (Option D) is incorrect.


Coeliac disease is associated with hypo- rather than hypersplenism.
45439

Next Question

Previous Question

Tag Question

Feedback

End Review

Difficulty: Average
Peer Responses

Session Progress
Responses Correct:

Responses Incorrect:

298

Responses Total:

298

Responses - % Correct:

0%

Blog (https://www.pastest.com/blog) About Pastest (https://www.pastest.com/about-us)


Contact Us (https://www.pastest.com/contact-us) Help (https://www.pastest.com/help)
Pastest 2016

https://mypastest.pastest.com/Secure/TestMe/Browser/429893#Top

2/2

8/22/2016

MyPastest

Back to Filters (/Secure/TestMe/Filter/429893/QA)

Question 266 of 298

A 17-year-old boy presents with poor growth, weight loss and diarrhoea. His duodenal biopsy
showed subtotal villous atrophy and intraepithelial lymphocytes.
Which of the following conditions is associated with coeliac disease?
A

Erythema marginatum

Molluscum contagiosum

Perianal fistulae

Pyoderma gangrenosum

Recurrent mouth ulcers

Explanation

The answer is Recurrent mouth ulcers


Recurrent mouth ulcers, diarrhoea and failure to thrive are typical clinical manifestations
of coeliac disease in younger people. Mouth ulcers and an intensely pruritic vesicular rash
over extensor surfaces are also found in dermatitis herpetiformis; the skin rash responds
to a gluten-free diet or to dapsone.

Erythema marginatum (Option A) is incorrect.


Erythema marginatum presents as a pink-red rash that may affect the torso and limbs, often
with pale centres and round edges; it occurs in the context of rheumatic fever, but has no
association with coeliac disease.
Molluscum contagiosum (Option B) is incorrect.
Molluscum contagiosum is a poxvirus infection of the skin that manifests as dome-shaped
papules; it also has no association with coeliac disease.
Perianal fistulae (Option C) is incorrect.
Perianal fistulae are a well-recognised potential complication of Crohns disease (but not
coeliac disease).
https://mypastest.pastest.com/Secure/TestMe/Browser/429893#Top

1/2

8/22/2016

MyPastest

Pyoderma gangrenosum (Option D) is incorrect.


Pyoderma gangrenosum has a number of associations (including leukaemia, vasculitides and
inflammatory bowel disease), but is not associated with coeliac disease.
45441

Next Question

Previous Question

Tag Question

Feedback

End Review

Difficulty: Average
Peer Responses

Session Progress
Responses Correct:

Responses Incorrect:

298

Responses Total:

298

Responses - % Correct:

0%

Blog (https://www.pastest.com/blog) About Pastest (https://www.pastest.com/about-us)


Contact Us (https://www.pastest.com/contact-us) Help (https://www.pastest.com/help)
Pastest 2016

https://mypastest.pastest.com/Secure/TestMe/Browser/429893#Top

2/2

8/22/2016

MyPastest

Back to Filters (/Secure/TestMe/Filter/429893/QA)

Question 267 of 298

A 76-year-old man presents with progressive dysphagia and weight loss. Endoscopy shows
an oesophageal malignancy of the lower third of the oesophagus.
Which of the following conditions has a recognised association with oesophageal
adenocarcinoma?
A

Barretts oesophagus

Crohns disease

Duodenal ulceration

Partial gastrectomy

Ulcerative colitis

Explanation

The answer is Barretts oesophagus


Aetiological factors for oesophageal carcinoma include:
Alcohol
Tobacco
Prolonged, severe gastro-oesophageal reflux
Caustic strictures
Barretts oesophagus
Dietary factors
Coeliac disease
Tylosis
Achalasia is associated particularly with squamous-cell carcinoma of the oesophagus,
although may also cause a very small increased risk of adenocarcinoma of the
oesophagus.

https://mypastest.pastest.com/Secure/TestMe/Browser/429893#Top

1/2

8/22/2016

MyPastest

Crohns disease (Option B) is incorrect.


Crohns disease does not have an association with oesophageal adenocarcinoma.
Duodenal ulceration (Option C) is incorrect.
Duodenal ulceration does not appear to confer any increased risk of oesophageal
adenocarcinoma.
Partial gastrectomy (Option D) is incorrect.
Partial gastrectomy is a risk factor for gastric rather than oesophageal carcinoma, making
this option incorrect.
Ulcerative colitis (Option E) is incorrect.
Ulcerative colitis does not have an association with oesophageal carcinoma.
45443

Next Question

Previous Question

Tag Question

Feedback

End Review

Difficulty: Average
Peer Responses

Session Progress
Responses Correct:

Responses Incorrect:

298

Responses Total:

298

Responses - % Correct:

0%

Blog (https://www.pastest.com/blog) About Pastest (https://www.pastest.com/about-us)


Contact Us (https://www.pastest.com/contact-us) Help (https://www.pastest.com/help)
Pastest 2016

https://mypastest.pastest.com/Secure/TestMe/Browser/429893#Top

2/2

8/22/2016

MyPastest

Back to Filters (/Secure/TestMe/Filter/429893/QA)

Question 268 of 298

A 43-year-old man presented with bloody diarrhoea and weight loss.


Which one of the following findings would favour a diagnosis of Crohns disease on rectal
biopsy?
A

Crypt abscesses

Crypt distortion

Inflammatory infiltrates

Patchy inflammation

Superficial ulceration

Explanation

The answer is Patchy inflammation


Crohns disease is characterised histologically by transmural (full-thickness)
inflammation, in contrast to ulcerative colitis. The inflammation is patchy with skip
lesions. Non-caseating granulomas are present in 2040% of cases. In ulcerative colitis,
inflammation is confined to the mucosa and submucosa only. The inflammation is
confluent and histological changes to the crypts are common, including crypt distortion,
neutrophilic crypt distraction and crypt abscesses.

Crypt abscesses (Option A) is incorrect. Crypt abscesses are found much less commonly in
Crohns disease than in ulcerative colitis.
Crypt distortion (Option B) is incorrect. Crypt distortion is found much less commonly in
Crohns disease than in ulcerative colitis.
Inflammatory infiltrates (Option C) is incorrect. Inflammatory infiltrates are found in both
conditions, and are therefore not helpful in distinguishing one condition from the other.
Superficial ulceration (Option E) is incorrect. As described, Crohns is characterised by
transmural inflammation.
45814

https://mypastest.pastest.com/Secure/TestMe/Browser/429893#Top

1/2

8/22/2016

MyPastest

Next Question

Previous Question

Tag Question

Feedback

End Review

Difficulty: Average
Peer Responses

Session Progress
Responses Correct:

Responses Incorrect:

298

Responses Total:

298

Responses - % Correct:

0%

Blog (https://www.pastest.com/blog) About Pastest (https://www.pastest.com/about-us)


Contact Us (https://www.pastest.com/contact-us) Help (https://www.pastest.com/help)
Pastest 2016

https://mypastest.pastest.com/Secure/TestMe/Browser/429893#Top

2/2

8/22/2016

MyPastest

Back to Filters (/Secure/TestMe/Filter/429893/QA)

Question 269 of 298

A 32-year-old man was referred with gastro-oesophageal reflux disease and commenced on a
proton-pump inhibitor.
Which of the following is true of the gastric K+/H+-ATPase proton pump?
A

It also occurs in other tissues

It is an acute antigen in pernicious anaemia

It is made up of alpha-, beta- and gamma-subunits

It is situated in chief cells

Omeprazole binds to it irreversibly

Explanation

The answer is Omeprazole binds to it irreversibly


It is the alpha-subunit of the proton-pump that catalyses the enzymatic activity, and
which forms an irreversible covalent complex with omeprazole. However, as the half-life
of the pump is 2436 hours, the duration of the effect of proton-pump inhibitors is
limited by the degradation of these pumps.

It also occurs in other tissues (Option A) is incorrect. The proton pump is only contained in
the tubulovesicles of the parietal cell.
It is an acute antigen in pernicious anaemia (Option B) is incorrect. Pernicious anaemia is
associated with chronic autoantibody production (IgG) against the gastric K+/H+-ATPase
proton pump, but it is not an acute antigen.
It is made up of alpha-, beta- and gamma-subunits (Option C) is incorrect. The proton pump
has only alpha and beta subunits.
It is situated in chief cells (Option D) is incorrect. The proton pump is only contained in the
tubulovesicles of the parietal cell.
45816

https://mypastest.pastest.com/Secure/TestMe/Browser/429893#Top

1/2

8/22/2016

MyPastest

Next Question

Previous Question

Tag Question

Feedback

End Review

Difficulty: Average
Peer Responses

Session Progress
Responses Correct:

Responses Incorrect:

298

Responses Total:

298

Responses - % Correct:

0%

Blog (https://www.pastest.com/blog) About Pastest (https://www.pastest.com/about-us)


Contact Us (https://www.pastest.com/contact-us) Help (https://www.pastest.com/help)
Pastest 2016

https://mypastest.pastest.com/Secure/TestMe/Browser/429893#Top

2/2

8/22/2016

MyPastest

Back to Filters (/Secure/TestMe/Filter/429893/QA)

Question 270 of 298

A 76-year-old man underwent investigation for abdominal pain. A liver ultrasound


demonstrated the presence of multiple hepatic lesions. Further tests revealed an elevated
urinary 5-hydroxyindoleacetic acid level.
Which of the following statements about carcinoid tumours is true?
A

Fibrosis of the heart valves is a recognised feature

Fifty percent of patients die within 2 months of diagnosis

Octreotide is not useful at controlling the diarrhoea

Presentation only occurs after metastasis

They most commonly involve the colon

Explanation

The answer is Fibrosis of the heart valves is a recognised feature


Carcinoid tumours may arise from either the gastrointestinal (GI) tract or from the
bronchi. Of those arising from the GI tract, the majority originate in the jejunum and
ileum (rather than the colon). These are neuroendocrine tumours that secrete a variety of
vasoactive agents, including serotonin. The carcinoid syndrome typically occurs when
these tumours spread, and a high burden of liver metastasis becomes established; at this
stage, so much normal liver parenchyma has been replaced by tumour that the ability of
the remaining healthy liver to catabolise these vasoactive agents is exceeded, and they
may then enter the central circulation and cause systemic effects. This can be detected
via a number of means, including a urinary test for 5-hydroxyindoleacetic acid (5-HIAA),
a breakdown product of serotonin. Carcinoid syndrome is also characterised by a number
of other clinical features which relate to the vasoactive agents released by the tumour.
These include flushing, bronchospasm and fibrous depositions on heart valves. It is the
valves of the right side of the heart that are most frequently affected, as inactivation of
vasoactive agents by the lungs causes relative sparing of the left side of the heart. This is
the reasoning behind the the correct answer here.

https://mypastest.pastest.com/Secure/TestMe/Browser/429893#Top

1/2

8/22/2016

MyPastest

Fifty percent of patients die within 2 months of diagnosis (Option B) is incorrect. These
tumours are usually slow-growing, with a 3-year survival rate of 50% (clearly ruling out this
option).
Octreotide is not useful at controlling the diarrhoea (Option C) is incorrect. Somatostatin
analogues (such as octreotide) may be useful for treating the diarrhoea associated with
carcinoid syndrome.
Presentation only occurs after metastasis (Option D) is incorrect. Presentation of these
tumours does not only occur with metastasis; tumours may also present because of mass
effect (eg intestinal obstruction), as appendicitis, or with a pellagra syndrome (ie dermatitis,
dementia and diarrhoea, due to niacin deficiency).
They most commonly involve the colon (Option E) is incorrect. As outlined, the majority of
carcinoid tumours originate in the jejunum and ileum.
45817

Next Question

Previous Question

Tag Question

Feedback

End Review

Difficulty: Average
Peer Responses

Session Progress
Responses Correct:

Responses Incorrect:

298

Responses Total:

298

Responses - % Correct:

0%

Blog (https://www.pastest.com/blog) About Pastest (https://www.pastest.com/about-us)


Contact Us (https://www.pastest.com/contact-us) Help (https://www.pastest.com/help)
Pastest 2016

https://mypastest.pastest.com/Secure/TestMe/Browser/429893#Top

2/2

8/22/2016

MyPastest

Back to Filters (/Secure/TestMe/Filter/429893/QA)

Question 271 of 298

A 52-year-old woman presented to her general practitioner complaining of tiredness and


itching. She had no history of alcohol excess, and had taken no medication. She had
xanthelasma. Her general practitioner noticed that her alkaline phosphatase level was raised,
and referred her for a gastroenterological opinion.
What would be the best test to confirm the likely diagnosis?
A

Alkaline phosphatase origin estimation (bone or liver)

Anti-mitochondrial antibodies

Bone scan

GGT (gamma-glutamyltransferase) testing

Hepatic ultrasound scan

Explanation

The answer is Anti-mitochondrial antibodies


The scenario described is highly suggestive of primary biliary cholangitis (PBC; formerly
known as primary biliary cirrhosis). The condition occurs most frequently in women
between the ages of 30 and 65 years of age, and the aetiology remains unclear. Antimitochondrial antibodies (AMA; in particular, the M2 variant) are found in 95% of cases,
and are 98% specific for this condition. The combination of typical clinical features of
PBC and a positive AMA is sufficient in itself to confirm the diagnosis without a liver
biopsy; however, a liver biopsy may still be useful for assessing the degree of hepatic
fibrosis. As such, anti-mitochondrial antibodies is the correct answer here.

Alkaline phosphatase origin estimation (bone or liver) (Option A) is incorrect. Although


raised cholestatic liver enzymes (alkaline phosphatase) are a characteristic feature of PBC,
they are not specific for the condition, meaning this is not the best option offered here.
Bone scan (Option C) is incorrect. PBC is associated with vitamin D malabsorption and
consequent osteomalacia, but a bone scan will not be useful in making the diagnosis, which
rules out this option.
https://mypastest.pastest.com/Secure/TestMe/Browser/429893#Top

1/2

8/22/2016

MyPastest

GGT testing (Option D) is incorrect. Although raised cholestatic liver enzymes (gammaglutamyltransferase) are a characteristic feature of PBC, they are not specific for the
condition, meaning this is not the best option offered here.
Hepatic ultrasound scan (Option E) is incorrect. A hepatic ultrasound may show features of
portal hypertension in advanced disease (ie splenomegaly, ascites), but there are no
ultrasound features specific to PBC that might be useful in confirming the diagnosis, and this
is therefore incorrect.
45819

Next Question

Previous Question

Tag Question

Feedback

End Review

Difficulty: Average
Peer Responses

Session Progress
Responses Correct:

Responses Incorrect:

298

Responses Total:

298

Responses - % Correct:

0%

Blog (https://www.pastest.com/blog) About Pastest (https://www.pastest.com/about-us)


Contact Us (https://www.pastest.com/contact-us) Help (https://www.pastest.com/help)
Pastest 2016

https://mypastest.pastest.com/Secure/TestMe/Browser/429893#Top

2/2

8/22/2016

MyPastest

Back to Filters (/Secure/TestMe/Filter/429893/QA)

Question 272 of 298

A 51-year-old man presented to his general practitioner complaining of tiredness. He had no


history of alcohol excess and took no medication. There had been a history of intermittent
bloody diarrhoea over the past few years. His general practitioner noticed that his alkaline
phosphatase level was raised, and that his antineutrophil cytoplasmic antibody (ANCA) was
positive. He was then referred for a gastroenterological opinion.
What is the most likely diagnosis?
A

Alcohol-related cirrhosis

Familial hypercholesterolaemia

Haemochromatosis

Primary biliary cholangitis

Primary sclerosing cholangitis

Explanation

The answer is Primary sclerosing cholangitis


The combination of a history of bloody diarrhoea, cholestatic liver enzymes and positive
ANCA is collectively suggestive of underlying inflammatory bowel disease (IBD)
complicated by primary sclerosing cholangitis (PSC). PSC occurs more commonly in
men, and IBD is strongly associated with the condition.

Alcohol-related cirrhosis (Option A) is incorrect. In the absence of an alcohol history, alcoholrelated cirrhosis is unlikely.
Familial hypercholesterolaemia (Option B) is incorrect. Familial hypercholesterolaemia is not
associated with any of the clinical features described in this scenario.
Haemochromatosis (Option C) is incorrect. Haemochromatosis may manifest with very
variable liver enzymes but does not have an association with either IBD or a positive ANCA,
making this unlikely.

https://mypastest.pastest.com/Secure/TestMe/Browser/429893#Top

1/2

8/22/2016

MyPastest

Primary biliary cholangitis (Option D) is incorrect. Primary biliary cholangitis is characterised


by cholestatic liver enzymes but does not have an association with either IBD or a positive
ANCA, making this unlikely.
45820

Next Question

Previous Question

Tag Question

Feedback

End Review

Difficulty: Average
Peer Responses

Session Progress
Responses Correct:

Responses Incorrect:

298

Responses Total:

298

Responses - % Correct:

0%

Blog (https://www.pastest.com/blog) About Pastest (https://www.pastest.com/about-us)


Contact Us (https://www.pastest.com/contact-us) Help (https://www.pastest.com/help)
Pastest 2016

https://mypastest.pastest.com/Secure/TestMe/Browser/429893#Top

2/2

8/22/2016

MyPastest

Back to Filters (/Secure/TestMe/Filter/429893/QA)

Question 273 of 298

A 44-year-old man who runs a bar in Alicante returned to the UK to visit relatives. Over the
past few months, he had been troubled by itching, and had noticed that his fluid intake had
increased markedly. On presentation to his general practitioner, it was noted that he was
extremely tanned, had loss of body hair, and also had gynaecomastia. Blood tests revealed
elevated alanine aminotransferase (ALT), aspartate aminotransferase (AST) and alkaline
phosphatase levels. A fasting plasma glucose was measured at 8.8 mmol/l.
What is the most likely diagnosis?
A

Alcohol-related cirrhosis

Haemochromatosis

Pancreatic carcinoma

Primary biliary cholangitis

Type 1 diabetes mellitus

Explanation

The answer is Haemochromatosis


The scenario given is very suggestive of haemochromatosis. Also sometimes called
bronze diabetes, haemochromatosis most commonly presents in men in their fifth
decade of life. Examination may be normal, but other recognised clinical presentations
include increased skin pigmentation, hepatosplenomegaly, ascites, reduced body hair,
amenorrhoea (in 25% of females) and loss of libido (in 50% of males). This mans
polydipsia is likely related to the development of diabetes mellitus, caused by iron
deposition within the pancreas. Haemochromatosis is inherited in an autosomal recessive
manner; heterozygotes may present with accelerated progression towards cirrhosis due
to an additional cause, often the consumption of alcohol. Laboratory investigations
usually reveal abnormal liver function tests (different patterns may be found), along with
biochemical evidence of iron overload, including elevated ferritin and transferrin
saturations. The diagnosis may now be confirmed by genetic/ mutation screening.

https://mypastest.pastest.com/Secure/TestMe/Browser/429893#Top

1/3

8/22/2016

MyPastest

Alcohol-related cirrhosis (Option A) is incorrect. Although information is given that this man
runs a bar, there is no clear-cut alcohol history; furthermore, alcohol-related liver disease
would not explain certain elements of this mans case, such as his tanned appearance and
loss of body hair. As such, alcohol-related cirrhosis is unlikely.
Pancreatic carcinoma (Option C) is incorrect. Pancreatic carcinoma may be associated with
diabetes (as very extensive replacement of normal pancreatic parenchyma with tumour may
cause insufficient pancreatic endocrine function); however, pancreatic carcinoma would not
explain features such as gynaecomastia or loss of body hair.
Primary biliary cholangitis (Option D) is incorrect. Primary biliary cholangitis (PBC; formerly
known as primary biliary cirrhosis) is associated with raised cholestatic liver enzymes, but
not with elevations in transaminases; furthermore, it has no association with the other clinical
features in this scenario, thus it can be ruled out.
Type 1 diabetes mellitus (Option E) is incorrect. Pancreatic carcinoma may be associated with
diabetes (as very extensive replacement of normal pancreatic parenchyma with tumour may
cause insufficient pancreatic endocrine function); however, type 1 diabetes mellitus would
not explain features such as gynaecomastia or loss of body hair.
45822

Next Question

Previous Question

Tag Question

Feedback

End Review

Difficulty: Average
Peer Responses

Session Progress
Responses Correct:

Responses Incorrect:

298

Responses Total:

298

Responses - % Correct:

0%

Blog (https://www.pastest.com/blog) About Pastest (https://www.pastest.com/about-us)


Contact Us (https://www.pastest.com/contact-us) Help (https://www.pastest.com/help)
Pastest 2016
https://mypastest.pastest.com/Secure/TestMe/Browser/429893#Top

2/3

8/22/2016

https://mypastest.pastest.com/Secure/TestMe/Browser/429893#Top

MyPastest

3/3

8/22/2016

MyPastest

Back to Filters (/Secure/TestMe/Filter/429893/QA)

Question 274 of 298

A 24-year-old female farm worker presented to her general practitioner with abdominal
swelling. She remembered an episode of abdominal pain a few months earlier, but nothing
else of note. Her medications included the oral contraceptive pill. She was single and did not
drink alcohol. Liver function testing revealed elevated alanine aminotransferase (ALT) and
bilirubin levels. Examination of the abdomen demonstrated mild ascites and splenomegaly.
What is the most likely diagnosis?
A

Alcohol-related cirrhosis

BuddChiari syndrome

Hydatid disease

Organophosphate toxicity

Viral hepatitis

Explanation

The answer is BuddChiari syndrome


This scenario is suggestive of BuddChiari syndrome. This rare condition usually presents
chronically, with evidence of ascites splenomegaly hepatomegaly other signs of liver
disease on examination. Acute and fulminant forms of BuddChiairi syndrome are also
recognised, and tend to present with acute abdominal pain and hepatomegaly; however,
these are much less common. BuddChiari syndrome is caused by obstruction of hepatic
venous flow. This is frequently caused by thrombosis, but external compression of the
vessels may also be a cause. The condition occurs more commonly in people with a
hypercoagulable state, eg those with myeloproliferative disorders, those with protein C,
antithrombin III or protein S deficiency, or patients taking the oral contraceptive pill.
Treatment includes venous shunting and anti-coagulation, although some patients
progress and require liver transplantation.

Alcohol-related cirrhosis (Option A) is incorrect. This can be ruled out in the absence of an
alcohol history.
https://mypastest.pastest.com/Secure/TestMe/Browser/429893#Top

1/2

8/22/2016

MyPastest

Hydatid disease (Option C) is incorrect. Hydatid disease would be expected to follow a


longer time course, with symptoms related to cyst enlargement, coupled with an obstructive
picture of liver enzymes.
Organophosphate toxicity (Option D) is incorrect. The information that this woman works on
a farm may increase suspicion for organophosphate poisoning, but this typically manifests
with neurological symptoms rather than features of liver disease.
Viral hepatitis (Option E) is incorrect. No risk factors for viral hepatitis are described in the
scenario, and viral hepatitis does not typically cause abdominal pain, making it unlikely.
45824

Next Question

Previous Question

Tag Question

Feedback

End Review

Difficulty: Average
Peer Responses

Session Progress
Responses Correct:

Responses Incorrect:

298

Responses Total:

298

Responses - % Correct:

0%

Blog (https://www.pastest.com/blog) About Pastest (https://www.pastest.com/about-us)


Contact Us (https://www.pastest.com/contact-us) Help (https://www.pastest.com/help)
Pastest 2016

https://mypastest.pastest.com/Secure/TestMe/Browser/429893#Top

2/2

8/22/2016

MyPastest

Back to Filters (/Secure/TestMe/Filter/429893/QA)

Question 275 of 298

A 21-year-old student, who had recently returned from summer travels in the Far East,
presented with acute-onset bloody diarrhoea, fever and abdominal pain. She had a raised, red
rash on her shins. She described two previous attacks of acute bloody diarrhoea in the past 2
years that resolved over a period of a few weeks.
What is the most appropriate initial treatment plan?
A

Oral antibiotics

Oral antibiotics and stool culture

Plain abdominal film, stool culture and intravenous antibiotics

Plain abdominal film, stool culture, sigmoidoscopy and biopsy, intravenous


corticosteroid therapy

Plain abdominal film, stool culture, sigmoidoscopy and intravenous antibiotics

Explanation

The answer is Plain abdominal film, stool culture, sigmoidoscopy and biopsy, intravenous
corticosteroid therapy
This woman presents with symptoms of an acute colitis. The major differential diagnosis
is between an infective aetiology (particularly given her recent travels), or inflammatory
bowel disease most likely ulcerative colitis (UC). There are clues given that the latter is
the case here. First, there is the description of a rash that sounds consistent with
erythema nodosum, which is associated with UC. Second, she has had episodes of
bloody diarrhoea previously, which again is consistent with a relapsingremitting
condition such as UC. A plain abdominal film is needed to rule out a toxic megacolon.
Stool culture is indicated to rule out an infective cause. Sigmoidoscopy is useful to
confirm the type of colitis present and to assess the severity of it. Given that intravenous
corticosteroids are the mainstay of treatment of acute severe colitis, the plan that has
intravenous corticosteroids in it is the best choice here. The other options are all less
appropriate initial management plans, since they do not include the use of intravenous
corticosteroids. Although there is an argument to be made for the administration of

https://mypastest.pastest.com/Secure/TestMe/Browser/429893#Top

1/3

8/22/2016

MyPastest

empirical antibiotics pending the result of a stool culture, the scenario given is more
suggestive of UC than infective colitis, and withholding corticosteroids could allow a
severe flare of ulcerative colitis to progress.

Oral antibiotics (Option A) is incorrect. As described, the scenario is more suggestive of UC


and the absence of intravenous corticosteroids could allow a severe flare of UC to progress.
Oral antibiotics and stool culture (Option B) is incorrect. As described, the scenario is more
suggestive of UC and the absence of intravenous corticosteroids could allow a severe flare of
UC to progress.
Plain abdominal film, stool culture and intravenous antibiotics (Option C) is incorrect. As
described, the scenario is more suggestive of UC and the absence of intravenous
corticosteroids could allow a severe flare of UC to progress.
Plain abdominal film, stool culture, sigmoidoscopy and intravenous antibiotics (Option E) is
incorrect. As described, the scenario is more suggestive of UC and the absence of
intravenous corticosteroids could allow a severe flare of UC to progress.
45825

Next Question

Previous Question

Tag Question

Feedback

End Review

Difficulty: Average
Peer Responses

Session Progress
Responses Correct:

Responses Incorrect:

298

Responses Total:

298

Responses - % Correct:

0%

Blog (https://www.pastest.com/blog) About Pastest (https://www.pastest.com/about-us)


Contact Us (https://www.pastest.com/contact-us) Help (https://www.pastest.com/help)
Pastest 2016
https://mypastest.pastest.com/Secure/TestMe/Browser/429893#Top

2/3

8/22/2016

https://mypastest.pastest.com/Secure/TestMe/Browser/429893#Top

MyPastest

3/3

8/22/2016

MyPastest

Back to Filters (/Secure/TestMe/Filter/429893/QA)

Question 276 of 298

A 52-year-old man with a long history of ulcerative colitis was found by his general
practitioner to have abnormal liver function tests, with a raised alkaline phosphatase. He did
not drink alcohol, and took no regular medications. On examination, there were no features of
chronic liver disease.
What are the most appropriate next investigations?
A

Computed tomography of the abdomen and viral hepatitis serology

Liver biopsy and anti-mitochondrial antibodies

Magnetic resonance cholangiopancreatography and liver biopsy

Ultrasound scan and anti-mitochondrial antibodies

Ultrasound scan and liver biopsy

Explanation

The answer is Magnetic resonance and cholangiopancreatography (MRCP) and liver


biopsy
The scenario given is very suggestive of primary sclerosing cholangitis (PSC). PSC is a
condition of unclear aetiology that is associated with inflammation, stricture formation
and associated biliary dilatation within both the intra- and extrahepatic biliary ductal
systems. The major clues that this is PSC are the cholestatic pattern of liver enzymes and
the history of ulcerative colitis (~75% of patients with PSC also have ulcerative colitis).
About 70% of patients are men, with an average age at onset of 40 years. Symptoms can
include pruritus, intermittent jaundice and attacks of cholangitis. PSC has a number of
associated complications, including colonic carcinoma (meaning that annual
colonoscopy is required as surveillance), as well as cholangiocarcinoma. Up to 20% of
people with PSC may eventually develop cholangiocarcinoma, and regular biliary
imaging is required as surveillance. Chronic inflammation of the intrahepatic biliary tree
may also lead to hepatic fibrosis, and over time the development of cirrhosis and its
complications.

https://mypastest.pastest.com/Secure/TestMe/Browser/429893#Top

1/3

8/22/2016

MyPastest

The major means of diagnosing PSC is by undertaking imaging that looks for the intraand extrahepatic duct stricture and dilatation that characterises the condition; MRCP and
ERCP are both sensitive for this, but MRCP is non-invasive and is therefore the preferred
investigation. Liver biopsy may also be useful for a number of reasons. First, it allows
assessment for the degree of hepatic fibrosis that has occurred in association with PSC.
Second, some cases of PSC may only affect the intrahepatic biliary tree (so-called smallduct PSC), and liver biopsy is, in these cases, the most appropriate way of confirming
the diagnosis. Findings on liver biopsy in PSC include inflammation of the intrahepatic
biliary radicals with associated scar tissue, often described as onion skin. The most
appropriate choice is therefore MRCP and liver biopsy.

Computed tomography (CT) of the abdomen and viral hepatitis serology (Option A) is
incorrect. No risk factors for viral hepatitis are described in the scenario, and such infections
tend to result in rises in hepatic transaminases rather than alkaline phosphatase. CT scans are
useful for looking for hepatic mass lesions, but do not give the same detail as MRCP when
imaging the biliary tree.
Liver biopsy and anti-mitochondrial antibodies (Option B) is incorrect. Anti-mitochondrial
antibodies are associated with primary biliary cholangitis but not PSC.
Ultrasound scan and anti-mitochondrial antibodies (Option D) is incorrect. Antimitochondrial antibodies are associated with primary biliary cholangitis but not PSC.
Ultrasound scan and liver biopsy (Option E) is incorrect. Ultrasound is useful for
demonstrating biliary dilatation, but is not as good as MRCP at defining whether the
dilatation is because of intrahepatic obstruction (eg gallstones) or because of inflammatory
strictures.
45826

Next Question

Previous Question

Tag Question

Feedback

End Review

Difficulty: Average
Peer Responses

Session Progress
Responses Correct:

Responses Incorrect:

298

Responses Total:

298

https://mypastest.pastest.com/Secure/TestMe/Browser/429893#Top

2/3

8/22/2016

Responses - % Correct:

MyPastest

0%

Blog (https://www.pastest.com/blog) About Pastest (https://www.pastest.com/about-us)


Contact Us (https://www.pastest.com/contact-us) Help (https://www.pastest.com/help)
Pastest 2016

https://mypastest.pastest.com/Secure/TestMe/Browser/429893#Top

3/3

8/22/2016

MyPastest

Back to Filters (/Secure/TestMe/Filter/429893/QA)

Question 277 of 298

A 47-year-old woman presented to her general practitioner with chronic fatigue. Liver
function testing was abnormal, with raised aspartate aminotransferase (AST) and alanine
aminotransferase (ALT) levels, accompanied by smaller rises in bilirubin and alkaline
phosphatase levels. Her gamma-globulin was also elevated, and there was a normochromic,
normocytic anaemia. Hepatitis B and C serology were negative, while liver biopsy revealed
chronic hepatitic change.
Which auto-antibody screen is most likely to be positive?
A

Anti-gliadin and anti-endomysial

Anti-LKM1

Anti-mitochondrial

Anti-smooth muscle and anti-nuclear

Rheumatoid factor

Explanation

The answer is Anti-smooth muscle and anti-nuclear


This scenario is very suggestive of this woman having type I autoimmune hepatitis; type I
autoimmune hepatitis may present with fatigue, more commonly in women in the perimenopausal period, whereas type II autoimmune hepatitis is more likely to present with
an acute episode of jaundice in young women. Type I autoimmune hepatitis is associated
with anti-smooth muscle and anti-nuclear antibody positivity, whereas type II is
associated with anti-LKM1 (liver-kidney-muscle 1) activity; as such, the best choice here
is anti-smooth muscle and anti-nuclear. Treatment of autoimmune hepatitis is typically
with prednisolone during the acute period, followed by a steroid-sparing agent (often
azathioprine) long-term. Corticosteroid and azathioprine therapy induce remission in
around 80% of cases.

Anti-gliadin and anti-endomysial (Option A) is incorrect. Anti-gliadin and anti-endomysial


antibodies are positive in coeliac disease, ruling out this option.
https://mypastest.pastest.com/Secure/TestMe/Browser/429893#Top

1/2

8/22/2016

MyPastest

Anti-LKM1 (Option B) is incorrect. As described, anti-LKM1 is associated with type II, rather
than type I, autoimmune hepatitis.
Anti-mitochondrial (Option C) is incorrect. Anti-mitochondrial antibodies are associated with
primary biliary cholangitis, but not with autoimmune hepatitis.
Rheumatoid factor (Option E) is incorrect. Rheumatoid factor is a test for rheumatoid
arthritis.
45828

Next Question

Previous Question

Tag Question

Feedback

End Review

Difficulty: Average
Peer Responses

Session Progress
Responses Correct:

Responses Incorrect:

298

Responses Total:

298

Responses - % Correct:

0%

Blog (https://www.pastest.com/blog) About Pastest (https://www.pastest.com/about-us)


Contact Us (https://www.pastest.com/contact-us) Help (https://www.pastest.com/help)
Pastest 2016

https://mypastest.pastest.com/Secure/TestMe/Browser/429893#Top

2/2

8/22/2016

MyPastest

Back to Filters (/Secure/TestMe/Filter/429893/QA)

Question 278 of 298

A 32-year-old woman was referred for endoscopy and found to have a duodenal ulcer and a
positive urease test. She was given lansoprazole, amoxicillin and clarithromycin for 7 days.
She says that she does not really have time to attend for an appointment with her GP or at
the hospital.
Which of the following is the most appropriate way of determining the successful eradication
of Helicobacter pylori?
A

14C

Blood serology testing

Endoscopy and antral histology

Endoscopy and CLO test

Faecal antigen testing

urea breath test

Explanation
The answer is

14

C urea breath test -

NICE consider this the only reliable option for re-testing after eradication therapy.

Blood serology testing (Option B) is incorrect. Serological testing for H. pylori remains
positive for at least 612 months after successful eradication of the infection, so would not be
helpful here.
Endoscopy and antral histology (Option C) is incorrect. Endoscopy with histology is invasive;
such tests also have a high risk of giving false-negative results after eradication therapy.
Endoscopy and CLO test (Option D) is incorrect. Endoscopy with CLO testing is invasive;
such tests also have a high risk of giving false-negative results after eradication therapy.
Faecal antigen testing (Option E) is incorrect. Faecal antigen has now largely replaced the
14
C urea breath, although NICE do not consider it reliable enough to determine eradication
yet.
46127

https://mypastest.pastest.com/Secure/TestMe/Browser/429893#Top

1/2

8/22/2016

MyPastest

Next Question

Previous Question

Tag Question

Feedback

End Review

Difficulty: Average
Peer Responses

Session Progress
Responses Correct:

Responses Incorrect:

298

Responses Total:

298

Responses - % Correct:

0%

Blog (https://www.pastest.com/blog) About Pastest (https://www.pastest.com/about-us)


Contact Us (https://www.pastest.com/contact-us) Help (https://www.pastest.com/help)
Pastest 2016

https://mypastest.pastest.com/Secure/TestMe/Browser/429893#Top

2/2

8/22/2016

MyPastest

Back to Filters (/Secure/TestMe/Filter/429893/QA)

Question 279 of 298

A 28-year-old woman presented to the Gastroenterology Clinic with weight loss, intermittent,
oily diarrhoea and malaise. Blood testing revealed folate and iron deficiency. There was also
mild hypocalcaemia on biochemistry screening. She had type 1 diabetes of 10 years duration
and was stable on a basal-bolus insulin regime. Otherwise, her past medical history was
unremarkable.
What is the most specific antibody test for confirming the likely diagnosis?
A

Antigliadin IgA and IgG antibodies

Anti-nuclear antibodies

Anti-smooth muscle antibodies

Anti-thyroid antibodies

Anti-tissue transglutaminase antibodies

Explanation

The answer is Anti-tissue transglutaminase antibodies The scenario is very suggestive of underlying coeliac disease. Coeliac disease is slightly
more common in women, with the peak ages of presentation being in the third and
seventh decades. The condition is often found in association with other conditions with
an immunological basis, such as type 1 diabetes or Graves disease. Pathologically, the
condition is characterised by autoimmune small-bowel villous atrophy, which can be
confirmed on duodenal biopsy. Although antigliadin, anti-endomysial and anti-tissue
transglutaminase antibodies are all relatively specific tests for coeliac disease, it is antitissue transglutaminase that has the highest overall sensitivity and specificity for the
condition, and is therefore the preferred test at present.
The association between coeliac disease and other autoimmune/immunological
conditions means that it is not unusual to detect either anti-thyroid or anti-nuclear
antibodies in the serum of people with coeliac disease. However, neither antibody is
specific for the diagnosis of coeliac disease, and therefore they have no role in making
the diagnosis of the condition.

https://mypastest.pastest.com/Secure/TestMe/Browser/429893#Top

1/2

8/22/2016

MyPastest

Antigliadin IgA and IgG antibodies (Option A) is incorrect. Anti-tissue transglutaminase


antibody has a higher sensitivity and specificity for coeliac disease.
Anti-nuclear antibodies (Option B) is incorrect. Anti-nuclear antibody is not specific for the
diagnosis of coeliac disease.
Anti-smooth muscle antibodies (Option C) is incorrect. Anti-tissue transglutaminase
antibody has a higher sensitivity and specificity for coeliac disease.
Anti-thyroid antibodies (Option D) is incorrect. Anti-thyroid antibody is not specific for the
diagnosis of coeliac disease.
46416

Next Question

Previous Question

Tag Question

Feedback

End Review

Difficulty: Average
Peer Responses

Session Progress
Responses Correct:

Responses Incorrect:

298

Responses Total:

298

Responses - % Correct:

0%

Blog (https://www.pastest.com/blog) About Pastest (https://www.pastest.com/about-us)


Contact Us (https://www.pastest.com/contact-us) Help (https://www.pastest.com/help)
Pastest 2016

https://mypastest.pastest.com/Secure/TestMe/Browser/429893#Top

2/2

8/22/2016

MyPastest

Back to Filters (/Secure/TestMe/Filter/429893/QA)

Question 280 of 298

A 45-year-old man presented to his general practitioner with epigastric pain. He worked as a
company sales manager, drank 35 units of alcohol per week and smoked 10 cigarettes per
day. He denied using NSAIDs. Screening for Helicobacter pylori was negative. He succeeded
in giving up smoking, reduced his alcohol intake and was given a trial of lansoprazole by his
doctor. Three months later, he presented to the Emergency Department with haematemesis.
Endoscopy showed multiple duodenal ulcers.
What is the most appropriate management plan?
A

Consider screening for multiple endocrine neoplasia type 2

Discharge home and continue low-dose lansoprazole

Measure his serum gastrin level and consider an octreotide scan or endoscopic
ultrasound, then give him a high-dose proton-pump inhibitor

Measure his serum vasoactive intestinal polypeptide level

Treat for Helicobacter pylori, even if repeat screen is negative

Explanation

The answer is Measure his serum gastrin level and consider an octreotide scan or
endoscopic ultrasound, then give him a high-dose proton-pump inhibitor
The presence of recurrent peptic ulcer disease in a young person without an obvious
alternative explanation raises the possibility of gastrinoma; when a gastrinoma leads to
hypersecretion of gastric acid and multiple duodenal ulcers this is known as the
ZollingerEllison syndrome. Gastrinoma is rare (accounting for 1 in 1000 cases of
duodenal ulceration), but should always be considered in scenarios such as this; as such,
measuring serum gastrin level with consideration of an octreotide scan for endoscopic
ultrasound, then giving him a high-dose proton-pump inhibitor, is the correct answer.
High-dose proton-pump inhibitors can suppress symptoms, but surgical resection is the
only curative option.

Consider screening for multiple endocrine neoplasia type 2 (MEN 2) (Option A) is incorrect.
Gastrinomas may also occur as part of the autosomal dominant multiple endocrine neoplasia
https://mypastest.pastest.com/Secure/TestMe/Browser/429893#Top

1/2

8/22/2016

MyPastest

1 (MEN-1) syndrome (i.e. the three Ps parathyroid, pituitary and pancreatic tumours), but
are not associated with MEN-2 syndrome, ruling out this option.
Discharge home and continue low-dose lansoprazole (Option B) is incorrect. Given that this
man reduced his risk factors for peptic ulceration and used lansoprazole, but still had
significant peptic ulcer disease, it is clear that this would not be appropriate.
Measure his serum vasoactive intestinal polypeptide (VIP) level (Option D) is incorrect.
VIPomas are like gastrinomas a form of neuroendocrine tumour that may arise from the
pancreas; unlike them, however, the major clinical manifestation they give is diarrhoea rather
than peptic ulcer disease.
Treat for Helicobacter pylori, even if repeat screen is negative (Option E) is incorrect.
Currently available tests for H. pylori are so sensitive that it is unlikely that he has underlying
infection which has been hitherto missed; this option may therefore be ruled out.
46417

Next Question

Previous Question

Tag Question

Feedback

End Review

Difficulty: Average
Peer Responses

Session Progress
Responses Correct:

Responses Incorrect:

298

Responses Total:

298

Responses - % Correct:

0%

Blog (https://www.pastest.com/blog) About Pastest (https://www.pastest.com/about-us)


Contact Us (https://www.pastest.com/contact-us) Help (https://www.pastest.com/help)
Pastest 2016

https://mypastest.pastest.com/Secure/TestMe/Browser/429893#Top

2/2

8/22/2016

MyPastest

Back to Filters (/Secure/TestMe/Filter/429893/QA)

Question 281 of 298

A 78-year-old woman returned for her third visit in 3 months to the Emergency Department,
complaining of extremely severe dull epigastric pain that radiated to her back. Her
haemoglobin was 10.4 g/dl and her alkaline phosphatase level was elevated.
What is the most appropriate initial management plan?
A

Arrange an abdominal ultrasound scan

Arrange an upper gastrointestinal endoscopy in the first instance

Check her CA-19.9 level

Check her serum ferritin level

Discharge home for a trial of proton-pump inhibitor therapy

Explanation

The answer is Arrange an abdominal ultrasound scan


The combination of epigastric pain, anaemia and raised alkaline phosphatase in a woman
of this age is most likely to be explained by pancreatic carcinoma. Pain arising from the
pancreas is often partially relieved by sitting forwards. Bile duct obstruction with
jaundice is a late feature of the condition. A mass might be palpable if the carcinoma
occurs in the head of the pancreas, but this would be more unusual if the tumour is in the
body or tail. The most appropriate next step is an abdominal ultrasound scan; ultrasound
is very good at imaging the biliary tree, so is useful at demonstrating biliary dilatation if
there is extrinsic compression/internal obstruction because of malignancy. Ultrasound
does always image the normal pancreas (because of bowel in the way), but would be
able to detect a large pancreatic mass. As such, arranging an abdominal ultrasound is
correct here.

Arrange an upper gastrointestinal endoscopy in the first instance (Option B) is incorrect.


Severe epigastric pain and anaemia could be explained by an upper gastrointestinal tract
malignancy; however, pain radiating to the back and a raised alkaline phosphatase are much
more suggestive of primary pancreatic pathology. As such, upper GI endoscopy should not
be considered before an abdominal ultrasound.
https://mypastest.pastest.com/Secure/TestMe/Browser/429893#Top

1/2

8/22/2016

MyPastest

Check her CA-19.9 level (Option C) is incorrect. CA-19.9 (CA, carbohydrate antigen) might be
useful as a marker of disease progression in pancreatic carcinoma, but cannot be used for
diagnosis as there is a high false-positive rate, making this option incorrect.
Check her serum ferritin level (Option D) is incorrect. Although checking her serum ferritin
may be helpful (as a low level would confirm iron-deficiency anaemia), this does not in itself
help to identify the unifying diagnosis it is therefore not the best next step.
Discharge home for a trial of proton-pump inhibitor therapy (Option E) is incorrect. Severe
epigastric pain and anaemia could be explained by an upper gastrointestinal tract
malignancy; however, pain radiating to the back and a raised alkaline phosphatase are much
more suggestive of primary pancreatic pathology. As such, trial of proton-pump inhibitor
therapy should not be considered before an abdominal ultrasound.
46418

Next Question

Previous Question

Tag Question

Feedback

End Review

Difficulty: Average
Peer Responses

Session Progress
Responses Correct:

Responses Incorrect:

298

Responses Total:

298

Responses - % Correct:

0%

Blog (https://www.pastest.com/blog) About Pastest (https://www.pastest.com/about-us)


Contact Us (https://www.pastest.com/contact-us) Help (https://www.pastest.com/help)
Pastest 2016

https://mypastest.pastest.com/Secure/TestMe/Browser/429893#Top

2/2

8/22/2016

MyPastest

Back to Filters (/Secure/TestMe/Filter/429893/QA)

Question 282 of 298

A 65-year-old vegetarian woman who ate a diet rich in a wide range of different fruits and
vegetables presented to the clinic complaining of facial flushing and diarrhoea. The flushing
episodes could last from a few minutes to a few hours, and the diarrhoea was often
accompanied by a feeling of peristaltic rushing. Her general practitioner was worried that she
may have an enlarged liver and sent off for a 24-h urinary 5-hydroxyindoleacetic acid (5HIAA) test to assess for carcinoid syndrome. The 5-HIAA result came back above the normal
range.
What would be the most appropriate next step?
A

Arrange an echocardiogram to rule out right-sided cardiac disease

Proceed to computed tomography of the abdomen

Repeat the 5-HIAA test after dietary restriction

Scan with radiolabelled octreotide

Try subcutaneous somatostatin for symptomatic relief

Explanation

The answer is option Repeat the 5-HIAA test after dietary restriction
The clinical features described together with the raised 5-hydroxyindoleacetic acid (5HIAA) result could be consistent with carcinoid syndrome. However, this woman is a
vegetarian; bananas, aubergines, avocados, pineapples and walnuts may all lead to a
falsely elevated urinary 5-HIAA level. As such, the most appropriate next step is to repeat
the 5-HIAA assay after removal of dietary factors that may falsely elevate it.
Should the test still be positive when repeated after dietary restriction, appropriate tests
to identify and characterise the carcinoid tumour include computed tomography,
radiolabelled octreotide scanning, and echocardiography (with the latter particularly to
assess for the valvular heart disease associated with the condition). However, none of
these tests would be appropriate at this stage.

https://mypastest.pastest.com/Secure/TestMe/Browser/429893#Top

1/2

8/22/2016

MyPastest

Arrange an echocardiogram to rule out right-sided cardiac disease (Option A) is incorrect.


This would not be an appropriate next step at this stage. It is important to first rule out a
dietary cause for the raised urinary 5-HIAA.
Proceed to computed tomography of the abdomen (Option B) is incorrect. This would not be
an appropriate next step at this stage. It is important to first rule out a dietary cause for the
raised urinary 5-HIAA.
Scan with radiolabelled octreotide (Option D) is incorrect. This would not be an appropriate
next step at this stage. It is important to first rule out a dietary cause for the raised urinary 5HIAA.
Try subcutaneous somatostatin for symptomatic relief (Option E) is incorrect. Somatostatin
and its analogues are recognised to provide symptomatic relief in the carcinoid syndrome,
but would not be initiated without a confirmed diagnosis, meaning this option may be
excluded.
46419

Next Question

Previous Question

Tag Question

Feedback

End Review

Difficulty: Average
Peer Responses

Session Progress
Responses Correct:

Responses Incorrect:

298

Responses Total:

298

Responses - % Correct:

0%

Blog (https://www.pastest.com/blog) About Pastest (https://www.pastest.com/about-us)


Contact Us (https://www.pastest.com/contact-us) Help (https://www.pastest.com/help)
Pastest 2016

https://mypastest.pastest.com/Secure/TestMe/Browser/429893#Top

2/2

8/22/2016

MyPastest

Back to Filters (/Secure/TestMe/Filter/429893/QA)

Question 283 of 298

A 55-year-old Vietnamese businessman now living in the UK presented with several months
of diarrhoea, anorexia and weight loss. On examination, he had some non-specific abdominal
tenderness and distension. He also had a little increased pigmentation, and glossitis was
noted on examining his oropharynx. Rectal examination revealed steatorrhoea stool. Blood
tests demonstrated megaloblastic anaemia, with vitamin B12 and folate deficiencies.
What is the most likely diagnosis?
A

Chronic pancreatitis

Coeliac disease

Small-bowel lymphoma

Tropical sprue

Ulcerative colitis

Explanation

The answer is Tropical sprue


The clue to the diagnosis here is that this man is a Vietnamese expatriate; South America,
India and Vietnam are common areas from which tropical sprue originates. Tropical sprue
may present several months after people native to tropical areas emigrate to a temperate
climate. The exact aetiology is unknown, but it is thought to be infective. Clinically, the
typical presentation is with diarrhoea; this represents malabsorption, and may manifest
as steatorrhoea in the worst cases. Up to 50% of affected people have megaloblastic
anaemia with associated vitamin B12 and folate deficiencies. Treatment usually consists
of 46 weeks of treatment with ampicillin or tetracycline.

Chronic pancreatitis (Option A) is incorrect. No risk factors for chronic pancreatitis are
described (e.g. alcohol excess), and there is no mention of the sort of epigastric pain that
typically defines the condition, so this option may be ruled out.
Coeliac disease (Option B) is incorrect. Although the clinical features described would fit
with coeliac disease, the information that this man was originally from Vietnam makes
https://mypastest.pastest.com/Secure/TestMe/Browser/429893#Top

1/2

8/22/2016

MyPastest

tropical sprue more likely.


Small-bowel lymphoma (Option C) is incorrect. Small-bowel lymphomas may present with a
wide variety of GI features, but often cause significant abdominal pain; as no mention is
made of abdominal pain, this is less likely to be the diagnosis here.
Ulcerative colitis (Option E) is incorrect. Ulcerative colitis typically presents with bloody
diarrhoea; it does not present with malabsorption (since it affects only the colon rather than
the small bowel), so is not the diagnosis here.
46420

Next Question

Previous Question

Tag Question

Feedback

End Review

Difficulty: Average
Peer Responses

Session Progress
Responses Correct:

Responses Incorrect:

298

Responses Total:

298

Responses - % Correct:

0%

Blog (https://www.pastest.com/blog) About Pastest (https://www.pastest.com/about-us)


Contact Us (https://www.pastest.com/contact-us) Help (https://www.pastest.com/help)
Pastest 2016

https://mypastest.pastest.com/Secure/TestMe/Browser/429893#Top

2/2

8/22/2016

MyPastest

Back to Filters (/Secure/TestMe/Filter/429893/QA)

Question 284 of 298

A 41-year-old man presented to his general practitioner with a history of intermittent fever,
cough and pleuritic chest pain. He was given a course of antibiotics and returned to his
general practitioner 3 weeks later complaining of bulky stools and malodorous diarrhoea up
to ten times per day (which, he now described, had pre-dated his chest symptoms). He also
described progressive arthralgia.
What is the most appropriate test to obtain a definitive diagnosis?
A

Chest X-ray

Human immunodeficiency virus testing

Small-bowel barium follow-through and biopsy of the small intestine

Sputum culture for tuberculosis

Stool assay for Clostridium difficile

Explanation

The answer is Small-bowel barium follow-through and biopsy of the small intestine
The history described is suggestive of Whipples disease. This is an uncommon illness
that occurs most frequently in men aged between 30 and 60 years. Extraintestinal
symptoms of Whipples disease include fever, arthralgia, pleuritic chest pain, pericarditis,
ophthalmoplegia (and, ultimately, dementia).
The gastrointestinal symptoms that occur within the condition (the most common of
which is malabsorptive diarrhoea) are caused by gastrointestinal infiltration by an
actinobacterium, Tropheryma whipplei. Small bowel follow-through may be useful for
helping to assess for/rule out other causes of diarrhoea, such as small-bowel Crohns
disease. Diagnosis is confirmed by intestinal biopsy, which will demonstrate periodic
acidSchiff- (PAS-) positive macrophages infiltrating the lamina propria. A polymerase
chain reaction (PCR) test is now also available for T. whipplei. Treatment is with a 46
months course of antibiotics, typically co-trimoxazole or penicillin.

https://mypastest.pastest.com/Secure/TestMe/Browser/429893#Top

1/2

8/22/2016

MyPastest

Chest X-ray (Option A) is incorrect. A chest X-ray would be worth considering given his chest
symptoms, but is clearly unlikely to reveal the unifying diagnosis given that he has a multisystemic disease.
Human immunodeficiency virus (HIV) testing (Option B) is incorrect. HIV testing would be
the most appropriate test if Mycobacterium avium intracellulare was suspected, but in itself
would not provide a definitive answer to the cause of the diarrhoea.
Sputum culture for tuberculosis (Option D) is incorrect. The scenario described would be
very unusual for tuberculosis (particularly the suggestion of malabsorptive diarrhoea), and
further investigations along this route are unlikely to be helpful.
Stool assay for Clostridium difficile (Option E) is incorrect. His diarrhoea was actually present
even before the antibiotics; this is the other way round to the course of events typically
found in those with C. difficile infection, and this option can be eliminated.
46421

Next Question

Previous Question

Tag Question

Feedback

End Review

Difficulty: Average
Peer Responses

Session Progress
Responses Correct:

Responses Incorrect:

298

Responses Total:

298

Responses - % Correct:

0%

Blog (https://www.pastest.com/blog) About Pastest (https://www.pastest.com/about-us)


Contact Us (https://www.pastest.com/contact-us) Help (https://www.pastest.com/help)
Pastest 2016

https://mypastest.pastest.com/Secure/TestMe/Browser/429893#Top

2/2

8/22/2016

MyPastest

Back to Filters (/Secure/TestMe/Filter/429893/QA)

Question 285 of 298

A 38-year-old man presented to his general practitioner complaining of bulky stools and
malodorous diarrhoea up to ten times per day. He had attended an Emergency Department 3
weeks earlier while on holiday in southern Spain because of pleuritic chest pain and a cough,
and was given antibiotics for a presumed chest infection. In light of the diarrhoea, he went on
to have a gastroscopy; a duodenal biopsy performed at this time demonstrated infiltration of
the lamina propria by periodic acidSchiff- (PAS-) positive macrophages.
What is the most likely diagnosis?
A

Chronic pancreatitis

Coeliac disease

Mycobacterium avium intracellulare

Tuberculosis with bowel involvement

Whipples disease

Explanation

The answer is Whipples disease


The clinical features described coupled with the biopsy results are highly suggestive
of Whipples disease. This is an uncommon condition, occurring most frequently in men
aged 3060 years. Extraintestinal symptoms of Whipples disease include fever,
arthralgia, pleuritic chest pain, pericarditis, ophthalmoplegia (and ultimately dementia).
The gastrointestinal symptoms found within the condition (the most common of which is
malabsorptive diarrhoea) are caused by gastrointestinal infiltration by an
actinobacterium, Tropheryma whipplei. Diagnosis is confirmed by a small intestinal
biopsy demonstrating periodic acidSchiff- (PAS-) positive macrophages infiltrating the
lamina propria. A polymerase chain reaction (PCR) test is also now available for T.
whipplei. Treatment is with a 46 months course of antibiotics, typically co-trimoxazole
or penicillin.

https://mypastest.pastest.com/Secure/TestMe/Browser/429893#Top

1/2

8/22/2016

MyPastest

Chronic pancreatitis (Option A) is incorrect. No risk factors for chronic pancreatitis (such as
alcohol excess) are described, and the histology result confirms that the diagnosis is
Whipples disease.
Coeliac disease (Option B) is incorrect. Coeliac disease may also be excluded given the
description of characteristic biopsy findings of Whipples disease.
Mycobacterium avium intracellulare (MAI) (Option C) is incorrect. MAI is an opportunistic
infection that occurs in human immunodeficiency virus (HIV)-positive patients; in the
absence of a history of HIV, this is unlikely.
Tuberculosis with bowel involvement (Option D) is incorrect. Tuberculosis may present with a
multi-systemic illness, but the scenario given here (and particularly the description of
malabsorptive diarrhoea) would be a very unusual manifestation of the condition.
46422

Next Question

Previous Question

Tag Question

Feedback

End Review

Difficulty: Average
Peer Responses

Session Progress
Responses Correct:

Responses Incorrect:

298

Responses Total:

298

Responses - % Correct:

0%

Blog (https://www.pastest.com/blog) About Pastest (https://www.pastest.com/about-us)


Contact Us (https://www.pastest.com/contact-us) Help (https://www.pastest.com/help)
Pastest 2016

https://mypastest.pastest.com/Secure/TestMe/Browser/429893#Top

2/2

8/22/2016

MyPastest

Back to Filters (/Secure/TestMe/Filter/429893/QA)

Question 286 of 298

An 18-year-old man presented with acute hepatitis, nausea, jaundice, elevated transaminases
and a prolonged prothrombin time. On examination, there were signs of chronic liver disease,
and a brown-green ring at the periphery of the iris in both eyes. His serum copper level was
low.
What other laboratory findings would be expected?
A

High serum caeruloplasmin level and low urinary copper excretion

High serum uric acid level and high urinary copper excretion

High serum uric acid and low serum caeruloplasmin levels

Low serum caeruloplasmin level and low urinary copper excretion

Low serum uric acid level and high urinary copper excretion

Explanation

The answer is Low serum uric acid level and high urinary copper excretion
The diagnosis here is Wilsons disease. This is a disorder of copper transport, caused
through an autosomal recessive inheritance of a mutation in the ATP7B gene, which
results in a defect in the incorporation of copper into caeruloplasmin. Pathologically, the
condition is characterised by inadequate biliary copper excretion, leading to
accumulation of copper in the liver, brain, kidneys and cornea. Clinically, the liver
manifestations of the condition may present very variably, ranging from acute hepatitis/
acute liver failure through to cirrhosis. The description of a brown-green ring at the
periphery of the iris is consistent with KayserFleischer rings, which are copper deposits
that occur in Descemets membrane in the cornea in people with the condition.
Laboratory findings in Wilsons disease include raised aspartate aminotransferase (AST),
low caeruloplasmin, low serum copper, raised urinary copper excretion and low serum
uric acid (also owing to increased urinary excretion). As such, option E is the only one
given that may be consistent with a diagnosis of Wilsons disease. Aminoaciduria,
glycosuria and calciuria also occur, with poor acidification of urine. The exact reason for

https://mypastest.pastest.com/Secure/TestMe/Browser/429893#Top

1/2

8/22/2016

MyPastest

these renal changes is unclear; although the Wilsons gene is expressed in renal tissue,
typically no specific histologic abnormalities are seen, and the renal problem appears to
be functional.

High serum caeruloplasmin level and low urinary copper excretion (Option A) is incorrect.
These findings are not consistent with the expected laboratory findings for Wilsons disease.
High serum uric acid level and high urinary copper excretion (Option B) is incorrect. These
findings are not consistent with the expected laboratory findings for Wilsons disease.
High serum uric acid and low serum caeruloplasmin levels (Option C) is incorrect. These
findings are not consistent with the expected laboratory findings for Wilsons disease.
Low serum caeruloplasmin level and low urinary copper excretion (Option D) is incorrect.
These findings are not consistent with the expected laboratory findings for Wilsons disease.
46423

Next Question

Previous Question

Tag Question

Feedback

End Review

Difficulty: Average
Peer Responses

Session Progress
Responses Correct:

Responses Incorrect:

298

Responses Total:

298

Responses - % Correct:

0%

Blog (https://www.pastest.com/blog) About Pastest (https://www.pastest.com/about-us)


Contact Us (https://www.pastest.com/contact-us) Help (https://www.pastest.com/help)
Pastest 2016

https://mypastest.pastest.com/Secure/TestMe/Browser/429893#Top

2/2

8/22/2016

MyPastest

Back to Filters (/Secure/TestMe/Filter/429893/QA)

Question 287 of 298

A 17-year-old girl presented with lethargy and chronic nausea. She had been unable to
complete her school examinations and her mother was worried that she might be depressed.
On examination, there were signs of chronic liver disease and a brown-green ring at the
periphery of the iris in both eyes. Her serum copper level was low.
What is the most likely diagnosis?
A

Acute autoimmune hepatitis

Alcoholic cirrhosis

Haemochromatosis

Hepatitis C

Wilsons disease

Explanation

The answer is Wilsons disease


The combination of liver disease, neurological disease and a disordered copper
metabolism is highly suggestive of Wilsons disease. The condition is caused through an
autosomal recessive inheritance of a mutation in the ATP7B gene, which results in a
defect in the incorporation of copper into caeruloplasmin. The result of this is inadequate
biliary copper excretion, leading to accumulation of copper in the liver, brain, kidneys and
cornea. The hepatic manifestations of Wilsons disease are very variable, ranging from
acute hepatitis/acute liver failure through to chronic liver disease and its complications.
KayserFleischer rings are almost always present when affected patients have
neurological symtoms; these are brown-green/ yellow rings that are found at the lumbus
of the cornea, and are best visualised on slitlamp examination. Long-term treatment of
the condition is with penicillamine (a copper chelator); the prognosis with early
intervention is good.
The typical laboratory findings of Wilsons disease include raised aspartate transaminase
(AST), low caeruloplasmin, low serum copper, raised urinary copper excretion and low
serum uric acid. Liver biopsy often reveals evidence of steatosis, focal necrosis,
inflammation and, later, cirrhosis.
https://mypastest.pastest.com/Secure/TestMe/Browser/429893#Top

1/3

8/22/2016

MyPastest

Haemochromatosis, alcohol excess, autoimmune hepatitis and hepatitis C may all clearly
be causes of chronic liver disease. However, the suggestion of this girl also having neuropsychiatric components to her illness (as implied by her decline in school
performance/mood disorder, as well as KayserFleisher rings) may only be explained by
Wilsons disease. In addition, Wilsons disease is the only one of the options given in
which low serum copper levels are consistently found.

Acute autoimmune hepatitis (Option A) is incorrect. The neuro-psychiatric components


described would not be explained by acute autoimmune hepatitis.
Alcoholic cirrhosis (Option B) is incorrect. The neuro-psychiatric components described
would nto be explained by alcoholic cirrhosis.
Haemochromatosis (Option C) is incorrect. The neuro-psychiatric components described
would not be explained by haemochromatosis.
Hepatitis C (Option D) is incorrect. The neuro-psychiatric components described would not
be explained by hepatitis C.
46424

Next Question

Previous Question

Tag Question

Feedback

End Review

Difficulty: Average
Peer Responses

Session Progress
Responses Correct:

Responses Incorrect:

298

Responses Total:

298

Responses - % Correct:

0%

Blog (https://www.pastest.com/blog) About Pastest (https://www.pastest.com/about-us)


Contact Us (https://www.pastest.com/contact-us) Help (https://www.pastest.com/help)
Pastest 2016
https://mypastest.pastest.com/Secure/TestMe/Browser/429893#Top

2/3

8/22/2016

https://mypastest.pastest.com/Secure/TestMe/Browser/429893#Top

MyPastest

3/3

8/22/2016

MyPastest

Back to Filters (/Secure/TestMe/Filter/429893/QA)

Question 288 of 298

A 32-year-old woman with known protein C deficiency presented to the Emergency


Department with a history of diarrhoea, steatorrhoea and weight loss. She suffered a Colles
fracture some years ago. On examination, there is an old, healed, midline abdominal scar.
Blood tests demonstrate anaemia with an increased red cell distribution width, a serum
calcium level just below the lower limit of normal, and a normal CRP.
What is the most likely diagnosis?
A

Active Crohns disease

Chronic pancreatitis

Coeliac disease

Short-bowel syndrome

Ulcerative colitis

Explanation

The answer is Short-bowel syndrome


The combination of protein C deficiency (ie a hypercoagulable state) with a previous
laparotomy scar hints at previous small-bowel infarction and resection. Her Colles
fracture is most likely to be due to osteoporosis/osteomalacia because of defective
calcium and vitamin D absorption, and a slightly reduced serum calcium level would be
consistent with this. The presence of anaemia with an increased red cell distribution
width may be suggestive of deficiency of vitamin B12, folate and iron. The apparent
severity of vitamin and electrolyte deficiency described suggests very extensive previous
short-bowel resection with consequent short-bowel syndrome.

Active Crohns disease (Option A) is incorrect. Active small-bowel Crohns disease may cause
malabsorptive disease, but the normal CRP level makes it unlikely that this is the case here.
Chronic pancreatitis (Option B) is incorrect. None of the risk factors for chronic pancreatitis
(such as alcohol excess) is described, and there is no description of typical symptoms of the
condition, such as abdominal pain; this condition is therefore also unlikely.
https://mypastest.pastest.com/Secure/TestMe/Browser/429893#Top

1/2

8/22/2016

MyPastest

Coeliac disease (Option C) is incorrect. Coeliac disease is more commonly associated with
iron deficiency/microcytic anaemia than the mixed anaemia described here, so may also be
ruled out.
Ulcerative colitis (Option E) is incorrect. As ulcerative colitis only affects the large bowel, it is
not associated with features of significant malabsorption, and cannot be the correct
diagnosis.
46425

Next Question

Previous Question

Tag Question

Feedback

End Review

Difficulty: Average
Peer Responses

Session Progress
Responses Correct:

Responses Incorrect:

298

Responses Total:

298

Responses - % Correct:

0%

Blog (https://www.pastest.com/blog) About Pastest (https://www.pastest.com/about-us)


Contact Us (https://www.pastest.com/contact-us) Help (https://www.pastest.com/help)
Pastest 2016

https://mypastest.pastest.com/Secure/TestMe/Browser/429893#Top

2/2

8/22/2016

MyPastest

Back to Filters (/Secure/TestMe/Filter/429893/QA)

Question 289 of 298

A 57-year-old man presented with a persistent history of heartburn, and was referred to the
gastroenterology clinic. His weight had been stable and he was otherwise constitutionally
well. A trial of low-dose proton-pump inhibitor (PPI) and lifestyle measures failed to alleviate
his symptoms. Screening blood test results were unremarkable, with no evidence of anaemia.
What is the most appropriate management?
A

Ask the general practitioner to discontinue PPI treatment and advise lifestyle
measures

Continue low-dose PPI long-term

Give intermittent high-dose PPI for symptom relief

Offer counselling for the non-sinister nature of his symptoms

Perform an upper gastrointestinal endoscopy

Explanation

The answer is Perform an upper gastrointestinal endoscopy


This man falls into the age range where upper GI tract symptoms have a significant
likelihood of representing serious pathology, and this needs to be assessed for; as such,
upper GI endoscopy is the best option here. The persistent nature of his symptoms
despite appropriate first-line therapy should also raise concerns. The crossover point at
which pathology becomes a statistically significant possibility compared with functional
disease is around 4550 years of age. In the over-50s, Barretts oesophagus,
oesophageal carcinoma, stomach carcinoma and peptic ulcer disease would all need to
be assessed for by endoscopy, even in people without other red flag features associated
with their presentation (such as weight loss and/or anaemia).

Ask the general practitioner to discontinue PPI treatment and advise lifestyle measures
(Option A) is incorrect. PPI helps relieve symptoms even in functional dyspepsia.
Continue low-dose PPI long-term (Option B) is incorrect. This may be appropriate to
consider if no sinister pathology is found by the gastroscopy, but should not replace
https://mypastest.pastest.com/Secure/TestMe/Browser/429893#Top

1/2

8/22/2016

MyPastest

performing a gastroscopy first.


Give intermittent high-dose PPI for symptom relief (Option C) is incorrect. This may be
appropriate to consider if no sinister pathology is found by the gastroscopy, but should not
replace performing a gastroscopy first.
Offer counselling for the non-sinister nature of his symptoms (Option D) is incorrect. This is
inappropriate as this neither helps obtain a diagnosis for this mans symptoms, nor provides
him any symptomatic relief.
46426

Next Question

Previous Question

Tag Question

Feedback

End Review

Difficulty: Average
Peer Responses

Session Progress
Responses Correct:

Responses Incorrect:

298

Responses Total:

298

Responses - % Correct:

0%

Blog (https://www.pastest.com/blog) About Pastest (https://www.pastest.com/about-us)


Contact Us (https://www.pastest.com/contact-us) Help (https://www.pastest.com/help)
Pastest 2016

https://mypastest.pastest.com/Secure/TestMe/Browser/429893#Top

2/2

8/22/2016

MyPastest

Back to Filters (/Secure/TestMe/Filter/429893/QA)

Question 290 of 298

A 57-year-old man presented with a history of persistent heartburn. He had seen his general
practitioner about this on several occasions and had now been referred to the
gastroenterology clinic for further evaluation. A trial of low-dose proton-pump inhibitor (PPI)
and lifestyle measures had failed to alleviate his symptoms. Screening blood test results were
unremarkable, with no evidence of anaemia. He subsequently underwent an upper
gastrointestinal endoscopy; this revealed an abnormally high junction between the columnar
epithelium of the stomach and the oesophageal squamous epithelium, with fronds of
columnar epithelium extending up into the oesophagus, consistent with Barretts oesophagus.
Which of the following is correct about this man?
A

Continuous low-dose PPI therapy would be appropriate in this case

He has a 3050 times increased risk of adenocarcinoma of the oesophagus


compared with someone without Barretts oesophagus

He has a 3050 times increased risk of carcinoma of the stomach

He has a 3050 times increased risk of squamous-cell carcinoma of the oesophagus


compared with someone without Barretts oesophagus

He has no increased risk of carcinoma

Explanation

The answer is He has a 3050 times increased risk of adenocarcinoma of the oesophagus
compared with someone without Barretts oesophagus
Barretts oesophagus is attributable to columnar metaplasia of the oesophageal
epithelium and is associated with a greatly increased risk of oesophageal
adenocarcinoma. It is thought to develop as a result of chronic severe reflux disease.
There is a 4:1 male to female ratio, with peak presentation at 5560 years. The condition
occurs more frequently in white populations, and has on average a prevalence of 9.6% in
patients undergoing endoscopy. Barretts oesophagus may be asymptomatic, or can be
discovered during investigations for chest pain, heartburn or occasional dysphagia to
solids. The differential diagnosis includes uncomplicated gastro-oesophageal reflux
disease (GORD), gastritis, hiatus hernia, benign oesophageal stricture and malignancy. All
affected patients should be on long-term, high-dose proton-pump inhibitors (PPIs), as
https://mypastest.pastest.com/Secure/TestMe/Browser/429893#Top

1/3

8/22/2016

MyPastest

these may prevent progression to malignancy. Endoscopic treatment of Barretts (such


as with radiofrequency ablation, or with endomucosal resection) may be indicated where
dysplasia occurs. Regular endoscopic surveillance of Barretts is required, with the
frequency of surveillance dependent upon the degree of dysplasia on histology. In total,
around 500 new oesophageal adenocarcinomas per year develop per 100 000 patients
with Barretts oesophagus.

Continuous low-dose PPI therapy would be appropriate in this case (Option A) is incorrect.
High-dose (rather than low-dose) PPI is preferred as the treatment for people with Barretts
oesphagus.
He has a 3050 times increased risk of carcinoma of the stomach (Option C) is incorrect.
Barretts oesophagus has no significant association with gastric cancer.
He has a 3050 times increased risk of squamous-cell carcinoma of the oesophagus
compared with someone without Barretts oesophagus (Option D) is incorrect. Barretts
oesophagus has no significant association with squamous-cell carcinoma of the oesophagus.
He has no increased risk of carcinoma (Option E) is incorrect. This option is clearly incorrect
because of the increased risk of oesophageal adenocarcinoma.
46427

Next Question

Previous Question

Tag Question

Feedback

End Review

Difficulty: Average
Peer Responses

Session Progress
Responses Correct:

Responses Incorrect:

298

Responses Total:

298

Responses - % Correct:

https://mypastest.pastest.com/Secure/TestMe/Browser/429893#Top

0%

2/3

8/22/2016

MyPastest

Blog (https://www.pastest.com/blog) About Pastest (https://www.pastest.com/about-us)


Contact Us (https://www.pastest.com/contact-us) Help (https://www.pastest.com/help)
Pastest 2016

https://mypastest.pastest.com/Secure/TestMe/Browser/429893#Top

3/3

8/22/2016

MyPastest

Back to Filters (/Secure/TestMe/Filter/429893/QA)

Question 291 of 298

A 17-year-old girl was admitted to the Emergency Department after being found drowsy at
home. Her parents had been to a party and found her lying on the sofa with an empty bottle
of vodka and empty packets of paracetamol around her when they returned home. They
mentioned that she had a row with her boyfriend the previous evening. Her blood tests on
admission to the Emergency Department revealed a paracetamol level of 70 mg/kg
(nomogram with treatment range not available) and an international normalised ratio (INR) of
2.1 (<1.0).
What is the appropriate management in this case?
A

Begin immediate N-acetylcysteine and arrange for her to be closely monitored on


the medical ward

Continue observation as her paracetamol level is below the treatment line

Give her oral methionine when she is awake enough to take it

Give her vitamin K and continue observation

Refer her immediately to a liver unit

Explanation

The answer is Begin immediate N-acetylcysteine and arrange for her to be closely
monitored on the medical ward
Since it is unclear when she took a paracetamol overdose, caution should be exercised
and intravenous N-acetylcysteine begun immediately. Her international normalised ratio
(INR) of 2.1 suggests that the overdose probably occurred the previous evening and that
some liver necrosis has already begun to develop. After optimal resuscitation (ie with
intravenous fluids), she should be very closely monitored for the development of adverse
prognostic features, including hepatic encephalopathy, worsening acidosis, progressive
renal failure, rise in bilirubin and prolongation of the INR. The paracetamol level itself and
liver enzyme values are not prognostic. Should these adverse features continue to
progress despite intravenous N-acetylcysteine and resuscitation, there should be a low
threshold to refer her to a liver unit in case she develops acute liver failure requiring
emergency liver transplantation.
https://mypastest.pastest.com/Secure/TestMe/Browser/429893#Top

1/3

8/22/2016

MyPastest

N-acetylcysteine works as an -SH group donator, so prevents some of the liver damage
caused by abnormal paracetamol metabolism via an alternative oxidative pathway that
results in the production of NAPQI (N-acetyl-p-benzoquinone imine), a highly toxic
metabolite. There is some evidence that continued N-acetylcysteine infusion may have
positive effects on mortality and morbidity in patients who already have established liver
failure.

Continue observation as her paracetamol level is below the treatment line (Option B) is
incorrect. The lack of clarity about when the overdose was taken, the implied intake of other
potential hepatotoxins at the same time (including alcohol), and the raised INR all imply that
this is a high-risk paracetamol overdose, regardless of where the paracetamol level falls on
the nomogram; N-acetylcysteine should be started immediately.
Give her oral methionine when she is awake enough to take it (Option C) is incorrect.
Methionine appears to be a less effective antedote to paracetamol overdose than Nacetylcysteine, and cannot be administered until the patient is less obtunded.
Give her vitamin K and continue observation (Option D) is incorrect. It is possible that part of
the reason for her prolonged INR is nutritional deficiency of vitamin K; however, this is clearly
too high-risk an overdose for this to be a safe and appropriate course of management.
Refer her immediately to a liver unit (Option E) is incorrect. As described, there is no
indication at present that she requires liver transplantation, but there should be a low
threshold to discuss her case with a specialist centre should she develop progressive adverse
prognostic factors despite optimal medical management with fluids and N-acetylcysteine.
46428

Next Question

Previous Question

Tag Question

Feedback

End Review

Difficulty: Average
Peer Responses

Session Progress
Responses Correct:

Responses Incorrect:

298

Responses Total:

298

Responses - % Correct:
https://mypastest.pastest.com/Secure/TestMe/Browser/429893#Top

0%
2/3

8/22/2016

MyPastest

Blog (https://www.pastest.com/blog) About Pastest (https://www.pastest.com/about-us)


Contact Us (https://www.pastest.com/contact-us) Help (https://www.pastest.com/help)
Pastest 2016

https://mypastest.pastest.com/Secure/TestMe/Browser/429893#Top

3/3

8/22/2016

MyPastest

Back to Filters (/Secure/TestMe/Filter/429893/QA)

Question 292 of 298

An obese 54-year-old woman presented to the Emergency Department. She had rigors and
reported a fever. On examination, she was jaundiced, and there was tenderness over the right
upper quadrant of her abdomen. She had an elevated white blood cell count, and markedly
raised bilirubin and alkaline phosphatase levels. Her transaminases were mildly increased, and
her amylase level was normal.
What is the most likely diagnosis?
A

Acute hepatitis

Ascending cholangitis

Pancreatitis

Peptic ulcer disease

Right ureteric calculus

Explanation

The answer is Ascending cholangitis


The clinical picture is likely to represent biliary obstruction due to gallstones, with
associated ascending cholangitis as such, ascending cholangitis is the correct answer
here. Gallstone disease is the most common cause of cholangitis, with other causes
including prior biliary tract surgery and endoscopic retrograde
cholangiopancreatography (ERCP). Patients usually have fever, rigors and abdominal
pain, with tenderness over the right upper quadrant and jaundice. The full collection of
signs of cholangitis are present in 5085% of affected patients. Bacteraemia and
systemic sepsis occur in up to 50% of cases. Hepatic abscess and pancreatitis are
recognised potential complications.
Investigation-wise, cholangitis is usually characterised by an elevated white blood cell
count with neutrophilia and abnormal liver function tests, with a markedly raised alkaline
phosphatase level and smaller rises in transaminases. Ultrasound, computed tomography
or magnetic resonance cholangiopancreatography (MRCP) are the favoured initial

https://mypastest.pastest.com/Secure/TestMe/Browser/429893#Top

1/2

8/22/2016

MyPastest

imaging modalities. The first-line treatment of choice is with antibiotics (which should
cover Gram-negative organisms); sphincterotomy via ERCP is indicated where gallstones
in the common bile duct have resulted in cholangitis.

Acute hepatitis (Option A) is incorrect. Although acute hepatitis is associated with a raised
bilirubin and abnormal liver enzymes, the abnormality is a rise in transaminases rather than
alkaline phosphatase. Furthermore, abdominal pain in acute hepatitis is almost always
relatively mild.
Pancreatitis (Option C) is incorrect. Since the amylase level is normal, pancreatitis is unlikely.
Peptic ulcer disease (Option D) is incorrect. Peptic ulcer disease may present with acute
severe abdominal pain, but this would not explain this womans abnormal blood test findings.
Right ureteric calculus (Option E) is incorrect. Ureteric calculus may present with acute
severe abdominal pain, but this would not explain this womans abnormal blood test findings.
46429

Next Question

Previous Question

Tag Question

Feedback

End Review

Difficulty: Average
Peer Responses

Session Progress
Responses Correct:

Responses Incorrect:

298

Responses Total:

298

Responses - % Correct:

0%

Blog (https://www.pastest.com/blog) About Pastest (https://www.pastest.com/about-us)


Contact Us (https://www.pastest.com/contact-us) Help (https://www.pastest.com/help)
Pastest 2016

https://mypastest.pastest.com/Secure/TestMe/Browser/429893#Top

2/2

8/22/2016

MyPastest

Back to Filters (/Secure/TestMe/Filter/429893/QA)

Question 293 of 298

A 75-year-old nun presented to her general practitioner with gradual-onset abdominal


swelling over some months and generalised abdominal tenderness, perhaps worse in her right
flank. On examination, there was inguinal lymphadenopathy. Blood tests revealed a
normochromic, normocytic anaemia, decreased serum albumin level and an elevated
creatinine of 180 mol/l. Her CA-125 level was raised.
What is the most likely diagnosis?
A

Cervical carcinoma

Cirrhosis of the liver

Haemochromatosis

Ovarian carcinoma

Wilsons disease

Explanation

The answer is Ovarian carcinoma


This scenario is very suggestive of ovarian carcinoma. Ovarian carcinoma has an annual
incidence of around 12.915.1/100 000 women per year, but the incidence rises with
increasing age: the median age of presentation is 61, with the peak occurring between
the ages of 75 and 79 years. There is an increased genetic predisposition to the condition
in carriers of the BRCA1 gene (breastovarian carcinoma syndrome). Low parity, delayed
childbearing, obesity and subfertility all appear to be risk factors for the condition.
CA-125 has reasonable sensitivity and specificity for ovarian carcinoma, although levels
may also be raised in certain benign gynaecological conditions (eg endometriosis), or
non-gynaecological conditions (eg cirrhosis, ascites, breast cancer, and so on). Crosssectional imaging is helpful in confirming the diagnosis and staging the disease, with
ultrasound, CT and MRI all being of use. This womans creatinine may be raised because
of urinary tract obstruction from the malignancy, either from the mass of the primary
tumour, or because of spread of the cancer into the omentum. It would be typical for
people with ovarian carcinoma to have a normochromic, normocytic anaemia and a low
albumin level (although any ascitic fluid is likely to be protein-rich).
https://mypastest.pastest.com/Secure/TestMe/Browser/429893#Top

1/3

8/22/2016

MyPastest

The initial treatment of choice in this scenario would be to drain the ascites (to relieve
symptoms), and to consider stenting of the ureters if they are being extrinsically
compressed by tumour mass. Chemotherapy (either with or without surgery) might be
appropriate in younger patients, but many patients present with very advanced disease,
which has a 5% 5-year survival rate.

Cervical carcinoma (Option A) is incorrect. Cervical carcinoma tends to occur in younger


women than this patient; furthermore, major risk factors include early onset of sexual activity,
multiple sexual partners and sexually transmitted infections, which are all clearly unlikely
here. In addition, the typical presenting symptom of cervical carcinoma is with
heavy/irregular vaginal bleeding, which has not been described.
Cirrhosis of the liver (Option B) is incorrect. Although this womans abdominal swelling may
suggest ascites, the absence of any obvious risk factors for or clinical features of chronic liver
disease make it very unlikely that the diagnosis is cirrhosis.
Haemochromatosis (Option C) is incorrect. Although haemochromatosis may cause cirrhosis
with subsequent ascites formation there is no specific information such as joint problems,
diabetes and skin discoloration to suggest this diagnosis.
Wilsons disease (Option E) is incorrect. Although Wilsons disease may cause cirrhosis with
subsequent ascites formation there is no specific information such as neuropsychiatric
disease to suggest this diagnosis.
46430

Next Question

Previous Question

Tag Question

Feedback

End Review

Difficulty: Average
Peer Responses

Session Progress
Responses Correct:

Responses Incorrect:

298

Responses Total:

298

Responses - % Correct:

https://mypastest.pastest.com/Secure/TestMe/Browser/429893#Top

0%

2/3

8/22/2016

MyPastest

Blog (https://www.pastest.com/blog) About Pastest (https://www.pastest.com/about-us)


Contact Us (https://www.pastest.com/contact-us) Help (https://www.pastest.com/help)
Pastest 2016

https://mypastest.pastest.com/Secure/TestMe/Browser/429893#Top

3/3

8/22/2016

MyPastest

Back to Filters (/Secure/TestMe/Filter/429893/QA)

Question 294 of 298

A 21-year-old student was admitted to an airport medical centre with severe diarrhea; he had
come directly off a flight from India. He spent most of the flight in the aircraft toilet before
collapsing just before the plane landed. He described eating shellfish from roadside stalls a
few days before travelling home. He had experienced some vomiting 2 days prior, but this had
soon settled, and he had not had any abdominal pain. Examination revealed him to be
hypovolaemic; he soiled the bed during the examination with watery diarrhoea, without
blood.
Blood test results were abnormal, with haemoglobin at the upper end of the normal range, a
raised haematocrit, and markedly elevated urea, with a smaller rise in creatinine. He also had
borderline hypoglycaemia, with a random blood glucose level of 3.4 mmol/l.
What is the most likely diagnosis?
A

Cholera

Crohns disease

Salmonella enteritidis infection

Typhoid fever

Ulcerative colitis

Explanation

The answer is Cholera


This scenario is very suggestive of cholera, caused by a strain of Vibrio cholerae,
probably contained in shellfish bought from a roadside stall. The illness is characterised
by an abrupt onset of voluminous watery diarrhoea, which in the absence of
appropriate rehydration is associated with a rapid descent into hypovolaemia, acidosis
and death. The diarrhoea is caused by the cholera toxin, which acts on intestinal
epithelial cells, causing a massive secretion of fluid and electrolytes. Although the
mortality rate of adequately hydrated patients is less than 1%, it is much higher in the
developing world, particularly in children under the age of 2 years. Treatment is with oral
or intravenous rehydration, and with with doxycycline or co-trimoxazole.

https://mypastest.pastest.com/Secure/TestMe/Browser/429893#Top

1/3

8/22/2016

MyPastest

Crohns disease (Option B) is incorrect. Given that this man has a very acute illness (without
any suggestion of previous GI symptoms, weight loss or other constitutional features), then
inflammatory bowel disease seems unlikely. In particular, the absence of abdominal pain
would be very unusual if this was a Crohns flare.
Salmonella enteritidis infection (Option C) is incorrect. S. enteritidis infection is typically
associated with significant abdominal pain and often blood and mucus mixed in with the
stool, so seems unlikely here. Additionally, Salmonella is typically transmitted by poultry,
meat, eggs or certain vegetables, rather than shellfish (as seems to be the source of
transmission here).
Typhoid fever (Option D) is incorrect. Typhoid fever (also caused by Salmonella infection) is
usually characterised by high fever but only modest GI symptoms, so also may be ruled out.
Additionally, Salmonella is typically transmitted by poultry, meat, eggs or certain vegetables,
rather than shellfish (as seems to be the source of transmission here).
Ulcerative colitis (Option E) is incorrect. Given that this man has a very acute illness (without
any suggestion of previous GI symptoms, weight loss or other constitutional features), then
inflammatory bowel disease seems unlikely. In particular, the absence of rectal bleeding
makes ulcerative colitis very unlikely.
46431

Next Question

Previous Question

Tag Question

Feedback

End Review

Difficulty: Average
Peer Responses

Session Progress
Responses Correct:

Responses Incorrect:

298

Responses Total:

298

Responses - % Correct:

0%

Blog (https://www.pastest.com/blog) About Pastest (https://www.pastest.com/about-us)


Contact Us (https://www.pastest.com/contact-us) Help (https://www.pastest.com/help)
https://mypastest.pastest.com/Secure/TestMe/Browser/429893#Top

2/3

8/22/2016

MyPastest

Pastest 2016

https://mypastest.pastest.com/Secure/TestMe/Browser/429893#Top

3/3

8/22/2016

MyPastest

Back to Filters (/Secure/TestMe/Filter/429893/QA)

Question 295 of 298

A 45-year-old man who was HIV-positive presented with diarrhoea. He had lost 12 kg in
weight during the past 2 months and looked cachetic. He was passing voluminous diarrhoea
610 times per day and during the night, but denied either abdominal pain or fever. He was
known to be only intermittently compliant with antiretroviral therapy. He had not travelled
abroad for over 10 years. His CD4 count on admission was 80.
What is the most likely causative organism?
A

A cryptosporidium

A microsporidium

Cytomegalovirus

Isospora belli

Mycobacterium avium intracellulare

Explanation

The answer is A cryptosporidium


Cryptosporidium infection in HIV-positive patients is typically associated with moderate
to severe diarrhoea, but there is often little or no abdominal pain and no fever. Severe
weight loss is also a common feature. Cryptosporidium infection particularly occurs when
the CD4 count is low (<100).

A microsporidium (Option B) is incorrect. Microsporidiosis is a possibility in this scenario but


tends only to occur in HIV-positive people with very severe immunodeficiency (ie CD4 count
<50), and is usually associated with abdominal pain.
Cytomegalovirus (CMV) (Option C) is incorrect. CMV infection is also a relatively common
cause of diarrhoea in people with HIV with low CD4 counts; however, CMV infection tends to
present with abdominal pain and fever, and a slightly lower stool frequency than that found
in Cryptosporidium infection.
Isospora belli (Option D) is incorrect. Isospora may certainly cause diarrhoea and weight loss
in people with HIV infection, but is usually acquired in tropical or subtropical regions; in the
https://mypastest.pastest.com/Secure/TestMe/Browser/429893#Top

1/2

8/22/2016

MyPastest

absence of a relevant travel history, this is less likely to be the correct diagnosis here.
Mycobacterium avium intracellulare (MAI) (Option E) is incorrect. MAI infection is a
possibility in this scenario, but tends to occur only in HIV-positive people with very severe
immunodeficiency (ie CD4 count <50), and is usually associated with abdominal pain.
Furthermore, MAI infection would also be often associated with disease in other organ
systems, and particularly a tuberculosis-like condition affecting the lungs.
46432

Next Question

Previous Question

Tag Question

Feedback

End Review

Difficulty: Average
Peer Responses

Session Progress
Responses Correct:

Responses Incorrect:

298

Responses Total:

298

Responses - % Correct:

0%

Blog (https://www.pastest.com/blog) About Pastest (https://www.pastest.com/about-us)


Contact Us (https://www.pastest.com/contact-us) Help (https://www.pastest.com/help)
Pastest 2016

https://mypastest.pastest.com/Secure/TestMe/Browser/429893#Top

2/2

8/22/2016

MyPastest

Back to Filters (/Secure/TestMe/Filter/429893/QA)

Question 296 of 298

A 31-year-old man presented with microcytic anaemia. He described a change in his bowel
habit over the past few months. His past medical history included hyperpigmented retinal
pigment epithelium, noted when he attended the optician; he had also had some teeth
removed for overcrowding during his teenage years, and had suffered from a number of
troublesome lipomas. His father had died at the age of 41 years from colonic carcinoma.
Colonoscopy revealed a number of dysplastic polyps and a right-sided colonic carcinoma.
What is the most likely diagnosis?
A

Gardner syndrome

Juvenile polyposis

Neurofibromatosis

PeutzJeghers syndrome

Sporadic colonic carcinoma

Explanation

The answer is Gardner syndrome


This scenario describes typical clinical features of Gardner syndrome. Gardner syndrome
has an autosomal dominant mode of inheritance and is characterised by multiple
adenomatous intestinal polyps, osteomas and soft-tissue tumours, including lipomas,
fibromas and epidermoid cysts. Congenital hypertrophy of the retinal pigment epithelium
is also a feature. It is caused by a mutation on the adenomatous polyposis coli gene
(APC) on chromosome 5q21. Polyps occur at a mean age of 16 years, and may be found
anywhere throughout the gastrointestinal tract. Colonic carcinoma develops in 7% of
affected individuals by the age of 21, in 50% by age 39 and in 90% by age 45, and the
average age of presentation with colonic carcinoma in unscreened people with Gardner
syndrome is 35 years. Total colectomy is the treatment of choice after the appearance of
colonic polyps.

Juvenile polyposis (Option B) is incorrect. Juvenile polyposis is an autosomal dominant


condition that presents first in childhood (rather than young adulthood), and is characterised
https://mypastest.pastest.com/Secure/TestMe/Browser/429893#Top

1/2

8/22/2016

MyPastest

by multiple hamartomatous polyps throughout the GI tract.


Neurofibromatosis (Option C) is incorrect. Neurofibromatosis-1 is associated with an
increased risk of GI stromal tumours, but the syndrome is not otherwise associated with GI
polyps or malignancies; furthermore, none of the typical dermatological manifestations of
neurofibromatosis (including neurofibromas, caf-au-lait macules, and so on) is described in
this scenario.
PeutzJeghers syndrome (Option D) is incorrect. PeutzJeghers syndrome is an autosomal
dominantly inherited condition that is associated with intestinal hamartomas as well as perioral pigmented macules however, it is not associated with any of the extra-intestinal
features found in Gardner syndrome.
Sporadic colonic carcinoma (Option E) is incorrect. Sporadic colonic carcinoma more usually
presents with a tumour in the left colon than on the right, and would be very unusual in a
patient as young as this.
46433

Next Question

Previous Question

Tag Question

Feedback

End Review

Difficulty: Average
Peer Responses

Session Progress
Responses Correct:

Responses Incorrect:

298

Responses Total:

298

Responses - % Correct:

0%

Blog (https://www.pastest.com/blog) About Pastest (https://www.pastest.com/about-us)


Contact Us (https://www.pastest.com/contact-us) Help (https://www.pastest.com/help)
Pastest 2016

https://mypastest.pastest.com/Secure/TestMe/Browser/429893#Top

2/2

8/22/2016

MyPastest

Back to Filters (/Secure/TestMe/Filter/429893/QA)

Question 297 of 298

A 24-year-old student presented to his general practitioner with lethargy and malaise
following a flu-like illness. His general practitioner noticed that he had jaundiced sclerae and
arranged blood tests. These revealed that his bilirubin was 65 mol/l; alkaline phosphatase,
albumin, transaminases and clotting were normal. A full blood count and haptoglobins were
also normal.
What is the most likely diagnosis?
A

Acute viral hepatitis

Cirrhosis

Gilbert syndrome

Haemolytic anaemia

Pancreatic carcinoma

Explanation

The answer is Gilbert syndrome


Gilbert syndrome is a genetic disorder of bilirubin conjugation, with an autosomal
pattern of inheritance. It affects more than 5% of the population and has a male to
female preponderance of 3:1. Alcohol consumption and periods of fasting (often
associated with illnesses like influenza) can be associated with worsening jaundice.
Phenobarbitone may help improve bilirubin conjugation; however, Gilbert syndrome is of
no pathological significance, and no treatment is required.

Acute viral hepatitis (Option A) is incorrect. Although the description of a flu-like illness
followed by jaundice may sound suggestive of acute viral hepatitis, the normal level of
transaminases helps to exclude this option.
Cirrhosis (Option B) is incorrect. No risk factors for or clinical features of cirrhosis are
described; furthermore, the normal albumin level and clotting would also be unusual for
someone with cirrhosis, and this option may therefore also be ruled out.

https://mypastest.pastest.com/Secure/TestMe/Browser/429893#Top

1/2

8/22/2016

MyPastest

Haemolytic anaemia (Option D) is incorrect. Haemolytic anaemia will cause an increase in


bilirubin, but the normal full blood count and haptoglobins mean that this cannot be the
diagnosis here.
Pancreatic carcinoma (Option E) is incorrect. Pancreatic cancer would be very unusual in
someone of this age; in addition, cancer of the head of the pancreas causing biliary
obstruction would be associated with a rise in alkaline phosphatase as well as bilirubin, and
this therefore can be ruled out.
46434

Next Question

Previous Question

Tag Question

Feedback

End Review

Difficulty: Average
Peer Responses

Session Progress
Responses Correct:

Responses Incorrect:

298

Responses Total:

298

Responses - % Correct:

0%

Blog (https://www.pastest.com/blog) About Pastest (https://www.pastest.com/about-us)


Contact Us (https://www.pastest.com/contact-us) Help (https://www.pastest.com/help)
Pastest 2016

https://mypastest.pastest.com/Secure/TestMe/Browser/429893#Top

2/2

8/22/2016

MyPastest

Back to Filters (/Secure/TestMe/Filter/429893/QA)

Question 298 of 298

A 22-year-old woman presented to her general practitioner after a Mediterranean holiday. Her
family often ate out during their holiday, particularly eating shellfish. She presented with
malaise, lack of appetite, jaundice and dark urine. She had a fever initially, but this subsided
once the jaundice appeared. On examination, she had hepatomegaly, as well as tenderness in
the right upper quadrant of her abdomen. Her alanine aminotransferase (ALT) and aspartate
aminotransferase (AST) levels were ten times the upper limit of the normal range; the
bilirubin was six times the upper limit of normal; and alkaline phosphatase was only mildly
elevated.
What is the most likely diagnosis?
A

Gallstones

Hepatitis A

Hepatitis B

Pancreatic carcinoma

Salmonella

Explanation

The answer is Hepatitis A


This scenario is very suggestive of hepatitis A infection, with the clue to the diagnosis
here being her holiday to the Mediterranean and her eating shellfish while she was there.
Hepatitis A is now relatively rare, but tends to occur in outbreaks; in the UK and USA,
there are currently approximately 945 cases/100 000 per year. The virus is transmitted
via undercooked food that has been subject to faecal contamination with hepatitis A
(shellfish is a common source of transmission), and poor standards of faecaloral hygiene
are likely to have played a part in this case. The acute disease is usually self-limiting and
lasts less than 6 weeks, but it can occasionally persist for up to 5 months. Infection may
be sub-clinical, or manifest as an acute hepatitic illness of variable severity; in less than
0.1% of cases, hepatitis A can cause fulminant liver failure. The diagnosis is made on the
basis of the patients history and the presence of hepatitis A virus antibodies.

https://mypastest.pastest.com/Secure/TestMe/Browser/429893#Top

1/2

8/22/2016

MyPastest

Gallstones (Option A) is incorrect. Although biliary obstruction with associated cholangitis


(from gallstones within the common bile duct) may present with fever, jaundice and right
upper quadrant pain, this condition is typically associated with a significant rise in alkaline
phosphatase and only a relatively modest increase in transaminases, ruling this out.
Hepatitis B (Option C) is incorrect. Acute hepatitis B may present with an acute hepatitic
illness; however, hepatitis B infection is spread through blood or sexual transfer rather than
faeco-orally (as is strongly implied to be the case here), and so may therefore be excluded.
Pancreatic carcinoma (Option D) is incorrect. Although biliary obstruction with associated
cholangitis (from extrinsic compression of the duct from pancreatic carcinoma) may present
with fever, jaundice and right upper quadrant pain, this condition is typically associated with
a significant rise in alkaline phosphatase and only a relatively modest increase in
transaminases, ruling this out.
Salmonella (Option E) is incorrect. Salmonella infection is faeco-orally spread, but is not
typically associated with significant hepatitis, ruling this out.
46435

End Session

Previous Question

Tag Question

Feedback

End Review

Difficulty: Average
Peer Responses

Session Progress
Responses Correct:

Responses Incorrect:

298

Responses Total:

298

Responses - % Correct:

0%

Blog (https://www.pastest.com/blog) About Pastest (https://www.pastest.com/about-us)


Contact Us (https://www.pastest.com/contact-us) Help (https://www.pastest.com/help)
Pastest 2016

https://mypastest.pastest.com/Secure/TestMe/Browser/429893#Top

2/2

Potrebbero piacerti anche